Download as pdf or txt
Download as pdf or txt
You are on page 1of 451

1.

Whi
choft
hef
oll
owi
ngst
atement
sregar
dingacut
evi
ral
hepat
it
isi
scor
rect
?
Low-
gradef
everi
smor
ecommonwi
thhepat
it
isA

2.Ri
skofmat
ernal
deat
hwi
thacut
efat
tyl
i
veri
srel
atedt
owhi
choft
hef
oll
owi
ng?
Al
loft
heabov
e

3.Mat
ernal
acut
efat
tyl
i
verofpr
egnancyi
sassoci
atedwi
thal
lexceptwi
thoft
he
f
oll
owi
ngr
ecessi
vel
yinher
it
edabnor
mal
i
tiesofmi
tochondr
ial
fat
tyaci
doxi
dat
ion?

Di
hydr
oli
poami
dedehy
drogenase

4.Af
terdel
i
ver
y,t
hel
i
verf
unct
iondet
eri
orat
ionofacut
efat
tyl
i
verofpr
egnancyhal
ted.
Nonethel
ess,
yourecommendcontinuedi
ntensi
vemater
nalsur
v ei
l
lanceduetothe
appr
ox i
matel
y20%riskofdev
elopi
ngwhichofthefol
l
owinginthedaysafterdel
i
very
?
Acut
epancr
eat
it
is

5.Whi
choft
hef
oll
owi
ngpr
egnancy
-rel
atedcompl
i
cat
ionshast
hecapaci
tyt
o
demonstr
atethemostprominental
ter
ati
onsofnor
mal
hepat
ic,
renal
,hemat
ologi
cal
,
andcoagul
ati
onlabor
atoryst
udies?
Acut
efat
tyl
i
verofpr
egnancy

6.Al
lEXCEPTwhi
choft
hef
oll
owi
ngl
i
ver
-r
elat
edchangesar
ephy
siol
ogi
cal
inpr
egnancy
?
Hy
pol
i
pidemi
a

7.Thi
simageshowst
het
ypi
cal
nodul
ar,
fibr
oti
cappear
anceofaci
rr
hot
icl
i
ver
.Whati
s
t
hemostcommoncauseoft
hiscondi
ti
oni
nthegener
alpopul
ati
on?

Al
cohol
exposur
e
8.From anet
iopat
hogenesi
sper
spect
ive,t
hepr
egnancyf
att
yli
verpi
ctur
ed
her
eis
anal
ogous t
o whi
ch chi
l
dhood
i
ll
ness?
Rey
e-l
i
kesy
ndr
omes

9.Whi
choft
hef
oll
owi
nghav
ethebestdat
afort
reat
mentofi
ntr
ahepat
icchol
est
asi
sof
pr
egnancy
?
Low-
chol
est
erol
diet

11.Youarecari
ngforapregnantpati
entwi t
hmi t
ral
stenosi
s.Sheisabletoper
form
mostacti
vit
iesofdai
lyl
i
vingwithoutsignif
icantl
i
mitati
on.Onechocardi
ogram,her
eject
ionf
racti
onis50%andhermi tr
alv al
veareameasures1.8cm2.Whi choft
he
f
oll
owi
ngv
ari
abl
esi
smostpr
edi
cti
veofahi
ghr
iskf
orcar
diaccompl
i
cat
ionsdur
ing
t
his
pr
egnancy
?
Herdegr
eeofl
eft
-si
dedobst
ruct
ion
12.Whi
choft
hef
oll
owi
ngcondi
ti
onsusual
l
yimpr
ovesdur
ingpr
egnancydue
t
o
pregnancy-
induced
hypervol
emia?
All
oft heabove
13.
Int
heUni
tedSt
ates,
whati
sthemostcommoncauseofaor
ti
cst
enosi
s
Bi
cuspi
daor
ti
cval
ve

14.
Whichofthef
oll
owi
ngdoesnotcont
ri
but
etot
he40%i
ncr
easei
ncar
diacout
put
seendur
ingnormal
pregnancy
?
I
ncr
easedl
eftv
ent
ri
cul
arcont
ract
il
it
y
15.I
napr
egnantwomanwhohaspr
evi
ousl
yunder
goneahear
ttr
anspl
ant
ati
on,
what
i
s
the approxi
mat
eri
sk f
orsuf
fer
ing a r
eject
ion epi
sode dur
ing
pregnancy
20%
16.
Whi chofthefol
l
owi
ngt
her
api
esi
ssaf
einpr
egnancyt
otr
eatacut
esupr
avent
ri
cul
ar
tachycar
dia?
Al
loft
heabov
e
17.
Whichofthefol
l
owingbestdescr
ibest
her
emodel
i
ngoft
hehear
tthatoccur
sin
nor
mal pr
egnancy?
Eccent
ri
clef
tvent
ri
cul
armassex
pansi
onwi
thspher
ical
remodel
i
ng

18.
Apat i
entwithaterm gestati
onandahi
storyofcr
it
icalaort
icst
enosispr
esentsi
n
acti
velabor.Whichofthef ol
lowi
ngmanagementstrat
egiesisadv
isabl
ewhilecar
ing
forherinherlaborcourse?
Al
loft
heabov
e
19.
Apr
egnantwomanpr
esent
stot
heemer
gencyr
oom i
nanobt
undedst
ate.Shehasa
fev
ert
o102.
6°F.Onexam,a4/
6mur
muri
snot
ed,shewi
thdr
awsf
rom pai
ndur
ing
her
abdominalexam,andt r
ackmar ksarenotedonherar msandl egs.Herf undal
hei
ghtis
approxi
matel
y32cm.Anemer gentechocardi
ogr
am i
sobtai
ned,whi
chdemonst ratesa
mitr
alvalv
evegetat
ion.Whati
sthemostlikel
yor
gani
sm under
lyi
ngherdiagnosis?
St
aphy
lococcusaur
eus

20.
Whatassoci
atedcondit
ionthepat
ientwit
hanunrepai
redvent
ri
cul
arseptal
def
ect
,
consi
der
ingpregnancy
,isathighr
iskfordev
elopi
ngbasedonherhi
story
?
Bact
eri
alendocar
dit
is

21.
Youobt
ainanechocar
diogr
am ont
hepat
ienti
npost
del
i
ver
yper
iodwi
thnew-
onset
cardi
omyopat
hy.Herej
ecti
onfracti
oni
sfoundtobe20%withnosi
gni
fi
cant
str
uctur
alorv
alvul
ardi
seasenoted.Whi
chhormonehasbeenimpl
i
cat
edinthe
devel
opmentofthi
scondit
ion?
Pr
olact
in
22.
Youar ecar
ingf
ora28- year
-ol
dG1wi t
hahistoryofmoder atemit
ralstenosiswho
presentedi
nacti
velaborandisnow8cm di l
ated.Shedesiresanaturallaborand
decli
nespainmedicat
ionsduri
nglabor.Ashercontract
ionsgetcl
osert ogether
,she
r
epor
tsincreasi
ng pai
n,i
svi
sibl
y uncomf
ort
abl
e,and i
nter
mit
tent
ly becomes
t
achy
car
dic
to130beat
spermi nut
e.Shesuddenl
ybegi
nscomplainingofshortnessofbreat
h
and
heroxy
gensatur
ati
ondecli
nest
o90%onr oom ai
r.Whatisyourbestcourseofacti
on?
Al
loft
heabov
e
23. Youobt
ainachestr
adi
ogr
aphf
ort
he30-
year
-ol
dG2P0A1pat
ientat14
weeks’
gestation.Shehasar heumaticheartdiseaseandamechani calval
verepl
acementat
age
20.Shewaspr eviousl
yonwar fari
n7.5mgdai l
y,butstoppedtaki
nghermedi cati
onwhen
shef oundoutshewaspr egnant.Nowshehasaper sist
entcoughandshortnessof
breath.Whi chofthefol
lowingparamet er
swouldindicateanextr
emelyelev
at ed
mat ernalandfetalri
skduri
ngpr egnancy?
Pul
monar
yar
ter
ial
pressur
e50mm Hg

24.Youar
etheon-
cal
lhospi
tal
i
stwheny
our
ecei
veanur
gentcal
lfr
om t
hepost
par
tum
fl
ooraboutapatientwhoi scomplaini
ngofchestpainandshort
nessofbreat
h.She
hadanuncompl i
catedv agi
naldel
iveryt
hepreviousdayandhasnosigni
fi
cantpast
medicalorsur
gicalhist
ory.Youobtainastatchestx-
ray
,whichi
sshown.Basedon
thef
indi
ngs,whatar eyoumostconcer nedfor
Car
diomy
opat
hy

25.Thecy
tochr
omeP450sy
stem i
sal
ter
edbywhi
choft
hef
oll
owi
ngdur
ingpr
egnancy
?
Al
loft
heabov
e
26.Lact
ati
oni
scont
rai
ndi
cat
edf
orwomeni
nfect
edwi
thwhi
choft
hef
oll
owi
ng
HI
V
27.Whati
sthei
nci
denceandr
ecur
rencer
iskofacut
efat
tyl
i
verofpr
egnancy
?
1i
n10,
000pr
egnanci
es,
andr
ecur
rencei
srar
e

28.Ri
skofmat
ernal
deat
hwi
thacut
efat
tyl
i
veri
srel
atedt
owhi
choft
hef
oll
owi
ng?
Al
loft
heabov
e
29.TheSoci
etyf
orMat
ernal
-Fet
alMedi
ciner
ecommendedwhi
choft
hef
oll
owi
ng
agent
sforwomenwi
thhi
ghhepat
it
isBv
iral
loadsi
npr
egnancy
?
Tenof
ovi
r
30.Whi
choft
hef
oll
owi
nghepat
icenzy
mesi
sincr
easedi
nnor
mal
pregnancy
?
Al
kal
i
nephosphat
ase

31.Whi
choft
hef
oll
owi
ngcombi
nat
ionsr
epr
esent
sappr
opr
iat
escr
eeni
ngf
orhepat
it
is
Ci
npr
egnancy
?
15-
year
-ol
dwomanwhosemotherwasachr
onici
ntr
avenousdr
uguseranddi
edofl
i
verf
ail
ure
whenthepat
ientwas4y
ear
soldscr
eenedv
iahepat
it
isCRNA

32.Whi
choft
hef
oll
owi
ngpr
egnancy
-rel
atedcompl
i
cat
ionshast
hecapaci
tyt
o
demonstr
atethemostprominental
ter
ati
onsofnor
mal
hepat
ic,
renal
,hemat
ologi
cal
,
andcoagul
ati
onlabor
atoryst
udies?

Acut
efat
tyl
i
verofpr
egnancy

33.Whi
choft
hef
oll
owi
ngv
iral
inf
ect
ionshasbeenassoci
atedwi
thamar
kedi
ncr
easei
n
t
her
iskf
ori
ntr
ahepat
icchol
est
asi
sofpr
egnancy
?
Hepat
it
isC
34.Mat
ernal
acut
efat
tyl
i
verofpr
egnancyi
sassoci
atedwi
thal
lexceptwi
thoft
he
f
oll
owi
ngr
ecessi
vel
yinher
it
edabnor
mal
i
tiesofmi
tochondr
ial
fat
tyaci
doxi
dat
ion

Di
hydr
oli
poami
dedehy
drogenase

35.Whi
choft
hef
oll
owi
ngv
aluesi
snotanor
mal
ser
um cr
eat
ini
nei
npr
egnancy1.
0

mg/
dL

36.Whi
choft
hef
oll
owi
ngst
atement
saboutpol
ycy
sti
cki
dneydi
seasei
s:

10%oft
hosewi
tht
hisdi
seasedi
efr
om r
upt
ureofani
ntr
acr
ani
alber
ryaneur
ysm.

37.Apr
egnantpat
ientwhopr
evi
ousl
ydonat
edaki
dneyandnowhasonl
yoneheal
thy
ki
dneyi
satr
iskf
orwhi
choft
hef
oll
owi
ngobst
etr
iccompl
i
cat
ions?

Gest
ati
onal
Hyper
tensi
on

38.Whati
sthel
eadi
ngcauseofsept
icshockdur
ingpr
egnancy
?

Py
elonephr
it
is

39.I
ncaseofur
inar
ytr
actobst
ruct
ionwhatdi
agnost
icopt
ioni
sthemet
hodofaf
ir
st
choi
ce?

Renal
sonogr
aphy
40.I
ntheset
ti
ngofaggr
essi
vef
lui
dhy
drat
ion,
whatper
cent
ageofpr
egnantwomenwi
th
py
elonephr
it
isdev
elopacut
eki
dneyi
njur
y?
5%

41.Whi
choft
hef
oll
owi
ngcompl
i
cat
ionsi
snoti
ncr
easedi
npr
egnantpat
ient
swi
th
nephr
oti
csy
ndr
ome?

Post
ter
m pr
egnancy

42.Whi
choft
hef
oll
owi
ngdoesnotchar
act
eri
zenephr
oti
csy
ndr
omes

Hy
per
albumi
nemi
a

43.A32-
year
-ol
dnul
l
igr
avi
dapr
esent
sforpr
econcept
ioncounsel
i
ngduet
oci
rr
hosi
s
wit
hesophageal
var
ices.Whi
choft
hef
oll
owi
ngshoul
dbei
ncl
udedi
nyour
counsel
i
ng?

Al
loft
heabov
e

44.Thef
oll
owi
ngar
evi
ral
ser
ologi
esoft
hepr
egnantpat
ient
.Whati
sherdi
agnosi
s?

1.I
gAAnti-
Hepat
it
isBcor
eantibody+
2.Hepat
it
isBsurf
aceant
ibody–
3.Hepat
it
isBeanti
gen+

Chr
oni
chepat
it
isBi
nfect
ion,
sti
l
lact
ive

45.A29-
year
-ol
dpr
imi
grav
idapr
esent
stoy
ouf
orpr
enat
alcar
e.Shei
sknownt
ohav
e
hepat
it
isCwit
hnoknownr i
skfactor
s,andyourprenatall
absconf
ir
m pr
esenceof
hepat
it
isCRNAandant i-
hepat
it
isCant i
body.Comparedtoawomanwi thanti
-
hepat
it
isCant
ibodywhoi sRNA-negati
ve,whichofthefol
lowi
ngismoreli
kel
yto
compli
cat
eherpregnancy?

Ver
ti
cal
transmi
ssi
onofhepat
it
isC
46.A27-
year
-ol
dmul
ti
grav
idacomest
oyouf
orherpost
par
tum v
isi
t.Bot
hof
her
pregnanci
eswerecompli
catedbyi
ntr
ahepati
ccholest
asisofpregnancy
.She
desir
es
anotherchi
ldi
nabout2y
ears.Whi
choft
hefoll
owingmethodsofcont
racept
iondoy
ou
recommend?

Copperi
ntr
aut
eri
nedev
ice
47. Howdoyoucounsel
theear
lypostdel
i
ver
ypat
ientr
egar
dingherf
oll
owi
ngv
iral
ser
ologi
es?

• I
gAAnti-
Hepat
it
isBcoreanti
body+
• Hepat
it
isBsur
faceant
ibody–
• Hepat
it
isBeanti
gen+

Shehasanacti
vev i
rusthatcanl
eadtoci
rr
hosi
sanddeath,
andy
ouwi
l
lref
erhert
oa
hepat
ologi
stf
orlong-t
erm managementr
ecommendat
ions.

48. Apr egnantwomanpr esentswithfl


ankpain.Sheisafebr
il
ebuttachycar
dicfrom
pai
n.Herserum creati
ninei
selevatedto5mg/ dL.Imagesfrom hermagnet
icresonance
imagingar
epr esent
edbelow-hy dronephr
osi
s.Whi chofthefol
l
owi ngi
sthebestplanof
care?

Per
cut
aneousnephr
ost
omyt
ube

49. A32-year-
oldG2P0presentsat19weeks’ gest at
ioncomplai
ningofbackpainthat
radi
atesforward.Sherepor
tsthatthepainisint
ense, andshelooksveryuncomf or
tabl
e.Shehas
notbeenfebril
e.Aur i
nal
ysi
sissignif
icantonl
yforredbl oodcell
s.Animagef r
om herrenal
ult
rasoundisprovidedbel
ow.Youdi agnosethepatientwi t
hakidneystone.Whichofthe
fol
lowingisthebestmanagementopt i
onforthepatientatthi
sti
me?

Anal
gesi
aandi
ntr
avenoushy
drat
ion

50. An18- year-


oldG1P0pr esent
sat30weeks’ gest
ati
onwit
hfeverof39oC,
chi
l
ls,
vomit
ing,dysuri
a,andfrequenturi
nati
onfor3days.Sheisal
soexper
ienci
ng
cont
racti
ons.Whati shermostl i
kel
ydiagnosi
s?

Py
elonephr
it
is

51. Youar econsul


tedabouta19-year
-oldpr
imigr
avi
daat18weeks’gest
ati
on.The
pat
ientwasdi
agnosedatherfi
rstpr
enatalcar
evisi
twit
hasympt
omat i
cbacter
iur
ia.Her
pr
ovi
dert
reat
edherwi
thni
tr
ofur
ant
oin100mgt
wicedai
l
yfor7day
s.Thepat
ient
was
seenamont haftertreat
ment,andherrepeaturi
necult
urewasnegati
ve.But
now, a
uri
neculturesenti
nerrorisposi
ti
vef
or>100,000col
ony-
for
minguni
ts/
mLgr amnegati
ve
rodsagain.Sheremainsasymptomati
c.Herprovi
deri
snotsur
ewhattodo.Whatisthe
bestresponsetothisconsul
t?

Recur
renceofasymptomat i
cbacter
iur
iai
s30%regardl
essofant
ibi
oti
cregimenandi
t
mayindicat
ecovertuppertr
acti
nfecti
on,
sotr
eatthepati
entwi
thni
trof
urantoi
n100mg
bymouthatbedtimef or21days
52.
Whi
choft
hef
oll
owi
ngphy
siol
ogi
cal
responsesi
sty
pical
l
yseeni
npr
eecl
ampt
icpat
ient
s?
I
ncr
easedsensi
ti
vi
tyt
oangi
otensi
nII

53.Amongfol
lowi
ngchoosecor
rectst
atementconcer
ninggest
ati
onal
hyper
tensi
on.
blood
pressur
eret
urnst
onormalby12weekspostpart
un
bl
oodpr
essur
eret
urnst
onor
mal
by12weekspost
par
tum

54.
Causeofconv
ulsi
oni
neclampsia
Cer
ebral
anoxi
aduetoar
ter
ialspasm

55.Whichofthefol
lowi
ngangiogenicf
actor
sar
eel
evat
edi
nwomenwhopr
oceedt
odev
elop
preecl
ampsia?
sFlt
-1(
sol
ubl
ef ms-l
i
ketyrosi
nekinase1)

56.Bestr
egi
menf
orecl
ampsi
ais
MgSo4

57.A39-
year
-ol
dG3P2pr
esent
sat30weeks’
gest
ati
onwi
thhy
per
tensi
on,
prot
einur
ia,
andheadache.Shei sdiagnosedwithsever
epreeclampsia,
andinducti
onoflabori
s
i
ndicated.Herpregnancyiscompl i
cat
edbythefetalkar
yotypeshownbelow.Whatist
he
possibl
eex pl
anati
onforherincreasedri
skforpr
eeclampsiawiththebel
owfetal
karyoty
pe?

I
ncr
easedant
iangi
ogeni
cfact
orl
evel
s

58.Thety
picalbl
oodvol
umeofagr
avi
daatt
ermis4500mL.I
npat
ient
swi
th
preecl
ampsia,
whichoft
hef
oll
owi
ngwoul
dbetheexpect
edbl
oodv
olume?

3200mL

59.Whichofthefollowi
ngnut
ri
ti
onal
suppl
ement
shasbeenshownt
oreducet
he
inci
denceofpreeclampsi
a?
Noneofthement ioned

60.Al
larefeat
uresofHELLPsy
ndr
omeexcept
Eosi
nophil
i
a

70.Whatper
cent
ageofpr
egnanci
esar
ecompl
i
cat
edbyhy
per
tensi
on?
10-20%

71.Thefol
lowi
ngcomputedtomographyi
mageshowscerebr
aledemainapostpar
tum
pati
entwit
hhypert
ensi
on,v
isi
onchanges,andconf
usi
on.Whatassoci
atedmor
bidi
tyi
s
thepati
entatr
iskf
or?
Tr
anst
ent
ori
alher
niat
ion

72.DOCf
orhy
per
tensi
oni
npr
egnancy
Labet
alol

73.Whi
choft
hef
oll
owi
ngi
sthoughtt
opl
ayasi
gni
fi
cantr
olei
nthedev
elopmentof
pr
eecl
ampsi
a?
Al
l heme
oft nt
ioned

74.Whatpr
opor
ti
onofmat
ernal
deat
hscanbeat
tri
but
edt
ohy
per
tensi
vedi
sor
der
sin
pr
egnancy
?
1i
n6

75.Whichstat
ementbestdescri
besrenalperfusionandglomerul
arfi
lt
rat
ionr
atesi
n
womenwi thpreecl
ampsi
athathasnoty etprogressedtosever
edisease?
Si
milarcomparedtonormalnonpregnantvvalues

76.Ant
ihyper
tensi
vecont
rai
ndi
cat
edi
npr
egnancy
ACEinhibi
tor
s

77.Whichoft hefol
lowi
ngisnotapathophy
siol
ogi
cal
changet
othecar
diov
ascul
ar
system seeninthesett
ingofpr
eecl
ampsia?
Incr
easedpr el
oad

78.A19-year-
oldG1P0womanat39weeks’ gestat
ionisdi agnosedwi thpr eecl
ampsia
basedonbloodpr essur
eof150/90mm Hgand2+pr oteinuriaonur i
nedi psti
ck.The
pati
entispl
acedonmagnesi um sul
fat
e,anddevel
opsflushi ngandf at
igue.Sheasks
abouttheneedf orthemagnesi
um sul
fat
e.Youexplai
nt hatitist opr
eventt heseizur
es
thatmaycompl i
catepreecl
ampsiaandmayevencausedeat h.Thepatientaskshow
seizur
esassociatedwit
hpreecl
ampsiacancausemor t
al i
ty.Whi chofthef oll
owingis
themostcommonmechani sm?
I
ntr
acer
ebr
alhemor
rhage

79.A33-
year
-ol
dwomanat29weeks’
gest
ati
oni
snot
edt
ohav
ebl
oodpr
essur
esof
150/90and2+prot
einuri
a.Theplat
eletcountandli
verfuncti
ontest
swer
enor
mal
.
Whichofthef
oll
owingisthebestmanagementf orthi
spatient?
Expect
antmanagement

80.Whati
stheunder
lyi
nget
iol
ogyoft
hepr
otei
nur
iaseenwi
thpr
eecl
ampsi
a?
I
ncr
easedcapi
l
lar
yper
meabi
l
ity

81.A21-
year
-ol
dpr
imi
grav
idapr
esent
sat36weeks’
gest
ati
onwi
thnew-
onsetheadache.
Herbl
oodpressureis150/
90mm Hg, herser
um cr
eat
ini
neis0.
8mg/ mL,
ASTi s32U/L,
andpl
atel
etcountis28,
000/μL.Whi
chofthefol
l
owingcri
ter
iaf
orsev
erepreecl
ampsi
a
i
smet?
Thr
ombocytopeni
a

82.A29-year-
oldG1P0womanat28weeks’ gest
ati
onisadmi
ttedtothehospi
tal
for
preecl
ampsia.Herbl
oodpressur
eis150/100mm Hgandherpr ot
einexcr
eti
onis500
mgi n24hours.Onhospi
talday7,shei
sdiagnosedwit
hsever
epreeclampsi
aandt he
decisi
onismadet oadmini
stermagnesium sul
fat
eanddeli
vert
hebaby.Whichoft
he
fol
lowingfi
ndingsi
smostlikel
ypresentint
hispati
entascr
it
eri
aforsev
ere
preecl
ampsia?
5gofpr ot
einuri
aexcret
edi
na24- hourperi
od

83.A21-year-
oldG1at36weeks’ gestat
ionpresentsforhercli
nicvi
sitandisnot
edto
haveabloodpr essureof148/88mm Hg.Ar epeatbloodpressure30mi nut
eslat
eri
s
146/92mm Hg.Herbl oodpressur
esthroughoutpregnancyhav ebeenbelow140/90
mm Hg.Shedeni esanycomplaint
s,andurinal
ysisi
snegat i
veforprotei
nuri
a.Whati
s
themostlikel
ydiagnosis?
Gestat
ionalHyper t
ensi
on

84.Whi
choft hefol
lowi
ngisthebestmanagementofan18-
year-
oldG1P0womanat28
weeks’
gestati
onwithabloodpressur
eof160/110mm Hg,el
evatedl
iv
erf
unct
iont
est
s,
andaplat
eletcountof60,
000/uL?

a.Magnesi
um sul
fat
ether
apyandi
nduct
ionofl
abor

85.A23- year
-ol
dprimigravi
dapresentswithabl oodpressureof160/ 104mm Hg, 3+
prot
einuria,
andrightupperquadrantdiscomf or
t.Shehasav aginaldel
i
verycomplicated
bybilater
alsul
caltearsandanest i
matedbl oodlossof1500mL.Shepr oduces110mL
ofurineinthefi
rst4hour spost
par t
um andherser um creat
ininerisesfr
om 0.98mg/ dL
to1.42mg/ dL.Whati sthemostlikel
yexplanati
onf orthi
sfinding?
Postpar
tum hemor
rhage

86.A21- year-
oldpri
mi gr
avi
daat32weeks’gest
ationhasnew-onsethyper
tensi
onand
protei
nur i
a.Sheinqui
resastowhet
herornotinit
iat
ionofananti
hypert
ensiveagentmay
behelpf ul
.Whichofthefol
lowi
ngcomplicat
ionsismoreli
kelywhenlabetal
oli
sinit
iat
ed
forpreeclampsia?
Fet
algr
owt
hrest
ri
cti
on

87.A24- year-
oldprimigr
avi
dapr esent
sat37weeks’gestati
onwithheadache,ablood
pressureof170/ 102mm Hg, andsever
eri
ghtupperquadrantpai
n.Sheisdiagnosed
withHELLPsy ndromeandunder goesanuncompli
catedinducti
onoflabor
.Herr i
ght
upperquadr antpainper
sists,andacomputedtomographyscanofherabdomen/ pel
vi
s
i
scompl et
edwi ththefi
ndingsasshownbelow.Whati sdenotedbytheasteri
sk(*)?
Subscapul
arhaemat
oma

88.A28-y
ear -ol
dG1at38weeks’ gestat
ionpresentswi
thcomplaintofcont
ract
ions.Her
bl
oodpr essureisnot
edtobe148/ 90mm Hgand152/ 96mm Hg.Shehasaur i
ne
pr
otei
n:creati
ninerat
ioof0.
4,acreati
nineof1.04mg/dL(baseli
ne0.48mg/dL),normal
ASTandALT,andpl
ateletcountof110,
000/
μL.Shedeni
esanysy
mpt
oms.Whatcr
it
eri
a
forsev
erepr
eecl
ampsiadoest hi
spati
entmeet
?
El
evat
edcr
eat
ini
ne

89.Apat
ientat28weeksgestat
ionwi
thsev
ereabdomi
nal
pai
n,bl
eedi
ngandhy
per
tensi
on.
Themostli
kel
ydiagnosi
sis
Abr
upt
iopl
acent
ae

90.Pl
acent
aisanchor
edt
othemy
omet
ri
um par
ti
all
yorcompl
etel
y

Pl
acent
aincr
eta

91.Pl
acent
apr
evi
aischar
act
eri
zedbyal
loft
hef
oll
owi
ngexcept

Pr
esent
sin1stt
ri
mest
er

92.Whi
chmaneuv
ersbel
owshoul
dbeper
for
medwhenr
epai
ri
ngacer
vical
lacer
ati
on?

Al
loft
hement
ioned

93.Thepostpar
tum pati
entwi
thsev
erehaemorrhageaft
eral
lnecessar
ymeasur
es
undergoesahyster
ectomyand5unit
sofpackedredbloodcel
lsaregiv
en,
butbl
eedi
ng
conti
nues.Whatisthemostli
kel
yeti
ologyoft
hebleedi
ng?

Di
l
uti
onal
coagul
opat
hy

94.Whi
choft
hest
atement
sbel
owaccur
atel
ydepi
ctst
hedi
ff
erencebet
ween
consumpt
ivecoagul
opat
hyanddi
ssemi
nat
edi
ntr
avascul
arcoagul
ati
on?

Al
loft
hement
ioned

95.Whati
stheappr
opr
iat
emanagementofv
ulv
ovagi
nal
hemat
omas?

I
fbl
eedi
ngceases,
smal
ltomoder
ate-
siz
edhemat
omascanbet
reat
edex
pect
ant
ly

96.Fr
om t
hef
oll
owi
nggr
aph,
whi
choft
hef
oll
owi
ngst
atement
sconcer
ningmor
bidl
y
adher
entpl
acent
a(MAP)i
saccur
ate?

Thehi
ghestr
iskf
orpl
acent
apr
evi
aiswi
thy
our5t
hpr
egnancy

97.A32-
year
-ol
dwomani
sseeni
ntheobst
etr
ical
uni
tatt
hehospi
tal
.Shei
sat29weeks’
gest
ati
on,wit
hachiefcompl
aintofsignif
icantv
aginal
bleedi
ng.Shehadastil
l
bir
thwi
th
herpr
iorpr
egnancyduetopl
acentalabrupti
on.Thepati
entasksthephy
sici
anaboutt
he
accur
acyoful
tr
asoundi
nthedi
agnosi
sofabr
upt
ion.Whi
choft
hef
oll
owi
ngst
atement
s
i
smostaccurat
e?

Fet
alul
tr
asoundi
snotsensi
ti
vei
ndi
agnosi
ngpl
acent
alabr
upt
ion

98.Whi
choft
hef
oll
owi
ngi
sthemostsi
gni
fi
cantr
iskf
act
orf
orabr
upt
iopl
acent
ae?

Tr
auma

99.A25-year-
ol dwomanat34weeks’ gestati
onisnot edtohav eapl acent
aprevi
a,af
ter
shepresentedwi thvagi
nalbleedi
ngandhasunder gonesonogr aphy.At37weeks,she
hasascheduledcesar ean.Uponcesar eansecti
on,bluishti
ssuedensel yadher
ent
betweentheut erusandmat ernal
bladderisnoted.Whi chofthefoll
owi ngi
sthemost
l
ikel
ydiagnosis?

Pl
acent
aper
cret
a

100.A22-year-
oldG1P0womanat34weeks’ gestationpr
esentswit
hmoderat
evagi
nal
bleedi
ngandnout er
inecont
ract
ions.Herbl
oodpr essure(
BP)is110/60mm Hgand
heartr
ate(HR)103beatsperminute(bpm).Theabdomeni snontender
.Whi
chofthe
foll
owingsequenceofexami
nati
onsi smostappropriat
e?
Ul
tr
asoundexami
nat
ion,
specul
um exami
nat
ion,
digi
tal
exami
nat
ion
101.A24-year
-ol
dwomanunder wentanormalvagi
nal
deli
ver
yofat er
minfantf
emale.
Aft
erthedeli
very
,thepl
acentadoesnotdel
i
verevenaf
ter30minut
es.Whichofthe
fol
lowi
ngwoul dbethenextst
epforthi
spati
ent
?
At
temptamanual
ext
ract
ionoft
hepl
acent
a

102.A34-
year
-ol
dwomani
snot
edt
ohav
esi
gni
fi
cantut
eri
nebl
eedi
ngaf
terav
agi
nal
deli
ver
ycompli
catedbyplacent
aabrupt
ion.Shei
snotedtobebl
eedingfr
om mult
ipl
e
veni
punct
uresi
tes.Whichofthefol
l
owingisthebest(
eti
opat
hol
ogycal)t
her
apy?

Cor
rect
ionofcoagul
opat
hy

103.A23-year-
oldG1P0womanat38weeks’ gestat
iondel
iv
ereda4.3kgbabyboy
vaginal
l
y.Upondeli
veryoftheplacent
a,t
her ewasnot edtobeaninvert
eduter
us,whi
ch
wassuccessfull
ymanagedi ncl
udingr
eplacementoft heuter
us.Whichoft
hef ol
l
owing
placent
ali
mplantat
ionsit
eswoul dmostlikel
ypr edi
sposetoaninv
ertedut
erus?

Fundal

104.A32-year-
oldwomanhassev erepostpart
um hemorr
hagethatdoesnotrespondto
medical
therapy.Theobstet
ricianst
atesthatsurgi
calmanagementi
sthebesttherapy.
Thepati
entdesiresfut
urechildbear
ing.
Whi chofthef
oll
owingismostappropri
ateto
achi
evethetherapeuti
cgoals?
Hy
pogast
ri
car
ter
yli
gat
ion

105.Whichoft
hefol
l
owingphy
siol
ogi
coranat
omi
cchar
act
eri
sti
cspr
edi
sposet
o
prepuber
tal
vul
vov
agini
ti
s?

Al
loft
hement
ioned

106Apr i
marygoal
oftherapyorcentr
alpr
ecoci
ouspuber
tyi
ncl
udespr
event
ionof
whichofthef
oll
owingconsequences?

Shor
tSt
atur
e

107.Whi
chofthef
oll
owinggener
all
yoccur
sfi
rstamongt
hemaj
ordev
elopment
al
event
soffemal
epubert
y?

Thel
arche

108.Whatappr
oxi
mat
eper
cent
ageofmal
i
gnantt
umor
sdi
agnosedi
nchi
l
dhoodar
eof
ovar
ianor
igi
n?

1per
cent

109.A14- year-
oldadolescentisbroughtinorexami nat
ionduet oherl ef
tbr eastbei
ng
noticeabl
ylargerthanherrightwhenshei sundr essed.Thebr eastsareot herwise
normal duri
ngex aminati
on.Thelarcheoccurredatt heageof11. 5y ears.Shecannot
thi
nkofanyspeci f
ictr
aumat otherightbr
eastorchestwal l
.Shehasnev erhadany
surgery.Cosmeticall
y,t
hedi f
fer
encei snotobv i
ouswhenshei sclothed.Whi chofthe
fol
lowingisthemostcommonet i
ologyofasy mmet ricbreastgrowthi naf emale
adolescent?

I
diopat
hic

110.Whi
choft
hef
oll
owi
ngi
sani
ndi
cat
ionorani
nter
nal
vagi
nal
exami
nat
ionofachi
l
d?

Al
loft
hement
ioned

111.Accessor
ynippl
es,whenpr
esent
,arel
ocatedal
ongtheembryoni
cmi
lkl
i
ne
ext
endingfrom t
heaxi
ll
atothegr
oinbi
lat
eral
l
y.Thei
rpresencei
ster
medwhi
choft
he
fol
lowi
ng?

Pol
ythel
i
a

112.Whichofthef
oll
owi
ngst
atement
sist
ruer
egar
dingpel
vi
canat
omyi
nthef
emal
e
i
nfantandchi
ld?
Presenceofanendomet
ri
alstr
ipeorf
lui
dwithi
ntheendomet
ri
alcav
ityoft
he
newbornuterusi
sanormalf
indingwi
thsonogr
aphy.

113.Whi
choft
hef
oll
owi
ngi
sthemostcommoncauseofcent
ral
precoci
ouspuber
ty?

I
diopat
hic

114.HowdoBMIcor
rel
atewi
thpuber
tal
dev
elopment
?
Hi
gherBMIcor
rel
atewi
thear
lydev
elopment
115.Whi
choft
hef
oll
owi
ngi
str
uer
egar
dingi
sol
atedpr
emat
uret
hel
arche?
I
tismostcommoni
ngi
rl
syoungert
han2y
ear
s.
116.Abnor
mal
breastdev
elopmentmaybeduet
oei
therf
asci
aladher
encet
othe
underl
yi
ngmusclel
ay erorduetohi
gh-
doseexogenoushormoneexposureduri
ng
pubert
y.I
nsuchcases,breast
shaveexcessiv
eforwar
dbutli
mitedl
ater
al gr
owth,
whi
ch
causesanabnor
mal shape.Thi
scondi
tionisref
err
edtoaswhichoft
hef oll
owi
ng?
Tuber
ousbr
east
s
117.Whi
choft
hef
oll
owi
ngor
gani
smsi
smostl
i
kel
ytocausepr
epuber
tal
vul
vi
ti
s?
Gr
oupAβ-
hemol
yti
cst
rept
ococcus
118.Whenanindiv
idual
exper
iencesdi
ff
icul
tyi
dent
if
yingwit
hthei
rphenot
ypi
cgender
,
percei
vi
ngthemsel
vestobeoftheopposit
egender
,thecondi
ti
onister
medwhichoft
he
fol
lowi
ng?
Genderdy
sphor
ia
119.A7-
year
-ol
dgi
rlhasexper
iencedv
ulv
ari
rr
it
ati
onwi
thext
ernal
itchi
ngandbur
ning
duri
ngurinat
ionorsever almonths.Sympt omshav epersi
steddespit
eat t
emptsto
el
iminat
epot ent
iali
rr
itantsandcont actall
ergensfr
om thebathandl aundrypr
oducts
usedbythef ami
ly.Exami nat
ion,asshownher e,
reveal
ssymmet r
icalhypopigmentat
ion
andapar chment-l
i
kethinningoft hevulvarandperi
analski
n.Whati sthemostlikel
y
di
agnosis?
Li
chenscl
erosus
120.A4-
year
-ol
dgi
rli
sbr
oughti
nbyherpar
entwi
tht
heconcer
nthat“
thel
i
psofher
vaginahavest
uckt
ogether
.”I
nit
ialmanagementoflabi
aladhesi
onoraggl
uti
nat
ioni
na
chil
d,asshownher
e,doesNOTi ncl
udewhichofthefol
lowingopt
ions?
Sur
ger
y
121.Pr
ecoci
ouspuber
tyassoci
atedwi
thbonydy
spl
asi
aandcaf
éaul
aitspot
sonski
nis
notedi
n:
McCune-
Albr
ightsy
ndr
ome
122.Af
etal
ovar
ianmassi
snot
edasani
nci
dent
alf
indi
ngdur
ingt
hir
d-t
ri
mest
er
sonographicassessmentoffetalgr
owth.Thefemalei
nfanti
sdeli
ver
edatt er
m wit
hout
i
ncident.Lowerabdominalf
ullnessispal
pateddur
ingneonat
alexaminat
ion,and
tr
ansabdomi
nal
sonographyshowsa4.
2-cm si
mplerightov
ari
ancyst
.Theinf
anti
s
afebr
il
eandi
nnodist
ress.Whatist
hemostappropr
iatemanagementoft
hisovar
ian
mass?
Obser
vat
ionwi
thr
epeatsonogr
aphyi
n6weeks
123.A6-
year
-ol
dgi
rli
snot
edt
ohav
ebr
eastdev
elopmentandv
agi
nal
spot
ti
ng.No
abnor
malhai
rgrowt
hisnoted.A10-
cm ov
ari
anmassi
spal
pat
edonr
ect
alexami
nat
ion.
Whichoft
hefol
l
owingist
hemostl i
kel
ydi
agnosi
s?
Gr
anul
osa-
thecacel
ltumor
124.An8-
year
-ol
dgi
rli
sbr
oughti
norev
aluat
ionof4day
sofi
nter
mit
tentv
agi
nal
bleedi
ngandf oul
discharge.Sheshowsnosi gnsofpubertaldevelopment.Thecauseof
thebleedi
ngisnotapparentduringphy
sicalexaminat
ion,andt hedecisi
onismadet o
proceedwithexaminati
onunderanesthesiaandsali
nev aginoscopy.Whatisthemost
l
ikelyfi
ndi
ng?
For
eignobj
ect
125.A16-y ear-
oldadolescentfemaleisbroughtint
ot hepediat
rici
an’soffi
ceduet ono
breastdevelopment.Thepat i
ent’
smot hernotesthatbothofpat i
ent’
ssistershadonset
ofbreastdev el
opmentatage10, andalsoallofherfri
endshav ealr
eadybegun
menst r
uati
ng.Exami nati
onrevealsTannerstageIbr eastandpubic/axil
laryhai
r,andi
s
otherwi
seunr emarkable.Whichoft hefol
lowi
ngi sthemostl i
kelydi
agnosis?
Del
ayedpuber
ty
126.A5-year
-oldfemal
echil
disbroughti
ntothephysici
an’
soffi
ceforbr
east
devel
opmentandmenses.Onexami nati
on,thechil
disfoundtohaveTannerIVbr
east
devel
opmentandTannerI Vpubi
candaxil
laryhai
r.Whichofthefol
l
owingisthemost
l
ikel
yther
apyf ort
hispat
ient
?
Gonadot
ropi
n-r
eleasi
nghor
mone(
GnRH)agoni
st
127.I
nanadolescentwit
hanot herwi
senor
mal
breastandchestwal
lexami
nat
ion,
what
i
sthebestapproachtobreastasymmetr
y?
Reassur
ethatmostcasesofbr
eastasy
mmet
ryr
esol
vebycompl
eti
onofbr
eastdev
elopment
128.Whyi
scar
eful
preoper
ati
vepl
anni
ngwar
rant
edwi
thcongeni
tal
vagi
nal
cyst
s,as
showni
nsonogr
am bel
ow?
Somemayext
endupt ot
hebr
oadl
i
gamentandanat
omi
cal
l
yappr
oxi
mat
ethedi
stal
cour
seoft
heuret
er.
129.A22- year-
oldG3P0womanpr esentsasanewpat ientforconsultationregarding
herhistor
yofmul tipl
emiscarri
ages.Hergynecologichistoryisotherwiseunr emarkable.
Duringphysicalexaminati
on,younotethattheuterusismar kedl
ydev i
atedt otheleft.
Transvagi
nal sonographyisperf
ormed,and3Di magesr eveal aut
erust hatisdeviated
andhasabanana- shapedcavity
,asshownbel ow.Whati syourdiagnosi s?
Uni
cor
nuat
eut
erus
130.Whi
choft
hef
oll
owi
ngar
eseeni
nof
fspr
ingofwomenwhot
ookdi
ethy
lst
il
best
rol
(
DES)dur
ingt
heassoci
atedpr
egnancy
?
Al
loft
hement
ioned
131.Whenshoul
dsurgical
reconst
ruct
ionofabi
cor
nuat
eut
erus(
Str
assman
metropl
ast
y)beper
formed?
Withr
ecurr
entpr
egnancyl
ossandnoot
heri
dent
if
iabl
ecauseexceptt
heut
eri
ne
anomaly
132.Oneofy ourpati
ent sbr
ingsher13- year -
olddaught
erf orconsul
tationr egardingthe
gi
rl’
scompl ai
ntsofpelvicpain.Thedaught erhasexper
iencednor mal puber tal
developmentt odatebuthasnoty etexperiencedmenarche.Shebeganhav ingmont hly
pelv
icpai nabout1y earago, andt hepainhasbeenpr ogressivel
ywor sening.Dur ing
physicalexaminati
on,herext ernalgenit
alia,incl
udi
nghymeneal ri
ng,appearnor mal .
Howev er
,proxi
mal t
ot hehymen, onlyav aginaldi
mplei
sf ound.Rectov aginal
exami nati
onconfir
mst hepresenceofaut erusandcervix.Whatisyourdi agnosi s?
Vagi
nal
atr
esi
a
133.Inapregnantpopul
ation,
whatisthemostcommoncauseoft
hecondi
ti
on
port
rayedi
nt hechestx-
rayimagebelow?
(
enl
argedpul
monar
yhi
l
arar
ter
iesandat
tenuat
edper
ipher
almar
kings)

134.TheWor l
dHeal t
hOrgani
zat
ioni
dent
if
ieswhi
choft
hef
oll
owi
ngcondi
ti
onsas
prohi
bit
iveofpregnancy
?

Pul
monar
yar
ter
yhy
per
tensi
on
135.A32- year-ol
dG2P1pr esentstoy ouroff
iceat17weeks’ gestat
ion.Shereport
sa
hist
oryoftetralogyofFal
lot,whichwasr epairedini nf
ancy .Sheappearscomf or
tabl
eon
theexam tablewi t
hahear tr
ateof92beat spermi nuteandabl oodpressureof96/62
mm Hg.Howev er,
shedoesr eportsigni
fi
cantpal pitat
ionsanddy spneawhencl i
mbing
thetwofli
ght sofstai
rstoherapar tmentont het hirdfl
oor.WhichNewYor kHeart
Associat
ioncl assbestdescri
besherf uncti
onal disabili
ty?
Cl
assI
I
136.A30-year
-ol
dG2P0A1pr esent
sat11weeks’gestati
ont oestabli
shcare.Shehasa
hi
storyofr
heumat i
chear
tdi seaseandhadamechani cal
valvereplacementatage20.
Shewasprev i
ousl
yonwarf ari
n7.5mgdaily
,butstoppedtakinghermedi cati
onwhen
shefoundoutshewaspregnant1weekago.Whati sherriskforwarfari
nembr yopat
hy
basedonherwar f
ari
ndosing?
8%
137.Forwhi
choft
hef
oll
owi
ngcar
diaccondi
ti
onsi
sconduct
ionanal
gesi
ahi
ghl
y
recommended?
Repai
redhy
popl
ast
icl
efthear
tsy
ndr
ome
138.Regar
dingdi
agnost
icst
udi
es,
whi
choft
hef
oll
owi
ngcor
rect
lydescr
ibesanor
mal
changet
hati
schar
act
eri
sti
cofpr
egnancy
?
Anav
erage15-
degr
eel
eftaxi
sdev
iat
iononel
ect
rocar
diogr
am
139.Aschemat
icofEi
senmengersy
ndr
omeduet
oav
ent
ri
cul
arsept
aldef
ecti
s
depi
cted.Whati
sthemostcommoncauseofdeat
hinpr
egnantwomenwi
tht
his
condi
tion?
Ri
ghtVent
ri
cul
arf
ail
ure
140.Whi
chofthef
oll
owi
ngt
her
api
esi
ssaf
einpr
egnancyt
otr
eatacut
esupr
avent
ri
cul
ar
tachy
car
dia?
Al
loft
heabov
e
141.Youar
ecaringf
ora24-year
-oldG1wi t
hsevereaort
icst
enosi
s.Wheni
npr
egnancy
areyoumostconcer
nedaboutcardiacdecompensati
on?
Per
ipar
tum
142.Youarecari
ngforcompl ainsofshor t
nessofbreathandchestpainwi thminimal
exer
ti
onanuncomplicatedprimi gravidaat32weeks’ gestat
ion.Anechocar di
ogram
demonstrat
esami t
ralvalvesur facear eaof2.1cm2, adil
atedlef
tat
rium, anaortic
val
veareaof3.7cm2,ar i
ghtv ent r
icularsyst
oli
cpressureof20mm Hg, andanej ecti
on
fr
acti
onof55%.Youalsoobt ainachestx- ray,whi
chisshown.Whati sthel i
kel
y
underl
yi
ngcauseofhersy mptoms?
Mi
tr
alSt
enosi
s
143.Whichoft
hef
oll
owi
ngpat
ient
sdonothav
eani
ndi
cat
ionf
orendocar
dit
is
prophy
laxi
s?
Apat
ientwi
thav
ent
ri
cul
arsept
aldef
ectr
epai
redwi
thasy
nthet
icpat
chwi
thoutacur
rentdef
ect
144.Youar
ecar
ingf
orapat
ientwi
thanunr
epai
redv
ent
ri
cul
arsept
aldef
ectwhoi
s
consi
deri
ngpregnancy.Youper
form anechocar
diogr
am toassessherfut
urer
isks.
Whichofthefol
lowingparamet
erswouldindi
cat
eanextremelyelev
atedmater
nal and
fet
alr
iskduri
ngpregnancy?
Ar
ightv
ent
ri
cul
arsy
stol
i
cpr
essur
eof75mm Hg
145.Whi choft hef
oll
owingmethodsofant
enat
alf
etal
survei
ll
ancehasbeen
conclusi
v elyshowntobeofbenefi
tint
hemanagementofpregnanci
escompl
icat
edby
chronichy pert
ensi
on?
Ser
ial
sonogr
aphi
cev
aluat
ionsoff
etal
growt
h
146.I
naddi
ti
ont
ochr
oni
chy
per
tensi
on,
whi
choft
hef
oll
owi
ngcondi
ti
onsar
e
consi
der
edhi
gh-
ri
skf
ordev
elopi
ngpr
eecl
ampsi
a?
Sy
stemi
clupuser
ythemat
osus
147.Forwomenwi
thchr
oni
chy
per
tensi
on,
whi
chmodi
fi
abl
efact
orf
urt
heri
ncr
easest
he
r
iskf
ort
hepl
acent
alabr
upt
ion?
Smoki
ng
148.Antihy
per
tensi
vet
her
apyi
smandat
oryf
orwhi
choft
hef
oll
owi
ngwomendur
ing
pregnancy?

30-
year
-ol
dG3P2wi
thabl
oodpr
essur
eof160/
110mm Hgandahi
stor
yofast
hma
149.Allexceptwhi
choft
hef
oll
owi
ngsuppor
tthedi
agnosi
sofsuper
imposed
preecl
ampsi a?

Decr
easedser
um al
kal
i
nephosphat
asel
evel

150.Whi
choft
hef
oll
owi
ngi
samaj
orf
act
orcont
ri
but
ingt
othei
nci
denceofper
inat
al
mor
tal
i
tynot
edi
npr
egnanci
escompl
i
cat
edbychr
oni
chy
per
tensi
on?

Al
loft
heabov
e

151.A33-
year
-ol
dG1at32weeks’
gest
ati
onhasbeendi
agnosedwi
thchr
oni
c
hypert
ensi
onwi thsuperi
mposedpr eeclampsiavi
awor seninghy pert
ension.Sheis
asymptomaticwithnormallabsandnor malfetal
gr owth.Anincreaseinthedoseofher
anti
hyper
tensiv
emedi cati
onhasr esul
tedinnonsev er
ebl oodpressures.Sheasksy ou
aboutexpect
antmanagementofsuper imposedpr eecl
ampsi a.Whichoft hefol
l
owingis
themostappropriat
e,evi
dence-basedresponse?

I
tisreasonabl
e,wi
thcl
oseinpat
ientsur
vei
l
lance,
topur
sueexpect
antmanagement
,butt
her
ear
e
st
il
lri
sksforadver
seoutcomes.

152.A37-
year
-ol
dG2P2wi
thnomedi
cal
diagnosesandanor
mal
bodymassi
ndexon
annualwell
womanex am f
oundtohav eBP142/ 92mm Hg, andaft
ercheckingsev
eral
ti
mesov erthenextweekathome, shecall
swithabloodpressurel
ogthatrangesfr
om
140–154/90–100mm Hg.Accor dingtotherecommendati
onsoft heEi
ghthJoint
Nati
onalCommi tt
ee,whatisyourgoalbl
oodpr essur
ewit
ht her
apy?

&l
t;
140/
90mm Hg

153.Ms.Thomaspr
esent
sforapr
econcept
ioncounsel
i
ngv
isi
t.Shei
sa37-
year
-ol
dG0
wit
habodymassi ndexof38kg/ m2, 4-
yearhistor
yofchroni
chy per
tensi
ontreat
edwit
h
amlodi
pine5mgperday ,anda2- y
earhistoryofdi
abeteswhichshemanageswi thdi
et
andexercise.Herbloodpressurerangeis128–144/ 80–94mm Hgf rom theweekpri
or.
Whatwor kupwouldy ouli
ket opursuereferentt
oherdiagnosisofchroni
chypert
ensi
on
andprepregnancyriskst
rati
ficat
ion?

Serum creati
nine,
uri
nepr
otei
n-t
o-cr
eat
ini
ner
ati
o,el
ect
rocar
diogr
am,
andmat
ernal
echocardiogr
am

154.Youar
eseei
ngMs.Bl
anchef
orpr
enat
alcar
e.Shei
sa37-
year
-ol
dG1at8weeks’
gestati
on.Shehasbeeny ourpat
ientf
orsever
alyear
s,andchar
trevi
ewshowsf ormer
bl
oodpr essurereadi
ngshav ebeen110–120/70–80mm Hgatpr i
orvisi
ts.Theblood
pressurenotedatherfi
rstobstet
ricvi
siti
s150/96mm Hg.Whichofthef ol
lowi
ngisthe
mostappr opri
atenextstep?

Requestt
hatshehav
eherbl
oodpr
essur
emeasur
edaf
ewt
imesov
ert
henex
t2weeksand
r
etur
nwi
thherl
ogf
orr
evi
ew.

155.A36-
year
-ol
dmul
ti
grav
idawi
thchr
oni
chy
per
tensi
onpr
esent
sforagr
owt
h
ult
rasoundat36weeks’ gestat
ion.Theult
rasoundreportshowsy outhatesti
matedf
etal
weightcorrespondto33weeks’ ,headsize-to34weeks’ ,
whileabdominal
cir
cumf er
enceandf emurlengt
h–t o32weeks’ gestat
ion.Whichofthef ol
l
owingist
rue
regardi
ngthedi agnosismadeont hegrowthultr
asound?

Fetal
-growt
hdysfuncti
oni
smorel
i
kel
yinchr
oni
cal
l
yhy
per
tensi
vewomenwhodev
elop
superimposedpr
eeclampsi
a.

156.Whi
choft
hef
oll
owi
ngst
atement
sregar
dingacut
evi
ral
hepat
it
isi
scor
rect
?

Low-
gradef
everi
smor
ecommonwi
thhepat
it
isA.

157.Af
terdel
i
ver
y,t
hel
i
verf
unct
iondet
eri
orat
ionofacut
efat
tyl
i
verofpr
egnancyhal
ted.
Nonethel
ess,
yourecommendcontinuedi
ntensi
vemater
nalsur
v ei
l
lanceduetothe
appr
ox i
matel
y20%riskofdev
elopi
ngwhichofthefol
l
owingintheday saf
terdel
i
very
?

Acut
epancr
eat
it
is

158.Al
lEXCEPTwhi
choft
hef
oll
owi
ngl
i
ver
-r
elat
edchangesar
ephy
siol
ogi
cal
in
pr
egnancy
?

Hy
pol
i
pidemi
a

159.Thi
simageshowst
het
ypi
cal
nodul
ar,
fibr
oti
cappear
anceofaci
rr
hot
icl
i
ver
.What
i
sthemostcommoncauseoft
hiscondi
ti
oni
nthegener
alpopul
ati
on?

Al
cohol
exposur
e

160.Fr
om anet
iopat
hogenesi
sper
spect
ive,
thepr
egnancyf
att
yli
verpi
ctur
edher
eis
anal
ogoust
owhi
chchi
l
dhoodi
l
lness?

Rey
e-l
i
kesy
ndr
omes

161.Whi
choft
hefol
lowingisnotar
ecommendedr
equi
sit
eforr
enal
transpl
antpat
ient
s
whowanttoat
temptpregnancy
?

Pr
otei
nur
ia&l
t;
300mg/
day

162.Thecl
ini
cal
present
ati
onofawhatcondi
ti
onusual
l
yincl
udeshy
per
tensi
on,
hematur
ia,r
ed-
cel
lcast
s,py
uri
a,pr
otei
nur
ia,
edema,
hyper
tensi
on,
andci
rcul
ator
y
congest
ion?
Gl
omer
ulonephr
it
is

163.Youngpr
egnantpat
ientwi
thpy
elonephr
it
isandl
aborcont
ract
ionst
ransf
err
edt
o
l
abordepart
mentformonitori
ng.Sheisgi
venint
ravenous( I
V)anti
bioti
cs,
IVfl
uids,
acet
aminophen,
andaβ- agoni
stbecauseofthecontract
ions.Whatcomplicat
ionis
moreli
kel
yinthesett
ingofβ-agoni
stuseinpati
ent
swi thpyelonephri
ti
s?

Pumonar
yedema

164.Whati
sthecut
offf
oraur
inar
ypr
otei
n-t
o-cr
eat
ini
ner
ati
oonaspotur
inesampl
ein
pr
egnantpat
ient
?

0.
3

165.Whi
choft
hef
oll
owi
ngi
snotcommonl
yfoundi
npat
ient
swi
thaur
ethr
al
di
ver
ti
cul
um?

Ur
inar
yRet
ent
ion

166.Whi
choft
hef
oll
owi
ngi
sthemostcommonf
orm ofacut
egl
omer
ulonephr
it
is
wor
ldwi
de?

Ber
gerdi
sease

167.A28-
year
-ol
dat19weeks’
gest
ati
onpr
esent
stot
heemer
gencyr
oom wi
th
complaint
sofr i
ghtupperquadr
antpai
n,fev
er,andvomiti
ng.Exam r
eveal
sri
ghtupper
quadranttender
ness,andshehasatemperatureof38.
7°C.Onediagnost
ici
mageis
shownher e.Commonbi l
eductobstr
uct
ionisnotsuspected.Whati
sthenextmost
appropri
atestepinhermanagement?

Lapar
oscopi
cchol
ecy
stect
omy

168.Yousendaper
ipher
albl
oodsmearoft
hepat
ientat34weeks’
gest
ati
onwi
than
acutefat
tyl
iver
,andarepr
esent
ati
vesl
i
dei
sshownbel
ow.Whati
stheunder
lyi
ng
eti
ologyoft
hebloodsmearfi
ndi
ngs?

Decr
easedchol
est
erol
product
ion

169.Whi
chcombi
nat
ionofl
abor
ator
yfi
ndi
ngsbel
owwoul
dbemostconcer
ningf
or
acut
efat
tyl
i
verofpr
egnancy
?

Hematocri
t32%, cr
eat
ini
ne2.
0mg/
dL,
plat
elet
s90/
L,AST400U/
L,f
ibr
inogen130
mg/dL,andglucose65mg/dL
170.A26-year-ol
dprimi
grav
idapresentsat34weeks’ gestat
ionwithnauseaand
vomiti
ng,f
atigue,andepi
gastr
icpain.Allexceptwhichofthef ol
lowingar
ecli
nical
charact
eri
sti
cst hati
ncr
easeherri
skf oracutefatt
yliverofpregnancyastheunderl
yi
ng
cause?

Femal
efet
us

171.A36- y
ear -
oldmul ti
grav
idapresentsforpr econcept i
oncounsel ing.Herobst etri
c
historyi ncl
udest wof ull
-t
ermv agi
nal del
iveri
esaf t
erpr egnanci escompl icatedby
gest ationaldiabetesandaf ir
st-t
ri
mest erpregnancyl oss.Shecomest oy outodi scuss
herr ecentdi agnosisofnonalcohol i
cfatt
yl i
verdisease( NAFLD)byherpr i
mar ycare
physi cian.Diagnosiswasbasedoni magingper formedf oranepi sodeofabdomi nal pai
n.
Herl iverfunctiontestsarenormal .Hermedi calhistoryisot herwisenot ablefor
longst andingobesi t
y( bodymassi ndex34kg/ m2)anddi abet es(hemogl obinA1c8. 2%)
diagnosed2y earsago.Whi choft hefoll
owingshoul dbei ncludedi nyourcounsel i
ng?

Thebestint
erv
enti
onstoreducet
her
iskt
oaf
utur
epr
egnancyar
ewei
ghtl
ossand
opt
imizi
ngherglucosecont
rol
.

172.A25-year-
oldG1P0at16weeks’gest
ati
onpr esent
sf orpr
enat al
care.Shehasa
hi
stor
yofaki dneytr
anspl
ant3year
sago.Shei sstabl
eonpr ednisoneandazat hi
opr
ine.
Thepati
enthasnotexperi
encedanyrej
ect
ion,andherbloodpr essureiswellcont
rol
l
ed.
Whichofthefoll
owingobst
etr
iccompli
cat
ionsisnotincreasedinher?
Per
ipar
tum car
diomy
opat
hy
173.Fort
hepati
entwit
hseverepreecl
ampsia,whi
chofthefol
l
owi
ngi
sani
ndi
cat
ionf
or
pri
marycesar
eandeli
verywi
thoutanattemptati
nduct
ion?

Mal
present
ati
on

174.A42-y ear-ol
dwomanpresentsf orprenat
al car
e.Shehasademandingprofessi
onal
careerandr eport
ssheisunwil
li
ngt oattendfrequentprenat
alvi
sit
satt
heendof
pregnancy.Youdi scour
aget
his,explai
ningthesev i
sit
sarerecommendedforwhichof
thefoll
owingr easons?

Al
loft
hement
ioned

175.Amongf
oll
owi
ngchoosecor
rectst
atementconcer
ningpr
otei
nur
iai
npr
eecl
ampt
ic
pati
ent

r
efl
ect
sthesy
stem-
wideendot
hel
i
all
eak

176.Amulti
parouswomanwit
hnoprenat
alcar
epresent
sinacti
velaborwi
thablood
pressur
eof156/92mm Hgandpr
otei
nuri
a.Herneonat
eisbornvaginal
l
yaf
teran
uncomplicat
edlaborcourse,witht
hesecondstageoflaborlast
ingonly15mi nut
es.The
i
nfanti
snot edtohavepet echi
aeonthescal
pandchest ,andoozingatthesit
eofhis
heelst
ick.Anini
ti
alplat
eletcounti
s32,000/
μL.Whichoft hefol
lowingdisor
dersisleast
l
ikel
ytobet hecauseofthei nf
ant’
sthr
ombocytopeni
a?

Mat
ernal
Preecl
ampsi
a

177.AG3P2at16weeks’ gest
ati
onpr esentst ot
heemergencydepart
mentcomplai
ning
ofvagi
nalbl
eeding.Herbloodpressureis148/ 96mm Hgandshehas3+pr otei
nuri
aon
aspeci
menobt aineddur
ingbladdercatheterizat
ion.Whi
chofthefol
l
owingmayexplain
thedev
elopmentofpreeclampsiainthispatient?

I
ncr
easedv
olumeofchor
ioni
cvi
l
li

178.Al
loft
hef
oll
owi
ngcanpr
edi
sposet
ocoagul
opat
hyEXCEPT

Pl
acent
apr
evi
a

179.Al
loft
hefoll
owingareindi
cat
ionsf
ort
ermi
nat
ionofpr
egnancyi
nAPH(
ant
epar
tum
haemorr
hage)pat
ientexcept

Tr
ansv
ersel
i
e

180.Theuseofwhol
ebl
oodf
ormassi
vehemor
rhagei
ssuppor
tedbywhi
choft
he
fol
lowi
ng?

Al
loft
hement
ioned

181.A34-year-ol
dG3P3beginshavingbri
skbrightredbleedi
ngf
oll
owi
ngcomplet
ionof
av agi
naldeli
very
.Yougivehercar
boprosttr
omet hamineandperf
ormthemaneuv
er
pictur
edbelow.Whatelseshouldbeimmediatelyconsi
dered?

Al
loft
hement
ioned

182.Al
lar
etr
ueaboutpl
acent
apr
evi
aexcept
:

I
ncr
easedut
eri
net
one

183.Rxofchoi
cei
npl
acent
aaccr
etai
nmul
ti
par
ousf
emal
e

Hy
ster
ect
omy

184.DI
Ciscommonwi
th:
.
Abr
upt
iopl
acent
a

185.Pl
acent
aaccr
etai
sassoci
atedwi
th:

Pl
acent
apr
evi
ainpr
esentpr
egnancy
Mul
ti
par
a
Ut
eri
nescar
186.Duri
ngev
aluat
ionofpost
part
um hemorrhagef
oll
owingav
agi
nal
del
i
ver
y,whi
choft
he
fol
l
owingmaneuver
sormedicati
onsmightbeused?
Bi
manual
uter
inecompr
essi
on
187.Whati
srepr
esent
edi
nthef
oll
owi
ngi
mage?
Fol
eycat
het
er
188.A28- year-ol
dwomanat32weeks’ gestat
ionisseeni nt
heobstetr
icaltri
ageareafor
vaginalbleedi
ngdescri
bedassigni
fi
cantwithclots.Shedeni
escrampi ngorpain.An
ult
rasoundi sper
formedreveal
i
ngthattheplacentaiscoveri
ngtheinternalosofthe
cervix.Whichofthefol
l
owingisari
skfactorforthispati
ent
’scondi
tion?
Mul
ti
plegest
ati
ons
189.A33- year-ol
dwomanG3P2whohadt wopriorcesareansiscurr
entl
yat38weeks’
gestation.Sheisnotedtohaveaposteriorpl
acenta.
Onul t
rasound,t
hereisevi
denceof
possibleplacentaaccr
eta.Thepat
ientiscounseledaboutthepossibl
eriskofneedfor
hyst
er ectomy.Whichofthefoll
owi
ngi sthemostaccur atestat
ement?
Thepost
eri
orpl
acent
aisassoci
atedwi
thl
essofar
iskf
oraccr
etat
hananant
eri
orpl
acent
a
190.A29-
year
-ol
dG1P0womanat39weeks’
gest
ati
ondel
i
ver
edv
agi
nal
l
y.Herpl
acent
a
doesnotdel i
vereasi
l
y.Amanual extract
ionoft heplacentaisatt
empt edandt he
placentaseemst obeadherentt
ot heuterus.Ahysterectomyiscontempl at
ed,butthe
patientref
usesduetost r
ongl
ydesiri
ngmor echil
dren.Thecordisl i
gatedwithsutureas
highaspossible.
Thepat i
enti
sgiventheopt i
onofmet hotr
exatetherapy.Whichofthe
foll
owingisthemostlikel
ycomplicati
onaf t
erthisint
erventi
on?
Coagul
opat
hicbl
eedi
ng

191.A33- y
ear-ol
dwomanat37weeks’gest
ati
on,confi
rmedbyfir
st-
tr
imester
sonography,pr
esent
swit
hmoderat
elysever
evaginalbl
eedi
ng.Sheisnotedon
sonographytohaveapl
acent
aprev
ia.Whichofthefol
lowi
ngisthebestmanagement
fort
hispatient
?

Caesar
eandel
i
ver
y

192.A26-year
-ol
dG2P1womanunder wentanormalvaginaldel
iver
y.Av
iabl
e3.9kg
maleinf
antwasdeli
ver
ed.Thepl
acent
adeliv
eredspontaneously
.Theobstet
ri
ciannot
ed
signi
ficantbl
oodlossfrom t
hev agina,tot
ali
ngapproximatel
y700mL.Theuter
ine
fundusappear edtobewellcontracted.Whichoft
hef oll
owingi
sthemostcommon
etiol
ogyf ort
hebleedi
nginthispatient?

Geni
tal
tractl
acer
ati
on

193.A32- year-
oldG1P0womanat40weeks’ gest
ati
onundergoesanormal vaginal
del
ivery.Deli
veryoft
heplacentai
scomplicatedbyaninvert
eduter
us,wit
hsubsequent
hemor r
hagel eadi
ngto1500mLofbl oodloss.Sheismanagedwi t
hatransfusionof
ery
throcytes.Whichoft
hef ol
lowi
ngisthebestexplanat
ionofthemechanism of
hemor r
hage?

I
nver
tedut
erusl
eadst
oinabi
l
ityf
oranadequat
emy
omet
ri
alcont
ract
ionef
fect
.

194.Allst
atesi
ntheUnit
edStat
eslegal
l
yall
owami norwomant
oconsenttomedical
examinati
onandtreat
mentonherownbehal
final
lofthefol
l
owi
ngsi
tuat
ionsEXCEPT:

Request
ingaschool
-sponsor
edspor
ts’
phy
sical
exami
nat
ion

195.Accor
dingtotheU.
S.FDAvaccinestar
geti
nghumanpapi
l
lomav
irusi
nfect
ionar
e
approv
edfrom whatage(i
nyear
s)forfemalesormal
es?

196.Wit
hprepuber
tal
v ul
vov
agini
ti
s,whatappr
oximateper
cent
ageofbact
eri
alcul
tur
es
obtai
nedfr
om vagi
nalswabsshowonlynormalgeni
talf
lor
a?

75per
cent

197.Asmoot h,fi
rm,mobil
e,nont
enderri
ght-
breastmassispal
pabl
eduri
ngexami
nati
on
ofa12-year-oldgir
l.I
tis1cm ini
tsgreat
estdimensi
onandappearssol
i
d
sonogr
aphi call
y,asshownhere.Shehashadnomaj orheal
thprobl
emstodat
e.Whati
s
themostlikelydiagnosi
s?

Fi
broadenoma

198.A14- y
ear-
oldwomanpr esentswithincreasingl
ypainf
ul,cy
clicmensessince
menar che,9mont hsago.Menstrual
flowi snor mali
namountanddur at
ion.Or
al
analgesicsandnonsteroi
dalanti
-i
nfl
ammat orymedicati
onsnol ongeradequat
elycontr
ol
hersev eredysmenorr
hea.Whichofthef oll
owi ngcongenit
alanomaliesismostli
kely
present?

I
mper
for
atehy
men
199.Medi
cat
ionusedi
nthet
reat
mentofi
diopat
hiccent
ral
precoci
ouspuber
tyi
ncl
ude:

GnRHagoni
sts

200.Mül
l
eri
ananomal
i
esar
eassoci
atedwi
thanomal
i
esofal
loft
hef
oll
owi
ngsy
stems
EXCEPT:

Hepat
ic

201.A16-year-
oldnul l
i
gravidapresentst ot
heemer gencydepar t
mentwi t
hcompl aints
ofabdominalandv aginalpain,
wor seningduringthepastsev eralmonths.Shedescr i
bes
thepainasbeingmost l
yonherr i
ghtsi deandmuchwor seduringmenstruati
on.Dur i
ng
yourphysi
calexami nati
on,apatentv aginaandcer vi
xarenot ed,butal
ateralvagi
nal and
pelv
icmassi spalpated.Transvagi
nal sonographydemonst ratesasingleuterusand
cervi
xbutalsoal ar
gepel vi
cmassf i
ll
edwi t
hcompl exf l
uid,asshownbel ow,andt he
absenceoftherightkidney.Whatist hemostl ikel
ydiagnosis?

OHVI
RAsy
ndr
ome

202.Thepat
hogenesi
sofpoorpr
egnancyoutcomeswi
thauni
cor
nuat
eut
erusi
s
thoughtt
oberel
atedt
owhichofthefol
lowi
ngactor
s?

Al
loft
hement
ioned

203.Septateut
erus,asshowninthesonogram bel
ow,hasasigni
fi
cant
lyhi
gher
spontaneousabort
ionrat
eandear l
ypregnancyl
ossrat
ethanbicor
nuateut
erus.Whatis
theprimarymechanism t
houghtt
ober esponsi
blef
orthi
sextr
aordi
nari
l
yhighpregnancy
wastage?

Par
ti
alorcompl
etei
mpl
ant
ati
onont
hel
argel
yav
ascul
arsy
stem

204.Comparedwi
thot
hermül
ler
ianductdef
ect
s,atransv
ersev
agi
nal
sept
um i
s
associ
atedwit
hal
owerrat
eofwhichofthef
oll
owing?

Ur
ologi
cabnor
mal
i
ties

205.Thepat
hogenesi
sofpoorpr
egnancyoutcomeswi
thauni
cor
nuat
eut
erusi
s
thoughtt
oberel
atedt
owhichofthefol
lowi
ngactor
s?

Al
loft
hement
ioned

206.Whichoft
hef
oll
owi
nganat
omi
cst
ruct
uresi
scor
rect
lypai
redwi
thi
tsembr
yol
ogi
c
ori
gin?
Al
loft
hement
ioned

207.Whenshoul
dsurgical
reconst
ruct
ionofabi
cor
nuat
eut
erus(
Str
assman
metropl
ast
y)beper
formed?

Withr
ecurr
entpr
egnancyl
ossandnoot
heri
dent
if
iabl
ecauseexceptt
heut
eri
ne
anomaly

208.Whi
chofthefol
l
owingstat
ement sist
rueregar
dingt
her
oleoft
hesex-
det
ermi
ning
regi
onY(SRY)i
nthesexual
diff
erent
iati
onofhumans?

TheSRYact
sast
het
est
is-
det
ermi
ningf
act
or.

209.Whi
choft
hef
oll
owi
ngst
atement
sregar
dingbl
adderexst
rophyi
sNOTt
rue?

Ext
ernal
geni
tal
i
aar
ety
pical
l
ynor
mal
exceptf
oroccasi
onal
def
ect
s.

210.Whatper
cent
ageofpregnancy-
rel
ateddeat
hsi
ntheUni
tedSt
ateswer
eat
tri
but
abl
e
tocar
diov
ascul
ardi
seasesbetween2011and2013?

10-
15%

211.Youar ecari
ngforapri
migravidawithknownhy pert
rophi
ccardi
omyopat
hy.She
tel
l
sy ouhermot heral
sosuff
eredf rom t
hecondit
ionandpassedawayatanear l
yage.
Thepatientisconcer
nedaboutherchi l
d’sf
utur
eriskfordevel
opingt
hesameconditi
on.
Whatdoy outel
lheri
sthemostlikelyri
skforpassingthecondi
ti
ontoherchi
l
d?

50%

212.Agraphicofpulmonar ycapil
l
arywedgepr essur
e( r
edline)isshownf orawoman
withmitr
alstenosi
s.“A”represent
sthefir
ststageoflabor,“B”mar ksthesecondst
age
oflabor
,“C”demonstratestheimmedi at
epostpartum peri
od,and“ D”showstheperi
od
4–6hour spostdel
i
very.Whataccount sf
orther i
seinpulmonar ycapil
l
arywedge
pressur
eimmedi at
elyfoll
owingdeliv
ery?

Pl
acent
alaut
otr
ansf
usi
on

213.Basedonthediagnosi
sofaperi
part
um cardiomyopat
hyandassumingarecov
ery
ofsyst
oli
cfunct
ionby6mont hspost
part
um, whatisthepat
ient
’sr
iskf
orsuf
feri
nga
rel
apseofthesameconditi
oninafut
urepregnancy?

30%
214.Thebenefi
tsandsafet
yofi
nsti
tut
inganti
hyper
tensi
vet
her
apyi
nwomenwi
th
chroni
chyper
tensi
onplanni
ngordur
ingpregnancyare:

Notcl
ear

215.Indi
cat
ionsf
oraspi
ri
nprophylaxistoreducether
iskfordevel
opi
ngpr
eecl
ampsi
ain
pregnantwomenwit
hchroni
chy pertensi
onincludeahist
oryof:

Al
loft
hement
ioned

216.Whichoft
hef
oll
owi
ngisNOTtr
ueregar
dingt
hecompl
i
cat
ionshownher
e,i
nthe
computeri
zedt
omogr
aphyscani
mage?

Theri
skisnoti
ncr
easedamongwomenwi
thchr
oni
chy
per
tensi
oni
ntheabsenceof
super
imposedpr
eecl
ampsi
a

217.Whyet
hni
cit
yisi
mpor
tanti
nrecommendat
ionst
oini
ti
ateanant
ihy
per
tensi
veagent
?
Amongf
oll
owi
ngchoosecor
rectst
atement
s

a.Angi
otensi
n-conv
ert
ingenzymeinhi
bit
orsandangiot
ensi
n-r
ecept
orbl
ocker
sar
e
r
ecommendedasi nit
ial
ther
apyintheblackpopul
ati
on.
b.Bet
abl
ocker
sar
enotr
ecommendedasi
nit
ial
ther
apyi
nthenonbl
ackpopul
ati
on.
c.Bet
abl
ocker
sar
erecommendedasi
nit
ial
ther
apyi
ntheAsi
anpopul
ati
on.
d.Angi
otensi
n-conv
ert
ingenzymeinhi
bit
orsandangi
otensi
n-r
ecept
orbl
ocker
sar
e
r
ecommendedasi nit
ial
ther
apyinthenonbl
ackpopulat
ion
218.Antihy
pert
ensi
vet
her
apyi
smandat
oryf
orwhi
choft
hef
oll
owi
ngwomendur
ing
pregnancy
?
30-
year
-ol
dG3P2wi
thabl
oodpr
essur
eof160/
110mm Hgandahi
stor
yofast
hma
219.Whi
choft
hef
oll
owi
ngi
samaj
orf
act
orcont
ri
but
ingt
othei
nci
denceofper
inat
al
mor
tal
i
tynot
edi
npr
egnanci
escompl
i
cat
edbychr
oni
chy
per
tensi
on?
Al
loft
hement
ioned
220.Whi
chant
ihy
pert
ensi
vedrugcommonl
yusedt
otr
eathy
per
tensi
oni
spr
oper
lymat
ched
wi
thi
tsmostnot
ablesi
deeff
ect?
Hy
dral
azi
ne—mat
ernal
tachy
car
dia
221.Ms.Al doisa30-year-ol
dG3P2whopr esent
sforestabl
i
shmentofpr enat
alcareat
9weeks’ gestat
ion.Herpriorpregnancieshavebothbeencompl i
catedbygestat
ional
hypert
ensionneart er
m.Herbl oodpr essur
ewas146/ 88mm Hg, whi chshest
atesis
concordantwithotherrecentmeasur ements.Whichofthefoll
owingi st
hemost
appropri
atenextstepinhermanagement ?

Recommendt
hatshekeepal
ogofherbl
oodpr
essur
esandbr
ingt
hem t
oher
appoi
ntment
s
222.A28- year-oldG3P1102at9weeks’ gestati
onpr esentsforestabli
shmentof
prenatalcare.Herf i
rstpregnancywascompl icat
edbygest ati
onalhypert
ensi
onatt erm,
andhersecondwascompl i
cat edbyi at
rogenicpreterm deli
veryat36weeks’ gest
ation
forpreeclampsi a.Herbl oodpr essur ewasnor mal atherpost par
tum visi
t2yearsago
buthasnotbeenassessedsi nce.Todayherbodymassi ndexis46kg/ m2andherbl ood
pressureis158/ 96mm Hg.Asi nstruct
ed,shecallsback2day safterhervi
sittor
epor t
thatbloodpr essur eshav ebeeni nthe146–160/ 94–100mm Hgr angeathome.I n
additi
ont oinit
iationofanant ihypertensi
veagent ,whatisthenextst epinmanagement
ofherbloodpr essurei nthi
spr egnancy?
Basel
i
neser
um cr
eat
ini
ne
223.Whichofthefoll
owingf
indi
ngsmakei
ncaseofel
evat
edl
i
verenzy
mesexcl
ude
int
rahepat
icchol
estasi
sofpregnancy
?
AST800U/
L
224.Whichimagingmodali
tyi
srecommendedindiscr
imi
nati
ngacut
efat
tyl
i
verof
pregnancyf
rom otheret
iol
ogi
esofel
evat
edli
verenzymes?
Noneoft
heabov
e
225.Whichofthefoll
owingfeatur
esarecr
it
eri
aforsev
eredi
seaset
hatshoul
dpr
ompt
hospi
tal
i
z at
ionforacutevi
ralhepat
it
is?
Hy
per
bil
i
rubi
nemi
a
226.Dur
ingt
hehospi
talcareofapat
ientwi
thacut
evi
ral
hepat
it
is,
whichoft
he
fol
lowi
ngper
sonalpr
otecti
veequi
pmentshoul
dbeusedatal
lti
mes?
Gl
oves
227.A27-year
-ol
dat32weeks’
gest
ati
onisbr
oughttotheemer
gencyr
oom by
ambulanceaft
erconsumi
ng6gramsofacet
aminophen.Whi
chofthef
oll
owi
ngist
rue?
Thi
sist
hemostcommoncauseofacut
eli
verf
ail
urei
ntheUni
tedSt
ates.
228.Whi
choft
hef
oll
owi
ngst
atement
sregar
dingphy
siol
ogi
cal
changesi
npr
egnancyi
s
tr
ue?
Ef
fect
iver
enal
plasmaf
lowi
ncr
eases
229.Whatper
centageofpregnantwomenexperi
encear
ecur
rentur
inar
ytr
acti
nfect
ion
aft
ercomplet
ionoftr
eatmentforpyel
onephr
it
is?
30-
40%
230.Whatpercentageofpr
egnantwomenwi
thasy
mpt
omat
icbact
eri
uri
adev
elopa
symptomati
cinfecti
onifl
eftunt
reat
ed?
25%
231.Thepregnantpati
entwi
thakidneystoneaskshowmanypregnantwomenactual
l
y
passthei
rstonewiththemanagementplanofanalgesi
aandi
ntr
avenoushy
drat
ion.
Whatisyourresponse?
65-
80%
232.Mostwomenwit
hnephr
oti
csy
ndr
omeswi
l
lhav
esuccessf
ulpr
egnancyout
comes
incasei
ftheyhav
e
pr
otei
nexcr
eti
ont
hatex
ceeded3g/
d
233.A27-
year
-ol
dnul
l
igr
avi
daandherhusbandpr
esentf
orpr
econcept
ioncounsel
i
ng.
ShehasWi
lsondiseasethati
sbeingtr
eat
edwi
thzi
ncsul
fat
e.Whi
choft
hef
oll
owi
ng
shoul
dbei
ncludedinyourcounsel
ing?
Herhusbandshoul
dconsi
dercar
ri
ert
est
ing
234.A32-
year
-ol
dnul
l
igr
avi
dapr
esent
sforpr
econcept
ioncounsel
i
ngduet
oci
rr
hosi
s
wi
thesophageal
var
ices.Whi
choft
hef
oll
owi
ngshoul
dbei
ncl
udedi
nyourcounsel
i
ng?
Al
loft
heabov
e
235.Yourpat
ienti
sadmi
tt
edt
othehospi
tal
forev
aluat
ionofnew-
onsethy
per
tensi
onat
30weeks’
gest
ati
on.Whi
choft
hef
oll
owi
ngt
est
sorev
aluat
ionsi
sleastappr
opr
iat
e?
Cel
l
-fr
eeDNAt
est
ingf
oraneupl
oidy
236.Whi
choft
hef
oll
owi
ngmedi
cal
ther
api
eshasbeenassoci
atedwi
thamodest
decr
easei
nthei
nci
denceofpr
eecl
ampsi
ainwomenati
ncr
easedr
isk?
Aspi
ri
n
237.Apr
egnantl
adypr
esent
swi
thv
agi
nal
bleedi
ngi
n1stt
ri
mest
er,
onexami
nat
ionOs
i
scl
osedandut
eri
nesi
zecor
respondst
othePOG(
Per
iodofGest
ati
on)
Thr
eat
enedabor
ti
on
238.Vi
ral
inf
ect
ionr
isksf
rom t
ransf
usi
onar
eaccur
atel
ydepi
ctedi
nwhi
choft
he
f
oll
owi
ngst
atement
s?
Al
loft
heabov
e
239.Whi
chhemost
ati
csur
gical
procedur
eisdemonst
rat
edi
nthepi
ctur
ebel
ow?
Ut
eri
near
ter
yli
gat
ion
240.Thefoll
owingpict
ureunfor
tunat
elydepi
ctsamaternaldeat
haf
terpost
par
tum
hemorrhage.Whichofthefol
l
owingarepossibl
econt
ributor
s?
Ut
eri
nei
nver
sion
241.Whi
choft
hebact
eri
abel
owar
eassoci
atedwi
thconsumpt
ivecoagul
opat
hy?
Gr
oupASt
rept
ococcus
242.Ligat
ionatwhi
choft hef
oll
owinglabel
edvessel
poi
ntsi
nther
etr
oper
it
oneum wi
l
l
decreasethepul
sepressurei
ntheuter
ineart
ery?
I
nter
nal
il
iacar
ter
y
243.Regar
dingt
hepati
entwi
thdi
l
uti
onal
coagulopat
hy,
whichoft
hef
oll
owi
ng
i
nterv
enti
onsmighthav
eavoi
dedoramel
ior
atedthi
scondi
ti
on?
Tr
ansf
usi
onofwhol
ebl
ood
244.Whichofthef
oll
owingmaneuv
ersshouldbeperf
ormedi
ntheset
ti
ngof
postpar
tum hemor
rhagefol
l
owi
ngav agi
naldel
iv
ery?
Al
loft
hement
ioned
245.A32- year-
oldwomanunder goesmy omectomyforsympt
omati
cuter
inefi
broi
ds,
all
ofwhi charesubserosal(outsi
desur f
aceoft heuter
us)
.Theendomet
ri
alcavi
tywasnot
enteredduringtheprocedure.Whichoft hefoll
owingstat
ement
sismostli
kel
ytobe
correctregardi
ngtheriskofplacentalaccret
a?
Herr
iskofaccr
etai
smostl
i
kel
ynotaf
fect
edbyt
hemy
omect
omy
246.A21-
year
-ol
dpat
ientat28weeks’
gest
ati
onhasv
agi
nal
bleedi
ngandi
sdi
agnosed
wi
thpl
acent
apr
evi
a.Whi
choft
hef
oll
owi
ngi
sat
ypi
cal
feat
ureoft
hiscondi
ti
on?
Associ
atedwi
thpost
coi
tal
spot
ti
ng
247.A35- year
-ol
dwomanpr esentswit
hbr i
ghtr
edvaginalbl
eedingat30weeks’
gestati
on.Herurinedr
ugscreenisposi
tiv
e.Whichofthefoll
owingismostl
ikel
ytobe
presentinherdrugscr
een?
Cocai
ne
248.Whi
choft
hefoll
owingstat
ementsisFALSEregardi
ngt
hedevelopmentoft
he
hypot
hal
ami
c–pit
uit
ary-
ovar
ianaxi
sinthefemal
ef et
usandneonat
e?
Thegonadot
ropi
n-r
eleasi
nghormone(GnRH)“
pul
segener
ator
”remai
nsf
unct
ional
l
y
dor
mantunti
lsever
al monthsaf
terbi
rt
h.
249.Pubarche,
withdevel
opmentofaxil
l
aryandpubi
chair,
isst
imul
atedbyt
he
androgensderi
vedpri
maril
yfr
om whichofthef
oll
owing?
Adr
enal
glands
250.Pri
marytherapyorcent
ral
(gonadot
ropi
n-dependent
)pr
ecoci
ouspuber
tyconsi
sts
ofwhichofthefoll
owing?
Gonadot
ropi
n-r
eleasi
nghor
mone(
GnRH)agoni
st
251.A6-y
ear
-ol
dCaucasi
angi
rli
snotedtohaveTanner3br
eastdev
elopmentand
menses.Whi
choft
hefoll
owi
ngistheprobabl
etr
eatmentf
orthi
spati
ent?
GnRHagoni
stt
her
apy
252.Lackofbreastdevelopmentisassoci
atedwi
thl
owest
rogenl
evel
sandmaybe
causedbywhichoft hefoll
owing?
Al
loft
hement
ioned
253.Whichmüll
eri
ananomal
yresul
tsf
rom complet
efail
ureoft
hepairedmüll
eri
an
ductst
ouse?Thisanomalyi
sshowninthi
ssonogram,andagestat
ionalsaci
sseenon
thei
mage’sl
eftsi
de.
Ut
eri
nedi
del
phy
s
254.Causesofnewbor
ncl
i
tor
omegal
yincl
udewhi
choft
hef
oll
owi
ng?
Al
loft
hement
ioned
255.Whi
choft
hef
oll
owi
ngi
sNOTt
rueofKl
i
nef
elt
ersy
ndr
ome?
Af
fect
edmenar
etal
landex
cessi
vel
yvi
ri
li
zed.
256.Thev
agi
naf
ormsi
npar
tfr
om bot
hthemül
l
eri
anduct
sandwhi
chot
herst
ruct
ure?
Ur
ogeni
tal
sinus
257.Oneofy ourpati
ent sbr
ingsher13- year -
olddaught
erf orconsul
tationr egardingthe
gi
rl’
scompl ai
ntsofpelvicpain.Thedaught erhasexper
iencednor mal puber tal
developmentt odatebuthasnoty etexperiencedmenarche.Shebeganhav ingmont hly
pelv
icpai nabout1y earago, andt hepainhasbeenpr ogressivel
ywor sening.Dur ing
physicalexaminati
on,herext ernalgenit
alia,incl
udi
nghymeneal ri
ng,appearnor mal .
Howev er
,proxi
mal t
ot hehymen, onlyav aginaldi
mplei
sf ound.Rectov aginal
exami nati
onconfir
mst hepresenceofaut erusandcervix.Whatisyourdi agnosi s?
Vagi
nal
atr
esi
a
258.Post
coi
tal
testi
susedt
oassess:
Cer
vical
fact
or
259.Art
ifici
alinsemi
nat
ionwi
thhusband’
ssemeni
sindi
cat
edi
nal
lthef
oll
owi
ng
si
tuat
ions,except
Azoosper
mia
260.A34-year-
oldi
nfer
ti
lewomani
snotedtohaveevi
denceofblockedfal
lopi
antubes
byhyst
erosalpi
ngogr
am.Whichoft
hefoll
owingi
sthebestnextstepfort
hispati
ent
?
Lapar
oscopy
261.Whi
choft
hef
oll
owi
ngi
str
ueoft
hepr
ocessofsper
m capi
tat
ion?
Al
loft
hement
ioned
262.Congeni
tal
bil
ater
alabsenceoft
hev
asdef
erensi
sgenet
ical
l
yrel
atedt
owhi
chof
thefol
l
owing?
Cy
sti
cfi
brosi
s
263.Howl
ongdoessper
mat
ogenesi
stake,
star
ti
ngf
rom st
em cel
ltomat
uresper
m?
90day
s
264.I
nrepr
oduct
ive-
agedwomen,
whi
choft
hef
oll
owi
ngi
ncr
easeswi
tht
hepassageof
ti
me?
Ri
skofspont
aneousabor
ti
on
265.Whi
choft
hef
oll
owi
ngi
sNOTt
ruer
egar
dingcl
omi
pheneci
tr
atet
her
apy
?

Thet
ypi
cal
star
ti
ngdosei
s200mgor
all
y.

266.A38-year-
oldwomanwi t
hinfert
il
it
ysecondaryt
odistalt
ubalblockageundergoes
invi
tr
of er
ti
li
zati
on.Herpeakestr
adioli
s4500pg/ mL.Subsequentl
y,17oocytesare
ret
ri
eved,and2embr yosaret
ransfer
red.Sheconcei
vesasinglet
ongest at
ionbut
devel
opsovari
anhyper
sti
mulat
ionsyndr
ome(OHSS).Asonogr
am ofherovar
iesi
s
shownhere.Whichoft
hefol
l
owi ngi
sapredi
sposi
ngact
ororherdevel
opingOHSS?

Hi
ghest
radi
oll
evel

267.Whi
chofthefol
l
owi
nghormonesist
houghtt
opl
ayacont
ri
but
ingr
oleear
lyi
nthe
devel
opmentofov
ari
anhyper
sti
mul
ati
onsyndr
ome?

Humanchor
ioni
cgonadot
ropi
n

268.A22-
year-ol
dG0P0womancompl ai
nsofirr
egul
armensesevery30t
o65days.The
semenanalysisi
snormal
.Thehyster
osalpi
ngogram i
snor
mal.Whichoft
hef
oll
owingis
themostl
ikelytr
eat
mentfort
hispati
ent?

Cl
omi
pheneci
tr
ate

269.Awomancompl ai
ningofinfer
ti
li
tyrepor
tsthatherhusbandhasalr
eadyundergone
eval
uati
onbyaurologi
sttoexcludemaleactors.Thi
sleadstothesurgi
calr
epairofa
unil
ater
alv
ari
cocel
e.Withregar
dt ovari
cocel
esandi nf
erti
l
ity
,whichofthef
oll
owingis
tr
ue?
Av
ari
cocel
eispr
esenti
nnear
ly20per
centoft
headul
tmal
egener
alpopul
ati
on.
270.Whati
stheexpect
edpercy
clef
ecundabi
l
ityr
ate?
25per
cent
271.Asshownhere,sonographyi
scli
nical
lymostusefulf
orthediagnosi
sofwhichof
thefol
l
owingcondi
ti
onst hatcont
ri
but
est oasi
gnifi
cantpercent
ageofinfer
ti
li
tycases?
Pol
ycy
sti
cov
ari
ansy
ndr
ome
272.Whichoft
hef
oll
owi
ngst
atement
sist
ruer
egar
dingt
hyr
oiddy
sfunct
ionand
i
nfert
il
it
y?
Ini
nfer
ti
lewomenseeki
ngpr
egnancy
,tr
eat
mentofanydegr
eeofhy
pot
hyr
oidi
sm i
s
advi
sabl
e.
273.Averyhealth-consci
ouspat
ientisseekingpregnancyandasksaboutthebestdi
et
tofol
lowatthi
st ime.Sheisover
whel medbyal ltheadvi
ceavail
abl
efr
om f r
iends,
books,
andmagazines.Whi chofthefol
l
owi ngnutri
tionalst
rat
egiesi
smoststrongly
recommendedf orwomenat t
emptingpregnancy ?
Fol
i
caci
dsuppl
ement
ati
on
274. CV PATHOLOHY
1
I
napregnantpopul
ati
on,
whati
sthemostcommoncauseoft
hecondi
ti
onpor
tray
ed
i
nthechestx-
rayi
magebelow?
(
enl
argedpul
monar
yhi
l
arar
ter
iesandat
tenuat
edper
ipher
almar
kings)
Ansmet
abol
i
cdi
sor
der

2Whati
sthemostcommoncauseofhear
tfai
l
urei
npr
egnancy
?
Ans Chr
oni
chy
per
tensi
onwi
thsuper
imposedpr
eecl
ampsi
a

3Youarecari
ngfora24-
year-
oldG1withsev
ereaort
icst
enosi
s.Wheni
npr
egnancyar
e
youmostconcer
nedaboutcardi
acdecompensat
ion?
Ansper
ipar
tum
4Forwhi
chofthef
oll
owi
ngcar
diaccondi
ti
onsi
sconduct
ionanal
gesi
ahi
ghl
y
r
ecommended?
Ans
5Aschemati
cofEisenmengersy
ndr
omeduet
oav ent
ri
cul
arseptaldef
ectisdepi
cted.
Whati
sthemostcommoncauseofdeat
hinpr
egnantwomenwiththiscondi
ti
on?
Ans
6diagnosti
cstudi
es,whichoft
hef
oll
owi
ngcor
rect
lydescr
ibesanor
mal
changet
hati
s
char
acter
ist
icofpregnancy?
Ans
7A32- year-ol
dG2P1pr esentstoy ouroffi
ceat17weeks’ gestati
on.Shereport
sa
hist
oryoft etr
alogyofFall
ot,
whi chwasr epair
edini nf
ancy .Sheappearscomfortabl
eon
theexam t abl
ewi t
hahear tr
ateof92beat spermi nuteandabl oodpressureof96/62
mm Hg.Howev er,
shedoesr eportsigni
fi
cantpalpitat
ionsanddy spneawhenclimbing
thetwof l
ightsofstai
rstoherapar tmentont hethirdfl
oor.WhichNewYor kHeart
Associati
oncl assbestdescri
besherf uncti
onaldisabili
ty?
Ans
8Agr aphicofpulmonarycapil
lar
ywedgepr essure(redli
ne)i
sshownf orawomanwi t
h
mitr
alstenosi
s.“A”repr
esentsthefirstst
ageofl abor,“
B”marksthesecondstageof
l
abor“C”demonst r
atestheimmedi atepostpartum peri
od,and“D”showstheperi
od4–6
hourspostdel
iv
ery.Whataccountsf ortheri
seinpul monarycapil
l
arywedgepressur
e
i
mmedi atel
yfoll
owingdeli
very?
Ans 50%
9Inapr
egnantwomanwhohaspr evi
ousl
yundergoneahearttr
ansplant
ati
on,
whati
s
t
heappr
oxi
materi
skforsuf
fer
ingarej
ecti
onepisodedur
ingpregnancy?
Ans
10Whati
sthegol
dst
andar
dfordi
agnosi
soft
hecondi
ti
oni
nQuest
ion49–24?
AnsRi
ghthear
tcat
het
eri
zat
ion

11TheWorldHeal
thOrgani
zat
ioni
dent
if
ieswhi
choft
hef
oll
owi
ngcondi
ti
onsas
pr
ohibi
ti
veofpr
egnancy?
Ans
12Whi choft
hef
oll
owi
ngt
her
api
esi
ssaf
einpr
egnancyt
otr
eatacut
esupr
avent
ri
cul
ar
tachy
cardi
a?
Ans
13Whi
chofthefol
lowi
ngbestdescr
ibest
her
emodel
i
ngoft
hehear
tthatoccur
sin
nor
malpr
egnancy?
AnsEccent
ri
clef
tvent
ri
cul
armassexpansi
onwi
thspher
ical
remodel
i
ng

14Youar ecar i
ngforapregnantpati
entwithmi t
ral
stenosis.Shei
sablet oper
for
m
mostact i
viti
esofdail
yli
vingwit
houtsignif
icantl
imit
ati
on.Onechocardiogram,her
eject
ionfracti
onis50%andhermi tralval
veareameasur es1.8cm2.Whi chofthe
foll
owingv ar
iabl
esismostpredict
iveofahi ghri
skforcardiaccompl
i
cat i
onsduri
ngthi
s
pregnancy?
Ans.Herdegr
eeofl
eft
-si
dedobst
ruct
ion

15Thepat i
entwhoseheartisshownint hisi
magehassy st
emiclupuseryt
hemat osus
andhasahi st
oryofapriorstr
oke.Theleftatr
ium (
LA)andl ef
tventr
icl
e(LV)ar e
identi
fi
ed,
andthearrowspointtononinfecti
ousvegetat
ionsonthemi t
rall
eaflets.
Whi choft
hefoll
owingistheli
kel
yconditionassoci
atedwi t
hhermi t
rali
nsuff
iciency?
Ans
16Whichofthefol
l
owingcondi
ti
onsusual
l
yimpr
ovesdur
ingpr
egnancyduet
o
pr
egnancy
-i
nducedhyper
vol
emia?
AnsAl
loft
heabov
e

17A30- y
ear-oldG2P0A1pr esentsat11weeks’
gestat
iontoestabl
ishcare.Shehasa
hi
storyofrheumat i
chear
tdi seaseandhadamechanicalv
alverepl
acementatage20.
Shewaspr eviousl
yonwarf ari
n7.5mgdaily
,butst
oppedtakinghermedi cat
ionwhen
shefoundoutshewaspr egnant1weekago.Whati sherriskforwarfar
inembr y
opat
hy
basedonherwar f
ari
ndosing?
Ans.8%
18Whatpercentageofpregnancy
-rel
ateddeat
hsi
ntheUni
tedSt
ateswer
eat
tri
but
abl
e
tocar
diov
asculardi
seasesbetween2011and2013?
Ans15–20%
19Whi choft
hefol
lowi
ngdoesnotcont
ri
but
etot
he40%i
ncr
easei
ncar
diacout
put
seenduri
ngnormal
pregnancy
?
Ans I
ncr
easedl
eftv
ent
ri
cul
arcont
ract
il
it
y

20I
ntheUni
tedSt
ates,
whati
sthemostcommoncauseofaor
ti
cst
enosi
s?
AnsBi
cuspi
daor
ti
cval
ve

21A32- y
ear-ol
dG2P1pr esentst oyouroffi
ceat17weeks’ gestati
on.Shereport
sa
hist
oryoftetr
alogyofFal
lot,
whi chwasr epair
edininf
ancy.Sheappear scomfortabl
eon
theexam tabl
ewi t
hahear tr
ateof92beat spermi nuteandabl oodpressureof96/62
mm Hg.Howev er,
shedoesr eportsigni
fi
cantpal
pitat
ionsanddy spneawhenclimbing
thetwofli
ghtsofstai
rstoherapar tmentont het
hirdfl
oor.WhichNewYor kHeart
Associ
ati
oncl
assbestdescr
ibesherf
unct
ional
disabi
l
ity
?
AnsCl
assI
I

22Youar
ecari
ngfora24-
year-
oldG1withsev
ereaorti
cstenosi
s.Wheni
npr
egnancy
ar
eyoumostconcer
nedaboutcardi
acdecompensat
ion?
Ans.Per
ipar
tum

23Whatpercentageofpregnancy
-rel
ateddeat
hsi
ntheUni
tedSt
ateswer
eat
tri
but
abl
e
tocar
diov
asculardi
seasesbetween2011and2013?
Ans
24Whichofthefoll
owingcondi
ti
onsusual
l
yimpr
ovesdur
ingpr
egnancyduet
o
pr
egnancy-
inducedhyperv
olemi
a?
Ans Al
loft
heabov
e

25Youar ecar i
ngforapregnantpati
entwithmi t
ral
stenosis.Shei
sablet oper
for
m
mostact i
viti
esofdail
yli
vingwit
houtsignif
icantl
imit
ati
on.Onechocardiogram,her
eject
ionfracti
onis50%andhermi tralval
veareameasur es1.8cm2.Whi chofthe
foll
owingv ar
iabl
esismostpredict
iveofahi ghri
skforcardiaccompl
i
cat i
onsduri
ngthi
s
pregnancy?
Ans.Herdegr
eeofl
eft
-si
dedobst
ruct
ion

26Whichoft
hefol
l
owingdoesnotcont
ri
but
etot
he40%i
ncr
easei
ncar
diacout
putseen
dur
ingnor
malpr
egnancy
?
Ans.I
ncr
easedl
eftv
ent
ri
cul
arcont
ract
il
it
y

27Whi
chofthefol
lowi
ngbestdescr
ibest
her
emodel
i
ngoft
hehear
tthatoccur
sin
nor
malpr
egnancy?
Ans.Eccent
ri
clef
tvent
ri
cul
armassexpansi
onwi
thspher
ical
remodel
i
ng

28Forwhichoft
hef
oll
owi
ngcar
diaccondi
ti
onsi
sconduct
ionanal
gesi
ahi
ghl
y
recommended?
Ans
29I
napr egnantpopul
ati
on,whati
sthemostcommoncauseoft
hecondi
ti
onpor
tray
ed
i
nthechestx-r
ayimagebelow?
(
enl
argedpul
monar
yhi
l
arar
ter
iesandat
tenuat
edper
ipher
almar
kings)
Ans

t
estf
orCV-
pat
hol
ogy-cl
i
nical
cases

30A32-
year
-ol
dG2P1t
ransf
erst
oyourcar
eat18weeks’
gest
ati
on.Shehasa
hist
oryofNYHAclass2hear
tdisease.Whatamongf
oll
owi
ngar
ecommon
recommendati
onstothi
spat
ient?

Anstoavoidcontactwi
thpersonswhohav erespi
rat
oryi
nfect
ions,
incl
udi
ngt
hecommon
col
d,
andtoreportatonceanyevi
dencefori
nfecti
on

31Youar et
heon- cal
lhospit
ali
stwheny ourecei
veanurgentcal l
from t
he
post
part um f
looraboutapat i
entwhoi scompl ai
ningofchestpainandshor t
ness
ofbreath.Shehadanuncompl icatedvaginaldel
i
verythepreviousdayandhasno
si
gnifi
cantpastmedi calorsurgicalhi
story.Youobtai
nast atchestx-
ray,whichi
s
shown.Basedont hefindi
ngs,whatar eyoumostconcer nedfor?

Ans

32Youar ecaringf
orcompl ainsofshort
nessofbreathandchestpai nwith
minimalexer
tionanuncompl i
cat
edprimigr
avi
daat32weeks’ gestat
ion.An
echocardi
ogram demonst r
at esamitr
alval
vesurf
acear eaof2.1cm2, adil
ated
lef
tatr
ium,anaor t
icval
vear eaof3.7cm2,ari
ghtv ent
ri
cularsystoli
cpressureof
20mm Hg, andanej ecti
onfracti
onof55%.Youalsoobt ai
nachestx- r
ay,which
isshown.Whati stheli
kel
yunder l
yi
ngcauseofhersy mptoms?

AnsMi
tr
alst
enosi
s

33At24weeks’ gest
ati
onthepat
ientwit
hahistor
yofpulmonaryhypert
ension
duetochroni
cthromboemboli
cdiseasei
nherpastbeginstodevel
oporthopnea
anddyspneaonex er
ti
on.Whattr
eatmentwouldyouconsidert
odecreaseher
symptoms?

Ans Al
loft
heabov
e

34Apr egnantwomanpr esentstotheemer gencyroom inanobt undedstate.She


hasafev erto102.6°
F.Onexam, a4/ 6mur murisnoted,shewithdrawsfrom pain
duri
ngherabdomi nalexam,andtrackmar ksarenotedonherar msandl egs.Her
fundalheighti
sapproxi
mately32cm.Anemer gentechocar
diogram isobtained,
whichdemonst rat
esami t
ralval
vev eget
ation.Whatisthemostl ikel
yorganism
underl
yingherdiagnosi
s?

Ans

t
estf
orChr
oni
chy
per
tensi
on–t
heor
y

35Whichintr
avenousanti
hyper
tensi
vedr
ugcommonl
yusedtotr
eatintr
apart
um
sev
ererangehyper
tensi
onisproperl
ymat
chedwi
thi
tsmostnot
ablesideeff
ect?

Ans.Labet
alol
—neonat
alhy
pot
ensi
onandbr
ady
car
dia

36Forwomenwi
thchr
oni
chy
per
tensi
on,
whi
chmodi
fi
abl
efact
orf
urt
her
i
ncr
easest
her
iskf
ort
hepl
acent
alabr
upt
ion?

Anssmoki
ng

37Amongwomenwhohav epersi
stent
lywor
ri
somehyper
tensi
ondespi
teusual
ther
apywhatant
agoni
zi
ngsubst
ancesmaybeused:

AnsChr
oni
cingest
ionofnonst
eroi
dal
ant
ii
nfl
ammat
orydr
ugs

38Antihyper
tensi
vet
her
apyi
smandat
oryf
orwhi
choft
hef
oll
owi
ngwomen
dur
ingpregnancy?

Ans.30-
year
-ol
dG3P2wi
thabl
oodpr
essur
eof160/
110mm Hgandahi
stor
yof
ast
hma

39Al
lexceptwhi
choft
hef
oll
owi
ngsuppor
tthedi
agnosi
sofsuper
imposed
pr
eecl
ampsia?

AnsDecr
easedser
um al
kal
i
nephosphat
asel
evel

40Whyethni
cit
yisimport
antinr
ecommendat i
onstoi
nit
iat
eanant
ihy
per
tensi
ve
agent
?Amongfoll
owingchoosecor
rectst
atements

Ans.Angi
otensi
n-conv
ert
ingenzy
mei nhi
bit
orsandangi
otensi
n-r
ecept
or
bl
ocker
sarerecommendedasi ni
ti
alt
herapyint
heblackpopul
ati
on.

Bet
abl
ocker
sar
erecommendedasi
nit
ial
ther
apyi
ntheAsi
anpopul
ati
on.

Angiot
ensi
n-conv
erti
ngenzy
meinhi
bit
orsandangiotensi
n-r
ecept
orbl
ocker
sar
e
recommendedasi ni
ti
alt
her
apyi
nthenonblackpopulat
ion.

41Whichofthef
oll
owi
ngst
atementsisnott
rueregar
dingint
rapar
tum
managementofawomanwit
hchronichyper
tensi
onwithsuperi
mposed
pr
eecl
ampsia?

AnsPostpar
tum anal
gesi
ashouldincl
udeschedul
ednonster
oidal
ant
ii
nfl
ammat or
ydrugsforatl
east72hourstoreducenar
coti
cuse.

42Riskfort
heplacent
alabr
upt
ioni
sincr
easedinpregnanci
escompl
i
cat
edby
chr
onichypert
ensi
on.Howwouldyoubestchar
acteri
zetheri
sk?

Ans2-
3-f
oldi
ncr
easeov
ert
hegener
alobst
etr
icpopul
ati
on

43Whichoft
hefol
l
owingappr
oximatestheri
skf orper
inat
almor
tal
i
tyi
n
pr
egnanci
escompl
icat
edbychr
onichypert
ension?

AnsAl
loft
hement
ioned

44Thet
hreshol
dof140/90mm Hgast
heupperl
i
mitofnor
mal
isbasedon
whi
chofthefol
l
owing?
AnsAct
uar
ial
tabl
esbasedondat
afr
om whi
teadul
tmal
es

t
estf
orChr
oni
chy
per
tensi
oni
nPr
egnancy-cl
i
nical
cases

45A36- y
ear-ol
dmul t
igr
avidawithchronichypert
ensionpresentsforagr owth
ult
rasoundat36weeks’ gestat
ion.Theul t
rasoundreportshowsy outhatestimatedf
etal
weightcorrespondto33weeks’ ,headsi ze-to34weeks’ ,
whileabdominal
cir
cumf er
enceandf emurlengt
h–t o32weeks’ gestati
on.Whichofthef oll
owingist
rue
regardi
ngthedi agnosismadeont hegrowthultr
asound?

AnsFetal-
growthdysf
uncti
onismor
eli
kel
yinchr
oni
cal
l
yhy
per
tensi
vewomenwho
dev
elopsuperi
mposedpreecl
ampsi
a

46A28- year-ol
dG3P1102at9weeks’ gestati
onpr esentsforestabli
shmentofpr enatal
care.Herfir
stpregnancywascompl icatedbygest at
ionalhypert
ensionatterm, andher
secondwascompl icat
edbyiatrogenicpr et
erm deliv
eryat36weeks’ gest
ationfor
preecl
ampsi a.Herbloodpr
essur ewasnor malatherpost part
um v i
sit2yearsagobut
hasnotbeenassessedsi nce.Todayherbodymassi ndexis46kg/ m2andherbl ood
pressureis158/96mm Hg.Asi nstructed,shecallsback2day saf t
erhervisitt
or eport
thatbloodpressureshavebeeni nthe146–160/ 94–100mm Hgr angeathome.I n
additi
ontoiniti
ati
onofanantihypertensiveagent ,
whati sthenextstepinmanagement
ofherbloodpr essurei
nthi
spr egnancy ?

AnsBasel
i
neser
um cr
eat
ini
ne

47Ms.Edwar dsisa26- year-


oldG2P2, now3day spostpar
tum f
rom av aginaldeli
very
.
Shehaschroni chy per
tensionanddev elopedsuper i
mposedseverepreeclampsi aat36
weeks’gestation.Shedidwel li
nitiall
y,butinthelast12hoursherbloodpr essur
ehas
beentrendi
ngup.Mostr ecentbloodpr essurewas156/ 90mm Hg.Thenur sehascall
ed
youbecauseMs.Edwar dsisfeelingshor tofbreath.Arepr
esentat
ivechestr adi
ograph
isshownher e.Whati stheappr opriatetreat
mentf orthi
scondi
ti
on?

AnsI
ntr
avenousf
urosemi
de

48Whi
chant
ihyper
tensi
vedrugcommonlyusedt
otr
eathy
per
tensi
oni
spr
oper
ly
mat
chedwi
thit
smostnot abl
esideef
fect
?

AnsHy
dral
azi
ne—mat
ernal
tachy
car
dia

49Forthepati
entwit
hmoderat
echroni
chyper
tensi
on,
whichoft
hefol
lowi
ng
compl
icat
ionsi
sreducedwi
thuseofant
ihy
pert
ensiv
ether
apydur
ingpr
egnancy
?

AnsDev
elopmentofsev
erehy
per
tensi
on
Li
ver
,ki
dneyandendocr
inal
pat
hol
ogyi
nPr
egnancy

50From anet
iopat
hogenesisperspect
ive,
thepr
egnancyf
att
yli
verpi
ctur
edher
eis
anal
ogoustowhichchi
ldhoodil
lness?

Ans

51.Whi
choft
hef
oll
owi
nghepat
icenzy
mesi
sincr
easedi
nnor
mal
pregnancy
?

Ans

52Whi chofthef
oll
owi
ngfi
ndingsmakei
ncaseofel
evat
edl
i
verenzy
mesexcl
ude
int
rahepat
icchol
est
asi
sofpregnancy
?

Ans

53Ri
skofmat
ernal
deat
hwi
thacut
efat
tyl
i
veri
srel
atedt
owhi
choft
hef
oll
owi
ng?

Ans.Al
loft
heabov
e

54Whati
sthei
nci
denceandr
ecur
rencer
iskofacut
efat
tyl
i
verofpr
egnancy
?

Ans

55Lact
ati
oni
scont
rai
ndi
cat
edf
orwomeni
nfect
edwi
thwhi
choft
hef
oll
owi
ng?

Ans HI
V

56Whi
choft
hef
oll
owi
ngi
str
uer
egar
dingpancr
eat
it
isi
npr
egnancy
?

AnsThepr
imar
ypr
edi
sposi
ngcondi
ti
oni
npr
egnancyi
schol
eli
thi
asi
s.

57.AllEXCEPTwhi
choft
hef
oll
owi
ngl
i
ver
-r
elat
edchangesar
ephy
siol
ogi
cal
in
pregnancy?

Ans

58Whichofthef
oll
owi
nghav
ethebestdat
afort
reat
mentofi
ntr
ahepat
icchol
est
asi
sof
pr
egnancy?

Ans

59Thecy
tochr
omeP450sy
stem i
sal
ter
edbywhi
choft
hef
oll
owi
ngdur
ingpr
egnancy
?

Ans

60Whichimagi
ngmodali
tyi
srecommendedi
ndiscri
minat
ingacut
efat
tyl
i
verof
pr
egnancyfr
om ot
heret
iol
ogi
esofel
evat
edli
verenzymes?

Ans
61A27-year-
oldat32weeks’
gest
ati
onisbroughttot
heemergencyr
oom byambul
ance
af
terconsuming6gramsofacet
aminophen.Whichofthef
oll
owingi
str
ue?

Ans Thi
sist
hemostcommoncauseofacut
eli
verf
ail
urei
ntheUni
tedSt
ates

62TheSociet
yforMat
ernal
-Fet
alMedi
cinerecommendedwhichoft
hef
oll
owi
ng
agent
sforwomenwit
hhighhepati
ti
sBviral
loadsi
npregnancy?

Ans

63Duri
ngthehospit
alcar
eofapatientwi
thacut
evir
alhepat
it
is,
whi
choft
hef
oll
owi
ng
per
sonalpr
otect
iveequi
pmentshouldbeusedatal
lti
mes?

Ans.Gl
oves

64Whi chofthefoll
owi
ngfeatur
esarecri
ter
iaf
orsev
eredi
seaset
hatshoul
dpr
ompt
hospi
tali
zat
ionforacut
evi
ralhepat
it
is?

Ans

65Whichofthefol
lowingpr egnancy-r
elatedcompl
i
cat
ionshasthecapaci
tyto
demonstr
atethemostpr omi nental
terati
onsofnor
malhepat
ic,
renal
,hematol
ogi
cal
,
andcoagul
ati
onlaboratorystudies?

Ans

66Chroni
chepat
iti
sBinf
ect
ioni
smostl
i
kel
ytodev
elopaf
teracqui
sit
ionbywhi
chof
thef
oll
owingpat
ient
s?

Ans

67Duri
ngnormalpregnancy
,whi
choft
hef
oll
owi
ngi
str
uer
egar
dinggal
l
bladder
phy
siol
ogyandfuncti
on?

Ans

68Thi
simageshowsthety
picalnodul
ar,
fibr
oti
cappearanceofacir
rhot
icl
i
ver
.Whati
s
themostcommoncauseofthi
scondit
ioninthegener
al popul
ati
on?

Ans

69Afterdel
iv
ery,t
hel i
verf
uncti
ondeter
iorati
onofacutefat
tyli
verofpr egnancyhal
ted.
Nonethel
ess,yourecommendcontinuedintensi
vematernalsur
v ei
l
lanceduet othe
appr
ox i
matel
y20%r iskofdevel
opi
ngwhi chofthefol
lowingintheday safterdel
i
very?

Ans Acut
epancr
eat
it
is

70Mat
ernalacut
efat
tyli
verofpr
egnancyisassoci
atedwit
hallexceptwi
thofthe
fol
l
owi
ngrecessi
vel
yinher
it
edabnormali
ti
esofmi t
ochondr
ial
fattyaci
doxidat
ion?
Ans

71Whichofthefol
lowingvi
ral
infect
ionshasbeenassoci
atedwi
thamar
kedi
ncr
easei
n
ther
iskf
orintr
ahepati
cchol
estasisofpregnancy
?

AnsHepat
it
isC

72I
mmuni
zat
ioni
snotav
ail
abl
eforwhi
choft
hef
oll
owi
ng?

Ans.Hepat
it
isB

73Concer
ningt
hegast
roi
ntest
inal
tractdur
ingpr
egnancy
,whi
choft
hef
oll
owi
ngi
s
cor
rect
?

Ans

74Whichofthef
oll
owi
ngcombi
nat
ionsr
epr
esent
sappr
opr
iat
escr
eeni
ngf
orhepat
it
isC
inpr
egnancy
?

Ans 15-year
-ol
dwomanwhosemotherwasachroni
cintr
avenousdr
uguseranddi
edof
l
iverf
ail
urewhenthepat
ientwas4y
earsol
dscr
eenedv i
ahepati
ti
sCRNA

75Whi
choft
hef
oll
owi
ngst
atement
sregar
dingacut
evi
ral
hepat
it
isi
scor
rect
?

Ans Low-
gradef
everi
smor
ecommonwi
thhepat
it
isA.

76Al
lEXCEPTwhi
choft
hef
oll
owi
ngl
i
ver
-r
elat
edchangesar
ephy
siol
ogi
cal
in
pr
egnancy
?

AnsHy
pol
i
pidemi
a

77Whichofthefol
lowi
ngf
eatur
esarecri
ter
iaf
orsev
eredi
seaset
hatshoul
dpr
ompt
hospi
tal
i
zat
ionforacut
evi
ral
hepat
it
is?

Ans Hyper
bil
i
rubi
nemi
a

78A27-year-
oldat32weeks’
gest
ati
onisbroughttot
heemergencyr
oom byambul
ance
af
terconsuming6gramsofacet
aminophen.Whichofthef
oll
owingi
str
ue?

AnsThi
sist
hemostcommoncauseofacut
eli
verf
ail
urei
ntheUni
tedSt
ates

79Whi
choft
hef
oll
owi
nghepat
icenzy
mesi
sincr
easedi
nnor
mal
pregnancy
?

Ans

ki
dneypat
hol
ogyt
heor
y

80Thecl
i
nical
present
ati
onofawhatcondi
ti
onusual
l
yincl
udeshy
per
tensi
on,
hematur
ia,r
ed-
cel
lcast
s,py
uri
a,pr
otei
nur
ia,
edema,
hyper
tensi
on,
andci
rcul
ator
y
congest
ion

Ans.Gl
omer
ulonephr
it
is

81Whatisthecut
offf
oraur
inar
ypr
otei
n-t
o-cr
eat
ini
ner
ati
oonaspotur
inesampl
ein
pr
egnantpati
ent
?

Ans

82Whichofthef
oll
owi
ngi
sthemostcommonf
orm ofacut
egl
omer
ulonephr
it
is
wor
ldwide?

Ans

83Apregnantpat
ientwhopr
evi
ouslydonat
edakidneyandnowhasonl
yoneheal
thy
ki
dneyi
satri
skforwhichoft
hefol
lowingobst
etr
iccompli
cat
ions?

Ans

84Thepr
egnantpatientwi
thaki
dneystoneaskshowmanypregnantwomenactual
l
y
passt
hei
rstonewiththemanagementplanofanal
gesi
aandi
ntrav
enoushydr
ati
on.
Whati
syourresponse?

Ans

85Whatper
centageofpregnantwomenexper i
encear
ecur
rentur
inar
ytr
acti
nfect
ion
af
tercompl
eti
onoftreat
mentf orpy
elonephr
it
is?

Ans

86Whi
choft
hef
oll
owi
ngst
atement
saboutpol
ycy
sti
cki
dneydi
seasei
str
ue?

Ans

87Whichofthef
oll
owi
ngst
atement
sconcer
ningpr
egnancyaf
terki
dneyt
ranspl
anti
s
notcor
rect
?

Ans

88Whi
choft
hef
oll
owi
ngi
sNOTaphy
siol
ogi
cal
changei
npr
egnancy
?

Ans

89Whi
chofthefol
l
owingisanacceptabl
etr
eat
mentr
egi
menf
ornewl
ydi
agnosed
asy
mptomat
icbact
eri
uri
ainpregnancy?

Ans

90Whi
choft
hef
oll
owi
ngst
atement
sregar
dingphy
siol
ogi
cal
changesi
npr
egnancyi
s
t
rue?

Ans

91Whati
sthel
eadi
ngcauseofsept
icshockdur
ingpr
egnancy
?

AnsPyel
onephr
it
is

92Youngpr egnantpat i
entwit
hpy elonephr
iti
sandlaborcontract
ionstransfer
redto
labordepart
mentf ormonitori
ng.Shei sgiv
enintr
avenous( I
V)antibi
oti
cs,IVfl
uids,
acetaminophen,andaβ- agoni
stbecauseoft hecontract
ions.Whatcompl icat
ionis
mor eli
kel
yintheset t
ingofβ-agonistuseinpati
entswithpyelonephri
ti
s?

Ans

93Whi
choft
hef
oll
owi
ngi
snotcommonl
yfoundi
npat
ient
swi
thaur
ethr
aldi
ver
ti
cul
um?

AnsUr
inar
yret
ent
ion

94Whichofthef
oll
owi
ngi
sthemostf
requentbact
eri
ali
nfect
ionscompl
i
cat
ing
pr
egnancy?

Ansasy
mpt
omat
icbact
eri
uri
a

95Oncey ouhav
etreat
edapregnantpat
ientf
orpyelonephr
it
isasani
npat
ient
,forhow
longshoul
dyouconti
nueor
alther
apyasanout pat
ient?

Ans

96Whenmanagingapatientwit
hpostpar
tum acut
ekidneyi
njur
y,whi
choft
hef
oll
owi
ng
medi
cat
ionsdoesnotneedt ohavei
tsdoseadjust
ed?

Ans

97Whichofthefol
l
owingcompl
i
cat
ionsi
snoti
ncr
easedi
npr
egnantpat
ient
swi
th
nephr
oti
csyndrome?

AnsPost
ter
m pr
egnancy

98Whati
sthel
eadi
ngcauseofacut
epy
elonephr
it
isdur
ingpr
egnancy
?

AnsEscher
ichi
acol
i

99Bywhatcr
eatini
nedoesonedef
inemoder
ater
enal
impai
rmenti
npat
ient
swi
th
chr
oni
crenal
disease?

Ans

Whatper
centageofpregnantwomenwit
hasy
mpt
omat
icbact
eri
uri
adev
elopa
sy
mptomaticinf
ect
ioniflef
tuntr
eat
ed?
Ans.25%

Whi
choft
hef
oll
owi
ngi
smostl
i
kel
ytol
eadt
oend-
stager
enal
disease?

Ans Di
abet
es

Loweruri
nar
ytractsy
mpt
omswi
thpy
uri
abutast
eri
l
eur
inecul
tur
ear
eli
kel
yduet
o
whichpat
hogen?

Ans Chl
amy
diat
rachomat
is

Whichofthefol
lowi
ngist
het
hreshol
dforpr
otei
nur
iai
npr
egnancy
,abov
ewhi
chl
evel
s
ar
econsideredabnor
mal?

Ans.300mg/
d

Ans.Pr
otei
nur
ia&l
t;
300mg/
day

I
ncaseofur
inar
ytr
actobst
ruct
ionwhatdi
agnost
icopt
ioni
sthemet
hodofaf
ir
stchoi
ce?

Ans.Renal
sonogr
aphy

I
nt hesetti
ngofaggressi
vefl
uidhydr
ation,whatper
cent
ageofpr
egnantwomenwi
th
pyelonephri
ti
sdevel
opacutekidneyi
njury?

Ans.5%

Whi
choft
hef
oll
owi
ngdoesnotchar
act
eri
zenephr
oti
csy
ndr
omes?

Ans

Whi
choft
hef
oll
owi
ngv
aluesi
snotanor
mal
ser
um cr
eat
ini
nei
npr
egnancy
?

Ans

Whati
sthemostcommonr
easonf
oracut
eki
dneyi
njur
yinobst
etr
ics?

Ans

Testf
orl
i
verpat
hol
ogf
ycl
i
nical
case

A32-year
-ol
dat26weeks’gestat
ionhasa3.4-
cm l
i
vermassnotedinci
dent
all
yona
ri
ghtupperquadr
antul
tr
asound.Sheunder
goesmagneti
cresonanceimagi
ngforf
urt
her
cl
ari
fi
cati
on.Whichoft
hefoll
owingistr
ue?

Ans
Whi
chcombinati
onoflabor
ator
yfi
ndi
ngsbel
owwoul
dbemostconcer
ningf
oracut
e
f
att
yli
verofpr
egnancy
?

Ans

Howdoy oucounsel
theear
lypostdel
i
ver
ypat
ientr
egar
dingherf
oll
owi
ngv
iral
ser
ologi
es?

 I
gAAnti-
Hepat
it
isBcoreanti
body+
 Hepat
it
isBsur
faceant
ibody–
 Hepat
it
isBeanti
gen+

Ans Shehasanact
ivev
irust
hatcanl
eadt
oci
rr
hosi
sanddeat
h,andy
ouwi
l
lref
erhert
o
ahepat
ologi
stf
orl
ong-
ter
m managementr
ecommendat
ions.

Yousendaperi
pheral bl
oodsmearoft
hepat
ientat34weeks’
gest
ati
onwithanacute
fat
tyl
iver
,andarepresentat
ivesl
i
dei
sshownbelow.Whatist
heunder
lyi
ngeti
ologyof
thebl
oodsmearfindings?

Ans

A36- y
ear -
oldmulti
gravi
dapr esent sforpr econceptioncounseli
ng.Herobst etri
chi st
ory
i
ncludest wofull
-t
ermv aginaldeliveri
esaf t
erpregnanciescompl i
catedbygest ati
onal
diabetesandaf i
rst-
tri
mest erpregnancyl oss.Shecomest oyoutodi scussherrecent
diagnosisofnonalcoholi
cf att
yliverdisease( NAFLD)byherpr i
mar ycarephy si
cian.
Diagnosiswasbasedoni magi ngper formedf oranepisodeofabdomi nalpai
n.Herl i
ver
functi
ont est
sarenor mal.Hermedi calhistoryisotherwisenotabl
ef orlongst
andi ng
obesity(bodymassi ndex34kg/ m2)anddi abetes(hemoglobinA1c8. 2%)diagnosed2
yearsago.Whi chofthefollowingshoul dbei ncludedinyourcounseling?

Ans

Thef
oll
owi
ngar
evi
ral
ser
ologi
esoft
hepr
egnantpat
ient
.Whati
sherdi
agnosi
s?

1.I
gAAnti-
Hepat
it
isBcor
eantibody+
2.Hepat
it
isBsurf
aceant
ibody–
3.Hepat
it
isBeanti
gen+

Ans.Chr
oni
chepat
it
isBi
nfect
ion,
sti
l
lact
ive

A19-year
-ol
dnulli
gravi
dawithSjögrensyndromepr esentsat26weeks’ gestati
onwith
nausea,f
ati
gue,andvagueabdomi nalpain.Onexam shehasabl oodpr essureof94/58
mmHgandy ounoticemildjaundi
ce.Laborator
yassessmentshowssev erelyel
evated
tr
ansaminasesandanti-
nuclearandanti
-smoot hmuscleant i
bodi
es.Basedony our
pri
marydiagnost
icsuspici
on,howdoy oucounsel herregardi
ngcomplicationsof
pr
egnancy
?

Ans

A32-y
ear-ol
dG1at32weeks’gest
ati
onisdi
agnosedwithintr
ahepaticchol
estasi
sof
pr
egnancy.Dur
ingt
hevisi
tshei
nquir
esaboutwhethert
hisdiagnosi
swi l
lal
terher
pr
enatal
careordel
iv
erypl
ans.Whatist
hebestevi
dence-basedadviceyoucanpr ov
ide?

Ans

A28- year
-ol
dat19weeks’ gestati
onpresent
stotheemer gencyroom wi
thcompl ai
nts
ofrightupperquadrantpai
n,fever
,andvomiti
ng.Exam r
ev eal
sri
ghtupperquadrant
tenderness,andshehasat emperatur
eof38.7°C.Onediagnosti
cimageisshownher e.
Commonbi leductobstr
ucti
onisnotsuspected.Whati
st henextmostappropri
atestep
i
nhermanagement ?

Ans.Lapar
oscopi
cchol
ecy
stect
omy

A27-year-
oldnull
igr
avidaandherhusbandpresentforpreconcept
ioncounsel
i
ng.She
hasWilsondiseasethatisbeingt
reat
edwit
hzi ncsulf
ate.Whichofthefoll
owi
ngshould
beincl
udediny ourcounsel
ing?

Ans

A32-year-
oldnull
igr
avi
dapr
esent
sforpr
econcept
ioncounsel
i
ngduetocir
rhosiswi
th
esophagealvar
ices.Whi
choft
hefol
lowi
ngshouldbeincl
udediny
ourcounsel
ing?

Ans Al
loft
heabov
e

A27-year-
oldmult
igr
avi
dacomest
oyouf orherpost
partum vi
sit
.Bothofher
pregnanci
eswerecompli
cat
edbyi
ntr
ahepaticchol
estasi
sofpregnancy.Shedesi
res
anotherchi
ldi
nabout2year
s.Whi
chofthefoll
owingmet hodsofcontr
acepti
ondoyou
recommend?

Ans

A26-year-ol
dpri
migravi
dapresentsat34weeks’gestati
onwithnauseaandvomiti
ng,
f
ati
gue, andepi
gastri
cpain.Al
lexceptwhichoft
hef oll
owingarecl
ini
calchar
act
eri
sti
cs
t
hatincreaseherri
skforacutefat
tyli
verofpr
egnancyast heunderl
yi
ngcause?

Ans

A29-year-
oldpr
imi
grav
idapresentstoy ouforprenatalcare.Sheisknownt ohave
hepati
tisCwit
hnoknownr i
skf act
ors,andyourpr enat
allabsconfir
m presenceof
hepati
ti
sCRNAandant i-
hepati
tisCant i
body.Compar edtoawomanwi t
hant i
-hepati
ti
s
Cant i
bodywhoisRNA-negat
ive,whichofthefollowingismor eli
kel
ytocompl icateher
pregnancy?

AnsVer
ti
cal
transmi
ssi
onofhepat
it
isC
Howdoy oucounsel
theear
lypostdel
i
ver
ypat
ientr
egar
dingherf
oll
owi
ngv
iral
ser
ologi
es?

 I
gAAnti-
Hepat
it
isBcoreanti
body+
 Hepat
it
isBsur
faceant
ibody–
 Hepat
it
isBeanti
gen+

AnsShehasanact
ivev
irust
hatcanl
eadt
oci
rr
hosi
sanddeat
h,andy
ouwi
l
lref
erhert
o
ahepat
ologi
stf
orl
ong-
ter
m managementr
ecommendat
ions.

Testf
orki
dneypat
hol
ogf
ycl
i
nical
case

Youareconsult
edabouta19- year-
oldpr i
migravidaat18weeks’ gest
ation.Thepati
ent
wasdiagnosedatherf i
rstprenatalcarev i
sitwit
hasympt omat icbacteri
uri
a.Her
provi
dertr
eatedherwithnitr
ofurantoin100mgt wicedai l
yf or7days.Thepat i
entwas
seenamont haft
ert r
eatment ,
andherr epeaturi
neculturewasnegat ive.Butnow,a
uri
necult
uresentinerrorispositi
vef or>100,000colony-formingunit
s/ mLgram-
negati
verodsagain.Sheremai nsasy mpt omati
c.Herpr ovideri
snotsur ewhatt odo.
Whatisthebestresponset othisconsul t
?

Ans.Recur
renceofasy
mpt
omat
icbact
eri
uri
ais30%r
egar
dlessofant
ibi
oti
cregi
menandi
tmay
i
ndicatecover
tuppertr
acti
nfect
ion,
sot
reatt
hepat
ientwi
thni
tr
ofur
ant
oin100mgbymout
hat
bedti
mef or21days.

A32- year
-oldG2P0pr esentsat19weeks’ gestati
oncompl ai
ningofbackpai nthat
radi
atesfor war
d.Sher epor
tsthatthepai
ni sintense,andshel ooksv eryuncomf ort
able.
Shehasnotbeenf ebril
e.Aurinal
ysisi
ssignifi
cantonlyforredbl oodcells.Animage
from herrenalul
trasoundi sprovi
dedbelow.Youdi agnosethepat ientwithakidney
stone.Whi chofthef oll
owingisthebestmanagementopt ionfort hepat i
entatthisti
me?

AnsAnal
gesi
aandi
ntr
avenoushy
drat
ion

Aseriesofant
eri
or-
poster
iorproj
ect
ionchestr
adiogr
aphsi
nasecond-
tr
imest
er
pregnantwomanwithseverepyel
onephri
ti
sincaseofaggr
essi
vef
lui
dhydr
ati
on
showingwhatofthefol
l
owi ng:

Ans.Respi
rat
orydi
str
esssy
ndr
ome

A25-year-
oldG1P0at16weeks’ gestat
ionpr esent
sf orprenatal
care.Shehasahi st
ory
ofakidneytranspl
ant3y earsago.Shei sstabl
eonpr ednisoneandazat hiopr
ine.The
pat
ienthasnotex peri
encedanyr ej
ecti
on,andherbl oodpr essur
eiswell cont
roll
ed.
Whichofthef ol
lowingobstetr
iccomplicati
onsisnoti ncr
easedinher?

Ans
Apregnantwomanpr esentswit
hflankpai
n.Sheisafebri
l
ebuttachycardi
cfrom pai
n.
Herserum cr
eat
ini
neiselevatedto5mg/ dL.Imagesfr
om hermagneticresonance
i
magingarepresent
edbel ow-hydronephr
osis.Whichofthef
oll
owingisthebestplanof
car
e?

AnsPer
cut
aneousnephr
ost
omyt
ube

1. Regardi
ngdiagnost
icst
udies,
whi
chofthefol
l
owi
ngcor
rect
lydescr
ibesanor
mal
changethati
scharact
eri
sti
cofpr
egnancy
?

Anav
erage15-
degr
eel
eftaxi
sdev
iat
iononel
ect
rocar
diogr
am

2. Thepatientwhosehear ti
sshowni nt
hisimagehassy stemi clupus
eryt
hemat osusandhasahi stor
yofapri
orstroke.Theleftatri
um (LA)andlef
t
ventr
icl
e( LV)arei dent
if
ied,andthear
rowspoi nttononinfecti
ousv eget
ati
onson
themi t
ralleafl
ets.Whichoft hefol
l
owingisthelikel
ycondi t
ionassociat
edwith
hermi t
ralinsuff
iciency
?
Ant
iphosphol
i
pidant
ibodi
es
3. Agr aphi
cofpul monarycapi
ll
arywedgepressure(r
edl i
ne)isshownf orawoman
withmi t
ralstenosi
s.“
A”repr
esentsthefi
rstst
ageofl abor
, “
B”mar ksthesecond
stageoflabor ,“
C”demonstr
atestheimmediatepostpart
um period,and“D”
showst heper i
od4–6hourspostdeli
ver
y.Whataccount sfortheri
sein
pulmonar ycapil
lar
ywedgepressureimmediat
elyfol
lowingdeliver
y?
Pl
acent
alaut
otr
ansf
usi
on
4. Whati
sthemostcommoncauseofhear
tfai
l
urei
npr
egnancy
?

Chr
oni
chy
per
tensi
onwi
thsuper
imposedpr
eecl
ampsi
a

5. Youarecari
ngforapri
migravi
dawi t
hknownhy pert
rophiccardi
omy opathy.She
tel
l
sy ouhermotheral
sosuff
eredfrom t
heconditi
onandpassedawayatan
earl
yage.Thepati
enti
sconcernedaboutherchil
d’sfut
ureriskfordevelopingt
he
sameconditi
on.Whatdoyoutellheri
sthemostl i
kel
yriskforpassingthe
condit
iont
oherchil
d?

50%

6. Youar ecari
ngf orapregnantpati
entwit
hmi tralstenosis.Sheisablet
oper form
mostact i
vit
iesofdail
ylivi
ngwithoutsi
gnif
icantlimitat
ion.Onechocardiogram,
herejecti
onf r
acti
onis50%andhermi t
ralvalv
ear eameasur es1.8cm2.Whi chof
thefoll
owingv ari
abl
esismostpr edi
cti
veofahi ghr i
skf orcardi
accompl i
cations
dur
ingt
hispr
egnancy
?

Herdegr
eeofl
eft
-si
dedobst
ruct
ion

7. Whati
sthegol
dst
andar
dfordi
agnosi
soft
hecondi
ti
oni
nQuest
ion49–24?
Ri
ghthear
tcat
het
eri
zat
ion

8. TheWor
ldHeal
thOr
gani
zat
ioni
dent
if
ieswhi
choft
hef
oll
owi
ngcondi
ti
onsas
pr
ohi
bit
iveofpr
egnancy
?

Pul
monar
yar
ter
yhy
per
tensi
on

1.Apr egnantwomanpr esent


st otheemer gencyr oom i
nanobt undedstat
e.She
hasaf everto102.6°
F.Onexam, a4/ 6mur murisnoted,shewithdr
awsfrom pain
duri
ngherabdomi nalexam, andtrackmar ksarenotedonherar msandl egs.Her
fundalheighti
sapproximately32cm.Anemer gentechocar
diogram i
sobtained,
whichdemonst rat
esami tr
al val
vev eget
ation.Whatisthemostl i
kel
yorganism
underlyi
ngherdiagnosi
s?

St
aphy
lococcusaur
eus

2.Youar ecar
ingforcomplainsofshortnessofbreathandchestpainwi t
hmi ni
mal
exert
ionanuncompl i
catedpr i
migr
avidaat32weeks’ gestati
on.An
echocardi
ogram demonst ratesamitral
val
vesur f
aceareaof2. 1cm2, adil
ated
l
eftatri
um,anaorti
cv al
vear eaof3.7cm2,arightventr
icularsyst
oli
cpressureof
20mm Hg, andanej ecti
onf r
acti
onof55%.Youal soobtainachestx- r
ay,which
i
sshown.Whati sthel i
kel
yunder l
yi
ngcauseofhersy mpt oms?

Mi
tr
alst
enosi
s

3.Youar ecari
ngforapat i
entwithanunr epai
redv
entri
cularsept
aldefectwhois
consider
ingpregnancy.Youperform anechocardi
ogram toassessherfutur
e
ri
sks.Whichoft hefol
lowingparameterswouldindi
cateanextremelyelev
ated
maternalandfetalr
iskduringpregnancy?

Ar
ightv
ent
ri
cul
arsy
stol
i
cpr
essur
eof75mm Hg
1.Whichoft
hefol
lowi
ngappr
oxi
matestheri
skf orper
inat
almor
tal
i
tyi
n
pr
egnanci
escompli
cat
edbychr
onichyper
tension?

Al
loft
hement
ioned

2.Forawomanwhodoesnotent erpregnancywit
haknowndiagnosi
s,t
he
di
agnosi
sofchr
onichypert
ensi
onissuppor t
edwhenhyper
tensi
onispresent
pri
ort
owhatgestat
ional
agethreshold?

20weeks’
gest
ati
on

3.Whichofthef
oll
owingchar
acter
izest herel
ationshi
pbet
weenchr
oni
c
hy
pertensi
onandsuper
imposedpr eeclampsia?

Approx
imatel
y80%ofthosewhoal
sohav
eatl
east300mg/
dayofpr
otei
nur
iaatbasel
i
ne
wil
ldevel
opsever
epr
eeclampsi
a

4.Amongwomenwithchroni
chyper
tensi
on,
whichoft
hef
oll
owi
ngi
str
uer
egar
ding
ri
skf
oradv
ersemater
nalandperi
natal
out
come?

Compli
cat
ionsar
emor eli
kel
ywit
hsev
erer
angehy
per
tensi
onatbasel
i
neandt
he
pr
esenceofend-
organdamage.

5.Riskfort
hepl
acentalabr
upt
ioni
sincr
easedinpregnanci
escompli
cat
edby
chroni
chyper
tensi
on.Howwouldyoubestchar
acteri
zetheri
sk?

2-
3-f
oldi
ncr
easeov
ert
hegener
alobst
etr
icpopul
ati
on

6.Inaddi
ti
ontochr
onichyper
tensi
on,
whichoft
hefol
lowi
ngcondi
ti
onsar
e
consi
deredhi
gh-
ri
skfordevel
opi
ngpreecl
ampsi
a?

Sy
stemi
clupuser
ythemat
osus

7.Ant
ihy
pert
ensi
vet
her
apyi
smandat
oryf
orwhi
choft
hef
oll
owi
ngwomendur
ing
pr
egnancy
?
30-
year
-ol
dG3P2wi
thabl
oodpr
essur
eof160/
110mm Hgandahi
stor
yofast
hma

1. Ms.Edwar
dsi
sa26-
year
-ol
dG2P2,
now3day
spost
par
tum f
rom av
agi
nal
deli
very.Shehaschr oni
chy pert
ensionanddev el
opedsuper imposedsev er
e
preecl
ampsi aat36weeks’ gestati
on.Shedi dwelli
nit
ial
l
y, butinthelast12hours
herbloodpr essurehasbeent r
endingup.Mostr ecentbl
oodpr essur
ewas
156/90mm Hg.Thenur sehascalledy oubecauseMs.Edwar dsisfeel
ingshor
t
ofbreath.Ar epresentat
ivechestradiographisshownher e.Whati sthe
appropriat
et r
eatmentf orthisconditi
on?

I
ntr
avenousf
urosemi
de

2. Fort
hepat
ientwi
thmoder
atechr
oni
chy
per
tensi
on,
whi
choft
hef
oll
owi
ng
compl
i
cat
ionsi
sreducedwi
thuseofant
ihy
per
tensi
vet
her
apydur
ingpr
egnancy
?

Dev
elopmentofsev
erehy
per
tensi
on

3. A33-
year
-ol
dG1at32weeks’
gest
ati
onhasbeendi
agnosedwi
thchr
oni
c
hypertensionwit
hsuper i
mposedpr eeclampsi
aviaworseninghy pertension.She
i
sasy mpt omati
cwi t
hnor mall
absandnor malf
etalgr
owth.Ani ncreasei nthe
doseofherant i
hypertensi
vemedicat i
onhasresult
edinnonsever ebl ood
pressures.Sheasksy ouaboutexpectantmanagementofsuper imposed
preecl
ampsi a.Whichofthefol
lowingist hemostappropr
iate,
ev idence-based
response?

I
tisreasonabl
e,wit
hcloseinpat
ientsur
vei
l
lance,
topur
sueexpect
antmanagement
,but
t
herearestil
lri
sksforadv
erseoutcomes.

1. Whi
choft
hef
oll
owi
ngv
iral
inf
ect
ionshasbeenassoci
atedwi
thamar
ked
i
ncr
easei
nther
iskf
ori
ntr
ahepat
icchol
est
asi
sofpr
egnancy
?

Hepat
it
isC

2. Whi
choft
hef
oll
owi
ngst
atement
sregar
dingacut
evi
ral
hepat
it
isi
scor
rect
?

Low-
gradef
everi
smor
ecommonwi
thhepat
it
isA.
3. Whi
choft
hef
oll
owi
ngcombi
nat
ionsr
epr
esent
sappr
opr
iat
escr
eeni
ngf
or
hepat
it
isCi
npr
egnancy
?

15-
year-
oldwomanwhosemot herwasachroni
cintr
avenousdr
uguseranddi
ed
ofl
iverf
ail
urewhent
hepat
ientwas4y ear
soldscreenedvi
ahepat
it
isCRNA

4. Al
lEXCEPTwhi
choft
hef
oll
owi
ngl
i
ver
-r
elat
edchangesar
ephy
siol
ogi
cal
in
pr
egnancy
?

Hy
pol
i
pidemi
a

5. Mat
ernal
acut
efat
tyl
i
verofpr
egnancyi
sassoci
atedwi
thal
lexceptwi
thof
thefol
lowingr
ecessi
vel
yinher
it
edabnor
mal
i
tiesofmi
tochondr
ial
fat
tyaci
d
oxidat
ion?

Di
hydr
oli
poami
dedehy
drogenase

6. Whi
choft
hef
oll
owi
ngf
eat
uresar
ecr
it
eri
aforsev
eredi
seaset
hatshoul
d
pr
ompthospi
tal
i
zat
ionf
oracut
evi
ral
hepat
it
is?
Hy
per
bil
i
rubi
nemi
a
7. Whi
choft
hef
oll
owi
ngpr
egnancy
-rel
atedcompl
i
cat
ionshast
hecapaci
tyt
o
demonstr
atethemostprominental
terati
onsofnor
mal
hepat
ic,
renal
,
hematol
ogical
,andcoagul
ati
onlaborator
ystudi
es?

Acut
efat
tyl
i
verofpr
egnancy

8. Whati
sthei
nci
denceandr
ecur
rencer
iskofacut
efat
tyl
i
verofpr
egnancy
?

1i
n10,
000pr
egnanci
es,
andr
ecur
rencei
srar
e

9. Lact
ati
oni
scont
rai
ndi
cat
edf
orwomeni
nfect
edwi
thwhi
choft
hef
oll
owi
ng?

HI
V
1.Whatpercentageofpr
egnantwomenexper
iencear ecur
rentur
inar
ytr
act
i
nfect
ionaftercompl
eti
onoftr
eatmentf
orpyelonephri
ti
s?

30–40%

2.Whichoft
hef
oll
owi
ngi
snotarecommendedr
equi
sit
eforr
enal
transpl
ant
pat
ient
swhowantt
oat
temptpr
egnancy
?

Pr
otei
nur
ia&l
t;
300mg/
day

3.Whi
chofthefol
l
owingcompl
i
cat
ionsi
snoti
ncr
easedi
npr
egnantpat
ient
s
wi
thnephr
oti
csyndr
ome?

Post
ter
m pr
egnancy

4.Whichofthef
oll
owingi
sanacceptabl
etr
eat
mentregi
menf
ornewl
y
di
agnosedasymptomat
icbact
eri
uri
ainpr
egnancy
?

Ni
tr
ofur
ant
oin100mgbymout
hatbedt
imef
or10day
s

5.Whi
choft
hef
oll
owi
ngst
atement
saboutpol
ycy
sti
cki
dneydi
seasei
str
ue?

10%oft
hosewi
tht
hisdi
seasedi
efr
om r
upt
ureofani
ntr
acr
ani
alber
ryaneur
ysm.

6.Whichoft
hefoll
owi
ngval
uesi
snotanor
mal
ser
um cr
eat
ini
nei
n
pr
egnancy
? 1.0mg/
dL

7.Whati
sthel
eadi
ngcauseofacut
epy
elonephr
it
isdur
ingpr
egnancy
?

Escher
ichi
acol
i

8.Whi
chofthefoll
owingi
sthet
hreshol
dforpr
otei
nur
iai
npr
egnancy
,abov
e
whi
chl
evelsareconsi
der
edabnormal
?
300mg/
d

9.Whichoft
hefoll
owingst
atement
sregar
dingphy
siol
ogi
cal
changesi
n
pr
egnancyi
strue?

Ef
fect
iver
enal
plasmaf
lowi
ncr
eases.

1. Whi
chcombi
nat
ionofl
abor
ator
yfi
ndi
ngsbel
owwoul
dbemostconcer
ning
f
oracut
efat
tyl
i
verofpr
egnancy
?

Hematocri
t32%, cr
eat
ini
ne2.
0mg/
dL,
plat
elet
s90/
L,AST400U/
L,f
ibr
inogen130
mg/dL,andglucose65mg/dL

2. A28-
year
-ol
dat19weeks’
gest
ati
onpr
esent
stot
heemer
gencyr
oom wi
th
compl ai
ntsofr
ightupperquadrantpai
n,f
ever,
andvomiti
ng.Exam r
eveals
ri
ghtupperquadranttender
ness,andshehasat emper
atur
eof38.7°C.One
diagnosti
cimageisshownher e.Commonbi l
eductobst
ruct
ionisnot
suspected.Whatisthenextmostappropri
atestepi
nhermanagement ?

Lapar
oscopi
cchol
ecy
stect
omy

3. A19-
year
-ol
dnul
l
igr
avi
dawi
thSj
ögr
ensy
ndr
omepr
esent
sat26weeks’
gestati
onwi t
hnausea, f
ati
gue,
andvagueabdomi nalpain.Onexam she
hasabl oodpressureof94/58mmHgandy ounot i
cemildj aundi
ce.
Laboratoryassessmentshowssever
elyelevat
edt r
ansami nasesandanti
-
nuclearandanti-
smoot hmuscl
eanti
bodies.Basedony ourpr i
mary
di
agnost i
csuspicion,
howdoy oucounselherregardi
ngcompl i
cat
ionsof
pregnancy?

Shei
sati
ncr
easedr
iskf
orpr
eter
m del
i
ver
y.

4.Thefol
lowingar
evi
ral
ser
ologi
esoft
hepr
egnantpat
ient
.Whati
sher
di
agnosis?
1.I
gAAnti-
Hepat
it
isBcor
eantibody+
2.Hepat
it
isBsurf
aceant
ibody–
3.Hepat
it
isBeanti
gen+

-Chr
Ansr: oni
chepat
it
isBi
nfect
ion,
sti
l
lact
ive

5. Howdoy oucounsel
theear
lypostdel
i
ver
ypat
ientr
egar
dingherf
oll
owi
ng
vir
alser
ologi
es?

 I
gAAnti-
Hepat
it
isBcoreanti
body+
 Hepat
it
isBsur
faceant
ibody–
 Hepat
it
isBeanti
gen+

Ansr-Shehasanac
: ti
vevi
rusthatcanleadtoci
rr
hosisanddeat
h,andyouwi
l
l
r
eferhert
oahepat
ologi
stf
orlong-t
erm managementrecommendati
ons.

1.Youar econsul tedabouta19- year-oldpri


mi gravi
daat18weeks’ gestat
ion.
Thepat ientwasdi agnosedatherf i
rstprenatalcarevi
sitwith
asympt omat icbact er
iur
ia.Herprovidertreatedherwithnitr
ofurantoi
n100
mgt wicedai lyfor7day s.Thepat i
entwasseenamont haf t
ertreatment,
andherr epeaturineculturewasnegat i
ve.Butnow, auri
neculturesentin
err
orisposi t
ivefor>100,000colony -
formi ngunit
s/mLgr am-negativerods
again.Sher emai nsasy mptomatic.Herpr ovideri
snotsur ewhatt odo.
Whati st hebestr esponset ot
hisconsult?

Recurr
enceofasymptomati
cbacter
iur
iais30%regardlessofantibi
oti
cregi
men
anditmayindi
catecover
tuppert
racti
nfect
ion,
sotreatt hepat
ientwit
h
ni
trof
urant
oin100mgbymout hatbedtimefor21day s.

2.An18-year
-oldG1P0pr esent
sat30weeks’gestati
onwi thfeverof39oC,
chil
l
s,vomit
ing,dy
suri
a, andfr
equentur
inat
ionfor3day s.Sheisalso
experi
enci
ngcontract
ions.Whatishermostli
kelydiagnosis?

Py
elonephr
it
is

4.Aser
iesofant
eri
or-
post
eri
orpr
oject
ionchestr
adi
ogr
aphsi
nasecond-
tr
imest
er
pregnantwomanwit
hsever
epyel
onephr
it
isi
ncaseofaggr
essi
vef
lui
dhy
drat
ion
showingwhatoft
hefol
l
owing:

Respi
rat
orydi
str
esssy
ndr
ome

5.Apr egnantwomanpresent
swithfl
ankpai
n.Sheisafebr
il
ebuttachy
cardicfr
om
pain.Herserum cr
eat
ini
neisel
evat
edto5mg/ dL.I
magesf r
om hermagnetic
resonanceimagingar
epresent
edbelow-hydr
onephrosi
s.Whichofthefol
lowing
i
st hebestplanofcar
e?

Per
cut
aneousnephr
ost
omyt
ube

1.Whichoft
hefol
l
owi
ngi
sthoughtt
opl
ayasi
gni
fi
cantr
olei
nthedev
elopmentof
preecl
ampsi
a?
Al
loft
hement
ioned
2.Ant
ihy
per
tensi
vecont
rai
ndi
cat
edi
npr
egnancy
ACEi
nhi
bit
ors
3.Al
lar
efeat
uresofHELLPsy
ndr
omeexcept
Eosi
nophi
l
li
a
4.Whatpropor
ti
onofmat
ernal
deat
hscanbeat
tri
but
edt
ohy
per
tensi
vedi
sor
der
sin
pregnancy
?
1i
n6
5.Whi
chst
atementbestdescr
ibesr
enal
per
fusi
onandgl
omer
ularf
il
tr
ati
onr
atesi
n
womenwi
thpr
eecl
ampsi
athathasnoty
etpr
ogr
essedt
osev
eredi
sease?
Si
mil
arcompar
edt
onor
mal
nonpr
egnantv
alues

6.Bestr
egi
menf
orecl
ampsi
ais
MgSo4

1.A19-year
-ol
dG1P0womanat39weeks’ gestat
ionisdi agnosedwi t
hpr eeclampsia
basedonbloodpr essur
eof150/90mm Hgand2+pr oteinuriaonur i
nedi pstick.The
pati
entispl
acedonmagnesi um sul
fat
e,anddevel
opsf l
ushi ngandf at
igue.Sheasks
abouttheneedforthemagnesium sul
fat
e.Youexplai
nt hatitist opr
eventt hesei zur
es
thatmaycomplicatepreecl
ampsiaandmayevencausedeat h.Thepatientaskshow
seizur
esassoci
atedwi t
hpreecl
ampsiacancausemor t
al i
ty.Whi chofthef oll
owi ngis
t
hemostcommonmechani
sm?

I
ntr
acer
ebr
alhemor
rhage

2.Yourpati
enti
sadmi
ttedt
othehospit
alforeval
uati
onofnew-onsethy
pertensionat30
weeks’gest
ati
on.Whi
chofthef
oll
owingtestsoreval
uat
ionsi
sleastappropri
ate?

Cel
l
-fr
eeDNAt
est
ingf
oraneupl
oidy

3.A19-
year
-ol
dG1P0womanat39weeks’
gest
ati
oni
sdi
agnosedwi
thpr
eecl
ampsi
a
basedonbloodpr essur
eof150/90mm Hgand2+pr oteinuriaonur i
nedi psti
ck.The
pati
entispl
acedonmagnesi um sul
fat
e,anddevel
opsflushi ngandf at
igue.Sheasks
abouttheneedforthemagnesium sul
fat
e.Youexplai
nt hatitist opr
eventt heseizur
es
thatmaycomplicatepreecl
ampsiaandmayevencausedeat h.Thepatientaskshow
seizur
esassoci
atedwi t
hpreecl
ampsiacancausemor t
al i
ty.Whi chofthef oll
owingis
themostcommonmechani sm?

I
ntr
acer
ebr
alhemor
rhage

4.A23-
year
-ol
dpr
imi
grav
idapr
esent
swi
thabl
oodpr
essur
eof160/
104mm Hg,
3+
prot
einuri
a,andrightupperquadrantdiscomf or
t.Shehasav aginaldel
i
verycompli
cated
bybilat
eralsul
caltearsandanest i
matedbl oodlossof1500mL.Shepr oduces110mL
ofurineinthefi
rst4hour spost
par t
um andherser um creat
ininerisesfr
om 0.98mg/dL
to1.42mg/ dL.Whatist hemostlikel
yexplanati
onf orthi
sfinding?
Post
par
tum hemor
rhage

5.A24-
year
-ol
dpr
imi
grav
idapr
esent
sat37weeks’
gest
ati
onwi
thheadache,
abl
ood
pressur
eof170/ 102mm Hg, andsever
eri
ghtupperquadrantpai
n.Sheisdiagnosed
withHELLPsy ndromeandundergoesanuncompli
catedinducti
onoflabor
.Herright
upperquadrantpainper
sists,
andacomputedtomographyscanofherabdomen/ pel
vi
s
i
scompl et
edwi ththefi
ndingsasshownbel
ow.Whati sdenotedbytheasteri
sk(*)?
Subcapsul
arhemat
oma

6.A21-y
ear-ol
dG1at36weeks’ gestati
onpr esentsforhercli
nicvi
sitandi snotedto
haveabloodpressureof148/88mm Hg.Ar epeatbloodpressure30mi nutesl
ateri
s
146/92mm Hg.Herbl oodpressuresthroughoutpr egnancyhavebeenbel ow140/ 90
mm Hg.Shedeni esanycomplaint
s,andur i
nalysisisnegativ
eforproteinuri
a.Whatis
themostli
kelydi
agnosis?
Gest
ati
onal
hyper
tensi
on
7.A29-
year
-ol
dG1P0womanat28weeks’
gest
ati
oni
sadmi
tt
edt
othehospi
tal
for
preecl
ampsia.Herbl
oodpressureis150/100mm Hgandherpr oteinexcret
ionis500
mgi n24hours.Onhospi
talday7,sheisdiagnosedwithseverepreeclampsiaandthe
decisi
onismadet oadmi
nistermagnesium sul
fat
eanddel i
verthebaby .Whichofthe
fol
lowingfi
ndingsi
smostlikel
ypresentint
hispati
entascrit
eri
af orsevere
preecl
ampsia?
5gofpr
otei
nur
iaexcr
etedi
na24-
hourper
iod
8.A21-
year-
oldpri
mi gr
avidapresentsat36weeks’gestat
ionwi
thnew-onsetheadache.
Herbl
oodpressureis150/ 90mm Hg, herser
um cr
eatini
neis0.
8mg/ mL,ASTi s32U/ L,
andpl
atel
etcountis28,000/μL.Whichofthefol
l
owi ngcri
ter
iaf
orsever
epr eecl
ampsia
ismet
?
Thr
ombocy
topeni
a
9.Whi
choft
hef
oll
owi
ngi
sthebestmanagementofan18-
year
-ol
dG1P0womanat28
weeks’
gestati
onwithabloodpressur
eof160/
110mm Hg,
elev
atedl
i
verf
unct
iont
est
s,
andaplat
eletcountof60,
000/
uL?

Magnesi
um sul
fat
ether
apyandi
nduct
ionofl
abor

1.A25-year
-ol
dwomanat34weeks’ gestati
onisnotedt
ohav eaplacentapr
evia,
after
shepresent
edwithvagi
nalbleedi
ngandhasunder gonesonography.At37weeks,she
hasascheduledcesar
ean.Uponcesar eansect
ion,
blui
shti
ssuedenselyadherent
betweentheuter
usandmat ernal
bladderisnot
ed.Whichofthefoll
owingist
hemost
li
kel
ydiagnosi
s?

Pl
acent
aper
cret
a

2.A24-
year
-ol
dG1P0womanat39weeks’
gest
ati
onhadi
nduct
ionofl
aborduet
o
gestati
onal hyper t
ension.Shewaspl acedonmagnesi um sulfat
eforseizureprophyl
axi
s.
Shewaspl acedonoxy tocinfor15hoursandr eachedacervicaldi
lat
ionof6cm.Af t
er
beingat6cm di lati
onf or3hoursdespiteadequat euteri
necontracti
onsasj udgedby
240Mont ev ideouni t
s,sheunder wentacesar eandeli
v er
y.Thebabywasdel iv
ered
withoutdi
ff i
cul t
ythroughal ow-tr
ansverseincisi
on.Upondeliver
yoft heplacenta,
profusebleedi ngwasnot edfrom t
heuterus,reaching1500mL.Whi choft hefoll
owi
ng
i
st hemostl ikelycauseofhemor rhageinthispatient
?
Ut
eri
neat
ony
3.A23-
year
-ol
dG1P0womanat38weeks’
gest
ati
ondel
i
ver
eda4.
3kgbabyboy
vaginal
l
y.Upondeli
veryoftheplacent
a,t
her ewasnotedtobeaninvert
eduter
us,whi
ch
wassuccessful
lymanagedi ncl
udingr
eplacementoftheuter
us.Whichoft
hef ol
l
owing
placent
ali
mplantat
ionsit
eswoul dmostlikel
ypredi
sposetoaninv
ertedut
erus?
Fundal

1.Abnor
mal
breastdev
elopmentmaybeduet
oei
therf
asci
aladher
encet
othe
underl
yi
ngmusclel
ay erorduetohi
gh-
doseexogenoushormoneexposureduri
ng
pubert
y.I
nsuchcases,breast
shaveexcessiv
eforwar
dbutli
mitedl
ater
al gr
owth,
whi
ch
causesanabnor
mal shape.Thi
scondi
ti
oni sref
err
edtoaswhichoft
hef oll
owi
ng?
Tuber
ousbr
east
s
2.Whi
choft
hef
oll
owi
ngi
str
uer
egar
dingi
sol
atedpr
emat
uret
hel
arche?
I
tismostcommoni
ngi
rl
syoungert
han2y
ear
s.
3.Whichofthefol
l
owi
nggener
all
yoccur
sfi
rstamongt
hemaj
ordev
elopment
alev
ent
s
offemalepuber
ty?
Thel
arche
4.I
ntheUni
tedSt
ates,
adol
escent
scanobt
ainmedi
cal
car
ewi
thoutpar
ent
alknowl
edge
orconsentorwhi
choft
hef
oll
owi
ng“
medi
cal
l
yemanci
pat
ed”condi
ti
ons?
Al
loft
hement
ioned
5.HowdoBMIcor
rel
atewi
thpuber
tal
dev
elopment
?
hi
gherBMIcor
rel
atewi
thear
lydev
elopment
6.Pubar
che,
wit
hdev
elopmentofaxi
l
lar
yandpubi
chai
r,i
sst
imul
atedbyt
heandr
ogens
der
ivedpr
imar
il
yfr
om whi
choft
hef
oll
owi
ng?
Adr
enal
glands
7.Whi
choft
hef
oll
owi
ngor
gani
smsi
smostl
i
kel
ytocausepr
epuber
tal
vul
vi
ti
s?
Gr
oupAβ-
hemol
yti
cst
rept
ococcus
8.Mast
it
isi
sunusual
inchi
l
drenandadol
escent
s.Whati
sthemostcommonpat
hogen
i
sol
atedf
rom br
eastabscessesi
nthepedi
atr
icpopul
ati
on?
St
aphy
lococcusaur
eus

1.Apat
ientpr
esent
stot
heEmer
gencyDepar
tmentwi
tha1-
dayhi
stor
yofheav
yvagi
nal
bleedi
ng.Sheistachy cardicbutnothy potensive,andpost ur
alvit
alsi
gnsarestablewith
changef r
om asupinet osit
ti
ngposi t
ion.Physical examinati
ondemonst r
atesbleedi
ng
fr
om abov et
heext ernal cer
vicalosandcont inuedpool i
ngofbl oodinthevagi
na.
Laborator
ystudi
esr ev ealsheisanemi c.Whichoft hefoll
owingisthemostappr opr
iat
e
fi
rst-
li
neagenttoat t
emptcont rolofheracuteut erinebleedi
ng?
I
ntr
avenousest
rogen
2.Cont
rol
ofbl
oodl
ossdur
ingmensesi
nvol
veswhi
choft
hef
oll
owi
ngmechani
sms?
Al
loft
hement
ioned
3.Thel
esi
onwi
thasi
ngl
efeederv
essel
(bycol
orDoppl
er)i
sseenhy
ster
oscopi
cal
l
y.
Ri
skact
orsormal
i
gnantt
ransf
ormat
ionoft
hisl
esi
oni
ncl
udewhi
choft
hef
oll
owi
ng?
Al
loft
hement
ioned
4.Whi
choft
hel
abor
ator
ycr
it
eri
ali
stedbel
owi
sconsi
stentwi
thi
ron-
def
ici
encyanemi
a?
Lowser
um f
err
it
inl
evel
5.Suggest
edet
iol
ogi
esorabnor
mal
uter
inebl
eedi
ngassoci
atedwi
thuseoft
hedev
ice
shownbel
owi
nthi
sthr
ee-
dimensi
onal
(3-
D)sonogr
am i
ncl
udewhi
choft
hef
oll
owi
ng?
Al
loft
hement
ioned
6.Whencounsel
i
ngapat
ientf
orendomet
ri
alabl
ati
on,
whi
choft
hef
oll
owi
ngpoi
ntsar
e
di
scussed?
Al
loft
hement
ioned
7.I
npremenarchi
algi
rl
s,whi
choft
hef
oll
owi
ngi
sthemostcommonsour
ceof
abnor
malbleedi
ng?
Vagi
na
8.Whi
choft
hef
oll
owi
ngi
sal
i
mit
ati
onofPi
pel
l
esampl
ersusedf
orendomet
ri
albi
opsy
t
oev
aluat
eabnor
mal
uter
inebl
eedi
ng?
Lowsensi
ti
vi
tyandhi
ghf
alse-
negat
iver
ateorf
ocal
endomet
ri
alpat
hol
ogy

9.Fort
het
reat
mentofdy
sfunct
ional
uter
inebl
eedi
ng,
evi
dencesuppor
tswhi
choft
he
fol
lowi
ngstat
ementsregardi
ngt
heef
fi
cacyoft
hedev
iceshowni
nthi
sthr
ee-
dimensi
onal(
3-D)sonogr
am?
Al
loft
hement
ioned
10.Anadv
ant
ageoft
ransv
agi
nal
sonogr
aphyf
ort
heev
aluat
ionofabnor
mal
uter
ine
bl
eedi
ngi
ncl
udeswhi
choft
hef
oll
owi
ng?
Al
loft
hement
ioned

1.Whatdef
ini
ti
vet
reat
menti
sthemostpr
efer
abl
eina48-
year
-ol
dpar
ous,
signi
fi
cant
ly
obese,
diabet
icandhy
per
toni
cpat
ientwi
thr
ecur
rentDUB?
endomet
ri
um abl
ati
on
2.A32-
year
-ol
dpat
ientcompl
ainsofbl
eedi
ngbet
weenherper
iodsandi
ncr
easi
ngl
y
heavymenses.Ov erthepast9mont hs,shehashadtwodilati
onandcurett
ages(D&Cs)
,
whichhavefail
edtor esol
vehersy
mpt oms,andcont
racept
ivesandanti
prostagl
andi
ns
havenotdecreasedtheabnormalbleeding.Ofthef
oll
owingoptions,
whichisthemost
appropr
iat
eatt hi
stime?
Per
for
m hy
ster
oscopy

1.Mostcommoncauseofpost
menopausal
bleedi
ngi
nIndi
ais:
Car
cinomacer
vix
2.Apat
ientpr
esent
swi
thCacer
vixwi
thst
ageI
II
b.Tr
eat
mentofchoi
cei
s:
I
ntr
acav
itat
orybr
achy
ther
apyf
oll
owedbyext
ernal
beam r
adi
other
apy
3.Cy
togenet
icsi
sdi
fficul
tinsol
i
dtumor
s,t
hatt
ooespeci
all
yincar
cinomacer
vix,
duet
o:
Of
tencont
ami
nat
edandi
nfeсt
edwi
thmi
croor
gani
sms
4.An18-y
ear-oldadolescentfemaleisbeingseenforahealthmai nt
enanceappoi
ntment.
ShehasnothadaPapsmearpr evi
ously
.Shecur r
entl
ytakesoralcontr
acept
ivepi
l
ls.She
begansexualint
ercourse1y earprevi
ously.Whichofthefol
lowingstat
ementsismost
accur
ateregardi
ngheal t
hmai ntenanceforthi
sindiv
idual
?
APapsmearshoul
dnotbeper
for
medi
nthi
spat
ientatt
hist
ime

1.I
nci
denceofchor
iocar
cinomai
sseenmor
eaf
ter
Spont
aneousabor
ti
on
2.Al
loft
hef
oll
owi
ngar
eindi
cat
ionsf
orpost
oper
ati
ver
adi
other
apyi
nacaseof
car
cinomaendomet
ri
um,
except
:
Tumorposi
ti
vef
orest
rogenr
ecept
ors
3.A35-
year
-ol
dG0P0pr
esent
stoherdoct
orf
ori
nfer
ti
li
ty.Sheal
sohasheav
yvagi
nal
bl
eedingt
hatpersi
stedfor3weeks.Shehasalonghist
oryofol
igomenor
rhea.An
endometr
ial
biopsyshowsGr ade1endometr
ial
carci
noma.Whichofthefol
lowingi
sthe
bestt
reat
mentforthispati
ent?
Pr
ogest
int
her
apy
4.Apat
ienti
srecei
vi
ngext
ernal
beam r
adi
ati
onf
ort
het
reat
mentofmet
ast
ati
c
endometr
ialcancer
.Thetr
eat
mentfieldi
ncl
udestheenti
repel
vi
s.Whi
choft
hef
oll
owi
ng
ti
ssueswit
hinthisr
adi
ati
onfiel
disthemostradi
osensi
ti
ve?
Ov
ary
5.Choi
ceofadj
uvantt
reat
mentf
orendomet
ri
alcar
cinomast
ageI
A,gr
adeIi
s:
Not
reat
ment

1.Thecutsur
faceofov
aryhas4l
ayer
s.Whi
chl
ayeri
sthenextf
rom t
hev
erysuper
fi
cial
?
connect
ivet
issue
2.Whi
chhi
stol
ogi
cal
groupofov
ari
ant
umour
sist
hemostcommon(
≈70%ofal
l
t
umor
s)?
epi
thel
i
alt
umour
s
3.A16-
year
-ol
dgi
rlpr
esent
swi
th6×6cm r
ightov
ari
anmasswi
thabsentAFP,
negat
ive
CA125,
andi
ncr
easedal
kal
i
nephosphat
ase.Di
agnosi
sis:
Dy
sger
minoma
4.Al
lthef
oll
owi
ngar
etr
ueaboutKr
ukenber
g’st
umor
,except
:
Noneoft
heabov
e
5.Thepr
esent
ati
onwi
tht
hedi
ff
erentt
ypesofbeni
gnov
ari
ant
umour
svar
ieswi
thage.
Mar
kthei
ncor
rectst
atement
.
ger
m cel
ltumour
soccurmor
ecommonl
yinol
derwomen
6.A5-
year
-ol
dgi
rli
snot
edt
ohav
ebr
eastenl
argement
,vagi
nal
bleedi
ng,
andan8-
cm
pel
vi
cmass.Whi
choft
hef
oll
owi
ngi
sthemostl
i
kel
yet
iol
ogy
?
Gr
anul
osa-
thecacel
ltumor
7.Mei
gs'
syndr
omei
sdi
agnosedbasedonat
ri
adoft
hef
oll
owi
ngsi
gns,
except
:
oedema
8.A55- y
ear-
oldwomani snotedt
ohav eanabdominalmassandi
ncr
easedabdominal
gir
th.Onexami nat
ion,
ther
eisshi
fti
ngdull
nessandaf l
uidwav
e.Whi
chofthefol
l
owing
malignanci
esismostlikel
ytobefoundinthi
spati
ent?
Ov
ari
ancancer
9.Whi
chbi
ochemi
cal
tumourmar
keri
suseddur
ingt
reat
mentandf
oll
ow-
upofov
ari
an
dy
sger
minoma?
bet
a-hCG
10.Condi
ti
onsdi
agnosedasapel
vi
cmassi
nwomenofr
epr
oduct
iveagear
ethe
f
oll
owi
ngex
cept
:
Pr
olact
inoma
CONTENTS

Preface ix

SECTION 1
BENIGN GENERAL GYNECOLOGY

1 Well Woman Care 2 8 Abnormal Uterine Bleeding 51


2 Techniques Used for Imaging in 9 Pelvic Mass 58
Gynecology 7 10 Endometriosis 67
3 Gynecologic Infection 14 11 Pelvic Pain 74
4 Benign Disorders of the Lower 12 Breast Disease 81
Genital Tract 23
13 Psychosocial Issues and
5 Contraception and Sterilization 30 Female Sexuality 90
6 First-Trimester Abortion 37 14 Pediatric Gynecology 95
7 Ectopic Pregnancy 44

SECTION 2
REPRODUCTIVE ENDOCRINOLOGY,
INFERTILITY, AND THE MENOPAUSE

15 Reproductive Endocrinology 104 19 Evaluation of the Infertile Couple 127


16 Amenorrhea 110 20 Treatment of the Infertile Couple 133
17 Polycystic Ovarian Syndrome 21 Menopausal Transition 140
and Hyperandrogenism 116 22 The Mature Woman 146
18 Anatomic Disorders 121

vii
viii Contents

SECTION 3
FEMALE PELVIC MEDICINE AND
RECONSTRUCTIVE SURGERY

23 Urinary Incontinence 154 26 Genitourinary Fistula


and Urethral Diverticulum 173
24 Pelvic Organ Prolapse 161
25 Anal Incontinence and Functional
Anorectal Disorders 168

SECTION 4
GYNECOLOGIC ONCOLOGY

27 Principles of Chemotherapy 180 33 Endometrial Cancer 217


28 Principles of Radiation Therapy 187 34 Uterine Sarcoma 223
29 Preinvasive Lesions of the Lower 35 Epithelial Ovarian Cancer 230
Genital Tract 193 36 Ovarian Germ Cell and
30 Cervical Cancer 200 Sex Cord-Stromal Tumors 236
31 Vulvar Cancer 205 37 Gestational Trophoblastic Disease 242
32 Vaginal Cancer 211

SECTION 5
ASPECTS OF GYNECOLOGIC SURGERY

38 Anatomy 250 41 Minimally Invasive Surgery


Fundamentals 269
39 Preoperative Considerations 257
42 Postoperative Considerations 274
40 Intraoperative Considerations 263

SECTION 6
ATLAS OF GYNECOLOGIC SURGERY

43 Surgeries for Benign 45 Surgeries for Pelvic Floor Disorders 303


Gynecologic Disorders 282 46 Surgeries for Gynecologic Malignancies 315
44 Minimally Invasive Surgery 293

Index 325
30

CHAPTER 5

Contraception and Sterilization

5–1. Which o the ollowing poses the highest risk o death 5–5. Your patient delivered a healthy in ant 2 weeks ago
in ertile women aged 35 to 44 years? and wishes to initiate use o a contraceptive method
a. Pregnancy during the next ew weeks. She is breast eeding
exclusively. For which o the ollowing is there strong
b. Oral contraceptive use
evidence that use decreases the quantity and quality o
c. Intrauterine device (IUD) use breast milk?
d. Surgical tubal sterilization procedure a. Progestin-only pills
b. Depot medroxyprogesterone acetate
5–2. Which o the ollowing is a rst-tier contraceptive
method with an expected ailure rate o 2 pregnancies c. Combination hormonal contraceptives
per 100 women during the rst year? d. None o the above
a. Spermicide
5–6. A 16-year-old nulligravida is requesting a contracep-
b. Male condom
tive method. She plans to become sexually active with
c. Intrauterine device (IUD) her boy riend soon. Which o the ollowing is legally
d. Depot medroxyprogesterone acetate (DMPA) required in most states prior to prescribing hormonal
contraception or adolescents below the age o consent?
5–3. Which o the ollowing is a second-tier contracep- a. Parental consent
tive method with an expected ailure rate o 3 to
b. Pelvic examination
9 pregnancies per 100 users during the rst year?
c. Cervical cancer screening
a. Spermicide
d. None o the above
b. Withdrawal
c. Intrauterine device 5–7. Which o the ollowing statements is NOT a proges-
d. Oral contraceptive pills tin-mediated mechanism by which a levonorgestrel-
releasing intrauterine system (LNG-IUS) may prevent
5–4. A 35-year-old woman presents to your o ce as a new pregnancy?
amily planning patient or initiation o contracep- a. Endometrial atrophy
tion. Her medical history is notable or migraines
b. Decrease in tubal motility
with aura. She wishes to use combination oral con-
traceptive pills (COCs). You re er to the United States c. T ickening o cervical mucus
Medical Eligibility Criteria (US MEC) guidelines d. Lysosomal activation and in ammation
and see that this method is rated a “4.” Which o the
ollowing corresponds to this score and describes the
sa ety pro le or COC use in this particular patient?
a. Use is not restricted.
b. Method advantages outweigh risks.
c. Method risks outweigh advantages.
d. Method poses an unacceptable high health risk.
Contraception and Sterilization 31

5–8. Which o the ollowing conditions is NOT listed by 5–9. Your patient is a 26-year-old multipara who presents
the manu acturer as a contraindication o this contra- or a well-woman examination. She has no complaints.
ceptive method? She is satis ed with her current method o contracep-
tion (shown here). It was inserted 2 years ago, and she

C
H
wishes to continue with this contraceptive method.

A
Her pelvic examination is normal. Cervical cytology

P
T
(Pap test) is obtained and is negative or malignancy,

E
but lamentous organisms consistent with Actinomyces

R
5
species are identi ed on the smear. Which o the
ollowing is NOT a reasonable treatment option or
managing this incidental nding according to current
recommendations rom the American College o
Obstetricians and Gynecologists (2011)?

a. Acute liver disease


b. Heavy menses due to coagulation disorder
c. Increased susceptibility to pelvic in ection
d. Uterine anomaly with distortion o the uterine
cavity

a. Hysterectomy
b. Expectant management (no intervention)
c. Extended course o antibiotics with intrauterine
device (IUD) le t in place
d. Intrauterine device removal and initiation o an
alternative contraceptive method

5–10. For which o the ollowing is intrauterine device use


contraindicated due to an increased complication
rate?
a. Adolescence
b. Human immunode ciency in ection
c. Insertion immediately a ter spontaneous or in-
duced abortion
d. None o the above

5–11. What is the spontaneous expulsion rate or the intra-


uterine device during the rst year a ter placement?
a. 0.05 percent
b. 0.5 percent
c. 5 percent
d. 15 percent
32 Benign General Gynecology

5–12. What is the approximate risk o the complication 5–13. A 34-year-old multipara with a copper-containing
shown here per intrauterine device insertion? intrauterine device (IUD) in place presents or IUD
removal because she plans to become pregnant soon.
Her last menstrual period was 8 weeks ago. Her urine
S
E
pregnancy test is positive, and transvaginal sonog-
C
raphy con rms an 8-week intrauterine gestation. She
T
I
wishes to continue this pregnancy, i possible. She
O
N
eels well, is a ebrile, and has no cervical discharge
1
or pelvic tenderness. You see the IUD tail strings
protruding rom the external cervical os. Which o
the ollowing management strategies is recommended
to optimize the outcome or your patient?

Used with permission rom Dr Kimberly Kho.

a. 1 in 100
b. 1 in 1000
c. 1 in 10,000
d. 1 in 100,000

a. Remove the IUD and evacuate the uterine


contents
b. Remove the IUD and plan expectant pregnancy
management
c. Leave the IUD in place and plan expectant preg-
nancy management
d. Leave the IUD in place and administer broad-
spectrum antibiotics or the next 4 weeks

5–14. For which o the ollowing contraceptive methods is a


history o previous ectopic pregnancy considered, by
its manu acturer, to be a contraindication to its use?
a. Depot medroxyprogesterone acetate
b. Copper-containing intrauterine device
c. Progestin-containing subdermal implant
d. Levonorgestrel-releasing intrauterine system
Contraception and Sterilization 33

5–15. Which o the ollowing is thought to be a mechanism 5–18. Which o the ollowing statements is true regarding
o action by which the etonogestrel implant provides the Essure permanent birth control system, which is
contraception? shown in this sonogram.

C
a. Ovulation suppression

H
b. Endometrial atrophic changes

A
P
c. Increased cervical mucus viscosity

T
E
d. All o the above

R
5
5–16. Which o the ollowing methods o tubal sterilization
has the highest cumulative probability o ailure at
5 years?
a. Bipolar sterilization
b. Unipolar sterilization
c. Band or clip placement
d. Puerperal sterilization

5–17. Which o the ollowing statements is true regarding


the ef ects o emale tubal sterilization?
a. Procedure time averages more than 1 hour.
b. Placement must be per ormed in the operating
room.
c. I coils are placed bilaterally and occlusion is
documented, contraceptive success rates exceed
95 percent.
d. ransvaginal sonography has been approved by the
Food and Drug Administration or con rmation
o postprocedural tubal occlusion.

5–19. Which o the ollowing statements regarding vasec-


tomy is true?
a. Semen analysis is per ormed 3 months a ter vasec-
tomy to con rm azoospermia.
b. T e ailure rate is 30 times less than that o emale
Used with permission rom Dr Deana Hussamy. tubal sterilization.
c. T e postoperative complication rate is 20 times
a. Ovarian cancer risk is increased. less than that o emale tubal sterilization.
b. T e incidence o menorrhagia and dysmenorrhea d. All o the above
is increased.
c. Approximately 10 percent o pregnancies occurring 5–20. Which o the ollowing is NOT a physiologic ef ect
a ter the procedure are ectopic. exerted by the progestin component o combination
d. By 5 years postprocedure, 50 percent o women hormonal contraceptives?
aged 30 years or younger at the time o sterilization a. Lowered serum ree testosterone levels
express regret. b. Suppressed serum levels o luteinizing hormone
c. Elevated serum levels o ollicle-stimulating
hormone
d. All are physiologic ef ects.
34 Benign General Gynecology

5–21. Which o the ollowing is NOT an absolute contrain- 5–23. Rates o which o the ollowing are increased with use
dication to use o this contraceptive method? o extended cycle hormonal contraception compared
with that o traditional cyclic hormonal contracep-
tion?
S
E
a. Headaches
C
T
b. Escape ovulation
I
O
c. Endometrial cancer
N
1
d. Unpredictable bleeding

5–24. Use o which o the ollowing drugs most clearly


decreases combined hormonal contraceptive e cacy?
a. Ri ampin
b. Penicillin
c. Doxycycline
d. Cipro oxacin

5–25. Low-dose combination hormonal contraceptives most


clearly increase the risk o developing which o the
a. T rombotic disorders
ollowing?
b. Cholestatic jaundice
a. Obesity
c. Migraines with ocal neurologic de cits
b. Clinically signi cant hypertension
d. Uncomplicated systemic lupus erythematosus that
c. Overt diabetes in women with prior gestational
includes negative testing or antiphospholipid
diabetes
antibodies
d. None o the above
5–22. Which o the ollowing statements regarding the
method o contraception shown here compared with 5–26. With combination hormonal contraceptive use,
combination oral contraceptive pills (COCs) is true? stroke risk is elevated by which o the ollowing being
coexistent?
a. obacco use
b. Hypertension
c. Migraine headaches with aura
d. All o the above

5–27. Your patient is concerned a ter reading that the risk


o deep-vein thrombosis and pulmonary embolism
is tripled or quadrupled in current users o combina-
tion hormonal contraceptives (CHCs) compared
with that o the general population. You explain that
this is true but that the risk is still lower than the risk
o venous thrombosis related to pregnancy. What is
the approximate rate o thromboembolic events per
10,000 woman years with CHC use?
a. 4 events
a. otal estrogen exposure is higher with this b. 40 events
method. c. 240 events
b. T is method is less likely to cause breast d. 440 events
tenderness.
c. T is method controls dysmenorrhea in a greater
percentage o women.
d. T e pregnancy rate with this method is slightly
higher than with COCs.
Contraception and Sterilization 35

5–28. Your patient is an 18-year-old nulligravida who is 5–32. Which o the ollowing is an advantage o lambskin
not sexually active but takes combination oral contra- condoms compared with latex rubber condoms?
ceptive pills (COCs) to achieve good control o her a. Fewer allergic reactions
irregular menses and dysmenorrhea. She has grand mal

C
b. Lower breakage and slippage rates

H
epilepsy. Her seizures are well controlled on medica-

A
tion. Serum levels o which anticonvulsant medication c. Better protection against sexually transmitted

P
in ections

T
are decreased signi cantly by concurrent COC use?

E
d. All o the above

R
a. Phenytoin

5
b. Lamotrigine
5–33. Which o the ollowing statements about spermicides
c. Carbamazepine is NOT true?
d. Phenobarbital a. Spermicides alone provide protection against sexu-
ally transmitted diseases.
5–29. Contemporary low-dose combination oral contracep-
b. T e chemical agents, such as nonoxynol-9 or
tive pills (COCs) are most strongly implicated as a
octoxynol-9, are spermicidal.
risk actor or the development o which o the
ollowing neoplasms? c. Spermicide preparations are marketed in various
orms, such as creams, jellies, suppositories, lm,
a. Breast cancer
and oams.
b. Cervical cancer
d. Even when inserted regularly and correctly, sper-
c. Benign hepatic adenomas micide preparations have a ailure rate o up to
d. Lymphoma, i human immunode ciency virus 12 pregnancies per 100 woman years o use.
(HIV) in ection coexists
5–34. Which o the ollowing is NOT a mechanism o ac-
5–30. Your patient has diabetes mellitus and hyperten- tion or hormone-based emergency contraception?
sion but pre ers to use “pills” or contraception. She a. Disruption o an implanted zygote
declines an intrauterine device and barrier methods.
b. Endometrial changes that prevent implantation
She is considering a progestin-only contraceptive and
avors progestin-only pills. You counsel her regarding c. Inter erence with sperm transport or penetration
the advantages. You also explain which o the ollow- d. Inhibition or delay o ovulation and impaired
ing disadvantages o progestin-only pills compared corpus luteum unction
with combination oral contraceptive pills (COCs)?
a. Higher ailure rate 5–35. Which o the ollowing is an advantage o progestin-
only emergency contraception regimens compared
b. High rate o irregular bleeding
with estrogen–progestin combinations or this
c. Higher relative ectopic pregnancy rate i pregnancy purpose?
occurs
a. More ef ective in preventing pregnancy
d. All o the above
b. Ef ective i taken beyond 5 days a ter exposure
5–31. Which o the ollowing is generally increased by the c. Provides better protection against sexually trans-
use o depot medroxyprogesterone acetate compared mitted in ections
with other contraceptive methods? d. None o the above
a. Acne
b. Bone ractures Reference
c. Hepatic neoplasms American College o Obstetricians and Gynecologists: Long-
d. Interval to resumption o ovulation a ter method acting reversible contraception: Implants and intrauterine devices.
Practice Bulletin No. 121, July 2011, Rea rmed 2013.
cessation
36 Benign General Gynecology

Chapter 5 ANSWER KEY


Question Letter Page Question Letter Page
number answer cited Header cited number answer cited Header cited
S
E
C
5–1 a p. 105 Table 5-1 5–20 c p. 118 Pharmacology
T
I
O
5–2 c p. 105 Chapter 5: Contraception 5–21 d p. 119 Table 5-6
N
and Sterilization 5–22 a p. 122 Transdermal System
1
5–3 d p. 105 Chapter 5: Contraception 5–23 d p. 123 Extended Cycle
and Sterilization Contraception
5–4 d p. 105 Medical Eligibility Criteria 5–24 a p. 124 Table 5-9
5–5 d p. 108 Table 5-3 5–25 d p. 124 Obese and Overweight
5–6 d p. 107 Adolescence and Women, Diabetes
Perimenopause Mellitus, Cardiovascular
5–7 d p. 108 Levonorgestrel-Releasing Disease
Intrauterine System 5–26 d p. 125 Cerebrovascular Disorders
(LNG-IUS) 5–27 a p. 125 Venous
5–8 b p. 109 Table 5-4 Thromboembolism
5–9 a p. 109 Infection 5–28 b p. 125 Seizure Disorders
5–10 d p. 110 Low Parity and 5–29 b p. 126 Neoplastic Diseases
Adolescents 5–30 d p. 126 Progestin-Only Pills
5–11 c p. 110 Expulsion or Perforation 5–31 d p. 127 Notable Effects
5–12 b p. 110 Expulsion or Perforation 5–32 a p. 128 Male Condoms
5–13 b p. 111 Marker Strings 5–33 a p. 130 Spermicides and
5–14 d p. 111 Ectopic Pregnancy Microbicides
5–15 d p. 112 Progestin Implants 5–34 a p. 131 Hormone-Based
5–16 a p. 107 Figure 5-7 Emergency
Contraception
5–17 c p. 117 Other Effects
5–35 a p. 132 Progestin-Only
5–18 c p. 116 Counseling Regimens
5–19 d p. 118 Male Sterilization
51

CHAPTER 8

Abnormal Uterine Bleeding

8–1. Which o the ollowing de nitions o abnormal uter- 8–6. Control o blood loss during menses involves which
ine bleeding is incorrect? o the ollowing mechanisms?
a. Intermenstrual bleeding is bleeding that occurs a. T rombus ormation
between cycles. b. Platelet aggregation
b. Oligomenorrhea re ers to cycles with intervals c. Vasoconstriction o endometrial arteries
shorter than 35 days.
d. All o the above
c. Hypomenorrhea re ers to menses with diminished
ow or shortened interval. 8–7. A 30-year-old patient presents to your o ce complain-
d. Heavy menstrual bleeding is de ned as prolonged ing o heavy menstrual bleeding. With additional
or heavy cyclic menstruation, with menses lasting questioning, she reports new onset dysmenorrhea.
longer than 7 days or exceeding 80 mL o blood Given this additional symptom, which o the ollowing
loss. etiologies or her abnormal bleeding is LEAST likely?
a. In ection
8–2. Which o the ollowing tools is NOT clinically use ul
b. Anovulation
to estimate menstrual blood loss?
c. Pregnancy complication
a. Pad counts
d. Structural abnormality o the uterus
b. Hemoglobin and hematocrit
c. Pictorial blood assessment chart 8–8. All o the ollowing diagnostic tests are typically
d. Sodium hydroxide extraction o hemoglobin obtained during the initial evaluation o abnormal
uterine bleeding EXCEPT:
8–3. In premenarchal girls, which o the ollowing is the a. Pap test
most common source o abnormal bleeding?
b. Pregnancy test
a. Ovary
c. Liver unction tests
b. Uterus
d. Complete blood count with platelets
c. Vagina
d. Urethra 8–9. Which o the laboratory criteria listed below is consis-
tent with iron-de ciency anemia?
8–4. Age groups have been correctly paired with their most a. Low serum erritin level
common etiology o abnormal genital tract bleeding
b. Normal hemoglobin and hematocrit
in all o the ollowing EXCEPT:
c. Low total iron-binding capacity ( IBC)
a. Adolescence—anovulation
d. Increase in mean corpuscular hemoglobin (MCV)
b. Childhood—vulvovaginitis
c. Perimenopause—anovulation 8–10. According to the American College o Obstetricians
d. Menopause—endometrial carcinoma and Gynecologists (2013), endometrial sampling
to assess abnormal uterine bleeding is NOT recom-
8–5. Which layer o the endometrium sloughs and there- mended or a woman with which o the ollowing
ore is responsible or observed menstrual discharge? characteristics?
a. Spiral a. Is 35 years old
b. Radial b. Has ailed medical management
c. Basalis c. Has persistent abnormal uterine bleeding
d. Functionalis d. Has history o unopposed estrogen exposure
52 Benign General Gynecology

8–11. Which o the ollowing is a limitation o Pipelle sam- 8–14. In the evaluation o abnormal uterine bleeding, what
plers used or endometrial biopsy to evaluate abnor- is the primary advantage o hysteroscopy compared
mal uterine bleeding? with saline in usion sonography?
a. Rate o inadequate sampling that exceeds 50 percent a. Hysteroscopy is less pain ul.
S
E
C
b. Inability to be per ormed in an o ce setting b. Hysteroscopy is less expensive.
T
I
c. Greater patient discom ort compared with a stif c. Hysteroscopy is more accurate in identi ying
O
metal curette global endometrial pathology.
N
1
d. Low sensitivity and high alse-negative rate or d. Hysteroscopy permits simultaneous identi cation
ocal endometrial pathology and removal o ocal endometrial lesions.

8–12. An advantage o transvaginal sonography or the 8–15. A 60-year-old postmenopausal woman presents or
evaluation o abnormal uterine bleeding includes evaluation o genital tract bleeding, which is con-
which o the ollowing? rmed as uterine in origin by physical examination.
a. Reduced use o endometrial biopsy Which diagnostic procedure is a logical rst step in
b. Simultaneous assessment o myometrium and her evaluation?
endometrium a. Colposcopy
c. Greater patient com ort compared with endome- b. Diagnostic hysteroscopy
trial biopsy or hysteroscopy c. ransvaginal sonography
d. All o the above d. Saline in usion sonography
8–13. What is the primary advantage o saline in usion 8–16. Which cause o abnormal uterine bleeding is NOT
sonography compared with transvaginal sonography? represented in the International Federation o Gyne-
cology and Obstetrics (FIGO) classi cation acronym
PALM-COEIN?
a. Leiomyoma
b. Pregnancy
c. Iatrogenic
d. Coagulopathy

a. Less patient discom ort


b. Superior detection o intracavitary masses
c. Ability to per orm at any time o the cycle
d. Simultaneous assessment o myometrium and
endometrium
Abnormal Uterine Bleeding 53

8–17. Pelvic sonography with applied color Doppler reveals 8–19. A 27-year-old woman who previously underwent
an intrauterine growth with a single eeder vessel. dilatation and curettage or an incomplete abortion
Which statement is true regarding this abnormality? presents with new-onset heavy menstrual bleeding.
ransvaginal sonography rst reveals a hypoechoic

C
H
tubular structure within the myometrium. With

A
application o color Doppler, large-caliber vessels are

P
T
seen in the second image. What is the most appropri-

E
ate next step in her evaluation?

R
8
a. T e main diagnostic tool is the Pap test.
b. It is an uncommon cause o abnormal uterine
bleeding.
c. In ertility has been linked directly to this
condition.
d. Use o oral contraceptive pills appears to be
protective.

8–18. T e lesion in Question 8–17 is seen hysteroscopically.


Risk actors or malignant trans ormation o this le-
sion include which o the ollowing?

a. Angiography
b. Hysteroscopy
c. Saline in usion sonography
d. Pelvic computed tomography (C ) with contrast
Used with permission rom Dr David Rogers.

a. amoxi en use
b. Postmenopausal status
c. Size greater than 1.5 cm
d. All o the above
54 Benign General Gynecology

8–20. Suggested etiologies or abnormal uterine bleeding 8–23. When should irregular spotting or bleeding be evalu-
associated with use o the device shown below in this ated in a postmenopausal patient using hormone
three-dimensional (3-D) sonogram include which o replacement therapy (HR )?
the ollowing? a. History o endometrial polyps
S
E
C
b. Continued bleeding a ter 6 months o HR use
T
I
c. Abnormal bleeding that develops a ter initial
O
amenorrhea
N
1
d. All o the above

8–24. Which o the ollowing statements regarding tamoxi-


en is true?
a. It acts as an estrogen agonist in the breast and
uterus.
b. amoxi en use has been linked to endometrial pol-
yps, hyperplasia, and carcinoma but not to uterine
sarcomas.
c. Women using tamoxi en should undergo evalu-
ation or endometrial cancer only i abnormal
bleeding develops.
a. Malpositioned device d. It is a selective estrogen-receptor modulator
(SERM) used as an adjunct or treatment o
b. Unbalanced ratio o prostaglandin to thromboxane
estrogen-receptor-negative breast cancer.
levels
c. Increased endometrial vascularity, congestion, and 8–25. A 60-year-old patient with breast cancer taking oral
degeneration anticoagulants or recent deep-vein thrombosis is ad-
d. All o the above mitted or acute heavy menstrual bleeding. Which o
the ollowing management options is LEAST suitable
8–21. A patient or whom you initiated combination oral or this patient?
contraceptive pills (COCs) 3 months ago returns to the a. Reversal o anticoagulation
o ce or a routine COC re-evaluation. She complains o
b. High-dose estrogen (intravenous Premarin)
light, irregular bleeding during this time but states that it
is diminishing. What is your diagnosis and plan o care? c. Intrauterine cavity insertion o a Foley catheter
a. COC intolerance requiring method d. All are suitable or this woman.
discontinuation
8–26. Systemic causes o abnormal uterine bleeding include
b. Breakthrough bleeding requiring counseling and
which o the ollowing?
reassurance
a. Liver disease
c. Endometrial pathology necessitating transvaginal
sonography b. Severe renal dys unction
d. Hormonal imbalance requiring selection o a c. Hypo- and hyperthyroidism
dif erent COC ormulation d. All o the above

8–22. A patient using the progestin-only implant (Nexpl- 8–27. A patient with heavy menstrual bleeding reveals a
anon) complains o unscheduled light bleeding since personal history o requent gingival bleeding and
her implant was placed 4 months ago. Which o the excessive bleeding during a recent tooth extraction.
ollowing treatment options would be LEAST likely You screen her or a coagulation disorder. Which o
to correct the bleeding? the ollowing laboratory tests is LEAST likely to be
a. Single dose o depot medroxyprogesterone acetate in ormative?
(DMPA) a. Bleeding time
b. Addition o combination oral contraceptives or b. Prothrombin time (P )
1 month c. Partial thromboplastin time (P )
c. Use o a nonsteroidal anti-in ammatory medica- d. Complete blood count (CBC) with platelets
tion or 1 week
d. Use o a daily estrogen-only supplement such as
ethinyl estradiol or conjugated equine estrogen or
1 month
Abnormal Uterine Bleeding 55

8–28. Which is an ef ective rst-line treatment or women 8–32. For the treatment o dys unctional uterine bleeding,
with heavy menstrual bleeding and von Willebrand evidence supports which o the ollowing statements
disease? regarding the e cacy o the device shown in this
three-dimensional (3-D) sonogram?

C
a. Endometrial ablation

H
b. Dilatation and curettage

A
P
c. Nonsteroidal anti-in ammatory drugs

T
E
d. Combination oral contraceptive pills

R
8
8–29. Which o the ollowing statements regarding abnor-
mal uterine bleeding-ovulatory (AUB-O) is true?
a. Since ovulation does not occur, a secretory endo-
metrium persists.
b. At the cellular level, the availability o arachi-
donic acid is increased, altering prostaglandin
production.
c. Women with anovulation may be amenorrheic
or long periods o time, which are ollowed by
irregular, prolonged, heavy bleeding episodes.
d. At the tissue level, the endometrial vascular
structure is altered and contains increased spiral a. It reduces menstrual loss by more than 75 percent
arteriole density and constricted venous capillaries. a ter 3 months o use.
b. Compared with endometrial ablation, this method
8–30. A patient presents to the Emergency Department
has similar therapeutic ef ects up to 2 years a ter
with a 1-day history o heavy vaginal bleeding. She
treatment.
is tachycardic but not hypotensive, and postural vital
signs are stable with change rom a supine to sitting c. It is more ef ective than nonsteroidal anti-in am-
position. Physical examination demonstrates bleed- matory drugs or oral progestins in decreasing
ing rom above the external cervical os and continued uterine blood loss.
pooling o blood in the vagina. Laboratory studies d. All o the above
reveal she is anemic. Which o the ollowing is the
most appropriate rst-line agent to attempt control o 8–33. Which o the ollowing statements regarding
her acute uterine bleeding? tranexamic acid is true?
a. Intravenous estrogen a. o be most ef ective, it requires administration or
b. Oral tranexamic acid 2 weeks be ore and during menses.
c. Gonadotropin-releasing hormone (GnRH) agonist b. Contraindications to its use include a history or
intrinsic risk o thromboembolic disease.
d. Combination oral contraceptive pill taper
c. It af ects blood coagulation parameters, such as
8–31. Which o the ollowing medications used to treat platelet count, prothrombin time (P ), and partial
abnormal uterine bleeding-endometrial (AUB-E) is thromboplastin time (P ).
correctly paired with its mechanism o action? d. It is an anti brinolytic drug that permanently
a. ranexamic acid—increases plasmin levels blocks lysine binding sites on plasminogen, thereby
increasing plasmin levels and brinolytic activity.
b. Nonsteroidal antiin ammatory drugs (NSAIDs)—
stimulate cyclooxygenase (COX-1 and 2)
production
c. Oral progestins—inhibit endometrial growth
and promote organized sloughing ollowing their
withdrawal
d. Combination oral contraceptive pills (COCs)—
induce endometrial atrophy but promote increased
prostaglandin synthesis and endometrial brinolysis
56 Benign General Gynecology

8–34. When counseling a patient or endometrial ablation, 8–35. Which o the ollowing is NOT a contraindication to
which o the ollowing points are discussed? endometrial ablation?
a. T ree ourths o women experience signi cantly a. Postmenopausal status
decreased menstrual volume a ter ablation.
S
b. Prior classical cesarean delivery
E
C
b. Approximately 25 percent o women will need a c. Anatomically normal endometrial cavity
T
hysterectomy by 5 years subsequent to ablation.
I
d. Desire to preserve ertility potential
O
c. Following ablation, evaluation o the endometrium
N
1
or recurrent abnormal bleeding can be di cult due
to resultant distortion o the endometrial cavity. Reference
d. All o the above American College o Obstetricians and Gynecologists: Management
o acute abnormal uterine bleeding in nonpregnant reproductive-
aged women. Committee Opinion No. 557, April 2013.
95

CHAPTER 14

Pediatric Gynecology

14–1. Which o the ollowing statements is FALSE regard- 14–5. Delayed puberty is characterized by a lack o initial
ing the development o the hypothalamic–pituitary- pubertal changes, usually thelarche, by what threshold
ovarian axis in the emale etus and neonate? age?
a. By 5 months’ gestation, 6 to 7 million oocytes a. 13 years
have been created rom accelerated germ cell b. 14 years
division.
c. 15 years
b. T e gonadotropin-releasing hormone (GnRH)
d. 16 years
“pulse generator” remains unctionally dormant
until several months a ter birth.
14–6. Compared with several decades ago, the age at which
c. At birth, ollicle-stimulating hormone (FSH) and U.S. girls currently experience thelarche and men-
luteinizing hormone (LH) concentrations rise and arche has shown which o the ollowing trends?
remain high during the rst 3 months o li e.
a. In general, later than in past
d. Neonatal breast budding, minor uterine bleeding,
b. In general, earlier than in past
and transient ovarian cysts may occur as a normal
response to initially high gonadotropin levels. c. For girls with a higher body mass index, later than
in the past
14–2. Which o the ollowing statements is true regarding d. No clear trend observed
pelvic anatomy in the emale in ant and child?
a. T e ovaries have obtained their normal adult size 14–7. All states in the United States legally allow a minor
by birth. woman to consent to medical examination and treat-
ment on her own behal in all o the ollowing situa-
b. At birth, the uterus and cervix are approximately
tions EXCEPT:
equal in size.
a. Pregnancy
c. Presence o an endometrial stripe or uid within
the endometrial cavity o the newborn uterus is a b. Emancipated minor status
normal nding with sonography. c. Living permanently apart rom parents
d. All the above are true statements. d. Requesting a school-sponsored sports’ physical
examination
14–3. Which o the ollowing generally occurs rst among
the major developmental events o emale puberty? 14–8. Which o the ollowing is an indication or an
a. Menarche internal vaginal examination o a child?
b. Pubarche a. Possible tumor
c. T elarche b. Vaginal bleeding
d. Growth spurt c. Suspected oreign body
d. All o the above
14–4. An alarmed parent brings her 8-year-old daughter in
or evaluation o breast budding. T e child shows no
other signs o puberty. Precocious puberty is de ned
as initial pubertal changes occurring prior to what
threshold age?
a. 6 years
b. 7 years
c. 8 years
d. 9 years
96 Benign General Gynecology

14–9. A 4-year-old girl is brought in by her parent with 14–10. A 14-year-old woman presents with increasingly
the concern that “the lips o her vagina have stuck pain ul, cyclic menses since menarche, 9 months ago.
together.” Initial management o labial adhesion or Menstrual ow is normal in amount and duration.
agglutination in a child, as shown here, does NOT Oral analgesics and nonsteroidal antiin ammatory
S
E
include which o the ollowing options? medications no longer adequately control her severe
C
dysmenorrhea. Which o the ollowing congenital
T
I
anomalies is most likely present?
O
N
1
Reproduced with permission rom Ho man BL, Schorge JO, Scha er JI, et al: Williams
Gynecology, 2nd ed. New York, McGraw-Hill, 2012. Photo contributor: Dr. Mary Jane
Pearson.
Used with permission rom Dr. Ellen Wilson.

a. Surgery
a. Imper orate hymen
b. Emollients
b. Müllerian agenesis
c. opical estrogen cream
c. Complete transverse vaginal septum
d. Await spontaneous resolution
d. Obstructed hemivagina with ipsilateral renal agen-
esis (OHVIRA) syndrome
Pediatric Gynecology 97

14–11. A 14-year-old woman presents with increasing 14–12. A 7-year-old girl has experienced vulvar irritation
abdominal pain. She has begun appropriate pubertal with external itching and burning during urination
development, although menarche has not occurred. or several months. Symptoms have persisted despite

C
During examination, a central abdominopelvic mass attempts to eliminate potential irritants and contact

H
is appreciated and is shown here. Physical examina- allergens rom the bath and laundry products used

A
P
tion ndings prompt magnetic resonance imaging by the amily. Examination, as shown here, reveals

T
with contrast, and the results are shown below. Which symmetrical hypopigmentation and a parchment-like

E
R
o the ollowing congenital anomalies is most likely thinning o the vulvar and perianal skin. What is the

1
present? most likely diagnosis?

4
Used with permission rom Dr. Mary Jane Pearson.

a. Eczema
b. Candidiasis
c. Sexual abuse
d. Lichen sclerosus

14–13. Which o the ollowing organisms is most likely to


cause prepubertal vulvitis?
a. Candida albicans
b. Group B streptococcus
c. Staphylococcus aureus
d. Group A β -hemolytic streptococcus

14–14. With prepubertal vulvovaginitis, what approximate


Used with permission rom Dr. Ellen Wilson.
percentage o bacterial cultures obtained rom vaginal
swabs show only normal genital ora?
a. Imper orate hymen
a. 25 percent
b. Müllerian agenesis
b. 50 percent
c. Unicornuate uterus with cavitary communicating
rudimentary horn c. 75 percent
d. Obstructed hemivagina with ipsilateral renal agen- d. 90 percent
esis (OHVIRA) syndrome
98 Benign General Gynecology

14–15. Which o the ollowing physiologic or anatomic char- 14–19. A unicornuate uterus and its noncavitary rudimentary
acteristics predispose to prepubertal vulvovaginitis? horn are shown here. What is the pre erred imaging
a. Poor vulvar hygiene modality when a congenital müllerian anomaly is
suspected?
S
b. Lack o labial at pads and labial hair
E
C
c. Nonestrogenized vulvovaginal epithelium
T
I
d. All o the above
O
N
1
14–16. An 8-year-old girl presents with a mucopurulent and
bloody vaginal discharge. Her parents report that,
while traveling recently, the girl was brie y hospital-
ized or treatment o severe diarrhea. Examinations o
the vagina in the o ce and under anesthesia reveal no
tumor or oreign body. What is the most likely organ-
ism involved?
a. Shigella spp.
b. Salmonella spp.
c. Candidia glabrata
d. Enterobius vermicularis
B = bladder.
14–17. What approximate percentage o malignant tumors
diagnosed in childhood are o ovarian origin? a. Computed tomography
a. 0.1 percent b. Hysterosalpinography
b. 1 percent c. ransabdominal sonography
c. 10 percent d. Magnetic resonance imaging
d. 30 percent
14–20. Accessory nipples, when present, are located along the
14–18. A etal ovarian mass is noted as an incidental nding embryonic milk line extending rom the axilla to the
during third-trimester sonographic assessment o etal groin bilaterally. T eir presence is termed which o
growth. T e emale in ant is delivered at term without the ollowing?
incident. Lower abdominal ullness is palpated during a. Polythelia
neonatal examination, and transabdominal sonog-
raphy shows a 4.2-cm simple right ovarian cyst. T e b. Multimomium
in ant is a ebrile and in no distress. What is the most c. Polymammoma
appropriate management o this ovarian mass? d. Areola duplicata
a. No urther surveillance
b. Percutaneous cyst aspiration 14–21. Which o the ollowing is true regarding isolated
premature thelarche?
c. Surgery i unresolved within 2 months
a. Bone age is advanced.
d. Observation with repeat sonography in 6 weeks
b. Gonadotropin levels are elevated.
c. It is most common in girls younger than 2 years.
d. None o the above
Pediatric Gynecology 99

14–22. A 14-year-old adolescent is brought in or examina- 14–26. A breast mass noted in an adolescent emale prompts
tion due to her le t breast being noticeably larger than you to order breast sonography. What is the most
her right when she is undressed. T e breasts are other- likely outcome o the lesion ound and shown here?

C
wise normal during examination. T elarche occurred

H
at the age o 11.5 years. She cannot think o any

A
P
speci c trauma to the right breast or chest wall. She

T
has never had any surgery. Cosmetically, the dif er-

E
R
ence is not obvious when she is clothed. Which o the

1
ollowing is the most common etiology o asymmetric

4
breast growth in a emale adolescent?
a. Idiopathic
b. Surgical trauma
c. Physical trauma
d. Strong right or le t handedness

14–23. In an adolescent with an otherwise normal breast and


chest wall examination, what is the best approach to
breast asymmetry?
a. Order breast sonography or diagnostic
Reproduced with permission rom Ho man BL, Schorge JO, Bradshaw KD, et al:
mammography Williams Gynecology, 3rd ed. New York, McGraw-Hill, 2016. Photo contributor: Dr. Stephen
J. Seiler.
b. Initiate combination low-dose oral contraceptive
pills
a. Spontaneous regression over time
c. Re er now or plastic surgery be ore asymmetry
worsens b. Persistence with little change in size
d. Reassure that most cases o breast asymmetry c. With hormonal stimulation, gradual increase in size
resolve by completion o breast development d. Development o more generalized brocystic changes

14–24. Abnormal breast development may be due to either 14–27. A smooth, rm, mobile, nontender right-breast mass is
ascial adherence to the underlying muscle layer palpable during examination o a 12-year-old girl. It is
or due to high-dose exogenous hormone exposure 1 cm in its greatest dimension and appears solid sono-
during puberty. In such cases, breasts have excessive graphically, as shown here. She has had no major health
orward but limited lateral growth, which causes an problems to date. What is the most likely diagnosis?
abnormal shape. T is condition is re erred to as which
o the ollowing?
a. Perithelia
b. uberous breasts
c. Aberrant breast hypertrophy
d. Asymmetric lacti erous hyperplasia

14–25. Lack o breast development is associated with low


estrogen levels and may be caused by which o the
ollowing?
a. Chronic disease
b. Gonadal dysgenesis
c. High levels o athletic activity
d. All o the above
Used with permission rom Dr. Stephen J. Seiler.

a. Malignancy
b. Fibroadenoma
c. Ductal ectasia
d. Asymmetric breast budding
100 Benign General Gynecology

14–28. Mastitis is unusual in children and adolescents. What 14–34. Pubarche, with development o axillary and pubic
is the most common pathogen isolated rom breast hair, is stimulated by the androgens derived primarily
abscesses in the pediatric population? rom which o the ollowing?
a. Escherichia coli a. Adrenal glands
S
E
C
b. Staphylococcus aureus b. Ovarian stroma
T
I
c. Streptococcus pyogenes c. Peripheral aromatization o estrogens
O
N
d. Staphylococcus epidermidis d. Physiology continues to be poorly understood.
1
14–29. An 8-year-old girl is brought in or evaluation o 14–35. Delayed puberty in women is de ned as the lack o
4 days o intermittent vaginal bleeding and oul dis- secondary sexual characteristics by age 13 or lack o
charge. She shows no signs o pubertal development. menarche by what age?
T e cause o the bleeding is not apparent during a. 14
physical examination, and the decision is made to
b. 15
proceed with examination under anesthesia and saline
vaginoscopy. What is the most likely nding? c. 16
a. Foreign object d. 18
b. Atrophic vaginitis
14–36. When an individual experiences di culty identi ying
c. Endocervical polyp with their phenotypic gender, perceiving themselves
d. Genital tract malignancy to be o the opposite gender, the condition is termed
which o the ollowing?
14–30. Which o the ollowing is the most common cause o a. Homosexuality
central precocious puberty?
b. Gender dysphoria
a. Idiopathic
c. Sexual identity con ict
b. Head trauma
d. Gender o rearing mismatch
c. Hydrocephalus secondary to surgery
d. Congenital central nervous system anomaly 14–37. Sexually active adolescents are particularly vulnerable
to partner violence. In a study by Abma and associates
14–31. Primary therapy or central (gonadotropin- (2010), what percentage o adolescent women who
dependent) precocious puberty consists o experienced coitarche prior to age 20 described their
which o the ollowing? rst sexual intercourse as nonvoluntary?
a. Bromocriptine a. 0.1 percent
b. Depot-medroxyprogesterone acetate b. 1 percent
c. Combination oral contraceptive pills c. 7 percent
d. Gonadotropin-releasing hormone (GnRH) agonist d. 17 percent

14–32. A primary goal o therapy or central precocious 14–38. Which o the ollowing contraceptive methods is
puberty includes prevention o which o the ollowing generally contraindicated in adolescents and women
consequences? younger than 21 years?
a. Short stature a. Vaginal ring
b. Excessive breast size b. Intrauterine devices
c. Learning disabilities c. Extended use o oral contraceptive pills
d. None o the above d. None o the above

14–33. A 5-year-old girl shows clinical signs o precocious 14–39. Which o the ollowing is required be ore initiating
puberty. Gonadotropin levels are low, even ollowing a contraceptive method or a healthy, sexually active
gonadotropin-releasing hormone (GnRH) stimula- adolescent?
tion testing. However, estrogen levels are elevated. a. Pelvic examination
Which o the ollowing could cause o her disorder?
b. Cervical cancer screening
a. Primary hypothyroidism
c. Human papillomavirus (HPV) vaccination
b. Ovarian granulosa cell tumor
d. None o the above
c. Congenital adrenal hyperplasia
d. All o the above
Pediatric Gynecology 101

14–40. Vaccines targeting human papillomavirus in ection Reference


are approved through what age (in years) or women
according to the U.S. Food and Drug Administra- Abma JC, Martinez GM, Copen CE: eenagers in the United
States: sexual activity, contraceptive use, and childbearing, National

C
tion? Survey o Family Growth 2006–2008. National Center or Health

H
Statistics. Vital Health Stat 23:30, 2010.

A
a. 12

P
b. 15

T
E
R
c. 21

1
d. 26

4
14–41. In the United States, adolescents can obtain medical
care without parental knowledge or consent or which
o the ollowing “medically emancipated” conditions?
a. Pregnancy
b. Contraception
c. Substance abuse
d. All o the above
102 Benign General Gynecology

Chapter 14 ANSWER KEY


Question Letter Page Question Letter Page
number answer cited Header cited number answer cited Header cited
S
E
C
14–1 b p. 318 Hypothalamic–Pituitary– 14–23 d p. 326 Breast Shape
T
I
Ovarian (HPO) Axis
O
14–24 b p. 326 Breast Shape
N
14–2 c p. 318 Anatomy 14–25 d p. 326 Absent Breast Development
1
14–3 c p. 319 Pubertal Changes 14–26 a p. 326 Breast Mass or Infection
14–4 c p. 319 Pubertal Changes 14–27 b p. 326 Breast Mass or Infection
14–5 a p. 319 Pubertal Changes 14–28 b p. 326 Breast Mass or Infection
14–6 b p. 319 Pubertal Changes 14–29 a p. 327 Table 14-2
14–7 d p. 320 Gynecologic Examination 14–30 a p. 328 Central Precocious
14–8 d p. 320 Gynecologic Examination Puberty (Gonadotropin
14–9 a p. 321 Labial Adhesion Dependent), Table 14-3
14–10 d p. 323 Congenital Anatomic 14–31 d p. 327 Central Precocious Puberty
Anomalies (Gonadotropin Dependent)
14–11 a p. 323 Congenital Anatomic 14–32 a p. 327 Central Precocious Puberty
Anomalies (Gonadotropin Dependent)
14–12 d p. 323 Lichen Sclerosus 14–33 d p. 328 Peripheral Precocious
Puberty (Gonadotropin
14–13 d p. 324 Infection Independent), Table 14-3
14–14 c p. 324 Vulvovaginitis 14–34 a p. 329 Variations of Normal Puberty
14–15 d p. 324 Table 14-1 14–35 c p. 329 Delayed Puberty
14–16 a p. 324 Vulvovaginitis 14–36 b p. 329 Gender Identity
14–17 b p. 325 Ovarian Tumors 14–37 c p. 330 Adolescent Sexuality
14–18 d p. 325 Ovarian Tumors 14–38 d p. 330 Contraception
14–19 d p. 325 Ovarian Tumors 14–39 d p. 330 Contraception
14–20 a p. 325 Polythelia 14–40 d p. 330 Contraception
14–21 c p. 326 Premature Thelarche 14–41 d p. 330 Contraception
14–22 a p. 326 Breast Shape
104

CHAPTER 15

Reproductive Endocrinology

15–1. Which o the ollowing most accurately describes 15–4. A 48-year-old woman presents to the Emergency
autocrine hormone communication? Department with severe upper abdominal pain and
a. A cell secretes a hormone that acts on a neighbor- a positive pregnancy test. She has had a prior tubal
ing cell. ligation. A computed tomography scan per ormed to
evaluate her pain is signi cant or multiple hetero-
b. A cell secretes a hormone that in uences its own
geneous hepatic lesions (shown below). Her serum
unction.
β -human chorionic gonadotropin (β -hCG) test result
c. T e hormone is secreted and acts upon a distant is 115 mIU/mL. ransvaginal sonography results are
target tissue. also shown below. Chest radiographs are normal.
d. T e hormone produced acts within the cell be ore What is the most likely source o her hCG?
it is secreted.

15–2. T e gure below portrays which type o hormone action?

Reproduced with permission rom Ho man BL, Schorge JO, Bradshaw KD, et al:
Williams Gynecology, 3rd ed. New York, McGraw-Hill, 2016. Figure 15-1B.

a. Autocrine
b. Endocrine
c. Paracrine
d. Intracrine

15–3. Which o the ollowing statements is true regarding


the gonadotropin luteinizing hormone (LH)?
a. It is secreted by the posterior pituitary gland.
b. Its unctional speci city is derived rom the
α -subunit.
c. It shares a common glycoprotein β -subunit with
ollicle-stimulating hormone (FSH).
d. Its β -subunit demonstrates approximately
80 percent homology with that o human chorionic
gonadotropin (hCG).
a. Liver tumors
b. Exogenous hCG injection
c. Partial molar pregnancy
d. Complete molar pregnancy
Reproductive Endocrinology 105

15–5. Which o the ollowing statements regarding steroido- 15–11. All o the ollowing will increase levels o sex hormone-
genesis is FALSE? binding globulin EXCEPT:
a. Steroid production primarily occurs in the Golgi a. Androgens

C
apparatus.

H
b. Estrogens

A
b. T e primary building block o sex steroid c. Pregnancy

P
hormones is cholesterol.

T
d. Hyperthyroidism

E
R
c. T e enzymes involved in steroid production are

1
members o the cytochrome P450 super amily. 15–12. Where in the cell are unliganded estrogen receptors

5
d. T e placenta is the only steroid-producing tissue located?
that cannot synthesize cholesterol rom its precur- a. Nucleus
sor, acetate.
b. Cytoplasm
15–6. T e last step in estrogen synthesis requires which o c. Mitochondria
the ollowing enzymes? d. Endoplasmic reticulum
a. Aromatase
15–13. Gonadotropin-releasing hormone agonists, such as
b. 21-hydroxylase
leuprolide acetate, reduce gonadotropin secretion by
c. 5α -reductase what mechanisms?
d. 11β -hydroxylase a. Receptor destruction
b. Receptor hydroxylation
15–7. All o the ollowing tissues express signi cant levels o
aromatase EXCEPT: c. Receptor downregulation
a. Skin d. Receptor phosphorylation
b. Brain
15–14. A 17-year-old adolescent presents with heavy vaginal
c. Ovary bleeding or 3 days. Her previous menstrual period
d. Muscle was 3 months ago, and she is sexually active without
contraception. She denies vision changes, galactorrhea,
15–8. What is the predominant estrogen during meno- or headaches. Physical examination is notable or a
pause? so t, 14-week size uterus. Her urine β-human chorionic
a. Estriol gonadotropin (β -hCG) level assay is negative. rans-
vaginal sonography is obtained and a sagittal view o
b. Estrone
the uterus reveals the ndings below. What is the next
c. Estradiol clinical step?
d. None o the above

15–9. Which o the ollowing androgens is NOT produced


by the ovary?
a. estosterone
b. Androstenedione
c. Dihydrotestosterone (DH )
d. Dehydroepiandrosterone (DHEA)

15–10. Approximately what percentage o androgens and


estrogens are ound unbound, or ree, in the
circulation?
a. 1 percent
b. 5 percent
c. 10 percent
d. 20 percent a. Endometrial biopsy
b. Serum-based β -hCG assay
c. Serum-based prolactin level
d. Head magnetic resonance imaging
106 Reproductive Endocrinology, Infertility, and the Menopause

15–15. For the patient in Question 15–13, what is the 15–21. Prolactin release is primarily regulated by which
mechanism by which signi cantly elevated hormone neurotransmitter?
levels can saturate assay antibodies and create a alse- a. Dopamine
negative reading?
S
b. Epinephrine
E
a. Hook ef ect
C
c. Norepinephrine
T
b. Werner ef ect
i
d. T yrotropin-releasing hormone
o
c. Layered ef ect
n
2
d. Maryland ef ect 15–22. Cells o the anterior pituitary primarily express which
o the ollowing dopamine receptors?
15–16. What ef ect do endogenous opioids have on the a. D1
release o gonadotropin-releasing hormone (GnRH)?
b. D2
a. No ef ect
c. D3
b. Variable ef ect
d. D4
c. Increase release o GnRH
d. Suppress release o GnRH 15–23. Which o the ollowing statements is true regarding
the corticotropin-releasing hormone–adrenocortico-
15–17. Endorphin levels in the brain peak during what phase tropic hormone (CRH-AC H) pathway?
o the menstrual cycle? a. Abnormalities in the pathway result in electrolyte
a. Menses disturbances.
b. Ovulation b. CRH secretion is under negative- eedback regula-
c. Luteal phase tion by circulating cortisol.
d. Follicular phase c. CRH binds to its receptors in the posterior pitu-
itary to stimulate AC H secretion.
15–18. Secretion o which anterior pituitary hormone is d. AC H stimulates glucocorticoid production by
NOT stimulated by hypothalamic neuroendocrine the adrenal zona reticularis.
secretion?
a. Prolactin 15–24. All o the ollowing stimulate release o growth hor-
mone EXCEPT:
b. Growth hormone
a. Sleep
c. Luteinizing hormone
b. Stress
d. T yroid-stimulating hormone
c. Adiposity
15–19. Sustained gonadotropin secretion requires which d. Hypoglycemia
o the ollowing patterns o gonadotropin-releasing
hormone (GnRH) secretion? 15–25. Which o the ollowing is NOT believed to be a unc-
a. Nocturnal release o GnRH tion o oxytocin?
b. Pulsatile release o GnRH a. Promotes vaginal distension during coitus
c. Continuous low levels o GnRH secretion b. Stimulates uterine contractions during labor.
d. Continuous high levels o GnRH secretion c. Promotes expression o milk during lactation.
d. Prompts uterine and tubal contractions to aid
15–20. Which o the ollowing statements regarding ertilization.
gonadotropin-releasing hormone (GnRH) pulsatility
in animal models is FALSE?
a. Progesterone decreases GnRH pulsatility.
b. Higher pulse requency avors luteinizing hormone
(LH) secretion.
c. Lower pulse requency avors ollicle-stimulating
hormone (FSH) secretion.
d. Estrogen increases GnRH pulse requency such
that FSH levels increase relative to LH levels.
Reproductive Endocrinology 107

15–26. What stage o olliculogenesis is shown here? 15–30. A 36-year-old woman is using a home ovulation-
predictor kit to help achieve pregnancy. She is given
speci c instructions by her physician to engage in sexual

C
intercourse a ter detecting the luteinizing hormone (LH)

H
surge. T is advice is based upon ovulation typically

A
P
occurring how many hours a ter the onset o this surge?

T
E
a. 12 to 16 hours

R
b. 24 to 30 hours

1
5
c. 36 to 40 hours
d. 48 to 52 hours

15–31. T e patient in Question 15–30 returns to her physician,


unsure i she detected a luteinizing hormone (LH)
surge with her home ovulation predictor kit. o
con rm ovulation, a progesterone level is drawn in
the midluteal phase (day 21 o the menstrual cycle).
At what minimum progesterone level can it be
Used with permission rom Dr. Kelley Carrick.
assumed that ovulation has occurred?
a. Corpus luteum a. 0.3 ng/mL
b. Antral ollicle b. 3 ng/mL
c. Corpus albicans c. 30 ng/mL
d. Primary ollicle d. 300 ng/mL

15–27. When is meiosis II completed during oocyte matura- 15–32. Steroidogenesis in the corpus luteum is mainly under
tion? the control o which hormone?
a. A ter ertilization a. Activin
b. A ter the luteinizing hormone (LH) surge b. Follistatin
c. Immediately preceding the LH surge c. Luteinizing hormone (LH)
d. During ollicular development in the preovulatory d. Follicle-stimulating hormone (FSH)
phase
15–33. A 28-year-old woman presents to the emergency
15–28. All o the ollowing statements regarding estrone are room reporting acute onset o pelvic pain. Her urine
true EXCEPT: pregnancy test result is positive; she reports her last
menses was 8 weeks ago; and a rst-trimester intra-
a. It is the principal serum estrogen in postmeno-
uterine pregnancy is seen sonographically. T e patient
pausal women.
is taken to the operating room to evaluate or torsion
b. T e major site or conversion o androstenedione a ter imaging demonstrates a large ovarian cyst. Ovar-
to estrone is adipose tissue. ian torsion is con rmed, and the ovary containing
c. Circulating levels o estrone are usually adequate a corpus luteum cyst must be removed. Until what
to protect against bone loss. gestational age must this patient receive progesterone
d. For a given body weight, conversion o andro- replacement?
stenedione to estrone is higher in postmenopausal a. 10 weeks
women than in premenopausal women. b. 14 weeks
c. 20 weeks
15–29. What hormone is responsible or the recruitment o
the ollicles destined or ovulation? d. No replacement is necessary a ter 8 weeks’ gestation.
a. Estrogen
15–34. What days o the menstrual cycle constitute the win-
b. Inhibin A dow o implantation or a human embryo?
c. Inhibin B a. Days 14 to 16
d. Follicle-stimulating hormone (FSH) b. Days 16 to 20
c. Days 20 to 24
d. Days 24 to 28
108 Reproductive Endocrinology, Infertility, and the Menopause

15–35. A 33-year-old woman has a 5-mm prolactin- 15–36. A 26-year-old woman presents with severe headaches.
producing pituitary adenoma, which is well con- Visual eld de ects are noted during evaluation, and
trolled on bromocriptine. She presents soon a ter a imaging con rms a pituitary adenoma. What is the
missed menstrual period with a positive pregnancy best next step in management or this patient?
S
E
test result. What is the best next step in management a. Radiation therapy
C
or this patient?
T
b. Neurosurgical evaluation
i
o
a. Decreasing bromocriptine dose
c. Somatostatin-agonist therapy
n
b. Increasing bromocriptine dose
2
d. Initiation o gonadotropin-releasing hormone
c. Discontinuing bromocriptine therapy (GnRH) agonist therapy
d. Visual eld testing in every trimester
15–37. Which o the ollowing progestins is NOT derived
rom 19-nortestosterone?
a. Drospirenone
b. Norgestimate
c. Norethindrone
d. Levonorgestrel
Reproductive Endocrinology 109

Chapter 15 ANSWER KEY


Question Letter Page Question Letter Page

C
number answer cited Header cited number answer cited Header cited

H
A
15–1 b p. 334 Reproductive Endocrinology 15–20 d p. 344 Pulsatile Gonadotropin-

P
T
15–2 c p. 334 Reproductive Endocrinology releasing Hormone

E
R
Secretion
15–3 d p. 334 Peptide Hormones: LH, FSH,

1
15–21 a p. 345 Dopamine and Prolactin

5
and hCG
15–4 a p. 335 Human Chorionic Gonadotropin 15–22 b p. 345 Dopamine and Prolactin
15–5 a p. 336 Steroidogenesis 15–23 b p. 345 Corticotropin-releasing
Hormone
15–6 a p. 337 Figure 15-5
15–24 c p. 345 Growth Hormone-releasing
15–7 d p. 336 Steroidogenesis Hormone
15–8 b p. 336 Steroidogenesis 15–25 a p. 346 Posterior Pituitary Peptides
15–9 c p. 336 Steroidogenesis 15–26 b p. 348 Figure 15-13
15–10 a p. 338 Steroid Hormone Transport 15–27 a p. 348 Oocyte Maturation
in the Circulation
15–28 c p. 350 Steroidogenesis Across the
15–11 a p. 338 Steroid Hormone Transport Life Span
in the Circulation
15–29 d p. 353 Follicular Phase
15–12 a p. 339 Estrogen, Progesterone, and
Androgen Receptors 15–30 c p. 353 Ovulation
15–13 c p. 340 Receptor Expression and 15–31 b p. 354 Luteal Phase
Desensitization 15–32 c p. 354 Luteal Phase
15–14 b p. 341 Immunoassays 15–33 a p. 354 Luteal Phase
15–15 a p. 341 Immunoassays 15–34 c p. 357 Implantation Window
15–16 d p. 343 Endogenous Opiates 15–35 c p. 360 Pregnancy and Pituitary
15–17 c p. 343 Endogenous Opiates Adenomas
15–18 a p. 343 Anterior Pituitary Hormones 15–36 b p. 360 Treatment of
Hyperprolactinemia and
15–19 b p. 344 Pulsatile Gonadotropin- Pituitary Adenomas
releasing Hormone
Secretion 15–37 a p. 363 Progestogens
121

CHAPTER 18

Anatomic Disorders

18–1. Congenital anatomic disorders o the emale repro- 18–6. Which o the ollowing statements is true regarding
ductive tract may result rom which o the ollowing the role o the sex-determining region Y (SRY) in the
mechanisms? sexual di erentiation o humans?
a. Genetic mutation a. T e SRY acts as the testis-determining actor.
b. Developmental arrest b. Without the in uence o SRY, gonads develop as
c. Abnormal hormonal exposure or exposure to envi- testes.
ronmental insults c. T e SRY is located on the Y chromosome and is
d. All o the above the only gene responsible or sex determination.
d. None o the above
18–2. Both urinary and genital systems develop rom which
o the ollowing cell types? 18–7. Antimüllerian hormone is involved in all o the
a. Ectoderm ollowing EXCEPT:
b. Endoderm a. Regression o the ipsilateral paramesonephric
system
c. Mesoderm
b. Peaking o testosterone production as a result o
d. None o the above
stimulation o the testes
18–3. Which o the ollowing organs is correctly paired with c. Rapid gubernacular growth necessary or the trans-
its embryologic origin? abdominal descent o testes
a. Uterus—mesonephric duct d. Assessment o ovarian reserve in women undergo-
ing assisted reproduction
b. Bladder—mesonephric duct
c. Kidney—paramesonephric duct 18–8. During emale sex development, when do the germ
d. estes and ovaries—genital ridge cells carrying two X chromosomes (oocytes) reach
their peak number o 5 to 7 million?
18–4. Up to what percentage o women with uterovaginal a. At birth
mal ormations have associated urinary tract anoma-
b. Age 5 to 7 years
lies?
c. Shortly be ore menarche
a. 5 percent
d. In utero at 20 weeks’ gestation
b. 25 percent
c. 50 percent 18–9. During a rst-trimester ultrasound or pregnancy
d. 75 percent dating, you discover that your patient has a subsep-
tate uterus. While counseling her, you explain that
18–5. Embryos o male or emale gender are indistinguish- this müllerian anomaly developed in utero secondary
able rom each other until which number o weeks o to ailure o the midline uterine septum to reabsorb
development? completely. Normally, reabsorption o the uterine
a. 7 septum in a etus should occur by how many weeks o
gestation?
b. 10
a. 20
c. 12
b. 28
d. 14
c. 32
d. 36
122 Reproductive Endocrinology, Infertility, and the Menopause

18–10. T e vagina orms in part rom both the müllerian 18–16. Which o the ollowing is NOT true o Kline elter
ducts and which other structure? syndrome?
a. Genital ridge a. A ected men are tall and excessively virilized.
S
b. Urogenital sinus b. A ected men have signi cantly reduced ertility.
E
C
c. Mesonephric duct c. T e syndrome occurs in 1 to 2 percent o all men.
T
I
d. Paramesonephric duct d. It con ers an increased risk or germ cell tumors,
O
N
osteoporosis, and breast cancer.
2
18–11. T e hymen is the partition that remains between
which structures? 18–17. rue statements about pure 46,XY gonadal dysgenesis
a. Cloacal membrane and genital tubercle include which o the ollowing?
b. Sinovaginal bulb and urogenital sinus a. It was ormerly named Swyer syndrome.
c. Un used cephalad portions o the two müllerian b. It results rom a mutation in sex-determining
ducts region Y (SRY) or in another gene with testis-
determining e ects.
d. None o the above
c. T e condition creates a normal prepubertal emale
18–12. In the male etus, dihydrotestosterone (DH ) is phenotype and normal müllerian system due to
involved in which o the ollowing? the absence o antimüllerian hormone (AMH).
a. Enlargement o the phallus d. All o the above
b. Lengthening o the anogenital distance
18–18. All o the ollowing statements regarding complete
c. Fusion o the labioscrotal olds to orm the androgen insensitivity syndrome (CAIS) are true
scrotum EXCEPT:
d. All o the above a. On outward examination, patients appear as phe-
notypically normal women.
18–13. Which o the ollowing anatomic structures is cor-
b. T ese women develop breasts during puberty due
rectly paired with its embryologic origin?
to abundant androgen-to-estrogen conversion.
a. Genital tubercle—clitoris
c. Surgical excision o the testes is recommended
b. Urethral olds—labia minora be ore puberty to decrease the associated risk o
c. Labioscrotal olds—labia majora germ cell tumors.
d. All o the above d. Estrogen replacement a ter removal o the testes is
important to maintain bone mass and to provide
18–14. Which o the ollowing disorders o sex development relie rom vasomotor symptoms.
is correctly paired with its category o abnormality?
a. Hypospadias—ambiguous genitalia 18–19. Which o the ollowing can cause virilization o a
46,XX in ant?
b. Streak gonad—gonadal dysgenesis
a. Placental aromatase de ciency
c. rue hermaphroditism–ovotesticular disorder
b. Fetal congenital adrenal hyperplasia due to de -
d. All o the above
ciency o 21-hydroxylase
18–15. rue statements regarding urner syndrome include c. Maternal virilizing ovarian tumors, such as Sertoli–
which o the ollowing? Leydig tumor, or maternal use o androgenic
medication
a. T ere are no classic physical stigmata.
d. All o the above
b. It is the least common orm o gonadal dysgenesis
that leads to primary ovarian ailure.
18–20. When aced with ambiguous external genitalia o a
c. Although the uterus and vagina are normal, streak newborn at delivery, the obstetrician should do which
gonads are present, and patients typically present o the ollowing?
with primary amenorrhea.
a. Examine the mother or signs o
d. All o the above hyperandrogenism.
b. Re er to the newborn as “your baby” and not as “it.”
c. Re rain rom gender assignment by explaining that
the genitalia are incompletely ormed.
d. All o the above
Anatomic Disorders 123

18–21. Which o the ollowing statements regarding bladder 18–24. Appropriate techniques or the surgical correction o
exstrophy is NOT true? the condition diagnosed in Question 18–23 involve
a. T is anomaly displays a predilection or women o all o the ollowing EXCEPT:

C
2:1. a. Hymenectomy

H
A
b. It is characterized by an exposed bladder lying b. Laparoscopy to exclude endometriosis

P
outside the abdomen.

T
c. Repair in in ancy or a ter thelarche

E
R
c. Surgical closure is per ormed early in li e and as a d. Needle aspiration o the hematocolpos

1
staged procedure.

8
d. It occurs rom ailure o the cloacal membrane to 18–25. Compared with other müllerian duct de ects, a trans-
be rein orced by an ingrowth o mesoderm. verse vaginal septum is associated with a lower rate o
which o the ollowing?
18–22. Causes o newborn clitoromegaly include which o a. Endometriosis
the ollowing?
b. Urologic abnormalities
a. Prematurity
c. Need or surgical correction
b. Neuro bromatosis
d. None o the above
c. Fetal exposure to excessive androgens
d. All o the above 18–26. A 16-year-old nulligravida presents to the emergency
department with complaints o abdominal and vagi-
18–23. A 14-year-old nulligravida presents to the emergency nal pain, worsening during the past several months.
department with complaints o worsening lower She describes the pain as being mostly on her right
abdominal pain over the past ew days. She states side and much worse during menstruation. During
that she has had a similar pain in the past, usually your physical examination, a patent vagina and cervix
or a ew days each month, but then it subsides. She are noted, but a lateral vaginal and pelvic mass is pal-
is a ebrile with stable vital signs. Although she has pated. ransvaginal sonography demonstrates a single
breasts and axillary and pubic hair, she has never had uterus and cervix but also a large pelvic mass lled
a period. Examination reveals a tender midline lower with complex uid, as shown below, and the absence
abdominal mass and a bluish bulging vaginal mass, as o the right kidney. What is the most likely diagnosis?
shown here. Based on her history and your physical
examination, which o the ollowing conditions is
your most likely diagnosis?

a. OHVIRA syndrome
b. Ovarian neoplasm
c. Imper orate hymen
d. ransverse vaginal septum
Reproduced with permission rom Ho man BL, Schorge JO, Bradshaw KD, et al (eds):
Williams Gynecology, 2nd ed. New York, McGraw-Hill, 2014. Photo contributor:
Dr. Ellen Wilson.

a. Bartholin cyst
b. Imper orate hymen
c. Gartner duct cyst
d. Longitudinal vaginal septum
124 Reproductive Endocrinology, Infertility, and the Menopause

18–27. Why is care ul preoperative planning warranted with 18–30. Appropriate techniques or surgical correction o
congenital vaginal cysts, as shown in sonogram below? the condition diagnosed in Question 18–29 include
which o the ollowing?
a. Hymenectomy
S
E
C
b. Hysterectomy
T
I
c. Uterine transplantation
O
N
d. Surgical creation o a neovagina
2
18–31. A 22-year-old G3P0 woman presents as a new patient
or consultation regarding her history o multiple
miscarriages. Her gynecologic history is otherwise
unremarkable. During physical examination, you
note that the uterus is markedly deviated to the le t.
ransvaginal sonography is per ormed, and three-
dimensional images reveal a uterus that is deviated
and has a banana-shaped cavity, as shown below.
What is your diagnosis?
a. T e recurrence rate a ter excision is high.
b. T ey are requently large, measuring greater than
8 cm in size.
c. T eir typical location is in the posterior-lateral wall
o the vagina.
d. Some may extend up to the broad ligament and
anatomically approximate the distal course o the
ureter.

18–28. Müllerian anomalies are associated with anomalies o


all o the ollowing systems EXCEPT:
a. Renal
b. Hepatic
c. Skeletal
d. Neurological

18–29. One o your patients brings her 13-year-old daughter a. Septate uterus
or consultation regarding the girl’s complaints o
b. Bicornuate uterus
pelvic pain. T e daughter has experienced normal
pubertal development to date but has not yet expe- c. Uterine didelphys
rienced menarche. She began having monthly pelvic d. Unicornuate uterus
pain about 1 year ago, and the pain has been progres-
sively worsening. During physical examination, her 18–32. T e pathogenesis o poor pregnancy outcomes with a
external genitalia, including hymeneal ring, appear unicornuate uterus is thought to be related to which
normal. However, proximal to the hymen, only a o the ollowing actors?
vaginal dimple is ound. Rectovaginal examination a. Cervical incompetence
con rms the presence o a uterus and cervix. What is
b. Reduced uterine capacity
your diagnosis?
c. Anomalous distribution o the uterine artery
a. Vaginal atresia
d. All o the above
b. Cervical atresia
c. Imper orate hymen
d. Müllerian agenesis
Anatomic Disorders 125

18–33. Which müllerian anomaly results rom complete 18–35. Septate uterus, as shown in the sonogram below,
ailure o the paired müllerian ducts to use? T is has a signi cantly higher spontaneous abortion rate
anomaly is shown in this sonogram, and a gestational and early pregnancy loss rate than bicornuate uterus.

C
sac is seen on the image’s le t side. What is the primary mechanism thought to be

H
responsible or this extraordinarily high pregnancy

A
P
wastage?

T
E
R
1
8
a. Associated cervical anomalies
a. Septate uterus b. Distortion o the uterine cavity
b. Bicornuate uterus c. Distortion o the uterine serosal contour
c. Uterine didelphys d. Partial or complete implantation on the largely
d. Unicornuate uterus avascular septum

18–34. When should surgical reconstruction o a bicornuate 18–36. Which o the ollowing are seen in o spring o
uterus (Strassman metroplasty) be per ormed? women who took diethylstilbestrol (DES) during the
associated pregnancy?
a. Prior to conception
a. Impaired conception rates in emale o spring
b. In all women with the diagnosis o bicornuate
uterus b. Cryptorchidism, testicular hypoplasia, and mal or-
mations o the penis in male o spring
c. Procedure no longer per ormed or repair o bicor-
nuate uterus c. -shaped uterus, “cockscomb” cervix, and
increased rates o clear cell adenocarcinoma in
d. With recurrent pregnancy loss and no other identi-
emale o spring
able cause except the uterine anomaly
d. All o the above

18–37. While staf ng labor and delivery, you are asked to


evaluate a pelvic mass discovered during the per or-
mance o a postpartum tubal ligation. T e mass is a
thin-walled, translucent cystic structure measuring
less than 1 cm in diameter. It is attached by a thin
pedicle to the distal end o the right allopian tube.
T e resident physician is concerned that the mass may
be neoplastic, but you assure her that it is not. What
is the likely diagnosis?
a. Hydrosalpinx
b. Accessory ovary
c. Supernumerary ovary
d. Paratubal cyst (hydatid o Morgagni)
126 Reproductive Endocrinology, Infertility, and the Menopause

Chapter 18 ANSWER KEY


Question Letter Page Question Letter Page
number answer cited Header cited number answer cited Header cited
S
E
C
18–1 d p. 404 Anatomic Disorders 18–19 d p. 413 Androgen Excess
T
I
O
18–2 c p. 404 Normal Embryology 18–20 d p. 413 Gender Assignment
N
18–3 d p. 404 Normal Embryology 18–21 a p. 414 Defects of the Bladder and
2
18–4 c p. 404 Normal Embryology Perineum
18–5 a p. 406 Gonadal Determination 18–22 d p. 414 Defects of the Clitoris
18–6 a p. 406 Gonadal Determination 18–23 b p. 415 Hymenal Defects
18–7 b p. 406 Gonadal Determination 18–24 d p. 415 Hymenal Defects
18–8 d p. 406 Gonadal Determination 18–25 b p. 416 Transverse Vaginal Septum
18–9 a p. 408 Ductal System Development 18–26 a p. 416 Longitudinal Vaginal Septum
18–10 b p. 408 Ductal System 18–27 d p. 417 Congenital Vaginal Cysts
Development 18–28 b p. 417 Müllerian Anomalies
18–11 b p. 408 Ductal System 18–29 a p. 419 Vaginal Atresia
Development 18–30 d p. 420 Müllerian Agenesis
18–12 d p. 409 External Genitalia 18–31 d p. 420 Unicornuate Uterus
18–13 d p. 409 External Genitalia 18–32 d p. 420 Unicornuate Uterus
18–14 d p. 409 Disorders of Sexual 18–33 c p. 420 Uterine Didelphys
Development, Definitions
18–34 d p. 421 Bicornuate Uterus
18–15 c p. 412 Gonadal Dysgenesis
18–35 d p. 422 Septate Uterus
18–16 a p. 411 Turner and Klinefelter
Syndromes 18–36 d p. 423 Diethylstilbesterol-Induced
Reproductive Tract
18–17 d p. 412 46, XYGonadal Dysgenesis Abnormalities
18–18 c p. 412 Abnormal Androgen 18–37 d p. 423 Fallopian Tube Anomalies
Production or Action
127

CHAPTER 19

Evaluation of the Infertile Couple

19–1. What is the expected per cycle ecundability rate? 19–6. O in ertility causes, which o the ollowing is most
a. 0.5 percent likely encountered?
b. 5 percent a. Unexplained
c. 25 percent b. Male actors
d. 85 percent c. ubal disease
d. Ovulatory dys unction
19–2. Regarding couples attempting pregnancy, what percent-
age o women are expected to have conceived at 1 year? 19–7. A 30-year-old nulligravida has been trying to conceive
a. 15 percent or the last 2 years. She has no medical problems.
She consumes ve alcoholic drinks weekly, smokes
b. 30 percent
hal a pack o cigarettes daily. She describes hersel as
c. 60 percent a heavy cof ee drinker. She works in a dry cleaning
d. 85 percent acility. Which o the ollowing exposures is LEAST
likely af ecting her ertility?
19–3. A 27-year-old married nulligravid resident physician a. Alcohol
has been married or 3 years. She is worried about
b. Caf eine
delaying childbearing or another ew years while she
pursues ellowship training. She can be reassured that c. Cigarettes
a signi cant decline in the ecundability rate does not d. Dry cleaning uid exposure
begin until what age (years)?
a. 26 19–8. Preconceptional carrier screening or cystic brosis is
currently recommended or which group?
b. 32
a. Ashkenazi Jewish descent
c. 38
b. Non-Hispanic white race
d. 44
c. All individuals considering pregnancy
19–4. In general, in ertility evaluation is initiated a ter a d. Family or personal history o cystic brosis
couple ails to conceive a ter 1 year o unprotected
intercourse o reasonable requency. When should 19–9. How long does spermatogenesis take, starting rom
earlier evaluation be considered? stem cell to mature sperm?
a. Woman older than 35 years a. 10 days
b. History o chronic anovulatory cycles b. 30 days
c. Prior severe pelvic in ammatory disease (PID) c. 60 days
d. All o the above d. 90 days

19–5. An in ertile couple presents or evaluation. Neither 19–10. Which o the ollowing is NOT known to impair
partner has ever conceived a pregnancy. Both believe sperm production or quality?
that the emale partner alone is the source o their a. ight underwear
ailure to achieve a pregnancy. It should be explained
b. Chronic hot tub use
that in ertility can be attributed to the male partner in
approximately one third o cases and to both partners c. Chemotherapy or local radiation
in approximately what percentage o cases? d. Illness accompanied by high ever
a. 1 percent
b. 10 percent
c. 33 percent
d. 66 percent
128 Reproductive Endocrinology, Infertility, and the Menopause

19–11. A woman complaining o in ertility reports that her 19–17. Which o the ollowing is NOT true o basal body
husband has already undergone evaluation by a urolo- temperature testing to identi y ovulation in adult
gist to exclude male actors. T is leads to the surgical women?
repair o a unilateral varicocele. With regard to varico- a. It can be an insensitive test in many women.
S
E
celes and in ertility, which o the ollowing is true?
C
b. With ovulation, the temperature rises rom 0.4°F
T
a. All should be repaired. to 0.8°F.
I
O
b. T ey are an established cause o male in ertility. c. It is an inexpensive and easy test or ovulation
N
2
c. Repair o subclinical varicoceles leads to correction monitoring.
o semen abnormalities. d. Once the temperature rises, a patient should
d. A varicocele is present in nearly 20 percent o the expect ovulation during the next 48 hours.
adult male general population.
19–18. Regarding commercially available urinary luteinizing
19–12. Use o which o these is most likely to damage testicu- hormone kits, when does ovulation take place in rela-
lar unction permanently? tion to a positive result?
a. Gentamycin a. Same day
b. Spironolactone b. Day a ter
c. Anabolic steroids c. Day be ore
d. Alcohol consumption d. 48 hours a ter

19–13. I not already up to date as a part o routine well- 19–19. Which o the ollowing midluteal progesterone level
woman care, which o the ollowing should be com- values signi es ovulation?
pleted during the initial in ertility evaluation? a. 0.05 ng/mL
a. Determine rubella immunity status b. 0.1 ng/mL
b. Screen or cervical cancer c. 1.0 ng/mL
c. Screen or Neisseria gonorrhoeae and Chlamydia d. 5.0 ng/mL
trachomatis
d. All o the above 19–20. Which o the ollowing is true o luteal phase endo-
metrial biopsy?
19–14. Gynecomastia in a male patient may suggest the pres- a. It shows high intra- and interobserver reliability.
ence o which o the ollowing conditions?
b. It does not reliably distinguish ertile women rom
a. Noonan syndrome in ertile women.
b. Kline elter syndrome c. An out-o -phase biopsy result predicts an increased
c. Pituitary prolactinoma risk o uture early pregnancy loss should concep-
d. 17β -hydroxysteroid dehydrogenase de ciency tion occur.
d. Its per ormance on day 21 o a menstrual cycle is
19–15. Congenital bilateral absence o the vas de erens is considered a standard component o an in ertility
genetically related to which o the ollowing? evaluation.
a. Cystic brosis
b. Noonan syndrome
c. Kline elter syndrome
d. Red–green color blindness

19–16. Which o the ollowing suggests ovulatory cycles?


a. Dysmenorrhea
b. Mittelschmerz
c. Moliminal symptoms
d. All o the above
Evaluation of the Infertile Couple 129

19–21. As shown here, sonography is clinically most use ul 19–25. Which o the ollowing statements regarding anti-
or the diagnosis o which o the ollowing conditions müllerian hormone level (AMH) is true?
that contributes to a signi cant percentage o in ertil-

C
ity cases?

H
A
P
T
E
R
1
9
Reproduced with permission Ho man BL, Schorge JO, Bradshaw KD, et al (eds): Williams
Gynecology, 3rd ed. New York, McGraw-Hill, 2016. Photo contributor: Dr. Kelley Carrick.

a. Anovulation a. AMH levels are increased in women with polycys-


b. Sex cord-stromal tumors tic ovarian syndrome.
c. Polycystic ovarian syndrome b. Compared with estradiol or ollicle-stimulating
hormone (FSH) levels, AMH levels correlate better
d. Periovarian adhesive disease
with the number o ovarian primordial ollicles
(seen in photomicrograph above).
19–22. In reproductive-aged women, which o the ollowing
increases with the passage o time? c. AMH expression is strong in preantral ollicles and
weak in larger ollicles.
a. Fertility
d. All o the above statements are true.
b. Ovarian reserve
c. Risk o spontaneous abortion 19–26. A 19-year-old woman is admitted with her second
d. Success o donor insemination bout o pelvic in ammatory disease (PID) in 18 months.
She is counseled that she is at increased risk o in er-
19–23. Reduced secretion o which o the ollowing hor- tility and ectopic pregnancy in the uture. Despite
mones is most likely responsible or the rising serum this ef ort, she is likely to continue her risk-taking
ollicle-stimulating hormone (FSH) level seen as a behaviors. What is the estimated tubal in ertility rate
woman ages? a ter three episodes o PID?
a. Activin a. 0.4 percent
b. Inhibin B b. 14 percent
c. Estradiol c. 34 percent
d. Follistatin d. 54 percent

19–24. For the evaluation o ovarian unction, which o the


ollowing laboratory tests is LEAST help ul?
a. Random inhibin B level
b. Cycle day 3 estradiol level
c. Random antimüllerian hormone level (AMH)
d. Cycle day 3 ollicle-stimulating hormone (FSH)
level
130 Reproductive Endocrinology, Infertility, and the Menopause

19–27. A 28-year-old woman presents with primary in er- 19–28. A 32-year-old woman undergoes in ertility evalua-
tility. She has no current health problems or his- tion a ter trying to conceive or 5 years. She has cyclic
tory o serious illness, including pelvic in ammatory but heavy menses. Prior to this, she experienced one
disease (PID). She has predictable, cyclic menses pregnancy that ended in a rst-trimester spontaneous
S
E
with mild dysmenorrhea, unchanged since men- abortion. What uterine abnormality is suggested by
C
arche. Her healthy male partner has been evaluated her hysterosalpingogram?
T
I
and has a normal physical examination and semen
O
N
analysis. What is your interpretation o the patient’s
2
hysterosalpingogram as shown below?

Reproduced with permission rom Ho man BL, Schorge JO, Bradshaw KD, et al (eds):
Williams Gynecology, 3rd ed. New York, McGraw-Hill, 2016. Photo contributor:
Dr. Diane wickler.

a. Normal
b. Asherman syndrome
c. Submucous leiomyoma
d. Bilateral hydrosalpinges

19–29. Hysterosalpingography is LEAST help ul or detect-


Reproduced with permission rom Ho man BL, Schorge JO, Bradshaw KD, et al (eds): ing which o the ollowing?
Williams Gynecology, 3rd ed. New York, McGraw-Hill, 2016. Photo contributor: Dr. Kevin
Doody. a. ubal patency
b. Pelvic adhesions
a. Normal c. Asherman syndrome
b. Uterine didelphys d. Congenital uterine anomalies
c. Bilateral hydrosalpinges
d. Unilateral proximal tubal blockage
Evaluation of the Infertile Couple 131

19–30. T e photomicrograph o an air-dried sample o cervi- 19–34. Asthenospermia re ers to which o the ollowing
cal mucus demonstrates the ef ect o high amounts o semen abnormalities?
which o the ollowing hormones? a. Sperm count

C
H
b. Semen volume

A
c. Sperm motility

P
T
d. Sperm morphology

E
R
1
19–35. Which o the ollowing tests can dif erentiate between

9
dead and alive nonmotile sperm?
a. Zona penetration assay
b. DNA ragmentation index
c. Hypoosmotic swelling test
d. Mannose uorescence assay

19–36. Which o the ollowing tests are currently recom-


mended components o a basic male actor in ertility
evaluation?
a. DNA integrity analysis
Reproduced with permission rom Halvorson LM: Evaluation o the in ertile couple. In
Ho man BL, Schorge JO, Bradshaw KD, et al (eds): Williams Gynecology, 3rd ed. New York, b. Sperm penetration assay
McGraw-Hill, 2016, Figure 19-10.
c. Antisperm antibody assay
a. Inhibin d. None o the above
b. Estrogen
19–37. A couple seeking pregnancy has undergone a thor-
c. Progesterone ough in ertility evaluation. Both partners are gener-
d. Follicle-stimulating hormone (FSH) ally healthy. T e emale partner is without apparent
problems that would contribute to in ertility. Semen
19–31. Which o the ollowing is true o the process o sperm analysis is per ormed. T e sperm count is 4 million/
capitation? mL, and semen volume is less than 1 mL. In addition
a. Involves sperm hyperactivation. to a serum testosterone level, which o the ollowing
should be measured during the hormonal evaluation
b. Aids sperm penetration o the ovum’s zona
o the male partner i an endocrinopathy is suspected?
pellucida.
a. Prolactin
c. Results in the sperm’s ability to release acrosomal
enzymes important to ertilization o the ovum. b. T yroid-stimulating hormone
d. All o the above c. Follicle-stimulating hormone (FSH)
d. All o the above
19–32. Lique action o the semen specimen a ter ejaculation
is due to secretions rom which o the ollowing? 19–38. Genetic testing should be per ormed in response to
a. Prostate which o the ollowing semen analysis results?
b. Epididymis a. Semen volume o 2.0 mL
c. Cowper gland b. Sperm concentration o 2 million/mL
d. Semini erous tubules c. Presence o ructose in the semen sample
d. 8-percent normal sperm morphology by Kruger
19–33. When treating hypertension with β -blocking agents, criteria
which o the ollowing semen abnormalities may be
seen due to retrograde ejaculation? 19–39. Which o the ollowing Y-chromosome deletions car-
a. Azoospermia ries the best prognosis or recovering sperm rom the
testes in an azoospermic patient?
b. Oligospermia
a. AZFa
c. eratospermia
b. AZFb
d. Low semen volume
c. AZFc
d. AZFd
132 Reproductive Endocrinology, Infertility, and the Menopause

Chapter 19 ANSWER KEY


Question Letter Page Question Letter Page
number answer cited Header cited number answer cited Header cited
S
E
C
19–1 c p. 427 Evaluation of the Infertile Couple 19–21 c p. 435 Sonography
T
I
O
19–2 d p. 427 Evaluation of the Infertile Couple 19–22 c p. 435 Reproductive Aging
N
19–3 b p. 427 Evaluation of the Infertile Couple 19–23 b p. 436 Follicle-Stimulating Hormone
2
19–4 d p. 427 Evaluation of the Infertile Couple and Estradiol
19–5 c p. 428 Etiology of Infertility 19–24 a p. 436 Antimüllerian Hormone
19–6 d p. 428 Table 19-1 19–25 d p. 436 Antimüllerian Hormone
19–7 d p. 428 Social, Table 19-2 19–26 d p. 437 Tubal and Pelvic Factors
19–8 c p. 430 Ethnicity and Family History 19–27 c p. 437 Tubal and Pelvic Factors
19–9 d p. 430 Male History 19–28 c p. 438 Hysterosalpingography
19–10 a p. 430 Male History 19–29 b p. 438 Hysterosalpingography
19–11 d p. 430 Male History 19–30 b p. 441 Cervical Factors, Figure 19-10
19–12 c p. 430 Male History 19–31 d p. 442 Normal Spermatogenesis
19–13 d p. 433 Examination of the Female 19–32 a p. 442 Semen Analysis
Patient 19–33 d p. 442 Semen Analysis Results
19–14 b p. 433 Examination of the Male Patient 19–34 c p. 442 Semen Analysis Results
19–15 a P. 433 Examination of the Male 19–35 c p. 442 Semen Analysis Results
Patient 19–36 d p. 444 Additional Sperm Testing
19–16 d p. 434 Clinical Evaluation 19–37 d p. 444 Hormonal Evaluation of the
19–17 d p. 434 Clinical Evaluation Male
19–18 b p. 435 Ovulation Predictor Kits 19–38 b p. 444 Genetic Testing of the Male,
19–19 d p. 435 Serum Progesterone Table 19-8
19–20 b p. 435 Endometrial Biopsy 19–39 c p. 444 Genetic Testing of the Male
133

CHAPTER 20

Treatment of the Infertile Couple

20–1. Which o these statements regarding in ertility is 20–5. A 27-year-old woman is diagnosed with a pituitary
FALSE? microadenoma during evaluation o secondary amenor-
a. Women are a ected more than twice as o ten as rhea. She subsequently begins a dopamine agonist and
men. becomes pregnant. Which o the ollowing is the best
management o her adenoma during this pregnancy?
b. Around 10 to 15 percent o the reproductive-aged
population is in ertile. a. Resect the pituitary adenoma.
c. Early evaluation a ter only 6 months may be justi- b. Suspend dopamine agonist therapy.
ed or certain medical conditions or or women c. Continue medical therapy with bromocriptine.
aged 35 years or older. d. Obtain pituitary imaging every trimester and
d. In ertility generally is de ned as the inability to continue medical therapy with cabergoline.
achieve success ul pregnancy a ter at least
12 months o regular, unprotected intercourse. 20–6. Which o the ollowing statements is true regarding
thyroid dys unction and in ertility?
20–2. A 28-year-old nullipara who has been seeking preg- a. Subclinical hypothyroidism is not associated with
nancy is diagnosed with polycystic ovarian syndrome ovarian dys unction.
(PCOS). She is generally healthy, but her body mass
b. T ere is no relationship between in ertility and
index (BMI) is in the obese range. Which o the
overt thyroid dys unction.
ollowing should be recommended as rst-line man-
agement o her anovulation? c. In in ertile women seeking pregnancy, treatment o
any degree o hypothyroidism is advisable.
a. Gonadotropins
d. T yroid-stimulating hormone levels are elevated in
b. Clomiphene citrate
more than hal o women with in ertility.
c. Weight loss and exercise
d. Insulin-sensitizing agents 20–7. Which o the ollowing is NOT true regarding clomi-
phene citrate therapy?
20–3. Obesity increases the risk o which o the ollowing a. T e typical starting dose is 200 mg orally.
pregnancy complications?
b. It can be initiated on the second day o the men-
a. Birth de ects strual cycle.
b. Cesarean delivery c. It is taken or ve consecutive days early in the
c. Unexplained stillbirth menstrual cycle.
d. All o the above d. It is classi ed as a category X drug by the Food and
Drug Administration (FDA).
20–4. A very health-conscious patient is seeking pregnancy
and asks about the best diet to ollow at this time. 20–8. Which o the ollowing statements is true regarding
She is overwhelmed by all the advice available rom clomiphene citrate and its use or the treatment o
riends, books, and magazines. Which o the ollow- anovulatory in ertile women?
ing nutritional strategies is most strongly recommend- a. It exerts its e ect as a pure estrogen agonist.
ed or women attempting pregnancy?
b. T e dose required to achieve ovulation shows no
a. High protein diet correlation with body weight.
b. Dietary antioxidants c. Failure to achieve pregnancy alone is a su cient
c. Folic acid supplementation indication to increase the dose in subsequent
d. Complex carbohydrate restriction cycles.
d. Most pregnancies achieved in response to clomi-
phene therapy will occur during the rst several
ovulatory cycles.
134 Reproductive Endocrinology, Infertility, and the Menopause

20–9. A 28-year-old woman with polycystic ovarian syndrome 20–11. Which o the ollowing gonadotropins is a recombi-
(PCOS) and in ertility asks about the potential use nant product?
o met ormin to help achieve pregnancy. She can be a. Menopur
counseled that current evidence LEAST supports
S
b. Bravelle
E
which o the ollowing bene ts o met ormin therapy
C
in women with PCOS? c. Repronex
T
I
d. Follistim
O
a. Improved live-birth rate
N
b. Improved menstrual cyclicity
2
20–12. Which o the ollowing is present in some gonadotro-
c. Higher requency o spontaneous ovulation pin preparations and provides the needed luteinizing
d. Increased ovulatory response to clomiphene citrate hormone (LH) activity by virtue o its shared receptor
therapy with LH?
a. Inhibin
20–10. Despite achieving ovulation in most patients,
b. Activin
clomiphene citrate results in pregnancy rates o 50
percent or less. T is disappointing pregnancy rate is c. Follistatin
attributed to which o the ollowing? d. Human chorionic gonadotropin
a. Long drug hal -li e
20–13. T e treatment protocol shown here is considered
b. Changes to cervical mucus quality
which o the ollowing?
c. Antiestrogenic e ects on the endometrium
d. All o the above

hCG
trigge r

GnRH a gonis t P roge s te rone


IM or va gina l
Gona dotropin inje ctions (8–10 days )

-hCG

Me ns e s Embryo
tra ns fe r
3 follicle s 17 mm Egg
re trieva l
36 h
pos t hCG
Reproduced with permission rom Ho man BL, Schorge JO, Bradshaw KD, et al (eds): Williams Gynecology, 3rd ed.
New York, McGraw-Hill, 2016. Figure 20-2C.

a. Gonadotropin-releasing hormone (GnRH) f are


protocol
b. GnRH antagonist protocol
c. Down-regulation GnRH agonist protocol
d. None o the above
Treatment of the Infertile Couple 135

20–14. Which o the ollowing is true regarding letrozole, an 20–17. A 38-year-old woman with in ertility secondary to
aromatase inhibitor, in the context o ovulation induc- distal tubal blockage undergoes in vitro ertilization.
tion? Her peak estradiol is 4500 pg/mL. Subsequently, 17

C
a. It is Food and Drug Administration (FDA) oocytes are retrieved, and 2 embryos are trans erred.

H
She conceives a singleton gestation but develops ovar-

A
approved or this indication.

P
ian hyperstimulation syndrome (OHSS). A sonogram
b. It is typically dosed daily or 4 to 6 weeks.

T
o her ovaries is shown here. Which o the ollowing

E
R
c. Data regarding its teratogenicity are is a predisposing actor or her developing OHSS?

2
contradictory.

0
d. It has become an accepted alternative to gonado-
tropins and clomiphene citrate.

20–15. Which o the ollowing hormones is thought to play a


contributing role early in the development o ovarian
hyperstimulation syndrome?
a. Progesterone
b. Luteinizing hormone (LH)
c. Human chorionic gonadotropin
d. Follicle-stimulating hormone (FSH)

20–16. Which o the ollowing is NOT a typical clinical


Reproduced with permission rom Doody KJ: reatment o the in ertile couple. In
mani estation o ovarian hyperstimulation Ho man BL, Schorge JO, Bradshaw KD, et al (eds): Williams Gynecology, 3rd ed.
syndrome? New York, McGraw-Hill, 2016, Figure 20-4A.

a. Ascites
a. Older maternal age
b. Seizures
b. High estradiol level
c. Hemoconcentration
c. Etiology o her in ertility
d. Hypercoagulability
d. Number o embryos trans erred

20–18. Suitable treatment o ovarian hyperstimulation syn-


drome may include all o the ollowing EXCEPT:
a. Paracentesis
b. Ovarian drilling
c. Fluid resuscitation
d. T romboembolism prophylaxis

20–19. A patient is now undergoing exogenous gonadotro-


pin ovulation induction. She developed mild ovarian
hyperstimulation syndrome during a previous attempt
at ovulation induction. Strategies to avert or minimize
ovarian hyperstimulation syndrome include which o
the ollowing?
a. Provide prophylactic volume expansion
b. Decrease ollicle-stimulating hormone (FSH) dose
c. Withhold the human chorionic gonadotropin
(hCG) trigger, resulting in cycle cancellation
d. All o the above
136 Reproductive Endocrinology, Infertility, and the Menopause

20–20. What approximate percentage o all multi etal births 20–26. Compared with in vitro ertilization, advantages o
in the United States, including twins, results rom in bilateral tubal reanastomosis to reverse tubal occlusion
vitro ertilization and related procedures? rom a prior sterilization procedure do NOT include
a. 10 percent which o the ollowing?
S
E
a. Ability to conceive naturally
C
b. 30 percent
T
b. Negligible risk o ectopic pregnancy
I
c. 60 percent
O
c. Decreased risk o multi etal gestation
N
d. 90 percent
2
d. Avoidance o ovarian stimulation with exogenous
20–21. Most adverse maternal or perinatal outcomes o hormonal therapy
higher-order multi etal pregnancies are the result o
which o the ollowing? 20–27. Which o the ollowing treatment options o ers the
a. Premature delivery best chance or intrauterine pregnancy in a patient
with the ollowing problem?
b. Advanced maternal age
c. Venous thromboembolic events
d. Fetal growth restriction and discordance

20–22. Which o the ollowing can limit most reliably


the risk o multi etal pregnancy resulting rom
ovulation induction and other assisted reproductive
technologies?
a. Monitor serum estradiol levels.
b. Sonographically measure ollicular size and
number.
c. Use in vitro ertilization to control the number o
embryos trans erred.
d. All o these options are equally reliable.

20–23. T e serum level o which o the ollowing hormones Reproduced with permission rom Ho man BL, Schorge JO, Bradshaw KD, et al (eds):
is most likely to increase a ter ovarian drilling? Williams Gynecology, 3rd ed. New York, McGraw-Hill, 2016. Photo contributor:
Dr. Kevin Doody.
a. Luteinizing hormone (LH)
b. Follicle-stimulating hormone (FSH) a. Fimbriectomy
c. Prolactin b. Neosalpingostomy
d. Androstenedione c. ubal reanastomosis
20–24. Which o the ollowing is the best treatment or a d. In vitro ertilization a ter bilateral salpingectomy
woman with a signi cant decline in ovarian reserve?
20–28. Which o the ollowing statements regarding the
a. Use o donor eggs management o uterine actors in in ertile women is
b. Clomiphene citrate ovulation induction supported by evidence rom a randomized trial?
c. Exogenous gonadotropin ovulation induction a. Endometrial polypectomy increases pregnancy rates.
d. In vitro ertilization with intracytoplasmic sperm b. Myomectomy o intramural myomas increases
injection pregnancy rates.
c. Asherman syndrome can be reversed with high-
20–25. Which o the ollowing tubal obstruction locations is dose estrogen therapy alone.
LEAST amenable to surgical repair?
d. None o the above
a. Isthmic
b. Fimbrial 20–29. Which o the ollowing is the pre erred surgical approach
c. Ampullary or ovarian endometriomas, usually via laparoscopy?
d. Interstitial a. Ovarian wedge resection
b. Cyst drainage and ablation
c. Cyst wall excision by a stripping technique
d. Unilateral oophorectomy i the contralateral ovary
appears normal
Treatment of the Infertile Couple 137

20–30. In women with inadequate cervical mucus quantity or 20–33. In cases o male hypogonadotropic hypogonadism,
quality as a suspected contributor to in ertility, what which o the ollowing semen analysis abnormalities is
is the most reasonable approach to achieving preg- most typically seen?

C
nancy? a. Aspermia

H
A
a. Cryotherapy b. Oligospermia

P
b. Ovulation induction

T
c. Asthenospermia

E
R
c. Intrauterine insemination d. eratozoospermia

2
d. Flax seed oil supplementation

0
20–34. When providing treatment to increase sperm counts,
20–31. Which o the ollowing assisted reproductive technol- it should be kept in mind that the process o sper-
ogies, typically is used to treat male actor in ertility, matogenesis takes approximately how long?
is shown here? a. 30 days
b. 100 days
c. 6 months
d. 9 months

20–35. During in vitro ertilization, prevention o a prema-


ture luteinizing hormone (LH) surge prior to oocyte
retrieval is important. Which o the ollowing medica-
tions helps achieve this goal?
a. Estradiol
b. Leuprolide acetate
c. Human menopausal gonadotropins
d. Recombinant ollicle-stimulating hormone (FSH)

20–36. In ertility thought secondary to which o the ollowing


Reproduced with permission rom Doody KJ: reatment o the in ertile couple. In scenarios warrants intracytoplasmic sperm injection
Ho man BL, Schorge JO, Bradshaw KD, et al (eds): Williams Gynecology, 3rd ed. (ICSI)?
New York, McGraw-Hill, 2016, Figure 20-9A.
a. Chronic anovulation
a. Embryo biopsy b. Severe oligospermia
b. Assisted hatching c. Diminished ovarian reserve in an older woman
c. Oocyte in vitro maturation d. Bilateral distal allopian tube occlusion with
hydrosalpinges
d. Intracytoplasmic sperm injection (ICSI)
20–37. Which o the ollowing assisted reproductive tech-
20–32. Oligospermia is de ned as less than which o the
niques avoids ertilization outside o the body,
ollowing sperm count thresholds per milliliter o
circumventing religious or ethical concerns or some
semen?
patients?
a. 15 million/mL
a. In vitro ertilization (IVF)
b. 50 million/mL
b. Intracytoplasmic sperm injection (ICSI)
c. 100 million/mL
c. Gamete intra allopian tube trans er (GIF )
d. 150 million/mL
d. Zygote intra allopian tube trans er (ZIF )
138 Reproductive Endocrinology, Infertility, and the Menopause

20–38. Which o the ollowing assisted reproductive tech- 20–39. Which o the ollowing complication is more com-
niques is shown here? mon in pregnancies resulting rom assisted repro-
ductive technologies compared with spontaneously
conceived pregnancies?
S
E
a. Cesarean delivery
C
T
b. Placental abruption
I
O
c. Gestational diabetes
N
2
d. All o the above

Reproduced with permission rom Doody KJ: reatment o the in ertile couple. In
Ho man BL, Schorge JO, Bradshaw KD, et al (eds): Williams Gynecology, 3rd ed.
New York, McGraw-Hill, 2016, Figure 20-13A.

a. Embryo biopsy
b. Assisted hatching
c. Oocyte in vitro maturation
d. Intracytoplasmic sperm injection (ICSI)
Treatment of the Infertile Couple 139

Chapter 20 ANSWER KEY


Question Letter Page Question Letter Page

C
number answer cited Header cited number answer cited Header cited

H
A
20–1 a p. 449 Treatment of the Infertile Couple 20–21 a p. 456 Multifetal Gestation

P
T
20–2 c p. 449 Weight Optimization 20–22 c p. 456 Multifetal Gestation

E
R
20–3 d p. 449 Weight Optimization 20–23 b p. 457 Ovarian Drilling

2
0
20–4 c p. 450 Nutrition 20–24 a p. 458 Correction of Diminished
20–5 b p. 450 Hyperprolactinemia Ovarian Reserve
20–6 c p. 450 Hypothyroidism 20–25 d p. 458 Proximal Tubal Obstruction
20–7 a p. 451 Clomiphene Citrate 20–26 b p. 458 Proximal Tubal Obstruction
20–8 d p. 451 Clomiphene Citrate 20–27 d p. 459 Distal Tubal Obstruction
20–9 a p. 452 Insulin-Sensitizing agents 20–28 a p. 460 Intrauterine Adhesions
20–10 d p. 452 Gonadotropins 20–29 c p. 460 Endometriosis
20–11 d p. 452 Gonadotropins, Table 20-1 20–30 c p. 461 Correction of Cervical
Abnormalities
20–12 d p. 452 Gonadotropins
20–31 d p. 462 Correction of Male Infertility
20–13 a p. 452 Gonadotropins, Figure 20-2
20–32 a p. 462 Abnormal Sperm Count
20–14 c p. 453 Aromatase Inhibitors
20–33 b p. 462 Abnormal Sperm Count
20–15 c p. 453 Ovarian Hyperstimulation
Syndrome 20–34 b p. 462 Abnormal Sperm Count
20–16 b p. 453 Ovarian Hyperstimulation 20–35 b p. 464 In Vitro Fertilization
Syndrome 20–36 b p. 465 Intracytoplasmic Sperm
20–17 b p. 453 Ovarian Hyperstimulation Injection
Syndrome 20–37 c p. 465 Gamete or Zygote
20–18 b p. 453 Ovarian Hyperstimulation Intrafallopian Transfer
Syndrome 20–38 a p. 466 Preimplantation Genetic
20–19 d p. 453 Ovarian Hyperstimulation Diagnosis or Screening
Syndrome 20–39 d p. 466 Complications of Assisted
20–20 a p. 456 Multifetal Gestation Reproductive Technology,
Table 20-6
140

CHAPTER 21

Menopausal Transition

21–1. Most women can now expect to live at least what 21–5. During menopausal transition, erratic uctuations
percentage o their li etime a ter menopause? in emale reproductive hormones lead to an array o
a. 5 percent physical and psychological symptoms. T ese symp-
toms include all o the ollowing EXCEPT:
b. 15 percent
a. Headache
c. 25 percent
b. Poor memory
d. 33 percent
c. Vision changes
21–2. Characteristically, menopausal transition begins with d. Urinary incontinence
a variable period o menstrual cycle irregularity and
extends to 1 year a ter permanent cessation o menses. 21–6. During the reproductive li e o a woman, gonado-
T e menopause transition typically spans how many tropin-releasing hormone is released in a pulsatile
years? ashion rom which o the ollowing?
a. 1 to 2 years a. Corpus lutea
b. 2 to 4 years b. Ovarian ollicles
c. 4 to 7 years c. Pituitary gonadotrophs
d. 7 to 10 years d. Arcuate nucleus o the hypothalamus

21–3. Premature ovarian ailure is associated with a persis- 21–7. During menopausal transition, which o the ollow-
tently elevated ollicle-stimulating hormone (FSH) ing results rom cessation o ovarian steroid hormone
level and is urther de ned as cessation o menses release?
be ore what age? a. A rise in luteinizing hormone (LH) levels
a. 35 years b. A rise in ollicle-stimulating hormone (FSH) levels
b. 40 years c. A maximal increase in the requency and ampli-
c. 45 years tude o gonadotropin-releasing hormone secretion
d. 51 years d. All o the above

21–4. Several actors can alter ovarian aging. Which o the 21–8. During the menopausal transition, antimüllerian
ollowing is LEAST likely to shi t menopause to an hormone (AMH) levels do which o the ollowing?
earlier age? a. Remain constant
a. Smoking b. Decrease progressively
b. Chemotherapy c. Increase progressively
c. Pelvic radiation d. Fluctuate erratically beyond menopause
d. Oral contraceptive pills
Menopausal Transition 141

21–9. T is photomicrograph illustrates which o the 21–12. Which o the ollowing transvaginal sonographic
ollowing? images demonstrates a typical premenopausal ovary?

C
H
A
P
T
E
R
2
1
A
Reproduced with permission rom Ho man BL, Schorge JO, Bradshaw KD, et al (eds):
Williams Gynecology, 3rd ed. New York, McGraw-Hill, 2016. Photo contributor:
Dr. Raheela Ash aq.

a. Corpus lutea
b. Corpora albicantia
c. Primordial ollicles
d. Polycystic ovarian syndrome

21–10. A emale etus has 6 to 7 million oocytes present dur-


ing the 20th week o gestation. Approximately, how
many oocytes are present at birth?
a. 70,000
b. 250,000
c. 700,000
B
d. 3,500,000

21–11. An average woman is expected to experience how a. A


many ovulatory events during her reproductive b. B
li etime?
c. Both A and B
a. 100
d. Neither o them is typical.
b. 400
c. 700,000 21–13. With advancing age, which o the ollowing adrenal
d. 6 to 7 million steroid levels declines?
a. Pregnenolone
b. Androstenedione
c. Dehydroepiandrosterone sul ate
d. All o the above
142 Reproductive Endocrinology, Infertility, and the Menopause

21–14. Anovulation is the most common cause o erratic 21–18. A tabulation o published epidemiologic studies
bleeding during menopausal transition. However, determined that up to what percentage o menstru-
as shown in this sonogram, endometrial cancer is ating women develop vasomotor symptoms during
suspected during this transition in those with abnormal menopausal transition?
S
E
uterine bleeding (AUB). Here, the thickened heteroge- a. 60 percent
C
neous endometrium measures 26 mm. T e overall risk
T
b. 70 percent
I
or endometrial cancer increases to what percentage in
O
c. 80 percent
N
women with AUB during menopausal transition?
2
d. 90 percent

21–19. T ermoregulatory changes that accompany a hot


ush have been well documented. An individual hot
ush typically lasts how many minutes?
a. < 1 minute
b. 1 to 5 minutes
c. 6 to 10 minutes
d. 11 to 15 minutes

21–20. Hot ushes and the concurrent increase in metabolic


rate may be accompanied by which o the ollowing?
a. Panic
b. Irritability
c. Palpitations
d. All o the above
a. 0.1 percent
b. 1 percent 21–21. Physiologic increases during a hot ush include all o
c. 10 percent the ollowing EXCEPT:
d. 30 percent a. Heart rate
b. Mean skin temperature
21–15. In postmenopausal women, unopposed estrogen may
c. Systolic blood pressure
be derived rom which o the ollowing sources?
d. Plasma levels o estradiol
a. Exogenous estrogen
b. Extragonadal endogenous estrogen production 21–22. A menstruating 48-year-old patient undergoes a total
c. Decreased sex hormone-binding globulin levels vaginal hysterectomy and bilateral salpingo-oophorec-
d. All o the above tomy or symptomatic uterine leiomyomas. Surgi-
cal menopause is associated with what percentage
21–16. A 51-year-old woman asks or relie rom an array o probability o hot ushes during the rst year a ter
recent-onset physical and psychological symptoms. oophorectomy?
What is the most common medical complaint o a. 40 percent
women during menopausal transition? b. 60 percent
a. Depression c. 90 percent
b. Vasomotor symptoms d. 100 percent
c. Pain ul intercourse
d. Stress urinary incontinence 21–23. A 55-year-old postmenopausal, thin, patient who
smokes asks or counseling regarding osteoporosis due
21–17. Longitudinal studies show thermoregulation dys unc- to her concern or uture bone racture risk. In addi-
tion is associated with which o the ollowing? tion to the vertebrae and emoral neck, which o the
ollowing is most commonly ractured?
a. Smoking
a. Rib
b. Low exercise levels
b. Wrist
c. Socioeconomic status
c. Fibula
d. All o the above
d. Humerus
Menopausal Transition 143

21–24. Osteopenia and osteoporosis are disorders character- 21–28. Genetic in uence on osteoporosis and bone mineral
ized by a progressive reduction in bone mass and density (BMD) is important. Heredity is estimated
predispose patients to ractures in the spine, hips, and to account or up to what percentage o bone mineral

C
other sites. What is the approximate mortality rate density variability?

H
rom hip racture alone?

A
a. 20 percent

P
a. 10 percent b. 40 percent

T
E
b. 30 percent

R
c. 60 percent

2
c. 50 percent d. 80 percent

1
d. 70 percent

21–25. Bone mineral density (BMD) values or sex, age, and


ethnicity have been determined. For diagnostic pur-
poses, results o BMD testing are reported as T-scores.
What -score value is associated with osteopenia?

Reproduced with permission rom Bradshaw KD: Menopausal transition. In Ho man BL, Schorge JO, Bradshaw KD, et al (eds): Williams Gynecology, 3rd ed. New York, McGraw-Hill,
2016, Figure 21-8B.

a. -score between + 2.5 and - 1.0 21–29. Modi able actors that may bene t emoral neck and
b. -score between + 1.0 and - 1.0 lumbar vertebral bone mineral density in postmeno-
pausal women include which o the ollowing?
c. -score between + 1.0 and - 2.5
a. Alcohol use
d. -score between - 1.0 and - 2.5
b. Low body mass
21–26. Age and race are nonmodi able risk actors or osteo- c. Sedative drug use
porosis. Menopausal osteoporosis is most common in d. Progressive resistance training
which racial group?
a. Asian 21–30. T erapy with glucocorticoids lasting more than 2 to 3
b. White months is a major risk actor or bone loss and racture,
particularly among postmenopausal women. O chron-
c. A rican–American
ic daily prednisone doses, which minimum threshold
d. Paci c Islander should prompt assessment and clinical intervention to
prevent or treat glucocorticoid-induced osteoporosis?
21–27. Which o the ollowing are secondary causes o os-
a. 5 mg
teoporosis that should be screened or in any patient
with osteoporosis? b. 10 mg
a. Hypothyroidism c. 15 mg
b. Hypoparathyroidism d. 20 mg
c. Chronic renal disease
d. None o the above
144 Reproductive Endocrinology, Infertility, and the Menopause

21–31. In women older than 50 years, what accounts or 21–36. T e North American Menopause Society term genito-
approximately 40 percent o deaths? urinary syndrome of menopause (GSM) encompasses
a. Stroke which o the ollowing signs and symptoms?
a. Dyspareunia
S
b. Lung cancer
E
C
c. Breast cancer b. Vulvar dryness
T
I
d. Atherosclerotic cardiovascular disease c. Urinary urgency and dysuria
O
N
d. All o the above
2
21–32. Be ore menopausal transition, women have a much
lower risk or cardiovascular events compared with 21–37. Estrogen receptors have been identi ed in the vulva,
men o their same age. Reasons or protection rom vagina, urethra, pelvic oor musculature, and en-
cardiovascular disease in premenopausal women are dopelvic ascia. Which o the ollowing is a result o
complex. However, a signi cant contribution can be decreased estrogen trophic in uence?
assigned to an e ect o estrogen that results in which a. Vaginal pH greater than 4.5
o the ollowing?
b. Lower rates o abnormal cervical cytology
a. Increased low-density lipoprotein levels
c. Increased vaginal concentrations o Lactobacilli
b. Decreased high-density lipoprotein levels
d. Higher rates o Bartholin gland duct cyst
c. Increased high-density lipoprotein levels ormation
d. Increased total cholesterol and low-density lipo-
protein levels 21–38. Symptoms o vulvovaginal atrophy include vaginal
dryness, itching, irritation, and dyspareunia. T ese
21–33. A 53-year-old multigravida postmenopausal patient complaints during menopausal transition have an
reports her physical activity level is less than when estimated prevalence that reaches what percentage?
she was younger and that she has not altered her a. 30 percent
eating habits. T is has resulted in weight gain. You
b. 50 percent
counsel her that gaining weight during menopausal
transition is associated with at deposition in the c. 70 percent
abdomen, increasing the likelihood o developing d. 90 percent
which o the ollowing?
a. Diabetes mellitus 21–39. A 38-year-old nulligravida complains o no men-
ses or the last 12 months and denies a history o
b. Insulin resistance
radiation or chemotherapy exposure, galactorrhea,
c. Cardiovascular disease headache, or cigarette smoking. A markedly elevated
d. All o the above ollicle-stimulating hormone (FSH) level is obtained
on repeated occasions. Which o the ollowing caus-
21–34. T e e ect o hormone de ciency on skin aging has ative categories should now be considered?
been widely studied but its contribution toward a. In ection
dermatologic changes is dif cult to determine. Skin
b. Autoimmune disorders
aging results rom the synergistic e ects o intrinsic
aging along with which o the ollowing? c. Chromosomal abnormalities
a. Photoaging d. All o the above
b. High body mass index
c. Reduced ollicle-stimulating hormone (FSH)
levels
d. None o the above

21–35. Decreasing estrogen levels seen in late menopausal


transition result in which o the ollowing dental
changes?
a. Increased salivation
b. Buccal epithelium atrophy
c. Increased oral alveolar bone osteoblasts
d. All o the above
Menopausal Transition 145

Chapter 21 ANSWER KEY


Question Letter Page Question Letter Page

C
number answer cited Header cited number answer cited Header cited

H
A
21–1 d p. 471 Menopausal Transition 21–21 d p. 475 Pathophysiology of Vasomotor

P
T
21–2 c p. 471 Definitions Symptoms

E
R
21–3 b p. 471 Definitions 21–22 c p. 476 Risk Factors for Vasomotor

2
Symptoms

1
21–4 d p. 471 Influential Factors
21–23 b p. 477 Osteopenia and Osteoporosis
21–5 c p. 471 Table 21-1
21–24 b p. 477 Osteopenia and Osteoporosis
21–6 d p. 472 Hypothalamus–Pituitary–
Ovarian Axis 21–25 d p. 479 Table 21-3
21–7 d p. 472 Hypothalamus–Pituitary– 21–26 b p. 481 Nonmodifiable Factors
Ovarian Axis 21–27 c p. 479 Table 21-4
21–8 b p. 472 Hypothalamus–Pituitary– 21–28 d p. 481 Nonmodifiable Factors
Ovarian Axis 21–29 d p. 482 Modifiable Factors
21–9 b p. 473 Figure 21-3 21–30 a p. 482 Modifiable Factors
21–10 c p. 473 Ovary 21–31 d p. 483 Cardiovascular Changes
21–11 b p. 473 Ovary 21–32 c p. 483 Cardiovascular Changes
21–12 a p. 473 Ovary 21–33 d p. 483 Weight Gain and Fat
21–13 d p. 473 Ovary Distribution
21–14 c p. 474 Menstrual Disturbances 21–34 a p. 484 Dermatologic Changes
21–15 d p. 474 Menstrual Disturbances 21–35 b p. 484 Dental Changes
21–16 b p. 474 Central Thermoregulation 21–36 d p. 486 Lower Reproductive Tract
21–17 d p. 474 Central Thermoregulation Changes
21–18 a p. 474 Central Thermoregulation 21–37 a p. 486 Vulvovaginal Changes
21–19 b p. 474 Vasomotor Symptoms 21–38 b p. 486 Vulvovaginal Changes
21–20 d p. 474 Vasomotor Symptoms 21–39 d p. 487 Patient Evaluation
200

CHAPTER 30

Cervical Cancer

30–1. Which o the ollowing statements regarding cervical 30–5. What does the human papillomavirus (HPV) onco-
cancer is NOT true? gene E6 bind and inactivate, leading to immortalization
a. It is the most common gynecologic cancer in o the cell?
women worldwide. a. Rb
b. It is the second most common cancer diagnosed in b. p16
women worldwide. c. p53
c. In the Unites States, Hispanic women have the d. Cyclin D1
highest incidence rate.
d. In the United States, A rican–American women 30–6. Which histologic subtype o cervical carcinoma is
have the highest mortality rate. represented in this micrograph? Arrows point to
keratin pearls.
30–2. Which human papillomavirus (HPV) subtype is
most strongly associated with adenocarcinoma o the
cervix?
a. HPV 6
b. HPV 16
c. HPV 18
d. HPV 31

30–3. Which o the ollowing is true regarding smoking and


cervical cancer risk?
a. Smoking may alter viral oncoprotein expression.
b. Passive smoking increases the risk o cervical
cancer.
c. Women who smoke are less likely to clear high-risk
human papillomavirus (HPV) in ection.
a. Melanoma
d. All o the above
b. Adenocarcinoma
30–4. Which o the ollowing statements regarding cervical c. Squamous cell carcinoma
cancer risk actors is FALSE? d. Neuroendocrine carcinoma
a. Cervical cancer is an acquired immune de ciency
syndrome (AIDS)-de ning illness. 30–7. Which histologic subtype o cervical cancer is associ-
ated with Peutz–Jeghers syndrome?
b. ransplant recipients have an increased risk o
cervical cancer. a. Adenoma malignum
c. In general, women who have an autoimmune dis- b. Clear cell adenocarcinoma
ease and take immunosuppressant therapy do not c. Glassy cell adenocarcinoma
have an increased risk o cervical cancer. d. Villoglandular adenocarcinoma
d. Combination oral contraceptives are associated
with an increased risk o cervical cancer due to the 30–8. Which o the ollowing should NOT be used or
higher number o sexual partners in users. hemostasis in cases o li e-threatening hemorrhage
rom cervical cancer?
a. Lugol solution
b. Emergent radiation
c. Uterine artery embolization
d. Monsel ( erric subsul ate) solution
Cervical Cancer 201

30–9. In patients with stage I cervical cancer, what percent- 30–13. A 45-year-old woman has a 6-cm adenocarcinoma o
age o Pap tests are read as consistent with cancer? the cervix. She has positive paraaortic nodes ound
a. 5 to 10 percent during positron emission tomography (PE ) scanning

C
and the ollowing nding during intravenous pyelog-

H
b. 10 to 20 percent
raphy. What stage is assigned to her cervical cancer?

A
c. 30 to 50 percent

P
T
d. 60 to 80 percent

E
R
3
30–10. A woman presents or a routine health maintenance

0
visit. T e ollowing is visualized during speculum
examination. What is the most appropriate next step?

a. IB2
b. IIB2
c. IIIB
d. IVB

30–14. A 38-year-old woman with stage IIB cervical cancer


a. Per orm a Pap test has positron emission tomography (PE ) scanning
that shows positive pelvic lymph nodes. In response,
b. Per orm a cervical biopsy
she undergoes a laparoscopic extraperitoneal para-
c. Re er to a gynecologic oncologist aortic lymph node dissection. Histologic review shows
d. Obtain abdominopelvic computed tomography that the paraaortic nodes are positive or tumor. What
is the most appropriate treatment?
30–11. Which o the ollowing tests is NOT used or staging a. Chemoradiation with extended eld radiation
cervical cancer per the International Federation o
b. Chemotherapy alone, since this represents meta-
Gynecology and Obstetrics (FIGO)?
static disease
a. Cystoscopy
c. Chemoradiation to pelvis alone, since involved
b. Chest radiograph nodes were removed
c. Intravenous pyelogram d. None o the above
d. Computed tomography scan
30–15. Among the ollowing, which is the most signi cant
30–12. A woman undergoes a radical hysterectomy, bilateral prognostic actor or early-stage cervical cancer?
pelvic and paraaortic lymph node dissection or a a. Grade
4-cm squamous cell carcinoma o the cervix. On nal
b. Histology
pathology report, the parametria are noted to contain
tumor. What stage is her cervical cancer? c. Depth o invasion
a. IB1 d. Lymph node metastasis
b. IB2
30–16. What is the most appropriate surgical approach or a
c. IIA1 woman who has a stage IA1 squamous cell carcinoma
d. IIB2 o the cervix and has completed childbearing?
a. Cold-kni e conization
b. Extra ascial hysterectomy
c. ype III radical hysterectomy
d. Modi ed ( ype II) radical hysterectomy
217

CHAPTER 33

Endometrial Cancer

33–1. O leading causes o cancer in women in the United 33–6. Which o the ollowing birth control methods does
States, endometrial cancer is ranked in which place? NOT reduce the risk o endometrial cancer?
a. Second a. Bilateral tubal ligation
b. Fourth b. Copper intrauterine device
c. Eighth c. Combination oral contraceptives
d. Sixteenth d. Levonorgestrel-releasing intrauterine system

33–2. Which o the ollowing does NOT increase a woman’s 33–7. Which o the ollowing patients does NOT require an
risk o developing endometrial cancer? endometrial biopsy?
a. Obesity a. A 27-year-old obese woman with polycystic
b. amoxi en ovarian syndrome and a long history o irregular
bleeding
c. Cigarette smoking
b. A 37-year-old obese woman with atypical glandu-
d. Unopposed estrogen
lar cells (AGCs) reported rom cervical cytology
testing
33–3. What is the primary mechanism by which obesity
increases the risk o endometrial cancer? c. A 55-year-old postmenopausal woman with post-
menopausal bleeding and an endometrial stripe
a. Androstenedione is aromatized by adipose tissue to
measuring 8 mm by transvaginal sonography
estrone.
d. A 52-year-old postmenopausal woman using
b. Androstenedione is aromatized by adipose tissue to
combination hormone replacement therapy, who
estradiol.
has postmenopausal bleeding and an endometrial
c. Higher levels o insulin growth actor lead to stripe measuring 2 mm by transvaginal sonography
anovulation, which results in unopposed estrogen.
d. None o the above 33–8. A 60-year-old woman has abnormal uterine bleeding.
ransvaginal sonography reveals a 15-mm endometri-
33–4. What percentage o endometrial cancer is diagnosed al stripe and an 8-cm, solid right adnexal mass. O ce
in women younger than 45 years? endometrial biopsy reveals grade 1 endometrioid
a. 1 percent adenocarcinoma o the uterus arising in a background
o complex atypical hyperplasia. What is the most
b. 8 percent
likely diagnosis?
c. 15 percent
a. Endometrial cancer metastatic to the ovary
d. 25 percent
b. Synchronous ovarian and endometrial cancer
33–5. What is the most common genetic syndrome associ- c. Pedunculated leiomyoma and endometrial cancer
ated with endometrial cancer? d. Ovarian granulosa cell tumor and endometrial
a. Cowden syndrome cancer
b. Li–Fraumeni syndrome
33–9. Which o the ollowing is an appropriate treatment o
c. Hereditary breast ovarian cancer syndrome a 35-year-old woman with the diagnosis o complex
d. Hereditary nonpolyposis colorectal cancer endometrial hyperplasia without atypia?
syndrome a. Medroxyprogesterone acetate
b. Combination oral contraceptive pills
c. Levonorgestrel-releasing intrauterine system
d. All o the above
218 Gynecologic Oncology

33–10. For simple and complex endometrial hyperplasia 33–15. Which o the ollowing patients should be re erred to
without atypia, what is the clinical success rate or a geneticist or possible hereditary nonpolyposis colon
resolution using progestin therapy? cancer (HNPCC/Lynch syndrome) screening?
a. 30 to 40 percent a. A 35-year-old patient with endometrial cancer
S
E
whose sister was diagnosed with breast cancer at
C
b. 50 to 60 percent
T
age 50 and maternal grandmother diagnosed with
I
c. 70 to 80 percent
O
colon cancer at age 61
N
d. 90 to 100 percent
b. A 39-year-old patient with endometrial cancer
4
whose mother was diagnosed with endometrial
33–11. A 30-year-old nulligravida has a body mass index o
cancer at age 50 and maternal grandmother diag-
35 kg/m2 and desires ertility. An endometrial biopsy
nosed with colon cancer at age 55
per ormed or abnormal bleeding reveals complex
atypical hyperplasia. Which o the ollowing is NOT c. A 39-year-old patient with endometrial cancer
an acceptable treatment option? whose maternal grand ather was diagnosed with
colon cancer at age 65 and paternal grandmother
a. Megestrol acetate 80 mg orally twice daily
diagnosed with endometrial cancer at age 58
b. Levonorgestrel-releasing intrauterine system
d. A 35-year-old patient with complex atypical
c. Medroxyprogesterone acetate 10 mg orally daily hyperplasia whose maternal grandmother was
d. otal laparoscopic hysterectomy with bilateral diagnosed with colon cancer at age 58 and mater-
salpingectomy nal grand ather diagnosed with small bowel cancer
at age 70
33–12. What is the approximate underlying risk o endo-
metrial cancer in a woman diagnosed with complex 33–16. Which o the ollowing patients does NOT need
atypical hyperplasia by endometrial biopsy? preoperative abdominopelvic computed tomography
a. 10 percent (C ) prior to surgical management?
b. 40 percent a. A 62-year-old patient with clear cell carcinoma
ound by o ce endometrial biopsy
c. 60 percent
b. A 75-year-old patient with uterine papillary serous
d. 80 percent
carcinoma ound during hysteroscopy plus dilation
and curettage
33–13. Which o the ollowing is the most appropriate surgi-
cal approach to a 51-year-old woman with a body c. A 58-year-old patient with grade 1 endometrioid
mass index o 35 kg/m2 and a preoperative diagnosis adenocarcinoma arising in a background o com-
o complex atypical hyperplasia? plex atypical hyperplasia that was ound by o ce
endometrial biopsy
a. ransvaginal hysterectomy, bilateral
salpingo-oophorectomy d. All o the above require C scanning prior to
surgical management.
b. otal abdominal hysterectomy, bilateral salpingo-
oophorectomy, pelvic washings
33–17. Which o the ollowing is true when a woman with
c. otal laparoscopic hysterectomy, bilateral salpingo- endometrial cancer is managed by a gynecologic
oophorectomy, pelvic washings oncologist compared with a general gynecologist?
d. None o the above a. Less surgical morbidity
b. More likely to undergo surgical staging
33–14. Which o the ollowing women should undergo
annual endometrial biopsy? c. Improved overall survival rate or high-risk types
a. A 36-year-old woman with hereditary nonpolypo- d. All o the above
sis colorectal cancer (HNPCC/Lynch syndrome)
b. A 45-year-old woman on tamoxi en or a personal
history o breast cancer
c. An obese 40-year-old woman with normal menses,
with a ather with colon cancer diagnosed at age
60 and a paternal aunt with endometrial cancer
ound at age 55
d. All o the above
Endometrial Cancer 219

33–18. T e histologic specimen shown below demonstrates 33–20. Which o the ollowing is true regarding the type o
< 5 percent solid growth but severe nuclear atypia. uterine cancer shown in the photomicrograph below?
What grade is assigned to this endometrioid adeno-

C
carcinoma o the uterus?

H
A
P
T
E
R
3
3
Reproduced, with permission, by Schorge JO, Scha er JI, Halvorson LM, et al (eds):
Williams Gynecology, 1st ed. New York, McGraw-Hill, 2008. Photo contributor:
Dr. Raheela Ash aq.
Used with permission rom Dr. Kelley Carrick.

a. It is the most common cell type o endometrial


a. Grade 1
cancer.
b. Grade 2
b. It is the least common cell type o endometrial
c. Grade 3 cancer.
d. T is is not endometrioid adenocarcinoma, but c. Most patients with this cell type are cured with
rather papillary serous carcinoma o the uterus. surgery alone.
d. T is is an aggressive type II endometrial cancer,
33–19. Which o the ollowing subtypes is NOT a type II
managed by a combination o surgery and chemo-
endometrial cancer?
therapy, with or without radiation therapy.
a. Clear cell carcinoma
b. Papillary serous carcinoma 33–21. What is the most common way endometrioid adeno-
c. Endometrioid adenocarcinoma with squamous carcinoma o the uterus spreads?
di erentiation a. Lymphatic
d. None o the above b. Hematogenous
c. Direct extension
d. Intraperitoneal ex oliation

33–22. A 60-year-old woman undergoes a robotic-assisted


hysterectomy, bilateral salpingo-oophorectomy, and
bilateral pelvic and paraaortic lymph node dissection
or endometrial cancer. Histologic evaluation reveals
a grade 2 endometrioid adenocarcinoma with > 50
percent myometrial invasion, lymphovascular space
invasion (LVSI), and positive washings. All other
surgical specimens are benign. What stage is she
assigned according to 2009 International Federation
o Gynecology and Obstetrics (FIGO) criteria?
a. Stage IB
b. Stage IC
c. Stage IIIA
d. Stage IIIC
220 Gynecologic Oncology

33–23. A 47-year-old woman undergoes total laparoscopic 33–25. What International Federation o Gynecology and
hysterectomy, bilateral salpingo-oophorectomy, and Obstetrics (FIGO) 2009 stage is represented below?
bilateral pelvic and paraaortic lymphadenectomy or
endometrial cancer. Histologic evaluation reveals a
S
E
grade 2 endometrioid adenocarcinoma arising in the
C
uterus, with 30 percent myometrial invasion and with
T
I
endocervical gland extension. What is her Inter-
O
N
national Federation o Gynecology and Obstetrics
4
(FIGO) stage?
a. Stage IA
b. Stage IB
c. Stage II
d. Stage IIIB

33–24. A 52-year-old woman undergoes a robotic hysterec-


tomy and bilateral salpingo-oophorectomy or grade
1 endometrioid adenocarcinoma o the uterus. T e
uterus is bisected, and one hal is shown to display Reproduced with permission rom Ho man BL, Schorge JO, Bradshaw KD, et al (eds):
the endocervical canal and endometrial cavity. With Williams Gynecology, 3rd ed. New York, McGraw-Hill Education, 2016. Figure 32-3 II.
rozen section analysis, no myometrial invasion and
no lymphovascular space invasion (LVSI) are ound. a. Stage IIIA
Which o the ollowing is FALSE? b. Stage IIIB
c. Stage IIIC1
d. Stage IIIC2

33–26. Which o the ollowing is NOT an advantage o lapa-


roscopy compared with laparotomy or the surgical
management o endometrial cancer?
a. Shorter hospital stay
b. Improved quality o li e
c. Lower rate o intraoperative injuries
d. Fewer moderate to severe complications

33–27. Which o the ollowing women needs continued Pap


tests o the vaginal cu a ter a hysterectomy, bilat-
eral salpingo-oophorectomy, and bilateral pelvic and
paraaortic lymph node dissection or a stage IB grade
Reproduced with permission by Schorge JO, Scha er JI, Halvorson LM, et al (eds): 1 endometrioid adenocarcinoma o the uterus?
Williams Gynecology, 1st ed. New York, McGraw-Hill, 2008. Photo contributor:
Dr. Raheela Ash aq. a. A 50-year-old woman with no history o abnormal
Pap test results
a. T e rozen section result may be inaccurate. b. A 50-year-old woman with oci o cervical
b. I f nal histologic analysis concurs with the rozen intraepithelial neoplasia (CIN) 2 in the hysterec-
section, the risk o lymph node metastasis in this tomy specimen
case is 1 percent. c. A 50-year-old woman with a remote history o
c. I f nal histologic analysis concurs with the rozen CIN 2 treated with a loop electrosurgical excision
section, she will need postoperative radiation i a procedure (LEEP) and no abnormal Pap tests in
lymph node dissection is not done. the last 10 years
d. All o the above d. All o the above
Endometrial Cancer 221

33–28. Which o the ollowing chemotherapeutic agents 33–31. Desiring uture ertility, a 34-year-old nulligravida
does not have def nitive activity against endometrial with polycystic ovarian syndrome has undergone a
cancer? hysteroscopy, dilatation and curettage (D&C), and

C
a. Cisplatin placement o a levonorgestrel-releasing intrauterine

H
system (LNG-IUS) or grade 1 endometrioid adeno-

A
b. opotecan

P
carcinoma o the uterus. Findings at the time o sur-

T
c. Paclitaxel gery are depicted below. How should she be ollowed?

E
R
d. Doxorubicin

3
3
33–29. An 80-year-old woman is diagnosed with a grade 1
endometrioid adenocarcinoma o the uterus by o ce
endometrial biopsy. She has a body mass index o
45 kg/m2, has uncontrolled type 2 diabetes mellitus,
has had a prior right below-the-knee amputation, and
is wheelchair bound. What is the most appropriate
management?
a. Radiation
b. amoxi en
c. Vaginal hysterectomy
d. Robotic hysterectomy, bilateral salpingo-
oophorectomy

33–30. A 65-year-old otherwise healthy woman has an


episode o postmenopausal bleeding. An endometrial Used with permission rom Dr. Mayra hompson.

biopsy reveals papillary serous carcinoma. At explo-


ration, she has carcinomatosis and an omental cake a. Remove IUD in 3 months and per orm endome-
(shown here). In addition to total abdominal hyster- trial biopsy.
ectomy with bilateral salpingo-oophorectomy, what b. Remove IUD in 3 months and per orm a hysteros-
other management is most appropriate? copy, D&C.
c. Repeat endometrial biopsy in 3 months with the
IUD in place.
d. None o the above

33–32. What is the most important prognostic actor or a


woman with endometrioid adenocarcinoma o the
uterus?
a. Age
b. Grade
c. Stage
d. Positive peritoneal washings

33–33. A 68-year-old woman has a history o a stage IA


grade 1 endometrioid adenocarcinoma o the uterus,
treated with surgery. She develops vaginal bleeding,
Used with permission rom Dr. David Miller. and biopsy o a 1-cm mass at her vaginal apex reveals
recurrent uterine cancer. What is the most appropri-
a. Omentectomy, adjuvant chemotherapy ate treatment, i no metastatic disease is ound during
urther evaluation?
b. Omental biopsy, adjuvant chemotherapy
a. Radiation
c. Omentectomy, maximal e ort at tumor debulking,
adjuvant chemotherapy b. Chemotherapy
d. Omentectomy, maximal e ort at tumor debulking, c. Pelvic exenteration
adjuvant chemotherapy, and tumor-directed d. Oral continuous progesterone therapy
radiation
222 Gynecologic Oncology

Chapter 33 ANSWER KEY


Question Letter Page Question Letter Page
number answer cited Header cited number answer cited Header cited
S
E
C
33–1 b p. 702 Epidemiology and Risk Factors 33–18 b p. 709 Histologic Grade, Table 33-6
T
I
O
33–2 c p. 702 Epidemiology and Risk Factors 33–19 c p. 710 Histologic Type
N
33–3 a p. 702 Epidemiology and Risk Factors 33–20 d p. 710 Clear Cell Carcinoma, Figure
4
33–4 b p. 702 Epidemiology and Risk Factors 33-9
33–5 d p. 702 Epidemiology and Risk Factors 33–21 c p. 712 Patterns of Spread
33–6 a p. 702 Epidemiology and Risk Factors 33–22 a p. 713 Table 33-9
33–7 d p. 705 Clinical Features and Diagnosis 33–23 a p. 713 Surgical Management,
Table 33-9
33–8 d p. 705 Clinical Features and Diagnosis
33–24 c p. 716 Adjuvant Radiation
33–9 d p. 706 Nonatypical Endometrial Therapy, Table 33-8
Hyperplasia
33–25 a p. 714 Figure 33-11
33–10 c p. 706 Response to Progestins
33–26 c p. 713 Treatment
33–11 c p. 706 Atypical Endometrial
Hyperplasia 33–27 b p. 715 Surveillance
33–12 b p. 706 Atypical Endometrial Hyperplasia 33–28 b p. 715 Chemotherapy
33–13 c p. 706 Atypical Endometrial 33–29 a p. 715 Radiation
Hyperplasia 33–30 c p. 716 Uterine Papillary Serous
33–14 a p. 707 Prevention Carcinoma Management
33–15 b p. 707 Prevention, Table 33-4 33–31 c p. 717 Fertility-Sparing Management
33–16 c p. 708 Imaging Studies 33–32 c p. 717 Prognostic Factors
33–17 d p. 709 Role of the Generalist 33–33 a p. 717 Recurrent Disease
230

CHAPTER 35

Epithelial Ovarian Cancer

35–1. In what place is ovarian cancer currently ranked 35–6. Which o the ollowing statements regarding ovarian
among causes o cancer-related death in US women? cancer screening is true?
a. First a. Women at average risk should undergo OvaSure
b. Fi th testing annually.
c. enth b. Women at average risk should undergo annual
cancer antigen 125 (CA125) and transvaginal
d. Fi teenth
sonography screening.
35–2. What percentage o epithelial ovarian cancers is c. Currently, there is no approved screening test or
hereditary? women at average risk in the United States.
a. 0.01 percent d. Women at average risk should undergo Risk o
Ovarian Cancer Algorithm (ROCA)-based CA125
b. 0.3 percent
screening and study-directed sonography.
c. 10 percent
d. 30 percent 35–7. A 40-year-old woman with a BRCA1 mutation
undergoes a laparoscopic bilateral salpingo-
35–3. Which o the ollowing is NOT a risk actor or oophorectomy or risk reduction. Subsequent to
ovarian cancer? this surgery, her breast cancer risk is reduced by
a. Nulliparity what percentage?
b. Late menopause a. 25 percent
c. Oral contraceptive pills b. 50 percent
d. Hereditary nonpolyposis colorectal cancer c. 75 percent
d. 90 percent
35–4. Which o the ollowing genetic mutations is associated
with the highest risk o developing ovarian cancer? 35–8. What is the main histologic dif erence between a
a. PTEN low-malignant-potential tumor and epithelial ovarian
cancer?
b. MLH1
a. Stromal invasion
c. BRCA1
b. Mitotic activity
d. BRCA2
c. Degree o nuclear atypia
35–5. Which o the ollowing statements regarding BRCA d. Nuclear to cytoplasmic ratio
variants o uncertain clinical signif cance is FALSE?
a. Women with this test result should undergo pro-
phylactic salpingo-oophorectomy.
b. T ey may be normal variants and may not repre-
sent an increased risk o ovarian and breast cancer.
c. One third o BRCA1 test results may be classi ed
as variant o uncertain clinical signi cance.
d. One hal o BRCA2 test results may be classi ed as
variant o uncertain clinical signi cance.
Epithelial Ovarian Cancer 231

35–9. A 35-year-old G1P1 undergoes surgical exploration 35–10. A 27-year-old nulligravida who desires uture ertility
or a large pelvic mass, shown here. T e rozen sec- underwent a laparoscopic le t salpingo-oophorectomy
tion analysis o her right ovary returns a diagnosis o or a persistent 7-cm cyst that was causing pressure

C
“mucinous tumor o low malignant potential, cannot and pain. T e cyst was removed intact, and peritoneal

H
exclude invasion.” Clinically, there is no other obvi- washings were negative or cancer. T e nal pathol-

A
P
ous disease. In addition to pelvic washings, multiple ogy report revealed a serous low-malignant-potential

T
peritoneal biopsies, omentectomy, and appendectomy, tumor. What is the most appropriate management?

E
R
what other surgical procedures should be per ormed?

3
5
a. Close observation
a. No other procedures b. Right salpingo-oophorectomy (RSO), omentec-
b. Bilateral pelvic and paraaortic lymph node tomy, multiple peritoneal biopsies
dissection c. RSO, omentectomy, multiple peritoneal biop-
c. otal abdominal hysterectomy, bilateral salpingo- sies, bilateral pelvic and paraaortic lymph node
oophorectomy, and bilateral pelvic and paraaortic dissection
lymph node dissection d. Hysterectomy, RSO, omentectomy, multiple
d. None o the above peritoneal biopsies, bilateral pelvic and paraaortic
lymph node dissection

35–11. Which o the ollowing is the worst prognostic eature


associated with low-malignant-potential tumors?
a. Advanced stage
b. Invasive implants
c. Mucinous histology
d. Stromal microinvasion

35–12. What percentage o women with recurrent low-


malignant-potential tumors have ovarian cancer?
a. 10 percent
b. 30 percent
c. 50 percent
d. 70 percent
232 Gynecologic Oncology

35–13. Which o the ollowing is FALSE regarding ovarian 35–18. In this photomicrograph, what are the arrows point-
cancer symptoms? ing to? (Hint: T is nding is pathognomonic or the
a. Most women with ovarian cancer experience most common histologic cell type among epithelial
persistent symptoms. ovarian cancers.)
S
E
C
b. Women with ovarian cancer do not have symp-
T
toms until the disease is very advanced.
I
O
c. T e most requent symptoms associated with ovar-
N
4
ian cancer include urinary urgency, pelvic pain,
and bloating.
d. None o the above

35–14. What percentage o patients with stage I ovarian can-


cer have a normal cancer antigen 125 (CA125) level?
a. 0.5 percent
b. 5 percent
c. 25 percent
d. 50 percent

35–15. Which o the ollowing conditions can cause an


Used with permission rom Dr. Kelley Carrick.
elevated cancer antigen 125 (CA125) level?
a. Leiomyomata
a. Signet ring
b. Menstruation
b. Hobnail cell
c. Endometriosis
c. Keratin pearl
d. All o the above
d. Psammoma body
35–16. Which o the ollowing radiologic tests is most
35–19. Which o the ollowing cell types o ovarian cancer is
help ul in the treatment planning or a patient with
NOT associated with endometriosis?
suspected advanced ovarian cancer?
a. Clear cell
a. ransvaginal sonography
b. Endometrioid
b. Positron emission tomography
c. Papillary serous
c. Magnetic resonance imaging o the pelvis
d. All o the above
d. Computed tomography o the abdomen and pelvis
35–20. A 27-year-old woman presents to the emergency
35–17. Which o the ollowing women does NOT need to be
department with a complaint o increasing abdominal
re erred to a gynecologic oncologist?
distention, constipation, and weight loss. Abdomino-
a. A 35-year-old woman with a complex 7-cm pelvic computed tomography reveals a 10-cm right
adnexal mass and a cancer antigen 125 (CA125) adnexal mass and retroperitoneal lymphadenopathy.
level o 75 U/mL Her cancer antigen 125 (CA125) level is 45 U/mL,
b. A 60-year-old woman with a complex 7-cm beta-human chorionic gonadotropin level is < 5 IU/mL,
adnexal mass and a CA125 level o 75 U/mL alpha- etoprotein (AFP) and lactate dehydrogenase
c. A 35-year-old woman with a complex 7-cm adnexal (LDH) levels are normal, and her serum calcium level
mass, ascites, and a CA125 level o 75 U/mL is 15 mg/dL. What is the most likely diagnosis?
d. A 40-year-old with a complex 7-cm adnexal mass, a. Lymphoma
a CA125 level o 275 U/mL, and a liver lesion seen b. Primary hyperparathyroidism
during abdominopelvic computed tomography c. Small-cell hypercalcemic-type ovarian cancer
d. Malignant germ cell tumor o the ovary with bone
metastasis
Epithelial Ovarian Cancer 233

35–21. Which o the ollowing is NOT a characteristic o 35–24. In the dissection shown here, which arrow correctly
Krukenberg tumors? indicates the cephalad border o paraaortic lymph
a. T ey are almost always bilateral. node dissection typically per ormed or ovarian cancer

C
that is clinically con ned to the ovary?

H
b. T ey usually arise rom primary gastric tumors.

A
c. T ey are usually the only site o metastatic disease.

P
T
d. T ey are composed o mucinous and signet ring

E
R
adenocarcinoma cells.

3
5
35–22. What is the most common method o ovarian cancer
spread?
a. Lymphatic
b. Hematogenous
c. Direct extension
d. umor ex oliation

35–23 A 40-year-old woman with a small right-sided pleural


ef usion undergoes an exploratory laparotomy or a
pelvic mass and elevated cancer antigen 125 (CA125)
level. During abdominal exploration, the nding Used with permission rom Dr. Jaya Lea.
below is seen. What International Federation o
Gynecology and Obstetrics (FIGO) stage is she? a. A
b. B
c. C
d. D

35–25. Approximately what percentage o women with ovar-


ian cancer clinically con ned to the ovaries will be
upstaged by surgery?
a. 1 percent
b. 30 percent
c. 60 percent
d. 90 percent

35–26. Which o the ollowing patients does NOT need


adjuvant chemotherapy or epithelial ovarian cancer
ollowing total abdominal hysterectomy, bilateral
Used with permission rom Dr. John Schorge.
salpingo-oophorectomy, omentectomy, and multiple
peritoneal biopsies?
a. Stage IIIA
a. A 65-year-old woman who underwent the above
b. Stage IIIB
surgeries plus pelvic and paraaortic lymph node dis-
c. Stage IIIC section or a stage IC clear cell carcinoma o the ovary
d. Stage IV b. A 45-year-old woman who underwent the above
surgeries plus pelvic lymph node dissection or a
stage IA, grade 1 endometrioid adenocarcinoma o
the ovary
c. A 55-year-old woman who underwent the above
surgeries plus pelvic and paraaortic lymph node
dissection or a stage IB, grade 1 papillary serous
carcinoma o the ovary
d. A 70-year-old woman who underwent the above
surgeries plus pelvic and paraaortic lymph node
dissection or a stage IC, grade 1 papillary serous
carcinoma o the ovary
234 Gynecologic Oncology

35–27. What is the goal o cytoreductive surgery or 35–31. Which o the ollowing statements regarding mainte-
advanced ovarian cancer? nance chemotherapy or advanced ovarian cancer is
a. Remove all tumors measuring > 1 cm FALSE?
a. Maintenance chemotherapy has not improved
S
b. Remove all tumors measuring > 2 cm
E
overall survival rates.
C
c. Remove all tumors measuring > 0.5 cm
T
b. Improvements in progression- ree survival rates
I
d. Remove all gross disease
O
have been demonstrated.
N
4
35–28. Which o the ollowing is a possible mechanism by c. Agents that have been tested include bevacizumab,
which primary debulking surgery is elt to improve pazopanib, and paclitaxel.
outcomes? d. Patients should undergo second-look surgery prior
a. Enhance tumor chemosensitivity to starting maintenance treatment.
b. Decrease risk o chemoresistance
35–32. Which o the ollowing is NOT a positive prognostic
c. Improve drug delivery to cancer cells a ter removal actor or ovarian cancer?
o necrotic tissue
a. Younger age
d. All o the above
b. Clear cell type
35–29. Patients with which o the ollowing cancers do NOT c. Absence o ascites
require lymphadenectomy? d. Debulking to no gross residual
a. Endometrioid adenocarcinoma o the ovary
clinically con ned to one ovary 35–33. A woman with a history o stage IIIC ovarian cancer
who underwent optimal cytoreductive surgery
b. Serous carcinoma o the ovary, with omental
ollowed by six cycles o intraperitoneal (IP) cisplatin
implants measuring > 2 cm and with bulky aortic
and paclitaxel has a recurrence 4 months a ter comple-
nodes
tion o chemotherapy. She complains o bloating and
c. Serous carcinoma o the ovary, with carcinomatosis decreased appetite. Which o the ollowing is the
that can be debulked to < 5 mm residual disease most appropriate treatment?
and with no grossly enlarged nodes
a. amoxi en
d. None o the above
b. Carboplatin
35–30. Which o the ollowing chemotherapy regimens is c. Gemcitabine
associated with the longest overall survival rate or d. Pegylated liposomal doxorubicin and bevacizumab
women with optimally debulked (< 1 cm residual
disease) stage III epithelial ovarian cancer? 35–34. Which o the ollowing chemotherapy regimens is an
a. Carboplatin alone appropriate treatment or a patient with platinum-
sensitive ovarian cancer?
b. Intraperitoneal (IP) cisplatin and paclitaxel
a. Carboplatin with paclitaxel
c. Intravenous (IV) carboplatin and paclitaxel
b. Carboplatin with gemcitabine
d. IV carboplatin, paclitaxel, and bevacizumab
c. Carboplatin with pegylated liposomal doxorubicin
d. Any o the above
Epithelial Ovarian Cancer 235

Chapter 35 ANSWER KEY


Question Letter Page Question Letter Page

C
number answer cited Header cited number answer cited Header cited

H
A
35–1 b p. 735 Epidemiology and Risk Factors 35–19 c p. 746 Clear Cell Adenocarcinoma

P
T
35–2 c p. 735 Epidemiology and Risk Factors 35–20 c p. 746 Other Histologic Types

E
R
35–3 c p. 735 Epidemiology and Risk Factors 35–21 c p. 747 Secondary Tumors

3
5
35–4 c p. 736 BRCA1 and BRCA2 Genes 35–22 d p. 747 Patterns of Spread
35–5 a p. 737 Genetic Testing 35–23 c p. 750 Table 35-5
35–6 c p. 738 New Biomarkers and 35–24 b p. 748 Surgical Staging
Proteomics 35–25 b p. 749 Postsurgical Management
35–7 b p. 738 Prophylactic Surgery 35–26 c p. 749 Postsurgical Management
35–8 a p. 739 Pathology 35–27 d p. 750 Residual Disease
35–9 a p. 739 Treatment 35–28 d p. 750 Residual Disease
35–10 a p. 739 Treatment 35–29 c p. 751 Surgical Approach to
35–11 b p. 740 Prognosis Cytoreductive Surgery
35–12 b p. 740 Prognosis 35–30 b p. 752 Intraperitoneal Chemotherapy
35–13 b p. 741 Symptoms and Physical 35–31 d p. 752 Management of Patients in
Findings Remission
35–14 d p. 742 Laboratory Testing 35–32 b p. 753 Table 35-9
35–15 d p. 742 Laboratory Testing 35–33 d p. 753 Management of Recurrent
35–16 d p. 742 Imaging Ovarian Cancer
35–17 a p. 742 Role of the Generalist 35–34 d p. 753 Management of Recurrent
Ovarian Cancer
35–18 d p. 744 Serous Tumors
236

CHAPTER 36

Ovarian Germ Cell and Sex Cord-Stromal Tumors

36–1. What approximate percentage o all ovarian cancers 36–6. An 18-year-old woman presents with severe, acute
are germ cell and sex cord-stromal tumors? abdominal pain. She is ound to have a large pelvic
a. 0.1 percent mass, and torsion o an enlarged ovary is suspected.
During the ensuing surgery, rozen section reveals a
b. 1 percent
dysgerminoma. T ere is no gynecologic oncologist at
c. 10 percent your hospital. What is the most appropriate next step?
d. 30 percent

36–2. Women with germ cell tumors are diagnosed most


commonly during which International Federation o
Gynecology and Obstetrics (FIGO) disease stage?
a. I
b. II
c. III
d. IV

36–3. Which o the ollowing invasive cancers is most


commonly diagnosed in women during their 20s
(third decade)?
a. Germ cell tumors
b. Sex cord-stromal tumors
c. Epithelial ovarian cancer
a. Per orm a bilateral salpingo-oophorectomy and
d. All o these occur with equal requency. close
b. Per orm a unilateral salpingo-oophorectomy and
36–4. What is the most common presenting symptom in a
close
woman with a germ cell tumor?
c. Close her abdomen and re er her to a gynecologic
a. Bloating
oncologist
b. Abdominal pain
d. Per orm a unilateral salpingo-oophorectomy,
c. Abdominal distention obtain peritoneal washings, care ully palpate the
d. Menstrual abnormality abdomen or metastases, and per orm biopsies as
indicated
36–5. Which o the ollowing is NOT a tumor marker or
germ cell tumors? 36–7. Which o the ollowing is the most commonly diag-
a. Inhibin nosed ovarian malignancy during pregnancy?
b. Alpha- etoprotein (AFP) a. Dysgerminoma
c. Lactate dehydrogenase (LDH) b. Choriocarcinoma
d. Human chorionic gonadotropin (hCG) c. Immature teratoma
d. Granulosa cell tumor
Ovarian Germ Cell and Sex Cord-Stromal Tumors 237

36–8. A 19-year-old emale nulligravid college student is 36–12. Which o the ollowing malignant germ cell tumors
ound to have a pelvic mass and an elevated lactate has the worst prognosis?
dehydrogenase (LDH) level. She reports that she was a. Dysgerminoma

C
amenorrheic until she started taking combination oral

H
b. Yolk sac tumor
contraceptive pills at age 17. She undergoes surgical

A
c. Immature teratoma

P
exploration, and rozen section analysis o the surgical

T
specimen reveals ovarian dysgerminoma. Which o d. Nongestational choriocarcinoma

E
R
the ollowing is FALSE?

3
36–13. Which o the ollowing is true regarding mixed germ

6
a. Her karyotype is most likely 46,XX.
cell tumors o the ovary?
b. Regardless o stage, she has an excellent prognosis.
a. Dysgerminoma is the most common component.
c. Her other ovary may contain a gonadoblastoma
and should be removed. b. reatment is based on the dysgerminoma
component.
d. She should undergo complete surgical staging, but
her uterus may be preserved. c. Women with a presumed pure dysgerminoma and
an elevated alpha- etoprotein (AFP) level are likely
36–9. What percentage o gonadoblastomas undergoes to have a component o immature teratoma.
malignant trans ormation? d. All o the above
a. 0.1 percent
36–14. What is the most common germ cell malignancy o
b. 10 percent
the ovary?
c. 40 percent
a. Dysgerminoma
d. 80 percent
b. Yolk sac tumor
36–10. Which germ cell tumor is most likely to be bilateral? c. Mature teratoma
a. Dysgerminoma d. Immature teratoma
b. Mature teratoma
36–15. Which o the ollowing characteristics is primarily
c. Immature teratoma used to grade immature teratomas?
d. Embryonal carcinoma a. Amount o solid component present
b. Amount o immature elements present
36–11. T e histologic structure shown in the photomicro-
graph below, a single papilla lined by tumor cells and c. Amount o immature neural tissue present
containing a central vessel, is pathognomonic or d. None o the above
which germ cell tumor?
36–16. An 18-year-old adolescent has a history o a stage IC
grade 3 immature teratoma. She received adjuvant
chemotherapy with bleomycin, etoposide, and cisplatin
(BEP). At her 6-month ollow-up, she is ound to
have a pelvic mass. Which o the ollowing is the
most appropriate management?
a. Chemotherapy
b. Radiation therapy
c. Continued observation
d. Exploratory laparotomy with removal o masses
ound

36–17. What is the most common cancer ound in a mature


teratoma?
a. Struma ovarii
Reproduced with permission rom Ho man BL, Schorge JO, Bradshaw KD, et al (eds):
Williams Gynecology, 3rd ed. New York, McGraw-Hill, 2016. Photo contributor: b. Basal cell carcinoma
Dr. Kelley Carrick. c. Neuroectodermal tumors
a. Dysgerminoma d. Squamous cell carcinoma
b. Yolk sac tumor
c. Immature teratoma
d. Nongestational choriocarcinoma
238 Gynecologic Oncology

36–18. Which o the ollowing patients does NOT need 36–23. Which o the ollowing is true regarding the tumor
adjuvant chemotherapy a ter unilateral salpingo- represented in this photomicrograph?
oophorectomy with surgical staging?
a. Stage IA dysgerminoma
S
E
C
b. Stage IA yolk sac tumor
T
I
c. Stage IA grade 3 immature teratoma
O
N
d. All o the above
4
36–19. What is the most common chemotherapy regimen
used to treat malignant germ cell ovarian cancers?
a. Carboplatin and etoposide
b. Carboplatin and paclitaxel
c. Bleomycin, etoposide, and cisplatin
d. Bleomycin, etoposide, and paclitaxel

36–20. Which o the ollowing is a prognostic actor or


malignant ovarian germ cell cancers?
Reproduced with permission rom Ho man BL, Schorge JO, Bradshaw KD, et al (eds):
a. Stage Williams Gynecology, 3rd ed. New York, McGraw-Hill, 2016. Photo contributor:
Dr. Raheela Ash aq.
b. Histologic subtype
c. Amount o residual tumor a. Inhibin B is o ten elevated.
d. All o the above b. T e tumor can o ten exceed 10 cm in size.
36–21. Which o the ollowing is NOT a protective actor c. T e characteristic microscopic eature is the
against the development o a sex cord-stromal tumor? Call-Exner body.
a. Smoking d. All o the above
b. Obesity 36–24. Which o the ollowing tumors is hormonally active
c. Multiparity and most o ten secretes estrogen?
d. Combination oral contraceptive pills a. T ecoma
b. Fibroma
36–22. What is the most common presenting symptom o a
sex cord-stromal tumor in a prepubescent girl? c. Sertoli-Leydig tumor
a. Hirsutism d. None o the above
b. Abdominal pain
c. Primary amenorrhea
d. Isosexual precocious puberty
Ovarian Germ Cell and Sex Cord-Stromal Tumors 239

36–25. A woman undergoes exploratory laparotomy, total 36–26. A 22-year-old woman has the vulvar f nding demon-
abdominal hysterectomy, and bilateral salpingo- strated below and an 8-cm pelvic mass palpable dur-
oophorectomy or a solid pelvic mass, ascites, and an ing bimanual examination. What is the most likely

C
elevated serum cancer antigen 125 (CA125) level. diagnosis?

H
Her preoperative chest radiograph is shown below.

A
P
Frozen section demonstrates a benign ovarian neoplasm.

T
What is the most likely diagnosis?

E
R
3
6
Reproduced with permission rom Ho man BL, Schorge JO, Scha er JI, et al (eds): Williams
Gynecology, 2nd ed. New York, McGraw-Hill, 2012. Photo contributor: Dr. Ben Li.

a. Sertoli tumor
a. Meigs syndrome b. Swyer syndrome
b. Benign ovarian neoplasm with cirrhosis c. Leydig cell tumor
c. Metastatic cancer with a benign adnexal mass d. Sertoli-Leydig tumor
d. Ovarian cancer with an inaccurate rozen section
diagnosis 36–27. A 30-year-old woman with Peutz-Jeghers syndrome is
ound to have a sex cord-stromal tumor. Which o the
ollowing is FALSE?
a. Her sex cord-stromal tumor is malignant.
b. Her sex cord-stromal tumor is likely bilateral.
c. She probably has hamartomatous polyps in her
colon.
d. She has a 15-percent chance o developing
adenoma malignum.
240 Gynecologic Oncology

36–28. A 35-year-old nulligravida who desires uture ertility 36–31. A 52-year-old woman with a history o granulosa cell
is ound to have a granulosa cell tumor during rozen tumor has a rising inhibin level. Computed tomogra-
section analysis o a right salpingo-oophorectomy phy reveals an isolated pelvic mass. What is the next
(RSO) specimen. What is the appropriate surgical most appropriate treatment?
S
E
management o this patient? a. Surgical resection
C
T
a. RSO alone b. Carboplatin and paclitaxel
I
O
b. RSO, omentectomy, multiple peritoneal biop- c. Medroxyprogesterone acetate
N
sies; consider pelvic and paraaortic lymph node
4
d. Bleomycin-etoposide-cisplatin chemotherapy
dissection
c. RSO, omentectomy, multiple peritoneal biopsies; 36–32. Which o the ollowing is a prognostic actor or sex
consider pelvic and paraaortic lymph node cord-stromal tumors?
dissection and dilatation and curettage o endo-
a. Age
metrial cavity
b. Stage
d. otal abdominal hysterectomy, bilateral salpingo-
oophorectomy, omentectomy, multiple peritoneal c. Amount o residual disease
biopsies; consider pelvic and paraaortic lymph d. All o the above
node dissection
36–33. Which o the ollowing is NOT true o sex cord-
36–29. Routine surveillance or malignant sex cord-stromal stromal tumors diagnosed during pregnancy?
tumors includes all o the ollowing EXCEPT: a. Granulosa cell tumors are most common.
a. Inhibin levels b. Chemotherapy, i indicated, should be given dur-
b. History and physical examination at regular ing pregnancy.
intervals c. A third o these cases are diagnosed incidentally at
c. Routine abdominopelvic computed tomography the time o cesarean delivery.
every 6 months d. Surgery should consist o unilateral salpingo-
d. Imaging as clinically indicated based on physical oophorectomy, omentectomy, and peritoneal
examination f ndings and tumor marker levels biopsies.

36–30. Which o the ollowing is NOT an appropriate adju-


vant treatment or a patient with a stage III granulosa
cell tumor?
a. Whole pelvic radiotherapy
b. Carboplatin and paclitaxel
c. Bleomycin-etoposide-cisplatin chemotherapy
d. All o the above are appropriate treatment options.
144

CHAPTER 21

Physiology of Labor

21–1. Which of the following patients are in labor? 21–5. Which of the following is not a function of the
a. A 23-year-old G1P0 at 38 weeks’ gestation with cervix during pregnancy?
painful contractions and a cervix that remains a. Preventing infection
closed over 2 hours b. Maintenance of competence
b. A 33-year-old G2P1 at 38 weeks’ gestation with c. Extracellular changes for greater compliance
nonpainful contractions and a cervix that changes
d. All of the above
to 3 cm dilated over 2 hours
c. A 33-year-old G2P1 at 38 weeks’ gestation 21–6. Which of the following is a characteristic of the
with one prior cesarean delivery with painful amnion?
contractions and a cervix that remains closed over
a. Provides immunological acceptance
2 hours
b. Enriched with enzymes that inactivate
d. A 33-year-old G2P1 at 38 weeks’ gestation with
uterotonins
one prior cesarean delivery with painful contrac-
tions and a cervix that remains 1 cm dilated over c. Provides all tensile strength to resist rupture of
2 hours membranes
d. All of the above
21–2. Which of the following describes current theories of
labor? 21–7. Which of the following supports the theory that
a. Synthesis of factors that induce parturition progesterone inhibits parturition?
b. Fetal source for commencement of parturition a. Progesterone withdrawal precedes labor
c. Functional loss of pregnancy maintenance factors b. Providing progesterone to some species delays
labor
d. All of the above
c. Administration of a progesterone-receptor
21–3. All except which of the following may result from antagonist promotes labor
abnormal parturition? d. All of the above
a. Dystocia
21–8. Of the four phases of parturition, phase 2 is
b. Preterm labor
characterized by which of the following?
c. Postterm pregnancy
a. Uterine activation, cervical ripening
d. Premature rupture of membranes
b. Uterine quiescence, cervical softening
21–4. Which of the following qualities of smooth muscles c. Uterine contraction, cervical dilatation
is advantageous for contractions and fetal delivery? d. Uterine involution, cervical remodeling
a. Greater shortening with contractions
21–9. Which phase of parturition corresponds to the
b. Forces are not exerted in multiple directions
clinical stages of labor?
c. Smooth muscle is organized in the same manner
a. Phase 1
as striated muscle
d. Unidirectional force generation in the fundus and b. Phase 2
lower uterine segment c. Phase 3
d. Phase 4
Physiology of Labor 145

21–10. Which of the following does not help achieve uterine 21–15. A 23-year-old primigravida presents for induction of
quiescence? labor at 41 weeks’ gestation. Her cervix is unfavor-
a. Uterotonin degradation able with a Bishop score of 4. You proceed with cer-

CHAPTER 21
vical ripening using the agent depicted below. This
b. Increased intracellular Ca2+
medication belongs to which class of agents?
c. Ion channel regulation of cell membrane potential
d. Activation of the uterine endoplasmic reticulum
stress response

21–11. During which stage of labor is the fetus delivered?


a. Stage 1
b. Stage 2
c. Stage 3
d. Stage 4

21–12. Cervical softening in phase 1 of parturition results in


part from which of the following?
a. Stromal atrophy
b. Increased stromal vascularity
c. Increased collagen monomer cross-linking
d. All of the above a. Oxytocin
b. Beta mimetics
21–13. Contraction-associated protein (CAP) within the
uterine smooth muscle prepares it to contract during c. Prostaglandins
labor. CAP concentrations increase during phase 2 d. Nonsteroidal inflammatory drugs
of parturition and include all except which of the
following proteins? 21–16. What is the primary source of corticotropin-releasing
a. Connexin 43 hormone in pregnancy?
b. Oxytocin receptor a. Placental
c. Progesterone A receptor b. Fetal adrenal
d. Prostaglandin F receptor c. Fetal hypothalamus
d. Maternal hypothalamus
21–14. Compared to the uterine body, the cervix has a
significantly lower percentage of which of the 21–17. Which of the following are fetal contributors to the
following? initiation of parturition?
a. Collagen a. Surfactant protein A
b. Proteoglycans b. Corticotropin-releasing hormone
c. Smooth muscle c. Senescent-associated secretory phenotype
d. Glycosaminoglycans d. All of the above
146 Labor

21–18. A fetus with a neural tube defect is shown below 21–21. During cesarean delivery, the hysterotomy incision
with the fetal orbits labeled O. Which of the is ideally made in the lower uterine segment, shown
following abnormalities of normal parturition has here prior to bladder flap creation. Which of the
been associated with this anomaly? following aids in development of this uterine segment
SECTION 7

during phase 3 of parturition?

a. Preterm labor
b. Prolonged gestation Used with permission from Dr. Sarah White.
c. Uterine tachysystole
d. None of the above a. Progressive thickening of the upper uterine segment
with labor progression
21–19. Which of the following are plausible causes of b. Smooth muscle cell fibers of the fundus relax to
uterine contraction pain?
their original length after each contraction
a. Myometrial hypoxia
c. Smooth muscle cell fibers of the lower uterine
b. Uterine peritoneum stretch segment relax to their original length after each
c. Compression of nerve ganglia in the cervix contraction
d. All of the above d. All of the above
21–20. A 30-year-old primigravida presents to your office 21–22. After the cervix is fully dilated, what is the most
at 39 weeks’ gestation. She is exhausted and ready important force in fetal expulsion?
to be delivered but does not want to be induced.
You suggest membrane stripping and inform her a. Uterine contractions
this will lead to a rise in which of the following b. Intraabdominal pressure
prostaglandins? c. Fetal head descending through the pelvis
a. I2 d. None of the above
b. E2
c. H
d. F2a
Physiology of Labor 147

21–23. As a result of contraction forces, the cervix effaces 21–26. What is the initial step that leads to placental
and dilates by mechanisms that include all except separation following delivery of the infant?
which of the following? a. Uterine contractions

CHAPTER 21
a. Contraction forces create lateral pull against the b. Tension pulls it away from the implantation site
cervix to open its canal. c. Formation of a hematoma behind the placenta
b. Contraction forces are transferred directly through with uterine involution
the presenting part to the cervix to dilate its canal. d. None of the above
c. Contraction forces pull smooth muscle fibers at
the internal os up into the adjacent upper uterine 21–27. In which mechanism of placental delivery does the
segment to efface the cervix. placenta leave the body before the retroplacental
hematoma?
d. Contraction forces are translated into hydrostatic
pressure within the amnionic sac, which presses a. Bandl mechanism
against the cervix to dilate the cervical canal. b. Duncan mechanism
c. Schultze mechanism
21–24. In which of the following women would you expect
d. Chadwick mechanism
rapid descent of the fetal head?
a. A 23-year-old G1P0 at 40 weeks’ gestation 21–28. Which of the following is accurate regarding
b. A 23-year-old G3P2 at 40 weeks’ gestation oxytocin?
c. A 33-year-old G1P0 at 28 weeks’ gestation a. Nanopeptide
d. A 33-year-old G1P0 at 37 weeks’ gestation b. Synthesized as a prohormone
c. Myometrial receptor numbers increase during
21–25. Which of the following is the most important phase 2
component of the pelvic floor?
d. All of the above
a. Vaginal wall
b. Piriformis muscle 21–29. Indomethacin, a nonsteroidal antiinflammatory drug
c. Coccygeus muscle (NSAID), has some tocolytic actions. As a group,
NSAIDs target which enzyme in prostaglandin
d. Levator ani muscle production?
a. Phospholipase A2
b. Cyclooxygenase-1
c. Prostaglandin isomerase
d. Prostaglandin dehydrogenase
148 Labor

21–30. A 34-year-old G1P0 is undergoing induction of


labor with misoprostol for cervical ripening. The
nurse calls you for fetal heart rate decelerations, and
you notice she is having prolonged contractions,
SECTION 7

as shown below. You administer terbutaline, a


β2 adrenergic receptor agonist, and initiate which
of the following cellular responses to cause uterine
relaxation?

a. Increased intracellular Ca2+ levels


b. Increased extracellular Mg2+ levels
c. Increased cyclic adenosine monophosphate levels
d. Decreased cyclic guanosine monophosphate levels

21–31. A 34-year-old woman is scheduled to return for her


postpartum visit 6 weeks after delivery. Given that
you will discuss birth control options at that visit,
what may be the optimal timing of that visit?
a. 6 weeks based on tradition
b. 4 weeks if she is breastfeeding
c. 6 weeks as long as she is breastfeeding
d. 4 weeks because ovulation returns 4–6 weeks
after birth

21–32. Which of the following uterotonins plays a role in


phase 3 of parturition?
a. Endothelin-1
b. Angiotensin II
c. Prostaglandins
d. All of the above
150

CHAPTER 22

Normal Labor

22–1. What is the most common fetal lie? 22–4. In the photo below, the neck is only partly flexed
a. Oblique and the anterior fontanel is presenting. What is the
fetal attitude?
b. Unstable
c. Transverse
d. Longitudinal

22–2. When the fetus lies with the long axis transversely,
what is the presenting part?
a. Knee
b. Head
c. Shoulder
d. Buttocks

22–3. What is the fetal attitude in the photo below?

Reproduced with permission from Cunningham FG, Leveno KJ, Bloom SL, et al (eds): Normal
labor. In Williams Obstetrics, 25th ed. New York, McGraw-Hill, 2018, Figure 22-1B.

a. Face
b. Brow
c. Occiput
d. Sinciput

22–5. What composes the cephalic pole?


a. Head
b. Head and upper extremities
c. Head, upper extremities, and trunk
Reproduced with permission from Cunningham FG, Leveno KJ, Bloom SL, et al (eds): Normal
labor. In Williams Obstetrics, 25th ed. New York, McGraw-Hill, 2018, Figure 22-1D. d. Head, upper extremities, trunk, and lower
extremities
a. Face
b. Brow
c. Vertex
d. Sinciput
Normal Labor 151

22–6. Why is there a high incidence of breech among 22–9. What is the fetal position in the drawing below?
hydrocephalic fetuses?
a. Increased intracranial pressure

CHAPTER 22
b. Podalic pole needs more room
c. Cephalic pole needs more room
d. Because the fetal head is small, it does not have
the force to turn

22–7. What is the incidence of breech at term?


a. 1%
b. 3%
c. 5%
d. 10%

22–8. What is the fetal position in the drawing below? Reproduced with permission from Cunningham FG, Leveno KJ, Bloom SL, et al (eds): Normal
labor. In Williams Obstetrics, 25th ed. New York, McGraw-Hill, 2018, Figure 22-2B.

a. Left occiput anterior


b. Left occiput posterior
c. Right occiput anterior
d. Right occiput posterior

22–10. What is the fetal position in the drawing below?

Reproduced with permission from Cunningham FG, Leveno KJ, Bloom SL, et al (eds): Normal
labor. In Williams Obstetrics, 25th ed. New York, McGraw-Hill, 2018, Figure 22-2A.

a. Left occiput anterior (LOA)


b. Left occiput posterior (LOP)
c. Right occiput anterior (ROA)
d. Right occiput posterior (ROP)

Reproduced with permission from Cunningham FG, Leveno KJ, Bloom SL, et al (eds):
Normal labor. In Williams Obstetrics, 25th ed. New York, McGraw-Hill, 2018, Figure
22-3A.

a. Left occiput anterior


b. Left occiput posterior
c. Right occiput anterior
d. Right occiput posterior
152 Labor

22–11. What is the fetal position in the drawing below? 22–14. What is the fetal position in the drawing below?
SECTION 7

Reproduced with permission from Cunningham FG, Leveno KJ, Bloom SL, et al (eds): Normal
labor. In Williams Obstetrics, 25th ed. New York, McGraw-Hill, 2018, Figure 22-3B.

a. Left occiput anterior


b. Right occiput anterior
c. Left occiput transverse
d. Right occiput transverse

22–12. Which of the following is least detrimental to the Reproduced with permission from Cunningham FG, Leveno KJ, Bloom SL, et al (eds):
performance of Leopold maneuvers? Normal labor. In Williams Obstetrics, 25th ed. New York, McGraw-Hill, 2018, Figure 22-6.
a. Obesity
b. Macrosomia a. Left sacrum anterior
c. Polyhydramnios b. Left sacrum posterior
d. Anterior placenta c. Right sacrum anterior
d. Right sacrum posterior
22–13. What is the fetal position in the drawing below?
22–15. Which Leopold maneuver determines degree of fetal
descent?
a. 1
b. 2
c. 3
d. 4

22–16. The term engagement refers to which of the


following?
a. Top of the head passing through the pelvic inlet
b. Top of the head passing through the pelvic outlet
c. Biparietal diameter passing through the pelvic
inlet
Reproduced with permission from Cunningham FG, Leveno KJ, Bloom SL, et al (eds): d. Biparietal diameter passing through the pelvic
Normal labor. In Williams Obstetrics, 25th ed. New York, McGraw-Hill, 2018, Figure 22-5. outlet

a. Left mento-anterior
b. Left mento-posterior
c. Right mento-anterior
d. Right mento-posterior
Normal Labor 153

22–17. What is the term for lateral deflection of the sagittal 22–19. Which of the following statements about the photo
suture toward the sacral promontory as depicted in provided below is false?
the drawing below?

CHAPTER 22
a. This is the product of rapid labor.
b. This can make it difficult to assess the sutures and
fontanels.
c. Most of the time, these changes to the fetal head
shape will resolve within a week of delivery.
d. These changes to the fetal head can facilitate
Reproduced with permission from Cunningham FG, Leveno KJ, Bloom SL, et al (eds):
a vaginal delivery in women with a contracted
Normal labor. In Williams Obstetrics, 25th ed. New York, McGraw-Hill, 2018, Figure
22-12. pelvis.

a. Normal asynclitism 22–20. During which portion of labor does the cervix dilate
very little but the connective tissue component
b. Oblique asynclitism
change considerably?
c. Anterior asynclitism
a. Active phase
d. Posterior asynclitism b. Acceleration phase
22–18. Which of the following statements about the occiput c. Dilational division
posterior position is true? d. Preparatory division
a. They are more commonly seen in the setting of a
posterior placenta. 22–21. Which of the following has been associated with
prolonging labor?
b. Epidural anesthesia may predispose to incomplete
rotation to the occiput anterior position. a. Maternal fear
c. The fetus is less likely to turn to the occiput b. Maternal obesity
anterior position if small for gestational age. c. Epidural analgesia
d. Extension of the fetal head improves the likelihood d. All of the above
that it will turn from occiput posterior to occiput
anterior. 22–22. Which stage of labor is defined as the period of time
from when the patient reaches complete cervical
dilation through delivery of the fetus?
a. First stage
b. Second stage
c. Third stage
d. Fourth stage
154 Labor

22–23. Which of the following has not been associated with 22–28. A 28-year-old primigravida at term presents in labor.
a prolonged second stage of labor? She is very uncomfortable. You go to examine the
a. Sedation patient, and you note that the fetal head is visible at
the introitus. What is the fetal station?
SECTION 7

b. Macrosomia
a. –5
c. Maternal obesity
b. –2
d. Contracted maternal pelvis
c. +2
22–24. What is the recommended nurse: patient ratio for a d. +5
low-risk laboring patient?
a. 1:1 22–29. A 39-year-old multigravida presents for induction at
38 weeks’ gestation. The patient has a history of two
b. 1:2
prior vaginal deliveries, cholestasis, pregestational
c. 1:3 diabetes, chronic hypertension requiring two
d. 2:1 medications, and a history of prior abruption in the
setting of preeclampsia with severe features. You
22–25. A 25-year-old primigravida presents at 38 weeks’ place the patient on continuous fetal monitoring.
gestation complaining of contractions every How often should the tracing be evaluated?
5–7 minutes. On exam, she is 1 cm dilated, 50% a. Every 15 minutes in the first and second stage.
effaced, and the fetal head is at –1 station. Her
b. Every 15 minutes in the first stage and every
membranes are intact and there is no vaginal bleeding
5 minutes in the second stage.
seen. After 2 hours of observation, the fetal tracing is
category I. The contractions have spaced out to every c. Every 15 minutes in the first stage and every
15 minutes. The patient’s cervix remains unchanged. 30 minutes in the second stage.
Her vitals are within normal limits, and she has no d. Every 30 minutes in the first stage and every
medical problems. What is the best management 15 minutes in the second stage.
plan for this patient?
a. Amniotomy induction 22–30. A 30-year-old multigravida presents in active labor
at term. She progresses from 4 cm to complete in
b. Continued monitoring
3 hours. She has the baby spontaneously 1 hour later.
c. Oxytocin augmentation She has a third-degree laceration that is repaired.
d. Discharge home with labor precautions and On postpartum day 1, she is found to have urinary
instructions to follow up with her doctor retention. What is her risk factor for urinary retention?
a. Age
22–26. A 17-year-old primigravida at 38 weeks’ gestation
b. Rapid labor
presents complaining of leakage of fluid. When
performing nitrazine testing to assess for the presence c. Multiparity
of amniotic fluid in the vagina, which of the following d. Perineal laceration
would you expect to give you a false-negative result?
a. Blood 22–31. A 22-year-old primigravida presents in active labor at
6 cm dilation. Her membranes are intact, the fetus
b. Semen
is cephalic, and the fetal tracing is category I. Two
c. Scant fluid hours later, she is still 4 cm dilated. Her contractions
d. Bacterial vaginosis are every 7 minutes. What is the best next step in
labor management?
22–27. When the cervix becomes as thin as the adjacent a. Amniotomy
lower uterine segment, what is the effacement?
b. Continued observation
a. 0%
c. Amniotomy and oxytocin
b. 25%
d. Cesarean section for failure to progress
c. 50%
d. 100% 22–32. Should the patient in Question 22–31 achieve
adequate uterine contractions, what is a reasonable
minimum rate of cervical dilation?
a. 1 cm in 2 hours
b. 1–2 cm per hour
c. 2–3 cm per hour
d. 3–4 cm per hour
Normal Labor 155

CHAPTER 22 ANSWER KEY

Question Letter Page

CHAPTER 22
number answer cited Header cited
22–1 d p. 422 Fetal Lie
22–2 c p. 422 Fetal Presentation
22–3 a p. 422 Figure 22-1
22–4 d p. 422 Figure 22-1
22–5 a p. 422 Cephalic Presentation
22–6 c p. 422 Cephalic Presentation
22–7 b p. 422 Breech Presentation
22–8 a p. 423 Figure 22-2
22–9 b p. 423 Figure 22-2
22–10 d p. 424 Figure 22-3
22–11 d p. 424 Figure 22-3
22–12 b p. 424 Leopold Maneuvers
22–13 a p. 425 Figure 22-5
22–14 b p. 425 Figure 22-6
22–15 d p. 426 Leopold Maneuvers
22–16 c p. 427 Engagement
22–17 c p. 429 Figure 22-12
22–18 b p. 431 Occiput Posterior Presentation
22–19 a p. 431 Figure 22-16
22–20 d p. 432 First Stage of Labor
22–21 d p. 433 Active Phase
22–22 b p. 434 Second Stage of Labor
22–23 c p. 434 Second Stage of Labor
22–24 b p. 434 Table 22-2
22–25 d p. 435 Identification of Labor
22–26 c p. 435 Ruptured Membranes
22–27 d p. 435 Cervical Assessment
22–28 d p. 436 Cervical Assessment
22–29 b p. 436 Intrapartum Fetal Monitoring
22–30 d p. 438 Urinary Bladder Function
22–31 a p. 439 Labor Management Protocols
22–32 b p. 439 Labor Management Protocols
156

CHAPTER 23

Abnormal Labor

23–1. Which of the following may be responsible for 23–6. Incoordinate uterine dysfunction refers to which of
dystocia in labor? the following?
a. Bony pelvis abnormalities a. The pressure gradient is distorted
b. Inadequate expulsive forces b. Basal uterine tone is appreciably elevated
c. Soft tissue abnormalities of the reproductive tract c. A more forceful contraction of the uterine
d. All of the above midsegment than the fundus
d. All of the above
23–2. The majority of labor dystocia is due to which of the
following? 23–7. The Montevideo group concluded that which of
a. Asynclitism the following was the lowest contraction pressure
necessary to cause cervical dilation?
b. Fetal macrosomia
a. 15 mmHg
c. Childhood rickets
b. 45 mmHg
d. Premature rupture of membranes
c. 60 mmHg
23–3. Which of these factors influences the progress of d. 180 mmHg
labor?
a. Prominent coccyx 23–8. When is the latent phase considered prolonged?
b. Pubic symphyseal separation a. 14 hours in nullipara
c. Forward pressure by the leading fetal part b. 20 hours in mulitpara
d. Cervical dilation at the time of rupture of c. 10 hours in multipara
membranes d. None of the above

23–4. Since the 1960s, what significant advancement has 23–9. Which of the following is a protraction disorder?
been made in treating dysfunctional labor? a. Prolonged active phase
a. Early artificial rupture of membranes b. Protracted latent phase
b. Use of prostaglandins for cervical ripening c. Prolonged deceleration phase
c. Movement toward vacuum-assisted delivery in d. Protracted active phase dilation
the setting of midpelvic arrest
d. Realization that prolonged labor may lead to 23–10. The World Health Organization has proposed a
increased perinatal and maternal morbidity labor management partograph in which protraction
is defined as ___ cm/hr cervical dilation for a
23–5. Uterine contractions in normal labor are minimum of ___ hours.
characterized by which of the following? a. 1, 3
a. Fundal dominance b. 2, 4
b. Contractions migrate from caudad to cephalad c. 1, 4
c. Pressure of an average spontaneous contraction is d. 2, 3
15 mmHg
d. Force of contractions is greatest at the lower
uterine segment
Abnormal Labor 157

23–11. What are the Montevideo units in the tracing below?

CHAPTER 23
a. 50
b. 109 23–14. Which of the following statements is true comparing
c. 142 the Zhang versus Friedman curve?
d. 199 10
Friedman (1955)
Zhang (2002)
23–12. What is the threshold for adequate uterine 8
contractions?
Cervical dilation (cm)

a. 180 Montevideo units


6
b. 200 Montevideo units
c. Contractions every 2 minutes
4
d. Five contractions in a 10-minute period

23–13. Which of the following statements comes from the 2


Obstetric Care Consensus Committee of 2016?
a. The threshold for active labor is 4 cm.
0
b. A protraction disorder is an indication for a cesar- 1 2 3 4 5 6 7 8 9 10 11 12 13
ean delivery. Duration of labor (hours)

c. A prolonged latent phase is not an indication for Reproduced with permission from Cunningham FG, Leveno KJ, Bloom SL, et al (eds):
Abnormal labor. In Williams Obstetrics, 25th ed. New York, McGraw-Hill, 2018, Figure 23-3.
cesarean delivery.
d. Active phase arrest cannot be considered until
a. The Friedman curve begins to flatten at 3–4 cm.
the patient is 4 cm dilated and membranes are
ruptured. b. In the Zhang curve the active phase of labor
begins at 6 cm.
c. In the Friedman curve the active phase of labor
begins at 6 cm.
d. All of the above

23–15. Which of the following accurately compares neur-


axial and intravenous analgesia?
a. Time to delivery is lower with neuraxial analgesia.
b. Oxytocin augmentation was higher in the intrave-
nous analgesia group.
c. Cesarean delivery rates are greater in the intrave-
nous analgesia group.
d. None of the above
158 Labor

23–16. A 26-year-old multigravida at 40 weeks’ gestation 23–21. Which of the following is false regarding premature
presents in labor at 6 cm. On repeat exam 4 hours rupture of membranes at term?
later, she is still 6 cm. What other piece of information a. Premature rupture of membranes complicates 8%
would you like to help determine your next step?
SECTION 7

of pregnancies.
a. If she has any analgesia b. Oxytocin induction leads to lower rates of
b. The estimated fetal weight chorioamnionitis.
c. If her membranes are ruptured c. Prophylactic antibiotics do not significantly lower
d. If her contractions are adequate the rates of chorioamnionitis.
d. The cesarean delivery rate is lower in women who
23–17. Regarding the patient in Question 23–16, you are induced rather than managed expectantly.
determine she has not had any analgesia, she
is unruptured, and her contractions total 100 23–22. Precipitous labor may result from which of the
Montevideo units. What is the next best course of following?
action? a. Absence of painful sensations
a. Place neuraxial anesthesia b. Abnormally strong uterine contractions
b. Augmentation with oxytocin c. Abnormally low resistance of the soft parts of the
c. Provide intravenous analgesia birth canal
d. Proceed with cesarean delivery d. All of the above

23–18. Which of the following statements regarding the 23–23. A 34-year-old multiparous woman presents in active
second stage of labor is false? labor at 38 weeks’ gestation. She reports onset of
a. A prolonged first stage of labor correlates with a contractions 1 hour ago and is 8 cm dilated on
longer second stage. admission. She delivers an infant 1 hour later. What
maternal risk is associated with precipitous labor?
b. No adverse maternal or neonatal outcomes are
linked to delayed pushing. a. Nerve injury
c. The maximum length of time all women should b. Uterine atony
spend in the second stage of labor is 4 hours. c. Retained placenta
d. Multiparous women should be allowed to push d. Bladder dysfunction
for 2 hours prior to diagnosing a second-stage
labor arrest. 23–24. The infant delivered in Question 23–23 is at
increased risk for which of the following?
23–19. In laboring nulliparous women, fetal station above 0 a. Sepsis
at the time of admission is associated with which of
the following? b. Brachial plexus palsy
a. A 25% cesarean rate c. Meconium aspiration
b. A 50% cesarean rate d. Continuous positive airway pressure at birth
c. An 86% cesarean rate 23–25. In obstetrics, which of the following defines a
d. A higher cesarean rate than if the head is engaged contracted pelvic inlet?
a. A transverse diameter <12 cm
23–20. Which of the following statements is true concerning
chorioamnionitis in labor? b. An obstetric conjugate <10 cm
a. Chorioamnionitis is associated with prolonged c. A diagonal conjugate <11.5 cm
labor. d. An anteroposterior diameter <12 cm
b. Chorioamnionitis in early labor is a consequence
of dysfunctional labor. 23–26. Which of the following statements is true regarding
contraction of the midpelvis?
c. Chorioamnionitis in the late stage of labor
increases the risk for cesarean delivery. a. It is less common than inlet contraction.
d. All of the above b. It causes transverse arrest of the fetal head.
c. It is suspected when the interspinous diameter is
<11 cm.
d. It can be inferred when there are parallel vaginal
sidewalls.
Abnormal Labor 159

23–27. Which interischial tuberous diameter measurement a. Most cesarean deliveries occurred in women with
serves as the threshold to define pelvic outlet macrosomic infants.
contraction? b. Fetal size appears to be the significant contributor

CHAPTER 23
a. 7 cm to failed forceps deliveries.
b. 8 cm c. Malposition of the fetal head is a contributing
c. 9 cm factor to obstruction through the birth canal.
d. 10 cm d. None of the above

23–28. A 29-year-old primigravida presents for prenatal 23–31. Which of the following statements is true concerning
care at 10 weeks’ gestation. She reports a history of a face mentum posterior presentations?
pelvic fracture 1 year ago. Which of the following is a. Most will convert to mentum anterior.
the most likely etiology of her fracture? b. Fetal macrosomia is the biggest risk factor.
a. Fall c. Forceps should be applied when the station is +2.
b. Assault d. A persistent mentum posterior presentation is
c. Gunshot wound never deliverable.
d. Automobile collision
23–32. This image illustrates which fetal presentation?
23–29. The patient in Question 23–28 reports she was told
she could never have a vaginal delivery. Which of the
following is true?
a. Healing requires 8–12 weeks, so she can have a
vaginal delivery.
b. If she has hardware in her pelvis she cannot have
a vaginal delivery.
c. Hyperflexion of her hips during pushing may
re-damage her pelvis.
d. If imaging shows any malalignment, she will have
to undergo cesarean delivery.

23–30. The graphic below demonstrates the prevalence


of cesarean deliveries after a failed forceps delivery
attempt plotted against fetal birthweight. Which of
the following is true regarding this data?

25

20
Percent

15
Reproduced with permission from Cunningham FG, Leveno KJ, Bloom SL, et al (eds):
10 Abnormal labor. In Williams Obstetrics, 24th ed. New York, McGraw-Hill, 2014, Figure 23-6.

5 a. Face presentation
b. Brow presentation
0 c. Occiput presentation
9

9
9

d. Anteroposterior presentation
79

09

99

39

89
39

69

59
–2

–3

–3

–4

–4
–3

–3

–4
00

00

00

00
00

00

00

00
25

28

37

40
31

34

43

46

Birthweight
Reproduced with permission from Cunningham FG, Leveno KJ, Bloom SL, et al (eds):
Abnormal labor. In Williams Obstetrics, 24th ed. New York, McGraw-Hill, 2014, Figure 23-5.
160 Labor

23–33. Which statement below is accurate concerning the 23–35. Which of the following are complications of labor
following presentation? dystocia?
a. Infection
SECTION 7

b. Uterine rupture
c. Hysterotomy incision extensions
d. All of the above

23–36. A 24-year-old multigravida at 41 weeks’ gestation


with a body mass index of 38 kg/m2 progresses
through the first stage of labor and becomes
complete at +2 station. She pushes for about 3 hours
with neuraxial anesthesia. She delivers a 4200-gram
infant with the assistance of forceps. The next day
your patient reports lower extremity weakness with
foot drop and pain. What is the most likely cause?
a. Obesity
b. Prolonged second stage
Reproduced with permission from Cunningham FG, Leveno KJ, Bloom SL, et al (eds):
Abnormal labor. In Williams Obstetrics, 25th ed. New York, McGraw-Hill, 2018, Figure 23-10.
c. Forceps-assisted vaginal delivery
d. Complication of neuraxial anesthesia
a. There is an increased risk of prolapsed cord.
b. This malpresentation will most likely be relieved
with classical cesarean.
c. This represents a complication of persistent
transverse presentation in labor.
d. All of the above

23–34. Which statement concerning the following labor


complication is true?

Used with permission from Dr. Elizabeth Mosier.

a. This is a composite presentation.


b. The forearm is frequently injured in this
presentation.
c. In most cases the infant will deliver vaginally
without difficulty.
d. The infant will sometimes withdraw its hand if
“pinched” by the delivering obstetrician.
162

CHAPTER 24

Intrapartum Assessment

24–1. Which portion of the fetal electrocardiogram is most 24–5. The NICHD Workshop standardized fetal heart
reliably detected? rate interpretation in 2008. Which of the following
a. P wave factors makes interpreting fetal heart rate tracings
difficult?
b. T wave
a. Paper speed at 3 cm/min
c. R-wave peaks
b. Interobserver agreement is low
d. QRS complex
c. Use of standardized terminology
24–2. Which of the following is true regarding fetal heart d. Use of external monitors instead of internal
monitoring through a fetal scalp electrode? monitors
a. Heart rate is determined via time between R waves.
24–6. Which of the following are descriptive characteristics
b. The maternal heart rate is also detected, but
of the fetal baseline heart rate?
masked.
a. Rate
c. In the setting of an intrauterine fetal demise, the
maternal heart rate will be recorded. b. Variability
b. All of the above c. Distinct patterns
d. All of the above
24–3. Current fetal monitors can detect which of the
following? 24–7. How is the fetal heart rate baseline determined?
a. Maternal heart rate a. Is determined over a 10-minute period
b. Maternal pulse oximetry b. Increases with advancing gestational age
c. Separate heart rates for a twin gestation c. Is rounded to increments of 10 beats/minute
d. All of the above d. Minimum duration needed in a 10-minute period
is 5 minutes
24–4. Which of the following statements is true regarding
the fetal heart rate pattern shown below?

a. Does not require gel to obtain


b. Is not more difficult in obese women
c. Can be placed anywhere on the maternal abdomen
d. Uses ultrasound waves to detect the fetal heart rate
Intrapartum Assessment 163

24–8. What fetal heart rates defines fetal bradycardia and


fetal tachycardia, respectively?
a. <100 and >170

CHAPTER 24
b. <110 and >170
c. <110 and >160
d. <120 and >160

24–9. A 24-year-old primigravida at 41 weeks’ gestation is


undergoing induction of labor. She is currently on
intravenous oxytocin and reports abdominal pain
that is not relieved by her epidural. The nurse calls
you for the fetal heart rate pattern seen below. What
is the likely etiology?

Used with permission from Dr. Travis Sims.

a. Fetal heart block


b. Rapid fetal descent
c. Placental abruption
d. Maternal hypothermia

24–10. What is the most common cause of fetal tachycardia?


a. Maternal pain
b. Chorioamnionitis
c. Maternal terbutaline
d. Fetal supraventricular tachycardia

24–11. What is the single most reliable sign of fetal


compromise?
a. Decelerations
b. Fetal tachycardia
c. Change in baseline
d. Reduced variability
164 Labor

24–12. A 28-year-old primigravida is undergoing induction


of labor. She experienced rupture of membranes
20 hours ago, and her cervical exam indicates she is
7 cm dilated. She receives intravenous butorphanol
SECTION 7

for pain. Which of the following is the etiology of


the fetal heart rate variability seen below?

a. Active labor
b. Butorphanol
c. Oligohydramnios
d. Chorioamnionitis

24–13. Which of the following statements concerning fetal 24–16. All except which of these statements is true regarding
arrhythmias is true? fetal heart rate accelerations?
a. Are usually tachyarrhythmias a. Represent intact neurohormonal cardiovascular
b. Are an indication for cesarean delivery control mechanisms
c. May hinder interpretation of the fetal heart rate b. Occur with fetal movement, scalp stimulation,
tracings and acoustic stimulation
d. All of the above c. Is considered prolonged if it is ≥2 minutes but
<10 minutes in duration
24–14. Sinusoidal fetal heart rate patterns are best described d. At all gestational ages the peak is ≥15 beats per
by which of the following statements? minute above the baseline
a. Can be due to fetal sucking
24–17. A deceleration that begins after the peak of the
b. Observed with mild fetal anemia contraction and returns to baseline after the
c. Frequency cycle of 2–5 cycles/minute contraction ends is which of the following?
d. Baseline heart rate must be <120 beats/minute a. Late
b. Early
24–15. Which of the following defines fetal heart rate
decelerations as recurrent? c. Variable
a. Occurring with ≥50% of contractions in a d. Prolonged
10-minute period.
b. Occurring with ≥50% of contractions in a
20-minute period.
c. Occurring with >50% of contractions in a
10-minute period.
d. Occurring with >50% of contractions in a
20-minute period.
Intrapartum Assessment 165

24–18. Which type of deceleration is pictured below?

CHAPTER 24
a. Late
b. Early
c. Variable
d. Prolonged

24–19. According to the American College of Obstetricians 24–22. Which of the following statements is true regarding
and Gynecologists, which of the following variable fetal heart rates in the second stage of labor?
decelerations are abnormal? a. Decelerations in the second stage of labor are
a. Deep variables ubiquitous.
b. Variables lasting >30 seconds b. If there is absent variability for longer than
c. Variables with absent variability 4 minutes, acidemia can be predicted.
d. Variables that take >30 seconds to reach their nadir c. As the number of decelerations <30 beats per
minute increases, the 5-minute Apgar score
24–20. Which physiologic event results in initial decreases.
compensatory rise in the fetal heart rate during d. All of the above
a variable deceleration?
a. Acute increase in uterine tone 24–23. What effect do admission fetal monitoring programs
have on low-risk women?
b. Occlusion of the umbilical vein
a. Improve neonatal outcomes
c. Occlusion of the umbilical artery
b. Decrease the number of interventions
d. Occlusion of all umbilical vessels
c. Increase the number of cesarean deliveries
24–21. A 25-year-old primigravida at 38 weeks’ gestation is d. Decrease the number of operative deliveries
in the second stage of labor. Her fetal heart tracing is
depicted below. Which of the following characteristics
of the tracing increases her risk for fetal compromise?

Used with permission from Dr. Noelle Zavala.

a. Normal baseline
b. Fetal tachycardia
c. Absent variability
d. All of the above
166 Labor

24–24. Which of the following is true for fetal scalp 24–29. Current guidelines from the American College
stimulation? of Obstetricians and Gynecologists recommend
a. Accelerations predict a pH >7.2 which of following regarding intrapartum neonatal
suctioning in the presence of meconium?
SECTION 7

b. Cannot be used in preterm infants


a. Should be routinely performed in all infants
c. Lack of an acceleration predicts a pH <7.2
b. A credentialed team in neonatal resuscitation
d. Cannot be used as a substitute for fetal scalp should be available
blood sampling
c. Routine intrapartum suctioning decreases the
incidence of meconium aspiration syndrome
24–25. The use of fetal pulse oximetry has been shown to
do which of the following? d. None of the above
a. Have no effect on the cesarean delivery rate
24–30. In which of the following scenarios does the
b. Increase the cesarean delivery rate for dystocia American College of Obstetricians and Gynecologists
c. Increase the cesarean delivery rate for nonreassuring endorse amnioinfusion?
status a. Meconium
d. Decrease the cesarean delivery rate for nonreassuring b. Oligohydramnios
status
c. Variable decelerations
24–26. Why are the terms reassuring and nonreassuring d. All of the above
imprecise and controversial?
a. Patterns change rapidly during labor. 24–31. The American College of Obstetricians and
Gynecologists recommends collection of umbilical
b. The terms are subjective and without definition. cord blood gases for which of the following
c. The terms reflect physiology rather than conditions?
pathology. a. Low 5-minute Apgar score
d. All of the above b. Abnormal fetal heart rate tracing
24–27. In 2008, the National Institute of Child Health c. Severe intrauterine growth restriction
and Human Development convened a conference d. All of the above
and constructed a three-tiered system for fetal heart
rate pattern classification. Which of the following 24–32. Which of the following statements are true
accurately characterizes the different tiers? concerning new uterine contraction terminology?
a. Category I: absence of early decelerations and a. Hyperstimulation has been abandoned.
presence of normal baseline variability b. Tachysystole is defined as >5 contractions in
b. Category II: presence of recurrent late 10 minutes averaged over 30 minutes.
decelerations and absent baseline variability c. Normal uterine activity is defined as ≤5
c. Category III: presence of recurrent variable contractions in 10 minutes averaged over a
decelerations and normal baseline variability 30-minute period.
d. None of the above d. All of the above

24–28. Since the introduction of the new National


Institute of Child Health and Human Development
classification of fetal heart rate patterns, which of the
following is true regarding its effect on perinatal and
maternal morbidity?
a. Cesarean delivery rates have declined.
b. Identification of fetal acidosis is easier.
c. Neonatal morbidity rates have declined.
d. There is not a consensus on interpretation and
management recommendations for fetal heart rate
patterns.
Intrapartum Assessment 167

CHAPTER 24 ANSWER KEY

Question Letter Page

CHAPTER 24
number answer cited Header cited
24–1 c p. 458 Internal (Direct) Electronic Monitoring
24–2 d p. 458 Internal (Direct) Electronic Monitoring
24–3 d p. 459 Internal (Direct) Electronic Monitoring
24–4 d p. 458 External (Indirect) Electronic Monitoring
24–5 b p. 459 Fetal Heart Rate Patterns
24–6 d p. 459 Baseline Fetal Heart Activity
24–7 a p. 459 Baseline Fetal Heart Activity
24–8 c p. 460 Table 24-1
24–9 c p. 461 Baseline Fetal Heart Activity
24–10 b p. 461 Baseline Fetal Heart Activity
24–11 d p. 462 Baseline Fetal Heart Activity
24–12 b p. 462 Baseline Fetal Heart Activity
24–13 c p. 462 Baseline Fetal Heart Activity
24–14 c p. 464 Baseline Fetal Heart Activity
24–15 b p. 465 Periodic Fetal Heart Rate Changes
24–16 d p. 465 Periodic Fetal Heart Rate Changes
24–17 a p. 466 Periodic Fetal Heart Rate Changes
24–18 c p. 466 Periodic Fetal Heart Rate Changes
24–19 c p. 467 Periodic Fetal Heart Rate Changes
24–20 b p. 467 Periodic Fetal Heart Rate Changes
24–21 c p. 469 Fetal Heart Rate Patterns During Second-Stage Labor
24–22 a p. 469 Fetal Heart Rate Patterns During Second-Stage Labor
24–23 c p. 469 Admission Fetal Monitoring in Low-Risk Pregnancies
24–24 a p. 470 Scalp Stimulation
24–25 a p. 471 Fetal Pulse Oximetry
24–26 d p. 472 Nonreassuring Fetal Status
24–27 a p. 473 Table 24-2
24–28 d p. 473 Diagnosis
24–29 b p. 474 Meconium in Amnionic Fluid
24–30 c p. 476 Management Options
24–31 d p. 477 Fetal Heart Rate Patterns and Brain Injury
24–32 d p. 480 Uterine Contraction Terminology
214

CHAPTER 32

The Newborn Infant

32–1. After delivery, clearance of the amnionic fluid that 32–4. Which of the following is true regarding neonatal
fills the fetal lungs occurs via which of the following resuscitation?
mechanisms? a. Approximately 1% of newborns need extensive
a. Physical compression of the fetal thorax resuscitation after delivery.
b. Absorption of fluid into the neonatal pulmonary b. Newborns delivered at home face risk of death
circulation that is 5-fold that of newborns delivered in the
c. Absorption of fluid into the neonatal pulmonary hospital.
lymphatic system c. Approximately 30% of newborns require some
d. All of the above degree of active resuscitation to stimulate
breathing.
32–2. Which of the following postnatal changes promotes d. All of the above
closure of the ductus arteriosus in the newborn?
a. Fall in pulmonary arterial blood pressure 32–5. How can primary and secondary apnea in the
newborn be distinguished from each other?
b. Fall in cardiac output from reduced preload after
umbilical cord clamping a. Stimulation usually reverses primary apnea.
c. Increase in pulmonary vascular compression after b. Secondary apnea is accompanied by a fall in heart
lung aeration from neonatal inspiration rate and primary is not.
d. All of the above c. Secondary apnea is accompanied by loss of
neuromuscular tone and primary is not.
32–3. Which of the following is not a benefit of delayed d. All of the above
cord clamping, as pictured below?
32–6. A 26-year-old multigravida presents in spontaneous
labor at 39 weeks 3 days. She had spontaneous
rupture of membranes with clear amnionic fluid and
delivered spontaneously with a category I fetal heart
rate tracing. The neonate was vigorous and crying
immediately after delivery. What is the next best step
in the management of this newborn?
a. Bulb suctioning of oropharyngeal secretions
b. Cord clamping and transfer to the radiant warmer
c. Placement on maternal abdomen for drying and
evaluation of tone, heart rate, and respiratory effort
d. Hold infant in fully supported prone position
and deliver 3 gentle back thumps to aid in initial
secretion clearance.

32–7. Although initially vigorous and crying, the newborn


a. Increased iron stores in Question 32–6 became apneic with a heart rate
b. Reduced risk of neonatal bradycardia that was 90 beats per minute at 50 seconds of life.
c. Reduced hyperbilirubinemia and need for The following is most appropriately initiated?
phototherapy a. Secretions are suctioned
d. Reduced necrotizing enterocolitis in preterm b. Positive pressure ventilation is administered
neonates c. Newborn is stimulated, and head placed in the
sniffing position
d. All of the above
The Newborn Infant 215

32–8. The newborn in Question 32–6 was given positive 32–11. Which site in the image below depicts the correct
pressure ventilation. Nonetheless, the newborn’s location of fingers for chest compressions in the
heart rate at 80 seconds of life is 86 beats per neonate?

CHAPTER 32
minute. What is the next most appropriate step?
a. Chest compressions
b. Placement of an endotracheal tube
c. Placement of an umbilical vein catheter
d. Mask adjustment and airway repositioning

32–9. A preterm neonate born at 29 weeks’ gestation is


undergoing resuscitation and requires endotracheal
ventilation. Which of the following is most correct
regarding endotracheal tube– assisted ventilation of
this newborn?
a. Positive pressure ventilation is delivered at a rate
of 30 breaths per minute.
b. Opening pressure of 30–40 cm H2O, greater than
that for a term neonate, is typical.
c. Methods of confirming tracheal intubation Used with permission from Dr. David Nelson.

include detection of end-tidal CO2, increasing


fetal heart rate, and visualization of symmetric a. A
chest wall motion. b. B
d. All of the above c. C
d. None of the above
32–10. When chest compressions are indicated for newborn
resuscitation, which of the following best describes 32–12. If the neonate’s heart rate remains less than or equal
the technique? to 60 beats per minute after adequate ventilation
a. Compressions are delivered at a 2:1 ratio with and chest compressions, which of the following is a
ventilation recommended pharmacological intervention?
b. Two hands encircle the chest of the neonate with a. Epinephrine delivered via endotracheal tube at
two thumbs used to deliver compressions 0.05–0.1 mg/kg
c. Compression depth is approximately one half of b. Epinephrine delivered intravenously at
the anterior-posterior diameter of the neonate’s 0.05–0.1 mg/kg
chest c. Ephedrine delivered via endotracheal tube at
d. All of the above 0.05–0.1 mg/kg
d. Ephedrine delivered intravenously at
0.01–0.03 mg/kg

32–13. Which of the following statements are true regarding


Apgar scores?
a. Apgar score assesses 5 characteristics and either
0 or 2 points are awarded.
b. Apgar score effectively assesses newborn health
and effectiveness of resuscitation.
c. Five-minute Apgar score is predictive of neonatal
survival in term but not preterm neonates.
d. In term neonates, Apgar score <7 at 5 minutes
can be used to correlate a hypoxic event as a cause
of cerebral palsy.
216 The Newborn

32–14. A term newborn has a heart rate of 128 beats per 32–18. A 27-year-old primigravida at 33 weeks’ gestation
minute, is pink except for her feet and hands, is has been on the antepartum unit for 6 days for
crying spontaneously and loudly, and is kicking preterm rupture of membranes. An image from
both legs. What is her Apgar score? her admission ultrasound is shown below. You
SECTION 9

a. 7 get a stat page from the nurse informing you the


patient is being wheeled over emergently with cord
b. 8
prolapse. Emergent cesarean delivery is performed
c. 9 with delivery approximately 24 minutes from the
d. 10 prolapse. Umbilical artery cord blood gas results are:
pH 6.95, pCO2 90 mmHg, HCO3- 18 mEq/L, and
32–15. Which of the following may impact the Apgar score? base deficit 8 mEq/L. One-minute Apgar score is 3
a. Prematurity and five-minute Apgar score is 8. After the case you
find her family to review the events. Which of the
b. Fetal malformation
following is an appropriate statement?
c. Maternal medications
d. All of the above

32–16. You receive an emergent consultation from the


trauma department. Ms. Omega was brought in via
emergency medical transport after a motor vehicle
collision. She has a tourniquet around her left upper
arm proximal to an apparent traumatic amputation
at the elbow. Another individual from the accident
is awake and alert and states that Ms. Omega is 39
weeks pregnant and was on the way to the hospital
for labor. You quickly perform an exam and note
that the fetal head is at the perineum. Delivery is
accomplished easily and on initial survey the neonate
is pale with no respiratory effort or response to
stimulation, and has a pulse of 80 beats per minute.
An umbilical cord blood gas is obtained. Which of
the following is most likely given the history? a. Findings are consistent with a metabolic acidosis
a. Base deficit is 14 mEq/L b. Findings suggest that the low pH was related to
an acute event
b. Umbilical artery pH is 7.19
c. One-minute Apgar score is 3 c. Findings suggest high likelihood of a
hypoxic-ischemic encephalopathy
d. Respiratory acidemia is present
d. All of the above
32–17. How do normal umbilical cord blood gas results
differ between umbilical venous and umbilical
arterial samples?
a. HCO3- is higher in a venous sample
b. The pCO2 is lower in a venous sample
c. The pH is higher in an arterial sample
d. The base excess is lower in an arterial sample
The Newborn Infant 217

32–19. A 23-year-old primigravida at 38 weeks’ gestation 32–21. Ms. Smith presents for her 6-week postpartum visit.
presents to labor and delivery after a witnessed seizure She is doing well, and is excited to show you her
at home. Her initial blood pressure is 130/80 mmHg, newborn daughter, who is doing well overall. She

CHAPTER 32
and pulse is 140 beats per minute. The fetal heart has noticed some eye drainage the last few days, and
rate tracing shows a heart rate of 100 beats per you observe the findings depicted below. Which of
minute, absent variability, and late decelerations. the following statements is true?
She is having frequent contractions, and exam shows
she is 3 cm dilated. She is taken back for emergent
cesarean delivery, and bloody fluid is noted upon
uterine entry. Placental findings are depicted below.
Umbilical artery blood gas pH is 6.9 and base deficit
is 14 mEq/L. Five-minute Apgar score is 2. Which
of the following statements is evidence-based?

Reproduced with permission from Levsky ME, DeFlorio P: Ophthalmologic complica-


tions. In Knoop KJ, Stack LB, Storrow AB, et l (eds): The Atlas of Emergency Medicine,
3rd ed. New York, McGraw-Hill, 2010, Figure 2-1.

a. Prophylactic topical eye treatments given to the


neonate do not reliably reduce the incidence of
chlamydial conjunctivitis.
b. 12–25% of neonates delivered vaginally of moth-
ers with chlamydia will develop conjunctivitis,
but this manifests within 3 weeks of delivery.
Used with permission from Dr. Anne Ambia. c. Some form of conjunctivitis affects 1–12% of all
neonates, but gonococcal and chlamydial infections
a. Speed of resolution of the acidemia after birth is are among the least common.
associated with outcome. d. All of the above
b. There is a high likelihood of multiorgan dysfunc-
tion in this neonate. 32–22. During her prenatal care, Ms. Oliver is noted to
c. These initial findings are poorly predictive of be positive for both hepatitis B surface antigen and
subsequent neurological impairment. hepatitis B e antigen. Which of the following is the
d. All of the above are evidence-based statements most appropriate evaluation and management of her
related to the details of this case. term newborn?
a. Neonatal hepatitis B viral load to guide
32–20. Ms. Cox was treated for Neisseria gonorrhoeae subsequent management
3 weeks prior to presentation for a term vaginal b. Administration of hepatitis B immune globulin as
delivery. She reports several episodes of unprotected passive immunization after delivery
intercourse since then, and does not feel confident c. Administration of hepatitis B immune globulin
that her partner sought treatment as recommended. after delivery and administration of the first dose
Which of the following is the most appropriate of the hepatitis B vaccine series prior to discharge
initial intervention for her neonate?
d. Administration of hepatitis B immune globulin if
a. Test for gonococcal and chlamydial infection mother is planning lactation, and administration
b. Single dose ceftriaxone 100 mg/kg intramuscular of the first dose of the hepatitis B vaccine series
injection prior to discharge
c. Single application of 1% silver nitrate solution to
both eyes
d. Single application of 0.5% erythromycin
ointment to both eyes
218 The Newborn

32–23. Why is the infant below receiving an injection of 32–25. During her first trimester, Ms. Harris traveled to
vitamin K within 1 hour of birth? a country that is known to be an endemic area for
Zika virus. Subsequently her serological testing was
positive. Ultrasound surveillance of the fetus has
SECTION 9

been performed, including the 32-week assessment


of fetal head biometrics shown here. Which of the
following is not included in the recommended neo-
natal evaluation?

a. To enhance newborn bone development


b. To prevent vitamin K–dependent hemorrhagic
disease of the newborn
c. To reduce the incidence of necrotizing enterocoli-
tis in premature infants
d. To augment the lower vitamin K levels noted in a. Newborn neurological assessment
breast milk compared to commercially produced
b. Neonatal Zika virus laboratory testing
infant formula
c. Neonatal magnetic resonance imaging
32–24 Which of the following is the most accurate d. Standard newborn hearing screen prior to
statement regarding newborn screening? discharge
a. Newborn screening panels are consistent across
the United States.
b. The United States federal government mandates
newborn screening.
c. The American Academy of Pediatrics dictates the
content of the newborn screening panel.
d. None of the above
The Newborn Infant 219

32–26. Ms. Blake comes to see you on postpartum day 5 for 32–27. A 31-year-old primigravida undergoes a spontaneous
a blood pressure check. She had a spontaneous vagi- vaginal delivery at 40 weeks’ gestation, delivering
nal delivery at term complicated by preeclampsia and a female infant weighing 7 lb 9 oz. They are both

CHAPTER 32
chorioamnionitis. The neonate never had a fever and discharged home on postpartum day 2. She has been
has been feeding, voiding, and stooling well since home for 2 days and calls your office worried that
discharge. Ms. Blake shows you the umbilical cord the baby weighs 7 lb today. What is the most
stump, which is depicted below. What is the appro- appropriate response?
priate response to her concern?

Used with permission from Kelly Yanes. Used with permission from Dr. David Nelson.

a. Provide reassurance of the normal finding with a. If any further weight loss occurs, supplement
no new recommendations. breastfeeding with formula until her weight
stabilizes.
b. Recommend covering the stump with petroleum
jelly and a gauze pad to hasten cord separation. b. Recommend converting to a high calorie
commercial formula and schedule a weight check
c. Recommend she proceed to the emergency
with the pediatrician in 3 days.
room due to concern for necrosis and need for
resection. c. This amount of weight loss is within expectations.
d. Recommend she proceed to her pediatrician’s Observe wet diapers and continue breastfeeding
office due to concern for omphalitis with need for with expectation to regain birthweight by
antibiotics. 10 days.
d. None of the above

32–28. The American College of Obstetricians and Gyne-


cologists recommend exclusive breastfeeding for how
long?
a. 1 month
b. 3 months
c. 6 months
d. 12 months

32–29. Passage of meconium in the delivery room is associ-


ated with which of the following?
a. Necrotizing enterocolitis
b. Gastrointestinal tract patency
c. Fetal distress and lower Apgar score
d. Increased risk for Hirschsprung disease
220 The Newborn

32–30. Ms. Lewis is pregnant with a male fetus. She asks 32–32. Which of the following is true regarding hospital
you about circumcision. Which of the following is stay after delivery?
not true? a. Early discharge is associated with increased
a. Newborn male circumcision rate is estimated to
SECTION 9

neonatal mortality.
be approximately 55% in the latest Centers for b. Early discharge is associated with increased
Disease Control report. neonatal readmission rates for dehydration and
b. Circumcision lowers the incidence of human jaundice.
papilloma virus related penile cancer in males and c. The Newborns’ and Mothers’ Health Protection
cervical cancer in their partners. Act of 1996 prohibits insurers from restricting
c. In their 2012 policy statement, the American hospital stays to less than 2 days for vaginal
Academy of Pediatrics Task Force on Circumcision delivery or 4 days for cesarean delivery.
concluded that health benefits of male circumcision d. All of the above
outweigh risks and recommended the procedure
for all newborns.
d. All of the above

32–31. You are teaching a new intern about male circumcision.


Which of the following points is least important in
safe surgical technique?
a. Never use injectable local analgesia with a
vasoactive compound such as epinephrine.
b. The Mogen technique is associated with less
apparent discomfort for the newborn.
c. Newborn circumcision is an elective procedure to
be performed only on healthy neonates with no
family history of a bleeding disorder.
d. Careful inspection of the external genitalia is
imperative to rule out any congenital disorders
that are contraindications to routine circumcision.
The Newborn Infant 221

CHAPTER 32 ANSWER KEY

Question Letter Page

CHAPTER 32
number answer cited Header cited
32–1 d p. 607 Transition to Air Breathing
32–2 a p. 607 Transition to Air Breathing
32–3 c p. 607 Transition to Air Breathing
Umbilical Cord Clamping
32–4 a p. 607 Newborn Resuscitation
32–5 a p. 608 Newborn Resuscitation
32–6 c p. 608 Resuscitation Protocol
32–7 d p. 609 Figure 32-2
32–8 d p. 610 Table 32-1
32–9 c p. 608 Alternative Airway
32–10 b p. 609 Chest Compressions
32–11 c p. 609 Chest Compressions
32–12 a p. 610 Epinephrine
32–13 b p. 610 Apgar Score
32–14 c p. 610 Apgar Score
32–15 d p. 611 Apgar Score
32–16 a p. 611 Fetal Acid–Base Physiology
32–17 b p. 612 Table 32-3
32–18 b p. 612 Figure 32-4
p. 613 Respiratory Acidemia
32–19 c p. 612 Clinical Significance of Acidemia
32–20 a p. 613 Eye Infection Prophylaxis
32–21 a p. 613 Eye Infection Prophylaxis
32–22 c p. 614 Hepatitis B Immunization
32–23 b p. 614 Vitamin K
32–24 d p. 614 Newborn Screening
32–25 c p. 614 Zika Virus
32–26 a p. 615 Care of Skin and Umbilical Cord
32–27 c p. 615 Feeding and Weight Loss
32–28 c p. 615 Feeding and Weight Loss
32–29 b p. 615 Stools and Urine
32–30 c p. 615 Male Circumcision
32–31 d p. 616 Male Circumcision
32–32 d p. 616 Rooming In and Hospital Discharge
228

CHAPTER 34

The Preterm Newborn

34–1. Compared to term infants, neonates born prematurely 34–6. What is the etiology of the diffuse reticulogranular
have higher associated rates of which of the following? infiltrate seen on the chest radiograph below?
a. Sleep apnea
b. Developmental delay
c. Congenital malformations
d. All of the above

34–2. What is a reason for the decrease in preterm birth


from 12% in 2007 to 10% in 2014?
a. Liberalized use of cerclage
b. Decrease in teen birth rate
c. Universal cervical length screening
d. Use of 17-alpha hydroxyprogesterone caproate

34–3. Respiratory distress syndrome contributes to which


of the following comorbidities?
a. Neurological damage Used with permission from Dr. Becky Ennis.
b. Pulmonary hypertension
c. Necrotizing enterocolitis a. Persistent fetal circulation
d. All of the above b. Bronchopulmonary dysplasia
c. Respiratory distress syndrome
34–4. What is the mechanism of action by which surfactant d. Transient tachypnea of the newborn
prevents respiratory distress syndrome in neonatal
lungs?
a. Clears fluid from the lungs
b. Dilates the pulmonary vasculature
c. Lowers surface tension in the alveoli
d. Aids in maturation of the terminal bronchioles

34–5. Respiratory distress syndrome can develop in the


term infant as surfactant can be inactivated by which
of the following substances?
a. Blood
b. Vernix
c. Meconium
d. All of the above
The Preterm Newborn 229

34–7. A 32-year-old primigravida delivers a neonate at 34–10. A 37-year-old multigravida presents for a 6-week
31 weeks’ gestation due to preeclampsia with postpartum visit after delivering her son prematurely
severe features. She received a course of antenatal at 28 weeks’ gestation. She informs you that he

CHAPTER 34
corticosteroids prior to delivery, but in the delivery developed severe respiratory distress syndrome and
room the neonate receives the intervention pictured was on a mechanical ventilator for many weeks.
below. The neonate likely displayed all except which She is worried he will develop bronchopulmonary
of the following clinical signs? dysplasia (BPD). Which of the following medications
have not been shown to prevent BPD?
a. Caffeine
b. Vitamin A
c. Glucocorticoids
d. Inhaled nitric oxide

34–11. Surfactant has been shown to reduce the rates of


which of the following complications?
a. Intubation
b. Pneumothorax
c. Childhood asthma
d. Necrotizing enterocolitis

34–12. In which of the following scenarios are you least


likely to give prophylactic surfactant?
a. Tachypnea a. A 33-week infant who did not receive antenatal
b. Arrhythmia corticosteroids.
c. Hypotension b. A 27-week infant who received a course of
d. Hypertension “rescue steroids” prior to delivery.
c. A 37-week infant with respiratory distress
34–8. The neonate in Question 34–7 continued to show syndrome requiring intubation.
a respiratory acidosis despite continuous positive d. A 26-week infant who received antenatal
airway pressure (CPAP). According to the American corticosteroids and continuous positive airway
Academy of Pediatrics, what is the next best course pressure in the delivery room.
of action?
a. Intubation 34–13. The American College of Obstetricians and
b. Nitric oxide Gynecologists considers women candidates for
antenatal corticosteroid therapy if they are at risk
c. Glucocorticoids for preterm delivery at what gestational ages?
d. Surfactant administration a. 22–34 weeks
34–9. Respiratory distress syndrome can be caused by b. 23–34 weeks
which of the following? c. 24–32 weeks
a. Infection d. 23–36 weeks
b. Heart failure
34–14. Antenatal corticosteroids have been shown to reduce
c. Meconium aspiration which of the following complications of prematurity?
d. All of the above a. Respiratory distress syndrome and intraventricular
hemorrhage
b. Bronchopulmonary dysplasia and intraventricular
hemorrhage
c. Respiratory distress syndrome and necrotizing
enterocolitis
d. Bronchopulmonary dysplasia and necrotizing
enterocolitis
230 The Newborn

34–15. At what gestational age does the concentration of 34–19. A 29-year-old primigravida underwent a cesarean
lecithin relative to sphingomyelin rise? delivery of a female infant at 32 weeks’ gestation
a. 23 weeks secondary to preeclampsia with severe features and
fetal-growth restriction. The infant weighed 1498
SECTION 9

b. 29 weeks
grams at birth. Which of the characteristics of this
c. 30 weeks infant, pictured below, is not a risk factor for the
d. 34 weeks development of necrotizing enterocolitis?

34–16. Which of the following are methods to assess fetal


lung maturity?
a. Foam stability test
b. Lamellar body count
c. Fluorescence polarization test
d. All of the above

34–17. Which of the following methods to assess fetal lung


maturity is considered the gold standard?
a. Foam stability test
b. Lamellar body count
c. Lecithin-sphingomyelin ratio
d. All of the above

34–18. A 26-year-old primigravida presents for her prenatal a. Female gender


visit at 36 weeks’ gestation and inquires about b. Very low birthweight
scheduling an induction of labor. At her first visit
c. Umbilical catheterization (U)
she was unsure of her last menstrual period, and
she was dated by a 22-week ultrasound. At what d. Supplementation of cow milk formula (G)
gestational age does the American College of
Obstetricians and Gynecologists (ACOG) 34–20. The neonate in Question 34–19 develops abdominal
recommend delivery? distention, fever, and intolerance of nasogastric feeds.
An abdominal radiograph is performed and shown
a. 39 weeks
below. What is thought to be the pathophysiology of
b. 41 weeks this condition?
c. 39 weeks after amniocentesis documenting fetal
lung maturity
d. 41 weeks after amniocentesis documenting fetal
lung maturity

Used with permission from Dr. Natalie Frost.

a. Intestinal immaturity
b. Exposure to enteral feeds
c. Highly immunoreactive intestinal mucosa
d. All of the above
The Preterm Newborn 231

34–21. How does hyperoxemia lead to retinopathy of 34–25. Which of the following is the preferred initial
prematurity? modality for identifying brain abnormalities in the
a. Causes centripetal vascularization of retina newborn?

CHAPTER 34
b. Causes hemorrhage from retinal vessels and a. Sonography
subsequent aberrant neovascularization b. Computed tomography
c. Causes severe retinal vasoconstriction, c. Magnetic resonance imaging
vessel obliteration, and subsequent aberrant d. Positron emission tomography
neovascularization
d. Causes dilation of the retinal vessels, increase 34–26. Which of the following contributes to the increased
in angiogenic factors, and subsequent aberrant risk of intraventricular hemorrhage in preterm
neovascularization infants?
a. Venous stasis and congestion
34–22. The infant pictured below was born at 34 weeks’
b. Impaired vascular autoregulation
gestation by cesarean delivery due to preeclampsia
with severe features. The infant’s birthweight c. Poor vessel support by the subependymal
was 1928 grams. Even at this gestational age, low germinal matrix
birthweight is a risk factor for which of the following d. All of the above
complications of prematurity?
34–27. A 34-year-old multigravida presents for her post-
partum visit and informs you her infant, born at
25 weeks’ gestation, has a bleed in its brain. She
reports she was told the bleeding has extended into
the tissue. What grade intraventricular hemorrhage
does her infant have?
a. Grade I
b. Grade II
c. Grade III
d. Grade IV

34–28. Which of the following medications has been shown


to reduce neurodevelopmental impairment in the
preterm infant?
a. Vitamin E
b. Indomethacin
a. Necrotizing enterocolitis
c. Phenobarbital
b. Retinopathy of prematurity
d. Magnesium sulfate
c. Intraventricular hemorrhage
d. Respiratory distress syndrome 34–29. Which of the following women are candidates
for a course of antenatal corticosteroids according
34–23. All except which of the following are cerebral lesions to the American College of Obstetricians and
seen in the preterm infant? Gynecologists?
a. Cerebellar hemorrhage a. 26 weeks’ gestation dilated to 3 cm contracting
b. Middle cerebral artery stroke every 2 minutes
c. Cystic periventricular leukomalacia c. 23 weeks’ gestation dilated to 2 cm contracting
d. Periventricular hemorrhagic infarction every 5 minutes
c. 33 weeks’ gestation dilated to 3 cm who received
34–24. Which of the following is seen more typically in a a course of steroids 21 days ago on presentation
term infant rather than a preterm infant? for preterm premature rupture of membranes
a. Subdural hemorrhage d. All of the above
b. Cerebellar hemorrhage
c. Subarachnoid hemorrhage
d. Intraventricular hemorrhage
232 The Newborn

34–30. All except which of the following are risk factors for 34–32. Which of the following statements about periven-
development of cerebral palsy? tricular leukomalacia are true?
a. Ischemia a. It is associated with cerebral palsy
SECTION 9

b. Chorioamnionitis b. Damage is irreversible because brain tissue does


c. Operative vaginal delivery not regenerate
d. Intraventricular hemorrhage c. Due to lack of gliosis the areas appear as echolu-
cent cysts on imaging
34–31. Which of the following types of cerebral palsy is the d. All of the above
least common?
a. Diplegia 34–33. All except which of the following substances have
been shown to be toxic to brain tissue?
b. Hemiplegia
a. Calcium
c. Spastic quadriplegia
b. Glutamate
d. Choreoathetoid types
c. Magnesium
d. Tumor necrosis factor
The Preterm Newborn 233

CHAPTER 34 ANSWER KEY

Question Letter Page

CHAPTER 34
number answer cited Header cited
34–1 c p. 636 Introduction
34–2 b p. 636 Introduction
34–3 d p. 636 Respiratory Distress Syndrome
34–4 c p. 637 Etiopathogenesis
34–5 c p. 637 Etiopathogenesis
34–6 c p. 637 Clinical Course
34–7 c p. 637 Treatment
34–8 d p. 637 Treatment
34–9 d p. 637 Clinical Course
34–10 d p. 637 Treatment
34–11 b p. 638 Surfactant Prophylaxis and Rescue
34–12 d p. 638 Surfactant Prophylaxis and Rescue
34–13 b p. 638 Prevention
34–14 a p. 638 Prevention
34–15 d p. 638 Prevention
34–16 d p. 638 Prevention
34–17 c p. 638 Prevention
34–18 b p. 638 Prevention
34–19 a p. 639 Necrotizing Enterocolitis
34–20 d p. 639 Necrotizing Enterocolitis
34–21 c p. 639 Retinopathy of Prematurity
34–22 a p. 639 Necrotizing Enterocolitis
34–23 b p. 639 Brain Disorders
34–24 a p. 639 Intracranial Hemorrhage
34–25 a p. 639 Brain Disorders
34–26 d p. 639 Periventricular-Intraventricular Hemorrhage
34–27 d p. 640 Periventricular-Intraventricular Hemorrhage
34–28 d p. 640 Periventricular-Intraventricular Hemorrhage
34–29 d p. 641 Periventricular-Intraventricular Hemorrhage
34–30 c p. 641 Risks
34–31 d p. 641 Cerebral Palsy
34–32 d p. 640 Periventricular Leukomalacia
34–33 c p. 641 Risks
242

CHAPTER 36

The Puerperium

36–1. What duration of time is encompassed in the 36–6. A 26-year-old G3P3 is postpartum day 1 following
puerperium? an uncomplicated vaginal delivery. She reports sharp,
a. 2–4 weeks intermittent lower abdominal pain, which is more
severe than in her prior deliveries. Her heart rate is
b. 4–6 weeks
84 beats per minute, blood pressure 110/60 mmHg,
c. 6–8 weeks and her temperature 99.3 °F. Her abdomen is soft
d. 10–12 weeks on exam and there is no uterine tenderness. A scant
amount of lochia is appreciated on bimanual exam.
36–2. What are myrtiform caruncles? What is the most likely diagnosis?
a. Vaginal rugae a. Afterpains
b. Scarred tags of hymenal tissue b. Endometritis
c. A sexually transmitted infection c. Bladder flap hematoma
d. Microscopic tears in the vaginal epithelium d. Septic pelvic thrombophlebitis

36–3. At what point in the puerperium does the 36–7. A 20-year-old G1P1 presents to the emergency room
endocervical canal reform? 10 days after an uncomplicated vaginal delivery
a. 1 week postpartum complaining of fever, nausea/vomiting, abdominal
pain, and increased vaginal discharge. On arrival, she
b. 2 weeks postpartum is febrile to 38.6 °C and she has fundal tenderness on
c. 3 weeks postpartum exam. Which of the following tests would be least
d. 4 weeks postpartum helpful for further evaluation?
a. A complete blood count
36–4. What percentage of women experience regression of b. A basic metabolic profile
high-grade dysplasia following delivery?
c. A transvaginal ultrasound
a. 10%
d. A Gram stain of her vaginal discharge
b. 33%
c. 35% 36–8. A 40-year-old G2P2 presents to your office 2 weeks
d. 50% after her scheduled cesarean delivery for follow-up.
She complains of intermittent episodes of heavy
36–5. How long does complete uterine involution take vaginal bleeding, occurring as recently as the day
following delivery? prior. Her temperature is 99.6°F, heart rate 98 beats
a. 1 week per minute, and blood pressure 120/80 mmHg. On
exam, her uterus is noted to be enlarged to 20 weeks
b. 2 weeks in size and boggy. Her incision appears well approxi-
c. 3 weeks mated without evidence of infection. The most
d. 4 weeks appropriate management includes which of the
following?
a. A complete blood count
b. Methergine administration
c. Empiric antibiotic treatment
d. All of the above
The Puerperium 243

36–9. Secondary postpartum hemorrhage is defined as 36–14. Your patient is postoperative day 2 following a primary
uterine hemorrhage occurring during what time- cesarean delivery for failure to progress. She is frustrated
frame after delivery? because she is only getting small drops of thick yellow

CHAPTER 36
a. 24 hours to 6 weeks liquid from her breasts. How long would you tell her
to expect colostrum production before beginning her
b. 48 hours to 6 weeks
conversion to a more mature milk?
c. 24 hours to 12 weeks
a. 2 to 5 days
d. 48 hours to 12 weeks
b. 2 to 10 days
36–10. What are common features of the urinary bladder in c. 5 to 14 days
the postpartum period? d. 5 to 21 days
a. Increased capacity
b. Incomplete emptying 36–15. What vitamin is virtually absent in human breast
milk?
c. Insensitivity to intravesical pressures
a. Vitamin C
d. All of the above
b. Vitamin A
36–11. A 24-year-old G1P1 complains of abdominal pain c. Vitamin K
and subjective fever 1 day following an uncomplicated d. All vitamins are equally represented in breast milk
vaginal delivery. Which of the following physical
exam signs or laboratory values would be most 36–16. Which hormone is most responsible for milk
helpful to make a diagnosis of endometritis in this expression during lactation?
postpartum patient? a. Prolactin
a. Fundal tenderness b. Oxytocin
b. An absolute neutrophilia c. Dopamine
c. White blood cell count of 25,000/µL d. Progesterone
d. All of the above
36–17. How would you advise a patient who is exclusively
36–12. After delivery, how long do cardiovascular parameters breastfeeding 4 weeks after delivery and develops
including cardiac output, heart rate, and blood nipple fissures?
pressure take to return to nonpregnant levels? a. Wash the area with mild soap and water daily.
a. 48 hours b. Apply a steroid cream and use a nipple shield
b. 72 hours temporarily.
c. 7 days c. Continue exclusive breastfeeding without
d. 10 days interruption.
d. Do not allow infant to feed on the affected side
36–13. You counsel your puerperal patient that most women and empty the breast regularly with a pump.
first approach their prepregnancy weight by which
time interval following delivery? 36–18. All except which of the following conditions are
a. 3 months contraindications to breastfeeding?
b. 6 months a. Infant with galactosemia
c. 9 months b. Maternal hepatitis B infection
d. 12 months c. Human immunodeficiency virus infection
d. Maternal active and untreated tuberculosis
244 The Puerperium

36–19. A 21-year-old G1P1 presents to your office 4 days fol- 36–20. Your patient presents 5 days postpartum with
lowing an uncomplicated vaginal delivery complaining an axillary mass. She noted it during pregnancy
of breast pain and difficulty with breastfeeding. Her although reports it was much smaller. Yesterday, she
temperature is 37.8oC, heart rate 102 beats per
SECTION 10

noticed that the mass became larger and more ten-


minute, and respiratory rate 22 breaths per minute. der. Her face is shielded and a sagittal photograph of
The appearance of her breasts is similar to that her breast, axilla, and arm is show below. She denies
pictured in the image. No erythema or fluctuance is fever and has no additional complaints. Management
appreciated on exam. What should you recommend of the patient should primarily include which of the
to the patient? following?

a. Initiation of dicloxacillin a. Needle aspiration


b. Admission to the hospital b. Observation and reassurance
c. Stop breastfeeding immediately and use a breast c. Axillary lymph node excision
binder d. Antibiotic therapy with gram-positive coverage
d. Oral analgesics, cool packs, and frequent feeding
or pumping 36–21. A 34-year-old G3P3 with a history of migraine head-
aches calls your office 1 week following an uncom-
plicated vaginal delivery complaining of severe
headaches. She did receive epidural anesthesia during
her labor course, but review of her chart confirms
that the epidural placement was easy and uncompli-
cated. The patient reports that the headaches began
a few days following delivery and are associated
with nausea, but no vomiting, and are minimally
improved with either rest or over-the-counter analge-
sics. What is the most likely trigger of this patient’s
postpartum headaches?
a. Dural puncture
b. Estrogen withdrawal
c. Progesterone withdrawal
d. Intracerebral hemorrhage
The Puerperium 245

36–22. Your patient had a 4-hour second stage of labor and 36–25. Which of the following is true regarding the postpar-
vaginal delivery without laceration. As her epidural tum blues?
analgesia subsides, she complains of perineal pain. Her a. May be affected by body image concerns

CHAPTER 36
temperature is a 37.0oC, pulse 84 beats per minute,
b. Usually lasts for no more than 10 days after delivery
and blood pressure 120/68 mmHg. Her first void
yielded 300 mL of urine. Management of the patient c. Effective treatment involves recognition and
should primarily include which of the following? reassurance
a. Perineal cool pack d. All of the above
b. Surgical evacuation 36–26. A 30-year-old G1P1 complains of difficulty with
c. Diagnostic needle aspiration ambulation on postpartum day 1 following a labor
d. Broad-spectrum intravenous antibiotic therapy and delivery course, which lasted 24 hours and
included 3 hours of pushing in stirrups. You perform
36–23. A 19-year-old G1P1 complains of worsening vul- an exam and note bilateral foot drop on exam.
var pain in the recovery room 6 hours following a Injury to what nerve is the most likely cause of the
vaginal delivery. The delivery was complicated by a patient’s ambulation difficulty?
prolonged second stage and a third degree laceration. a. Femoral nerve
On further evaluation, her temperature is 37.2oC, b. Ilioinguinal nerve
heart rate 130 beats per minute, blood pressure
86/52 mmHg, and respiratory rate 28 breaths per c. Common peroneal nerve
minute. She appears pale and mildly diaphoretic. d. Lateral femoral cutaneous nerve
Examination of the perineum reveals the findings
shown in the photo below. What is the next best 36–27. Which of the following is true regarding the
course of action? condition shown in the pelvic radiograph?

Reproduced with permission from Cunningham FG, Leveno KJ, Bloom SL, et al (eds): The
a. Urgent surgical evacuation Puerperium. In Williams Obstetrics, 25th ed. New York, McGraw-Hill, 2018, Figure 36-7.

b. Place a vaginal pack and Foley catheter


a. Symphyseal separation >1.5 cm is diagnostic.
c. Check a complete blood count and observe
closely b. Surgery is usually necessary for separation >3 cm.
d. Start intravenous antibiotics with gram-negative c. Treatment is generally conservative and consists
coverage of rest and placement of a pelvic binder.
d. All of the above
36–24. A 29-year-old G2P2 with an 18-hour labor course is
unable to void 5 hours following a vaginal delivery. 36–28. What is the mean time to ovulation resumption in a
What is the best management option? postpartum woman who elects not to breastfeed?
a. A single in-and-out catheterization a. 4 weeks
b. Place a Foley catheter for 24 hours b. 5 weeks
c. Continue close observation without intervention c. 6 weeks
d. Encourage increased fluid intake since the bladder d. 7 weeks
is likely empty following delivery
246 The Puerperium

36–29. Your patient presents for her 6-week follow-up 36–32. Approximately what percentage of postpartum
visit after having a primary cesarean delivery for women require readmission to the hospital within
breech presentation. She is trying to breastfeed 8 weeks of delivery?
SECTION 10

exclusively but reports issues with milk supply. What a. <1%


contraceptive option would you not recommend for
b. 3%
this patient?
c. 7%
a. Progestin-only pills
d. 10%
b. Estrogen-progestin patch
c. Depot medroxyprogesterone 36–33. A 29-year-old G1P1 presents for a follow-up visit
d. A levonorgestrel intrauterine device 4 weeks postpartum following an uncomplicated
spontaneous vaginal delivery. She is overall doing
36–30. Following an uncomplicated vaginal delivery, when well and has returned to most of her usual activities.
can women be advised to resume coitus based on However, she complains of low energy levels despite
desire and comfort? the fact that her baby is sleeping for long intervals at
a. 1 week postpartum night. What should you offer her?
b. 2 weeks postpartum a. Reassurance only
c. 3 weeks postpartum b. Psychiatry referral
d. 4 weeks postpartum c. Check her thyroid hormone levels
d. All of the above
36–31. What treatment can be offered to the postpartum
woman who is exclusively breastfeeding and
complains of vaginal dryness during intercourse
7 weeks following delivery?
a. Topical estrogen
b. Reassurance only
c. Topical testosterone
d. Intercourse is not recommended within the first
8 weeks of delivery.
248

CHAPTER 37

Puerperal Complications

37–1. What percentage of non-breastfeeding women 37–6. What organism has been implicated in late-onset,
develop fever from breast engorgement postpartum? indolent metritis?
a. 15% a. Proteus
b. 25% b. Klebsiella
c. 35% c. Chlamydia
d. 55% d. Peptostreptococcus

37–2. What is the most common etiology of persistent 37–7. What is the most important criterion for the
fevers after childbirth? diagnosis of postpartum metritis?
a. Atelectasis a. Fever
b. Pyelonephritis b. Leukocytosis
c. Breast engorgement c. Foul-smelling lochia
d. Genital tract infections d. Parametrial tenderness

37–3. What physiological process in the postpartum period 37–8. A 20-year-old G1P1 underwent a cesarean delivery
causes urinary tract infections to be uncommon? for failure to progress. Just prior to surgery, the
a. Diuresis patient was diagnosed with metritis and started on
broad-spectrum antibiotics, which were continued
b. Passage of lochia postpartum. The patient continues to have fever on
c. Uterine involution postoperative day 5. She does not appear septic. She
d. Immunosuppression is frustrated that she can’t go home. Which of the
following is the least likely diagnosis?
37–4. Which of the following is the single most significant a. Infected hematoma
risk factor for development of an uterine infection? b. Parametrial phlegmon
a. Route of delivery c. Septic pelvic thrombophlebitis
b. Use of internal monitors d. Antimicrobial-resistant bacteria
c. Artificial rupture of membranes
d. Group B streptococcus colonization 37–9. What is the difference in treatment for metritis in
cases of vaginal delivery versus cesarean section?
37–5. A 35-year-old multigravida at 39 weeks’ gestation a. Dosing of antibiotics
undergoes an induction for elevated blood pressures b. Coverage of anaerobes
at term. After 24 hours, placement of internal
monitors, and 10 cervical exams, the patient c. Duration of treatment
undergoes a cesarean delivery for failure to progress. d. Route of administration of antibiotics
Which of the following is not a risk factor for her
developing an uterine infection? 37–10. Which of the following regimens is the gold standard
a. Multiparity for treatment of a pelvic infection following a cesarean
delivery?
b. Prolonged labor
a. Meropenem
c. Cesarean delivery
b. Vancomycin
d. Placement of internal monitors
c. Clindamycin and aztreonam
d. Clindamycin and gentamicin
Puerperal Complications 249

37–11. A 22-year-old primigravida at term is diagnosed with 37–15. A 30-year-old G2P2 presents on postoperative day
failure to progress, and the plan is made for cesarean 6 to the emergency room complaining of drainage
delivery. The patient has no medical problems other from her cesarean incision. The cesarean delivery was

CHAPTER 37
than a body mass index of 50 kg/m2. She also denies for failure to progress after a long induction. The
any allergies. The anesthesiologist asks if you would patient is currently afebrile. Her body mass index
like routine perioperative antimicrobial prophylaxis. is 47 kg/m2. The patient reports that she sat down
Which antibiotic do you ask for to further reduce yesterday and felt a pop. Shortly thereafter, she
her chance of a postoperative infection? noticed pink/light brown drainage coming from
a. 3 grams instead of 2 grams of cefazolin her incision. She endorses chills and skin irritation
around the incision. Lochia has been normal. On
b. Vancomycin instead of the routine cefazolin
your exam, her skin is erythematous near the incision.
c. Routine 2 grams of cefazolin prior to skin There is serosanguinous drainage from the wound.
incision Although the patient has good pain tolerance, why
d. Ampicillin, gentamicin, and clindamycin for do you elect to take her to the operating room to
24 hours after the surgery evaluate the incision?
a. You plan to proceed with hysterectomy.
37–12. Which of the following has not been shown to lower
b. You want to open the wound, debride necrotic
the risk for infection after cesarean delivery?
tissue, and then close it back up using en bloc
a. Spontaneous separation of the placenta closure.
b. Chlorhexidine-alcohol skin preparation c. You want to place a negative-pressure wound
c. Single-dose antibiotics prior to skin incision therapy system and that can only be done in the
d. Surgeons changing gloves after delivery of the operating room.
placenta d. You are concerned that the fascia may not be
intact and if so, the fascia needs to be closed in
37–13. In more than 90% of women, metritis responds to the operating room.
treatment with antibiotics within what period of
time? 37–16. Which of the following statements about necrotizing
a. 12–24 hours fasciitis is false?
b. 24–36 hours a. It is common with low mortality rates.
c. 48–72 hours b. Three risk factors are diabetes, obesity, and
hypertension.
d. 72–96 hours
c. Surgical debridement of infected tissue should
37–14. Which of the following is an evidence-based statement leave wide margins of healthy bleeding tissue.
about the use of vacuum-assisted wound closure d. Early diagnosis, surgical debridement, anti-
devices in obstetrics? microbials, and intensive care are paramount to
a. It prevents wound infection. successful treatment.
b. It is superior to standard dressings.
37–17. Which of the following statements about an ovarian
c. Provider time is decreased substantially. abscess in the puerperium is true?
d. It is significantly more cost effective than stan- a. Rupture is rare
dard dressings.
b. Usually affects both ovaries
c. Women present 4–6 weeks after delivery
d. It is thought to be caused by bacterial invasion of
the ovary through a rent in the capsule.
250 The Puerperium

37–18. Which of the following is frequently the first sign/ 37–21. What is the overall incidence of septic pelvic
symptom of peritonitis in a postpartum woman? thrombophlebitis?
a. Diarrhea a. 1/100
SECTION 10

b. Skin erythema b. 1/200


c. Adynamic ileus c. 1/2000–1/3000
d. Abdominal rigidity d. 1/20,000–1/30,000

37–19. Which of the following statements about the 37–22. A 19-year-old primigravida undergoes a cesarean
phlegmon illustrated in the figure below is true? delivery for failure to progress. Her course is com-
plicated by chorioamnionitis for which she receives
broad-spectrum antibiotics. On postoperative day 5,
the patient is still having fevers. She feels well and is
becoming annoyed that she can not go home. Her
incision is healing nicely with no erythema or drain-
age. On computed tomography imaging, the patient
is noted to have a clot that extends to the ovarian
vein. An image is provided below. What is the next
step in your management?

Reproduced with permission from Worley KC: Postoperative complications. In Yeomans


ER, Hoffman BL, Gilstrap LC III, et al (eds). Cunningham and Gilstrap’s Operative
Obstetrics, 3rd ed. New York, McGraw Hill Education, 2017.

Reproduced with permission from Cunningham FG, Leveno KJ, Bloom SL, et al (eds):
Puerperal complications. In Williams Obstetrics, 25th ed. New York, McGraw-Hill, 2018, a. Therapeutic heparin
Figure 37-3.
b. Continuation of antibiotic therapy
a. This is usually a bilateral process. c. Consult interventional radiology about removing
the clot and/or placing a filter.
b. Rarely is this limited to the parametrium.
d. Stop all medications and discharge home, so she
c. Typically fever resolves in 5–7 days with broad- can walk, reducing further clot risk.
spectrum antibiotics.
d. The most common route of extension is posteri-
orly into the rectovaginal septum.

37–20. Which of the following is least likely to occur during


surgery for uterine incisional necrosis?
a. Transfusion
b. Hysterectomy
c. Surgical debridement
d. Bilateral salpingo-oophorectomy
Puerperal Complications 251

37–23. Which of the following is a risk factor for episiotomy 37–27. Which of the following is the best treatment for
dehiscence? toxic shock syndrome?
a. Smoking a. Supportive care

CHAPTER 37
b. Infection b. Supportive care and antibiotics
c. Genital warts c. Supportive care, antibiotics, and wound
d. All of the above debridement if necessary
d. There is no necessary treatment as it will resolve
37–24. A 22-year-old G1P1 presents 6 days after a on its own over time
vaginal delivery. Her course was complicated by
chorioamnionitis and a second-degree perineal 37–28. What is the incidence of mastitis?
laceration. The patient is complaining of pain and a. 1%
drainage from her vagina. On exam, her laceration
repair is open and draining purulent material. b. 3%
Which of the following would not be a step in your c. 10%
management? d. 15%
a. Intravenous antibiotics
b. Debridement of necrotic tissue 37–29. When is mastitis most likely to occur?
c. Establishment of adequate analgesia prior to a. Postpartum day 1
debridement b. Postpartum day 5
d. Intravenous antibiotics, debridement of necrotic c. 3–4 weeks postpartum
tissue in the operating room, and then immediate d. 6–9 months postpartum
closure of the laceration
37–30. What percentage of women with mastitis develop an
37–25. What is the case-fatality rate of toxic shock abscess?
syndrome? a. 1%
a. 1–2% b. 3%
b. 5–6% c. 10%
c. 10–15% d. 15%
d. 20–25%
37–31. Which of the following is not expected in cases of
37–26. A 20-year-old G1P1 presents 3 days postpartum mastitis?
after a vaginal delivery for fever, headache, nausea/ a. Fever
vomiting, and lower abdominal pain. The patient’s
b. Chills
boyfriend reports that she got sick very quickly and
is “not making sense when she talks.” On exam, the c. Breast firmness
patient has severe abdominal pain and foul-smelling d. Symptoms in both breasts
lochia. She is hypotensive and tachycardic. She
appears extremely ill. Which of the following is the
most likely diagnosis?
a. Listeriosis
b. Pyelonephritis
c. Gastroenteritis
d. Toxic shock syndrome
252 The Puerperium

37–32. A 32-year-old G4P2 presents 3 weeks after under-


going a cesarean delivery for breech presentation.
She has been exclusively breastfeeding and feeling
SECTION 10

well. She now complains of a 2-day history of fever,


chills, and breast pain. She finds it difficult to feed
the infant on the affected side. On exam, the breast
is warm, red, and tender. You expect mastitis at the
least, are not sure if there is an abscess. You obtain
an ultrasound and the image is shown below. What
is the best management plan?

Used with permission from Dr. Emily Adhikari.

a. Intravenous antibiotics, supportive care, suspend


breast feeding on the affected side for 48–72
hours
b. Oral antibiotics, discontinuation of breast feeding
on the affected side, follow up with OB/GYN next
week
c. Intravenous antibiotics, drainage of abscess,
culture, pumping or breast feeding continued for
both breasts
d. Milk culture, discharge home with pain
medication, encourage breast feeding on that
side, antibiotics if the milk culture is positive
Puerperal Complications 253

CHAPTER 37 ANSWER KEY

Question Letter Page

CHAPTER 37
number answer cited Header cited
37–1 a p. 666 Puerperal Fever
37–2 d p. 666 Puerperal Fever
37–3 a p. 667 Puerperal Fever
37–4 a p. 667 Uterine Infection
37–5 a p. 667 Uterine Infection
37–6 c p. 668 Uterine Infection
37–7 a p. 668 Uterine Infection
37–8 d p. 668 Uterine Infection
37–9 b p. 668 Choice of Antimicrobials
37–10 d p. 669 Table 37-2 Antimicrobial regimens for pelvic infections
following cesarean delivery
37–11 a p. 669 Perioperative Prophylaxis
37–12 d p. 669–670 Perioperative Prophylaxis
37–13 c p. 670 Complications of Uterine and Pelvic Infections
37–14 c p. 670–671 Vacuum-Assisted Wound Closure
37–15 d p. 671 Abdominal Incisional Infections
37–16 a p. 671 Necrotizing Fasciitis
37–17 d p. 671 Adnexal Abscesses and Peritonitis
37–18 c p. 672 Adnexal Abscesses and Peritonitis
37–19 c p. 672 Figure 37-3
37–20 d p. 672 Parametrial Phlegmon
37–21 c p. 673 Septic Pelvic Thrombophlebitis
37–22 b p. 673 Septic Pelvic Thrombophlebitis
37–23 d p. 674 Perineal Infections
37–24 d p. 675 Early Repair of Infected Episiotomy
37–25 c p. 675 Toxic Shock Syndrome
37–26 d p. 675 Toxic Shock Syndrome
37–27 c p. 675 Toxic Shock Syndrome
37–28 b p. 675 Breast Infections
37–29 c p. 675 Breast Infections
37–30 c p. 675 Breast Infections
37–31 d p. 675 Breast Infections
37–32 c p. 676 Breast Abscess
254

CHAPTER 38

Contraception

38–1. According to the World Health Organization, with 38–5. Which of the following statements properly describe
no contraceptive use, a sexually active woman has the intrauterine device?
what risk for pregnancy over a year? a. Mirena is a levonorgestrel-eluting device, and it is
a. 65% approved for 5 years of use following insertion.
b. 70% b. Liletta contains 52 mg of levonorgestrel, and it is
c. 75% approved for 5 years of use following insertion.
d. 85% c. Skyla is the largest of the intrauterine devices, and
it is approved for 3 years of use following insertion.
38–2. In contrast to the answer to Question 38–1, typical d. ParaGard is not considered a “chemically active”
use of combination oral contraceptive pills by a intrauterine device, and it is approved for 7 years
sexually active woman has what risk for pregnancy of use following insertion.
over the first year of use?
a. 3% 38–6. Contraceptive efficacy with the method pictured below
is not believed to result from which of the following
b. 9%
mechanisms?
c. 13%
d. 17%

38–3. Which of the following is not considered a


long-acting reversible contraceptive method?
a. Levonorgestrel implant
b. Copper intrauterine device
c. Depot medroxyprogesterone acetate
d. All of the above are long-acting reversible
contraceptive methods

38–4. What is the purpose of the United States Medical


Eligibility Criteria, published by the Centers for
Disease Control and Prevention?
a. Provides legal criteria for contraception provision
to minors
b. Provides codified data for insurance submission a. Ovulation inhibition
under the Affordable Care Act b. Decreased motility and viability of sperm
c. Provides financial stratification criteria for c. Reduced viability of egg and sperm preventing
Medicaid contraception eligibility fertilization
d. Provides guidance in contraceptive choice for d. Endometrial inflammation action directed toward
women with various comorbidities a blastocyst, if formed
Contraception 255

38–7. Which of the following statements properly 38–10. Ms. Thomas from Question 38–8 undergoes an
characterizes expulsion of an intrauterine device? ultrasound with the findings shown below. Which of
a. The cumulative expulsion rate after 3 years is 20%. the following is the next best clinical step?

CHAPTER 38
b. Expulsion is most common in the final years of
approved use.
c. If a woman is unable to palpate the trailing
strings, she should be evaluated.
d. All of the above

38–8. Ms. Thomas calls complaining of dull lower abdominal


pain for the last 2 months. You placed an intrauterine
device (IUD) 3 months ago, and she has never
attempted to palpate the strings. On speculum
examination you see the picture below. Which of the
following is the most appropriate next step?

a. Obtain a β-hCG level


b. Obtain a KUB radiograph
c. Attempt IUD removal by means of an IUD hook
or Randall stone clamp a. Hysterectomy
d. Continue the exam by twirling a cytologic brush b. Dilation and curettage
in her endocervical canal
c. Diagnostic laparoscopy
38–9. Ms. Thomas in Question 38–8 has a negative d. Operative hysteroscopy
β-hCG. Which of the following is the next best
management step? 38–11. You receive a phone call from an emergency medicine
physician. She is seeing a patient who reports having
a. Diagnostic laparoscopy a ParaGard in place. Which of the following imaging
b. Diagnostic hysteroscopy modalities is contraindicated in this patient?
c. Transvaginal ultrasound a. Computed tomography scan
d. Computed tomography scan b. 3-tesla magnetic resonance imaging
c. 1.5-tesla magnetic resonance imaging
d. None of the above
256 The Puerperium

38–12. Which of the following is true regarding the intra- 38–14. Which of the following is true regarding the
uterine device–related complication pictured here? risk for infection associated with an intrauterine
device (IUD)?
SECTION 10

a. An IUD is not a safe method of contraception for


women with human immunodeficiency virus.
b. The risk for upper genital tract device-related
infection increases in parallel with increasing time
of device use.
c. The American Heart Association recommends
single dose antibiotic prophylaxis prior to
IUD insertion for women at risk for bacterial
endocarditis.
d. An IUD may remain in situ for pelvic inflam-
matory disease that is responding to antibiotic
therapy, but removal is indicated in the case of a
tuboovarian abscess.

38–15. Ms. Cooper presents reporting 14 weeks of amenorrhea


Used with permission from Dr. Seth Hawkins. and she is found to have a positive pregnancy test.
Strings of her intrauterine device (IUD) are visible
a. Is more common with insertion during lactation at the cervical os, and her ultrasound findings are
b. Rate of occurrence is approximately 1 per 100 shown below. Which of the following is the correct
insertions course of action in this case?
c. Is suspected clinically when sound or insertion
device will not travel as far as expected based on
bimanual examination
d. None of the above

38–13. Which of the following best characterizes the


relationship between intrauterine device (IUD) use
and ectopic pregnancy?
a. Use of an IUD increases the risk for an ectopic
pregnancy.
b. IUD is contraindicated in women with a history
of an ectopic pregnancy.
c. Use of an IUD does not impact risk for an
ectopic pregnancy because the overall risk for
pregnancy is decreased.
d. Compared to pregnancy after failure of combination
oral contraceptive pills, a higher proportion of a. IUD removal
pregnancies after IUD failure will be ectopic b. IUD removal and laparoscopy
pregnancies. c. IUD removal and 10-day course of doxycycline
d. IUD remains in place, and continue routine
prenatal care
Contraception 257

38–16. Which of the following is most accurate regarding 38–20. Which of the following is true regarding placement
pregnancy with an intrauterine device (IUD) in situ? of the progestin implant?
a. Risk for spontaneous abortion is approximately a. For women who are certain they are not pregnant,

CHAPTER 38
four times as high if the device is left in situ. insertion can occur at any time with no need for
b. Retention of an IUD during pregnancy increases backup contraception.
risk for fetal malformation and preterm delivery. b. For women transitioning from combination oral
c. After viability has been reached, data is definitive contraceptives, it is inserted on what would be
that the IUD should be left in place even if the first day of a new pack.
strings are visible. c. Insertion can occur prior to discharge home
d. Evidence of pelvic infection during a pregnancy following miscarriage, abortion, or delivery with
with a retained IUD should be treated with no impairment of lactation.
antibiotics and uterine evacuation. d. For women not currently using hormonal
contraception, it is ideally inserted 5 days prior
38–17. Which of the following is true regarding insertion of to expected menses onset with contraception
an intrauterine device (IUD)? established within 24 hours.
a. Implanon cannot be easily identified for removal
with ultrasound and has lost approval by the 38–21. Which of the following is a suspected method of
Food and Drug Administration. efficacy for all progestin-only contraceptive methods?
b. For placement unrelated to pregnancy, the only a. Endometrial atrophy
recommended time is near the end of menstruation b. Inhibition of ovulation
as it is easier, and pregnancy is excluded. c. Thickening of cervical mucus
c. With immediate postabortion or postdelivery d. All of the above
placement, fewer women will receive and retain
their IUD compared to those scheduled to return 38–22 Ms. Hubbard is a 28-year-old G6P5 who is seeing
for traditionally timed placement. you for her postpartum exam. She had a repeat
d. All of the above cesarean delivery 2 weeks ago and desires reversible
contraception. Her pregnancy was complicated
38–18. Which of the following is true regarding progestin by gestational diabetes requiring insulin. She also
implants? takes medication for depression. She asks about
a. Implanon cannot be easily identified for removal Nexplanon. Which of the following is an appropriate
with ultrasound and has lost approval by the statement as part of her contraceptive counseling?
Food and Drug Administration. a. Nexplanon will further increase your risk for overt
b. Nexplanon provides 3 years of contraception by diabetes due to weight gain and increased insulin
releasing etonogestrel, and it is implanted in the resistance.
upper, medial surface of the arm. b. Nexplanon is a good choice, but may increase
c. The first implantable progestin contraceptive was your depression, so I would like you to call me if
Norplant, which released etonogestrel from six you notice changes.
subdermal rods, but it is no longer manufactured. c. According to the United Stated Medical Eligibility
d. All of the above Criteria, depression is a contraindication to pro-
gestin only contraceptive methods.
38–19. Ms. Bradley calls your office a few weeks after d. Nexplanon will reduce your breast milk
insertion of her Nexplanon and reports that she has production, but if you choose this method
had numbness and tingling in her arm. Which of the we should insert it today before ovulation can
following characterizes the most likely approximate resume.
region of her symptoms?
a. Anterior and posterior portions of her shoulder
b. Lateral surface of her forearm extending inferiorly
to the base of her thumb
c. Anterior and posterior portions of her 5th digit
and the medial half of her 4th digit
d. Medial aspect of her forearm extending superiorly
to the anterior surface of her upper arm
258 The Puerperium

38–23. Which of the following statements most accurately 38–27. Which of the following statements is true regarding
describes the injectable progestin contraceptive depot patient-specific factors that impact choice of
medroxyprogesterone acetate (DMPA)? combination oral contraceptive (COC) pill?
SECTION 10

a. When injection is given within 5 days of the a. Ongoing irregular bleeding may improve from
onset of menses, a backup method is needed for using a pill with a higher estrogen dose.
7 days. b. Women with significant menstrual symptoms
b. Although irregular bleeding leads to 25% of users may benefit from an extended cycle formulation.
discontinuing within the first year, amenorrhea c. Patients on heparin should have their potassium
develops in 80% of users after 1 year. monitored in the first month of using a COC
c. For women desiring only a brief period of with the progestin drospirenone.
contraception, DMPA is not a good choice due d. All of the above
to prolonged anovulation after discontinuation.
d. All of the above 38–28. Which of the following is not true regarding the
relationship of combination oral contraceptive
38–24. Combination oral contraceptive (COC) pills differ (COC) pills and thrombotic events?
from progestin-only “mini-pills” in which of the a. Etonogestrel-containing COCs are linked to
following ways? greater risk.
a. COC pills primarily inhibit ovulation, and progestin b. Highest risk is in women with a co-morbid
only pills do not reliably inhibit ovulation. thrombophilia.
b. COC pills are contraindicated in women with c. COCs are contraindicated for women over
breast cancer, but progestin only pills are not. 35 years of age who smoke.
c. When one COC dose is missed, a barrier method d. Thrombotic events are increased with COC use
should be used for 7 days. Whereas, when one in the 4 weeks prior to a major operation and in
progestin pill is taken >4 hours late, a barrier the first weeks postpartum.
method needs to be used for 48 hours.
d. All of the above 38–29. Relative risk is increased for which of the following
cancers with combination oral contraceptive (COC)
38–25. The combination oral contraceptive pill’s mechanism pill use?
of action is best described by which of the following a. Ovarian
statements? b. Cervical
a. The estrogen component suppresses luteinizing c. Endometrial
hormone. d. Hepatocellular carcinoma
b. Overall mechanism is stimulation of the hypo-
thalamic gonadotropin-releasing factors.
c. Pituitary secretion of follicle-stimulating hormone
and luteinizing hormone are blocked, and
ovulation is inhibited
d. All of the above

38–26. Undesirable effects attributed to the estrogen


component of combination oral contraceptive pills
include all except which of the following?
a. Headache
b. Hirsutism
c. Weight gain
d. Breast tenderness
Contraception 259

38–30. With use of the contraceptive method pictured, how 38–32. In counseling a woman who is considering the
frequently should it be replaced? contraceptive method pictured here, which of the
following is not true?

CHAPTER 38
a. Daily
b. Weekly a. The ring must be carefully inserted such that it
remains in an oblique lie around the cervix.
c. Every 9 days
b. The ring can be removed during intercourse but
d. Every 3 weeks
should be replaced within 3 hours of removal.
38–31. Ms. Adams is a 35-year-old multigravida who is c. The ring is more forgiving than combination oral
seeing you for her annual exam. She has hypertension contraceptive pills, as it remains efficacious if left
that is well controlled with a single agent. You are in place for a fourth week.
encouraged that three times per week she is still d. The ring is placed within 5 days of menses onset
doing the water aerobics you recommended last year. and stays in place for 3 weeks with removal
Her blood pressure is 130/78 mm Hg and weight is prompting withdrawal bleeding.
92 kg, which is down from 98 kg at last year’s visit.
She inquires about starting the contraceptive patch 38–33. Contraceptive efficacy of the male latex condom is
for her birth control method. Which characteristic enhanced by which of the following?
below is not a reason she cannot use the patch? a. Reservoir tip
a. Patient weight may reduce efficacy b. Oil-based spermicide
b. Regular immersion in water limits patch efficacy c. Concurrent female condom use
c. A 35-year-old with hypertension presents excess d. All of the above
cardiac risk
d. All of the above make this a poor choice 38–34. Ms. Burns is a 26-year-old G1P1001 with a partner
who has a history of herpes. She desires contracep-
tion, and advice on how she can reduce the risk for
getting genital herpes. In addition to counseling her
regarding symptoms and lesions, you discuss which
of the following?
a. The female condom is not an option if she is
allergic to latex.
b. The female condom has a lower contraceptive
failure rate than the male condom.
c. The female condom is a single-use barrier method
that provides both contraception and protection
against sexually transmitted diseases.
d. All of the above should be considered.
260 The Puerperium

38–35. Which of the following is not true regarding spermi- 38–38. Which of the following is false regarding emergency
cide use? contraception?
a. Spermicide enhances the contraceptive efficacy of a. The major mechanism of action with all methods
SECTION 10

other barrier methods. is inhibition or delay in ovulation.


b. Duration of maximal effectiveness is 6 hours, and b. Single dose regimen, such as Plan B One-Step is
they must be reinserted before repeat intercourse. available over the counter to all reproductive aged
c. Spermicides currently do not require prescription women.
in the United States and are available in several c. Outside of an allergy, there are no health
formats. comorbidities that contraindicate use of
d. Spermicides function by providing a physical emergency contraception.
barrier to sperm penetration and chemical d. The ParaGard method is most efficacious and
spermicidal action. carries the additional benefit of 10 subsequent
years of effective contraception.
38–36. Which of the following is not a variation of periodic
abstinence as a family planning method?
a. Withdrawal method
b. Standard days method
c. Cervical mucus method
d. Temperature rhythm method

38–37. Ms. Polly is a 24-year-old recently engaged G0 who


presents to discuss contraceptive methods in advance
of her wedding. She and her fiancé have not been
sexually active and do not intend to be until after the
wedding. She has no medical comorbidities and has
a normal body mass index. She has always had irreg-
ular menstrual cycles ranging from 28 to 40 days.
She is sure that they do not desire pregnancy for 1
to 2 years, and she has a religious objection to any
method that has prevention of implantation of a
fertilized embryo as a major mechanism of action.
Taking all these factors into account, what is the
most efficacious method you can recommend for
this patient?
a. Male condom
b. Intrauterine device
c. Fertility awareness-based method
d. Combination oral contraceptive pill
Contraception 261

CHAPTER 38 ANSWER KEY

Question Letter Page

CHAPTER 38
number answer cited Header cited
38–1 d p. 680 Table 38-1
38–2 b p. 680 Table 38-1
38–3 c p. 681 Introduction
38–4 d p. 681 Introduction
38–5 a p. 682 Intrauterine Devices
38–6 a p. 683 Intrauterine Devices
38–7 c p. 683 Intrauterine Devices
38–8 a p. 683 Intrauterine Devices
38–9 c p. 683 Intrauterine Devices
38–10 c p. 683 Intrauterine Devices
38–11 d p. 683 Intrauterine Devices
38–12 a p. 683 Intrauterine Devices
38–13 d p. 683 Intrauterine Devices
38–14 d p. 684 Intrauterine Devices
38–15 a p. 684 Intrauterine Devices
38–16 d p. 684 Intrauterine Devices
38–17 a p. 685 Intrauterine Devices
38–18 b p. 687 Progestin Implants
38–19 d p. 688 Progestin Implants
38–20 c p. 688 Progestin Implants
38–21 d p. 689 Progestin-Only Contraceptives
38–22 b p. 689 Progestin-Only Contraceptives
38–23 c p. 693 Injectable Progestin Contraceptives
38–24 a p. 690 Combination Oral Contraceptive Pills
p. 693 Progestin-Only Pills
38–25 c p. 689 Combination Hormonal Contraceptives Mechanism of Action
38–26 b p. 690 Combination Oral Contraceptive Pills
38–27 d p. 690 Combination Oral Contraceptive Pills
38–28 a p. 691 Combination Oral Contraceptive Pills
38–29 b p. 692 Combination Oral Contraceptive Pills
38–30 b p. 692 Transdermal Patch
38–31 b p. 692 Transdermal Patch
38–32 a p. 693 Transvaginal Ring
38–33 a p. 694 Barrier Methods
38–34 c p. 680 Table 38-1
p. 694 Barrier Methods
38–35 b p. 695 Spermicides
38–36 a p. 695 Fertility Awareness-Based Methods
38–37 d p. 680 Table 38-1
p. 683 Contraceptive Action
p. 689 Actions and Side Effects
p. 695 Fertility Awareness-Based Methods
38–38 c p. 696 Emergency Contraception
262

CHAPTER 39

Sterilization

39–1. Among women using contraception, what percentage 39–6. A 36-year-old G3P2 desires permanent sterilization
use sterilization? after delivery. She has read recently that the entire
a. 10% fallopian tube should be removed when perform-
ing a tubal ligation. What is the rationale for this
b. 15%
recommendation?
c. 20%
a. Decrease cancer risks
d. 33%
b. Decrease the failure rate
39–2. A 28-year-old G4P3 presents for prenatal care at c. Decrease bleeding complications
8 weeks’ gestation and reports she does not want any d. Decrease the risk for an ectopic pregnancy
more children. She asks for a postpartum bilateral
tubal ligation. All except which of the following are 39–7. What method of tubal ligation is displayed below?
accurate and should be included in your counseling?
a. It is permanent
b. There are failure rates
c. It can be reversed without consequence
d. An explanation of alternative options for
contraception

39–3. Which of the following aspects of postpartum anat-


omy are advantageous for a puerperal sterilization?
a. A noninvoluted uterus
b. Laxity of the abdominal wall
c. Fallopian tubes directly beneath the abdominal
wall
d. All of the above
Reproduced with permission from Cunningham FG, Leveno KJ, Bloom SL, et al
39–4. All except which of the following aspects of the (eds): Sterilization. In Williams Obstetrics, 25th ed. New York, McGraw-Hill, 2018,
intrapartum and postpartum periods should be Figure 39-1B.
accounted for when considering a puerperal tubal
sterilization? a. Uchida
a. Status of the newborn b. Pomeroy
b. Postpartum hemorrhage c. Parkland
c. Patient’s ability to ambulate d. Modified Pomeroy
d. Utilization of neuraxial anesthesia placed for labor
39–8. Sterilization in the puerperium is typically performed
39–5. Which of the following methods of puerperal using which of the following anesthetic methods?
sterilization carries the greatest morbidity? a. Spinal anesthesia
a. Hysterectomy b. General anesthesia
b. Salpingectomy c. Incision infiltration
c. Uchida method d. Transverse abdominis plane block
d. Parkland method
Sterilization 263

39–9. You are seeing a 33-year-old G3P2 for prenatal care. 39–12. Which of the following is true regarding identification
She had two prior vaginal deliveries and desires a of the structure being ligated?
bilateral tubal ligation after this pregnancy. She a. The distal fimbria must be seen prior to ligation.

CHAPTER 39
anticipates having another vaginal delivery and
b. Common reason for failure of sterilization is
wants to know what type of incision she will have
ligation of the wrong structure.
for her tubal ligation. Which of the answer choices
below depicts the best incision for a puerperal tubal c. The midportion of the fallopian tube can be
ligation? confused with that of the round ligament.
d. All of the above

39–13. What is the most common approach to the


nonpuerperal tubal ligation?
a. Colpotomy
b. Laparotomy
c. Laparoscopy
d. Minilaparotomy

39–14. What type of suture is typically used to complete the


tubal ligation picture below?

a. A
b. B
c. C
d. D

39–10. Why is an infraumbilical incision the most favorable


for a puerperal tubal ligation?
a. Smallest incision
b. Yields better cosmesis
c. Decreased risk for bleeding complications
d. It’s the thickest portion, which improves the Reproduced with permission from Cunningham FG, Leveno KJ, Bloom SL, et al (eds):
integrity of the incision. Sterilization. In Williams Obstetrics, 25th ed. New York, McGraw-Hill, 2018, Figure 39-2.

39–11. Which of the following are reasonable options to a. Vicryl


provide adequate exposure during a puerperal tubal
b. Chromic
ligation?
c. Monocryl
a. A larger incision
d. Plain catgut
b. Reverse Trendelenburg
c. Elevate the abdominal wall with your fingers 39–15. Why should that particular suture be used in
d. Tilt the table to same side as the tube being Question 39–14?
exposed a. Quick absorption allows separation of the severed
ends
b. Easier to tie knots with this suture compared to
others
c. Quick absorption leads to decreased adhesion
formation
d. Delayed absorption decreases the risk for bleeding
complications
264 The Puerperium

39–16. What is the cumulative failure rate for tubal 39–20. A 27-year-old multigravida tells you at her 28-week
sterilization? prenatal care visit that she desires a tubal ligation
a. 0.5% after delivery. For which of the following reasons do
SECTION 10

you counsel her on the permanence of the procedure


b. 1%
and encourage her to seek another method of
c. 3% contraception?
d. 5% a. It is associated with early menopause.
39–17. Which of the following are reasons for failure of b. Younger women have increased risk for regret.
tubal sterilization? c. It is associated with an increased risk for ovarian
cancer.
a. Spontaneous reanastomosis
d. There is a decrease in sexual interest after tubal
b. Transection of the round ligament
ligation.
c. Partial transection of the fallopian tube
d. All of the above 39–21. Which of the following factors favor successful tubal
reversal?
39–18. What benefits, other than contraception, does a a. Age <35 years
tubal ligation provide?
b. Isthmic-isthmic repairs
a. Decrease risk for menorrhagia
c. 7 cm of remaining tube
b. Decrease risk for breast cancer
d. All of the above
c. Decrease risk for ovarian cancer
d. All of the above 39–22. The following form of contraception is made of
what material?
39–19. A 37-year-old G3P3 underwent a bilateral tubal
ligation 2 years ago at the time of her third cesarean
section. She now presents with amenorrhea and nau-
sea and vomiting for the last month. You perform a
urine pregnancy test and it is positive. You perform
an ultrasound and discover the finding below. This
complication affects what percentage of pregnancies
after a tubal ligation?

a. Zinc
b. Copper
c. Progesterone
d. Nickel and titanium alloy

39–23. How does Essure work as a form of contraception?


a. Prevents ovulation
b. Alters cervical mucous
c. Tubal lumen occlusion
d. Thinning of the endometrium

39–24. What adverse effects are associated with Essure use?


a. 10%
a. Perforation
b. 20%
b. Allergic reactions
c. 30%
c. Abnormal bleeding
d. 70%
d. All of the above
Sterilization 265

39–25. When after Essure placement should confirmation 39–29. Which of the following is a reason why pregnancy
with a hysterosalpingography be performed? after a vasectomy may occur?
a. 3 weeks a. Recanalization

CHAPTER 39
b. 6 weeks b. Incomplete occlusion
c. 12 weeks c. Unprotected intercourse too soon
d. 16 weeks d. All of the above

39–26. During a vasectomy, which of the following structures 39–30. Pregnancy rates after vasectomy reversal increase
is ligated? with all except which of the following?
a. Epididymis a. Microsurgical technique
b. Spermatic cord b. Younger female partner age
c. Efferent ductile c. Longer duration from vasectomy to reversal
d. Ductus deferens d. Normal sperm quality during reversal procedure

39–27. Compared with tubal ligation, which of the 39–31. Compared to a traditional tubal ligation, which of
following is an advantage of vasectomy? the following is increased with a salpingectomy?
a. Less invasive a. Risk for bleeding
b. Easily reversed b. Risk for postoperative ileus
c. Immediately effective c. Risk for adhesion formation
d. Does not require confirmation d. All of the above

39–28. How long does it take for sperm to be completely 39–32. Why should the bladder be emptied prior to a
removed from the reproductive tract? puerperal tubal ligation?
a. 1 week a. Avoid bladder injury
b. 4 weeks b. Prevent postop urinary retention
c. 12 weeks c. Prevent the fundus from dropping to the pubic
d. 16 weeks symphysis
d. All of the above
266 The Puerperium

CHAPTER 39 ANSWER KEY

Question Letter Page


SECTION 10

number answer cited Header cited


39–1 d p. 702 Introduction
39–2 c p. 702 Introduction
39–3 d p. 702 Timing
39–4 c p. 702 Timing
39–5 a p. 703 Technique
39–6 a p. 703 Technique
39–7 c p. 703 Figure 39-1
39–8 a p. 703 Technique
39–9 d p. 703 Technique
39–10 b p. 703 Technique
39–11 a p. 703 Technique
39–12 d p. 703 Technique
39–13 c p. 704 Nonpuerperal Tubal Sterilization
39–14 d p. 704 Figure 39-2
39–15 a p. 704 Figure 39-2
39–16 a p. 704 Contraceptive Failure
39–17 d p. 705 Contraceptive Failure
39–18 c p. 705 Other Effects
39–19 c p. 705 Contraceptive Failure
39–20 b p. 705 Other Effects
39–21 d p. 705 Tubal Sterilization Reversal
39–22 d p. 705 Transcervical Sterilization
39–23 c p. 705 Transcervical Sterilization
39–24 d p. 705 Transcervical Sterilization
39–25 c p. 705 Transcervical Sterilization
39–26 d p. 706 Vasectomy
39–27 a p. 706 Vasectomy
39–28 c p. 706 Vasectomy
39–29 d p. 706 Vasectomy
39–30 c p. 706 Vasectomy
39–31 a p. 704 Technique
39–32 a p. 703 Technique
268

CHAPTER 40

Hypertensive Disorders

40–1. What percentage of pregnancies are complicated by 40–5. A 28-year-old G1 at 38 weeks’ gestation presents
hypertension? with complaint of contractions. Her blood pressure
a. 2–3% is noted to be 148/90 mm Hg and 152/96 mm Hg.
She has a urine protein:creatinine ratio of 0.4, a
b. 4–5%
creatinine of 1.04 mg/dL (baseline 0.48 mg/dL),
c. 5–10% normal AST and ALT, and platelet count of
d. 10–20% 110,000/µL. She denies any symptoms. What criteria
for severe preeclampsia does this patient meet?
40–2. What percentage of eclamptic seizures occur in a. Proteinuria
women without significant proteinuria?
b. Low platelets
a. 5–9%
c. Elevated creatinine
b. 10–17%
d. She does not meet criteria for severe preeclampsia.
c. 20–25%
d. 30–33% 40–6. Many conditions and factors are associated with
an increased risk for preeclampsia. Which of the
40–3. A 21-year-old G1 at 36 weeks’ gestation presents for following factors results in the greatest relative risk for
her clinic visit and is noted to have a blood pressure of a diagnosis of preeclampsia in the current pregnancy?
148/88 mm Hg. A repeat blood pressure 30 minutes a. Primigravida
later is 146/92 mm Hg. Her blood pressures through-
b. Advanced maternal age
out pregnancy have been below 140/90 mm Hg.
She denies any complaints, and urinalysis is negative c. Systemic lupus erythematous
for proteinuria. What is the most likely diagnosis? d. History of preeclampsia in a prior pregnancy
a. Delta hypertension
40–7. Which of the following is thought to play a
b. Chronic hypertension
significant role in the development of preeclampsia?
c. Preeclampsia syndrome
a. Genetic factors
d. Gestational hypertension
b. Immunological factors
40–4. What percentage of eclamptic seizures occur more c. Abnormal trophoblastic invasion
than 48 hours after delivery? d. All of the above
a. 2%
40–8. Which of the following angiogenic factors are
b. 5%
elevated in women who proceed to develop
c. 10% preeclampsia?
d. 20% a. sFlt-1
b. PIGF
c. VEGF
d. TGF-β
Hypertensive Disorders 269

40–9. Which of the following is not a pathophysiologi- 40–11. The patient Question 40–10 has a prolonged
cal change to the cardiovascular system seen in the induction receiving over 5 liters of intravenous fluids.
setting of preeclampsia? She requires 4 liters supplemental oxygen via nasal

CHAPTER 40
a. Increased preload cannula. On the below schematic, which position best
describes her hemodynamic state at this time?
b. Decreased preload
c. Increased afterload 120

d. Endothelial activation 110 Hyperdynamic

100
40–10. A 25-year-old primigravida at 36 weeks’ gestation D
presents with scotomata and headache, a blood 90
pressure of 168/102 mm Hg, and proteinuria. A

LVSWI (g·m·m–2)
diagnosis of severe preeclampsia is made. On the 80 Normal
below schematic, which position best describes her 70
hemodynamic state?
60
120 C
50 A
110 Hyperdynamic B
40 Depressed
100
D 30
90
LVSWI (g·m·m–2)

0
80 Normal 0 5 10 15 20 25 30
PCWP (mm Hg)
70
Reproduced with permission from Cunningham FG, Leveno KJ, Bloom SL, et al (eds):
60 Hypertensive disorders. In Williams Obstetrics, 25th ed. New York, McGraw-Hill, 2018,
C Figure 40-5.
50 A
B
40 Depressed a. A
b. B
30
c. C
0
0 5 10 15 20 25 30
d. D
PCWP (mm Hg)
40–12. A 21-year-old primigravida presents at 36 weeks’
Reproduced with permission from Cunningham FG, Leveno KJ, Bloom SL, et al (eds):
gestation with new-onset headache. Her blood
Hypertensive disorders. In Williams Obstetrics, 25th ed. New York, McGraw-Hill, 2018,
Figure 40-5. pressure is 150/90 mm Hg, her serum creatinine is
0.8 mg/mL, AST is 32 U/L, and platelet count is
a. A 28,000/µL. Which of the following criteria for severe
preeclampsia is met?
b. B
a. Hypertension
c. C
b. Liver dysfunction
d. D
c. Thrombocytopenia
d. Elevated serum creatinine
270 Obstetrical Complications

40–13. For the patient Question 40–12, which of the following 40–16. What proportion of maternal deaths can be attrib-
is an indication for primary cesarean delivery without uted to hypertensive disorders in pregnancy?
an attempt at induction? a. 1 in 2
SECTION 11

a. Malpresentation b. 1 in 6
b. Unfavorable cervix c. 1 in 10
c. Possible fetal thrombocytopenia d. 1 in 20
d. None of the above
40–17. A G3P2 at 16 weeks’ gestation presents to the emer-
40–14. Which statement best describes renal perfusion and gency department complaining of vaginal bleeding.
glomerular filtration rates in women with preeclampsia Her blood pressure is 148/96 mm Hg and she has
that has not yet progressed to severe disease? 3+ proteinuria on a specimen obtained during
a. Similar compared to normal pregnant values bladder catheterization. Which of the following may
explain the development of preeclampsia in this
b. Increased compared to normal pregnant values
patient?
c. Similar compared to normal nonpregnant values
d. Decreased compared to normal nonpregnant
values

40–15. A 24-year-old primigravida presents at 37 weeks’


gestation with headache, a blood pressure of
170/102 mm Hg, and severe right upper quadrant
pain. She is diagnosed with HELLP syndrome and
undergoes an uncomplicated induction of labor.
Her right upper quadrant pain persists, and a
computed tomography scan of her abdomen/pelvis is
completed with the findings as shown below. What
is denoted by the asterisk (*)?

a. Increased volume of chorionic villi


b. Extensive remodeling of the spiral arterioles
c. Increased invasion of extravillous trophoblastic
tissue
d. None of the above

40–18. What is the underlying etiology of the proteinuria


seen with preeclampsia?
a. Increased capillary permeability
b. Increased renal artery resistance
c. Increased glomerular filtration rate
a. Splenic infarction d. Increased systemic vascular resistance
b. Intrahepatic infarction
c. Subcapsular hematoma
d. Periportal hemorrhagic necrosis
Hypertensive Disorders 271

40–19. A 39-year-old G3P2 presents at 30 weeks’ gestation 40–23. A multiparous woman with no prenatal care presents
with hypertension, proteinuria, and headache. She is in active labor with a blood pressure of 156/92 mm Hg
diagnosed with severe preeclampsia, and induction and proteinuria. Her neonate is born vaginally after

CHAPTER 40
of labor is indicated. Her pregnancy is complicated an uncomplicated labor course, with the second
by the fetal karyotype shown below. What is the stage of labor lasting only 15 minutes. The infant
possible explanation for her increased risk for is noted to have petechiae on the scalp and chest,
preeclampsia with the below fetal karyotype? and oozing at the site of his heel stick. An initial
platelet count is 32,000/µL. Which of the following
disorders is least likely to be the cause of the infant’s
thrombocytopenia?
a. Aneuploidy
b. Maternal preeclampsia
c. Maternal autoimmune disorder
d. Alloimmune thrombocytopenia

40–24. A 23-year-old primigravida presents with a blood


pressure of 160/104 mm Hg, 3+ proteinuria, and
right upper quadrant discomfort. She has a vaginal
delivery complicated by bilateral sulcal tears and an
estimated blood loss of 1500 mL. She produces
110 mL of urine in the first 4 hours postpartum
and her serum creatinine rises from 0.98 mg/dL to
a. Increased antiangiogenic factor levels 1.42 mg/dL. What is the most likely explanation for
this finding?
b. Increased frequency of placenta mosaicism
c. Higher frequency of spiral arteriole atherosis
a. Postpartum hemorrhage
d. Increased levels of oxidative products in the b. Subcapsular liver hematoma
placenta c. Ureteral injury during laceration repair
d. Dehydration due to prolonged induction of labor
40–20. Which of the following nutritional supplements has
been shown to reduce the incidence of preeclampsia? 40–25. The following computed tomography image shows
a. Calcium cerebral edema in a postpartum patient with
b. Vitamin E hypertension, vision changes, and confusion. What
associated morbidity is the patient at risk for?
c. Ascorbic acid
d. None of the above

40–21. Which of the following physiological responses is


typically seen in preeclamptic patients?
a. Increased production of nitric acid
b. Increased sensitivity to angiotensin II
c. Decreased reactivity to norepinephrine
d. All of the above

40–22. The typical blood volume of a gravida at term is


4500 mL. In patients with preeclampsia, which of
the following would be the expected blood volume?
a. 2500 mL
b. 3200 mL
c. 4500 mL a. Hemiplegia
d. 5000 mL b. Cystic leukomalacia
c. Retinal artery occlusion
d. Transtentorial herniation
272 Obstetrical Complications

40–26. Which of the following interventions is least 40–30. A 42-year-old woman presents for prenatal care.
indicated for the patient in Question 40–25? She has a demanding professional career and reports
a. Mannitol she is unwilling to attend frequent prenatal visits at the
SECTION 11

end of pregnancy. You discourage this, explaining these


b. Dexamethasone
visits are recommended for which of the following
c. Antihypertensive medication reasons?
d. Intravenous immune globulin a. To aid in the early detection of preeclampsia
b. To detect possible malpresentation and discuss
40–27. Your patient is admitted to the hospital for evalua-
delivery planning
tion of new-onset hypertension at 30 weeks’ gesta-
tion. Which of the following tests or evaluations is c. To aid in the timely detection of fetal com-
least appropriate? plications such as growth restriction or
oligohydramnios
a. Maternal weight
d. All of the above
b. Protein:creatinine ratio
c. Cell-free DNA testing for aneuploidy 40–31. The patient in Question 40–30 presents to labor and
d. Sonographic evaluation of fetal growth delivery at 38 weeks’ gestation with contractions,
vaginal bleeding, and hypertension. She has a rapid
40–28. What is the sensitivity and specificity, respectively, of labor with recurrent late fetal heart rate decelerations and
hyperuricemia for the detection of preeclampsia? delivers a depressed infant within an hour of arrival.
a. 0–55%, 17–30% What is the most likely finding on pathological
examination of the placenta?
b. 0–55%, 77–95%
a. No abnormality
c. 65–90%, 20–55%
b. Chronic chorioamnionitis
d. 77–95%, 85–97%
c. Retroplacental clot and diffuse infarction
40–29. Which of the following medical therapies has been d. Small for gestational age with villous infarction
associated with a modest decrease in the incidence of
preeclampsia in women at increased risk? 40–32. A 21-year-old primigravida at 32 weeks’ gestation
a. Aspirin has new-onset hypertension and proteinuria. She
inquires as to whether or not initiation of an anti-
b. Metformin
hypertensive agent may be helpful. Which of the
c. Enoxaparin following complications is more likely when labetalol
d. Heparin sulfate is initiated for preeclampsia?
a. Preterm birth
b. Hypertensive urgency
c. Fetal growth restriction
d. None of the above
Hypertensive Disorders 273

40–33. A 24-year-old G4P2 at 26 weeks’ gestation presents


via emergency medical services after being found
unconscious at home by her 6-year-old child. She

CHAPTER 40
is oriented on arrival and found to have a blood
pressure of 180/110 mm Hg and 4+ proteinuria.
Upon questioning she reports a history of eclampsia
in her prior pregnancy at 34 weeks’ gestation. She
then has a 5-minute-long tonic-clonic seizure which
resolves with magnesium sulfate administration.
Once the patient is stabilized, she undergoes cesarean
delivery for malpresentation. The below magnetic
resonance imaging was performed postpartum. What
is the most likely finding on imaging?

Used with permission from Dr. Emily Calasanz.

a. Subdural hematoma
b. Parenchymal hemorrhage
c. Subarachnoid hemorrhage
d. Posterior reversible encephalopathy syndrome
274 Obstetrical Complications

CHAPTER 40 ANSWER KEY

Question Letter Page


SECTION 11

number answer cited Header cited


40–1 c p. 710 Introduction
40–2 b p. 711 Terminology and Diagnosis
40–3 d p. 711 Terminology and Diagnosis
40–4 c p. 712 Terminology and Diagnosis
40–5 c p. 712 Terminology and Diagnosis
40–6 d p. 714 Incidence and Risk Factors
40–7 d p. 715 Etiology
40–8 a p. 717 Pathogenesis
40–9 a p. 718 Pathophysiology
40–10 a p. 719 Pathophysiology
40–11 d p. 719 Pathophysiology
40–12 c p. 720 Pathophysiology
40–13 a p. 720 Pathophysiology
40–14 c p. 721 Pathophysiology
40–15 c p. 722 Pathophysiology
40–16 b p. 724 Incidence and Risk Factors
40–17 a p. 715 Etiology
40–18 a p. 721 Pathophysiology
40–19 a p. 716 Etiology
40–20 d p. 728 Prevention
40–21 b p. 717 Pathogenesis
40–22 b p. 719 Pathophysiology
40–23 b p. 720 Pathophysiology
40–24 a p. 721 Pathophysiology
40–25 d p. 725 Pathophysiology
40–26 d p. 725 Pathophysiology
40–27 c p. 727 Prediction
40–28 b p. 727 Prediction
40–29 a p. 729 Prevention
40–30 d p. 729 Preeclampsia
40–31 c p. 729 Preeclampsia
40–32 c p. 731 Preeclampsia
40–33 d p. 724 Pathophysiology
275

CHAPTER 41

Obstetrical Hemorrhage

41–1. What is the most important cause of maternal 41–4. For a woman measuring 5!0" and 120 lb, what is her
mortality worldwide? expected pregravid blood volume?
a. Infection a. 3000 mL
b. Hemorrhage b. 3250 mL
c. Pulmonary embolism c. 3500 mL
d. None of the above d. 3800 mL

41–2. Which of the following statements is accurate 41–5. Assuming a 50% increase in the blood volume of
concerning postpartum hemorrhage? a woman during pregnancy, what would the blood
a. The blood loss at delivery approaches the volume volume of a 5!2" woman who weighed 140 lb
of blood added during pregnancy. pregravid be at term?
b. Studies show that estimated blood loss reported is a. 4000 mL
often less than the actual blood loss. b. 4340 mL
c. The American College of Obstetricians and c. 4700 mL
Gynecologists defines postpartum hemorrhage as d. 4930 mL
cumulative blood loss of >100 mL accompanied
by symptoms and signs of hypovolemia. 41–6. Causes of uterine atony include which of the
d. None of the above following?
a. Obesity
41–3. Given the diagram below, which of the following
b. Placenta previa
statements is true?
c. Multiple fetuses
70
<500 mL d. Placental abruption
60
500–1000 mL
50 1000–1500 mL 41–7. A 33-year-old G4P3 at 35 weeks’ gestation presents
Percent of cases

>2500 mL to labor and delivery with a small amount of vaginal


40
bleeding. When you place her on the fetal monitor
30 there is a category III fetal heart rate tracing.
An emergent cesarean delivery is performed.
20
The 1-minute and 5-minute Apgars are 0 and 3,
10 respectively. Which of the following is the etiology?
0 a. Vasa previa
Vaginal Repeat cesarean Repeat cesarean
delivery delivery with hysterectomy b. Placenta previa
Reproduced with permission from Cunningham FG, Leveno KJ, Bloom SL, et al (eds): c. Placental abruption
Obstetrical hemorrhage. In Williams Obstetrics, 25th ed. New York, McGraw-Hill, 2018, d. All of the above
Figure 41-1.

a. Very few vaginal deliveries have blood loss less


than 500 mL.
b. All cesarean hysterectomies have blood loss
greater than 1000 mL.
c. A smaller percentage of vaginal deliveries have
blood loss greater than 1000 mL than cesarean
hysterectomies have blood loss less than 500 mL.
d. None of the above
276 Obstetrical Complications

41–8. Which of the following maneuvers should be 41–11. The patient in Question 41–10 continues bleeding
performed in the setting of postpartum hemorrhage after the interventions mentioned above. Which of
following a vaginal delivery? the following maneuvers might be employed?
SECTION 11

a. Evaluate birth canal for lacerations a. Hysterectomy


b. Evaluate the placenta for possible retained b. Bakri balloon placement
fragments c. Uterine compression sutures
c. The uterus should be manually explored, and d. All of the above
placental fragments removed
d. All of the above 41–12. The patient in Question 41–10 undergoes a
hysterectomy and 5 units of packed red blood cells
41–9. During evaluation of postpartum hemorrhage are given, but bleeding continues. What is the most
following a vaginal delivery, which of the following likely etiology of the bleeding?
maneuvers or medications might be used? a. Vaginal cuff bleeding
a. Bimanual uterine compression b. Dilutional coagulopathy
b. Ergot alkaloids for patients with hypertension c. Lacerated internal iliac artery
c. Carboprost tromethamine in patients with mild d. Placental implantation on the omentum
asthma
d. All of the above 41–13. Regarding the patient in Question 41–10, which of
the following interventions might have avoided or
41–10. A 34-year-old G3P3 begins having brisk bright red ameliorated this condition?
bleeding following completion of a vaginal delivery. a. Transfusion of platelets
You give her carboprost tromethamine and perform
b. Transfusion of whole blood
the maneuver pictured below. What else should be
immediately considered? c. Infusion of 5% albumin for initial resuscitation
d. All of the above

41–14. What is represented in the following image?

Reproduced with permission from Cunningham FG, Leveno KJ, Bloom SL, et al (eds):
Obstetrical hemorrhage. In Williams Obstetrics, 25th ed. New York, McGraw-Hill, 2018,
Figure 41-4.
Reproduced with permission from Cunningham FG, Leveno KJ, Bloom SL, et al (eds):
Obstetrical hemorrhage. In Williams Obstetrics, 25th ed. New York, McGraw-Hill, 2018,
a. Call for help Figure 41-5.

b. Ask for urgent help from anesthesia


a. Bakri balloon
c. Place large-bore intravenous lines, order blood,
and begin volume resuscitation b. Foley catheter
d. All of the above c. Blakemore tube
d. Jackson-Pratt drain
Obstetrical Hemorrhage 277

41–15. The following picture unfortunately depicts a maternal 41–19. The hematoma in the following picture is in which
death after postpartum hemorrhage. Which of the location?
following are possible contributors?

CHAPTER 41
Reproduced with permission from Cunningham FG, Leveno KJ, Bloom SL, et al (eds):
a. Ischiorectal hematoma
Obstetrical hemorrhage. In Williams Obstetrics, 25th ed. New York, McGraw-Hill, 2018, b. Periurethral hematoma
Figure 41-7.
c. Supralevator hematoma
a. Placenta previa d. Left-sided anterior perineal triangle
b. Uterine rupture
41–20. The following picture represents which of the
c. Uterine inversion following?
d. Chorioamnionitis

41–16. What percentage of women have lacerations at the


time of vaginal delivery?
a. 50%
b. 90%
c. 80%
d. 65%

41–17. Which maneuvers below should be performed when


repairing a cervical laceration?
a. Operator grasps lips of cervix with ring forceps.
b. Second assistant can provide better exposure with
vaginal wall retractors.
c. Assistant place downward pressure on the uterus
to expose the cervix better for the operator. Used with permission from Dr. Ed Wells.

d. All of the above a. Molar gestation


41–18. What is the appropriate management of vulvovaginal b. Placenta accreta
hematomas? c. Acute abruption
a. Surgical exploration in all cases d. Partial abruption
b. To prevent infection they should all undergo
ultrasound guided drainage. 41–21. Which of the following are risk factors for recurrent
abruption?
c. In a small hematoma, if pain is severe then ice
packs and analgesia are appropriate. a. Prior abruption
d. If bleeding ceases, small to moderate-sized b. Low birthweight
hematomas can be treated expectantly. c. Preterm rupture of membranes
d. None of the above
278 Obstetrical Complications

41–22. Your patient is a 22-year-old G2P1 at 36 weeks’ 41–26. How is a low-lying placenta defined?
gestation with a history of prior abruption during a a. The placenta is implanted in the lower uterine
spontaneous vaginal delivery. Which of the following segment.
SECTION 11

might be effective for preventing recurrence during


b. The placental edge covers the internal os, but not
this pregnancy?
completely.
a. Antepartum testing
c. The placental edge does not cover the internal os
b. Delivery at 38 weeks but lies within a 1-cm wide perimeter.
c. Cesarean delivery at 38 weeks d. The placental edge does not cover the internal os
d. Cesarean delivery at 39 weeks but lies within a 2-cm wide perimeter.

41–23. Which of the following is true concerning placental 41–27. What percentage of women who have a primary
abruption? cesarean with a placenta previa have hysterectomies
a. It may lead to a dilutional coagulopathy. performed?
b. Concealed abruption forces thromboplastin into a. 1%
large veins draining the implantation site. b. 2%
c. In most women with abruption severe enough to c. 6%
kill the fetus, the plasma fibrinogen level will be d. 10%
less than 150 mg/dL.
d. All of the above 41–28. Which of the following statements are true concern-
ing morbidly adherent placentas?
41–24. What is depicted in the following picture? a. Cesarean-scar pregnancies are a precursor to a
morbidly adherent placenta.
b. Placenta villi attach to smooth muscle fibers
rather than to decidual cells.
c. Abnormal placental adherence is in part due to
partial or total absence of Nitabuch layer.
d. All of the above

41–29. The picture below represents which of the following


placental pathologies?

a. Placenta previa
b. Placenta accreta
c. Couvelaire uterus
d. None of the above

41–25. A 22-year-old G2P1 at 28 weeks’ gestation presents


with rupture of membranes about 2 hours ago. What
is true about this patient concerning abruption?
a. She has a 5% risk for abruption. Used with permission from Dr. Ed Wells.
b. She has a 17% risk for abruption.
c. Her risk for abruption is not increased. a. Placenta increta
d. None of the above b. Placenta accreta
c. Placenta percreta
d. Placental abruption
Obstetrical Hemorrhage 279

41–30. From the following graph, which of the following 41–31. Which of the following can be seen in the ultra-
statements concerning morbidly adherent placenta sound picture below?
(MAP) is accurate?

CHAPTER 41
75
67
61
Accrete syndromes (percent)

50
40

25

11

3
a. Lacunae
0
0 1 2 3 4 5
b. Bridging vessels
Number of prior CDs in women c. Intraabdominal placental implantation
with current placenta previa d. None of the above
Reproduced with permission from Cunningham FG, Leveno KJ, Bloom SL, et al (eds):
Obstetrical hemorrhage. In Williams Obstetrics, 25th ed. New York, McGraw-Hill, 2018, 41–32. Which of the statements below accurately depicts the
Figure 41-26. difference between consumptive coagulopathy and
disseminated intravascular coagulation?
a. The highest risk for placenta previa is with your a. Abruption best represents a consumptive
5th pregnancy. coagulopathy.
b. Most women with MAP have had more than b. Loss of procoagulants with massive hemorrhage is
3 previous cesareans. the basis of dilutional coagulopathy.
c. With placenta previa, the more cesareans a c. A concealed abruption forces thromboplastin into
woman has had, the higher her risk for MAP. the circulation and leads to the consumption of
d. All of the above procoagulants.
d. All of the above
280 Obstetrical Complications

41–33. The slide pictured below is consistent with a fatal 41–37. Which hemostatic surgical procedure is to be per-
syndrome. Which of the following proposed diag- formed in the picture below?
nostic criteria are required to make the diagnosis?
SECTION 11

Reproduced with permission from Cunningham FG, Leveno KJ, Bloom SL, et al (eds):
Obstetrical hemorrhage. In Williams Obstetrics, 25th ed. New York, McGraw-Hill, 2018,
Figure 41-31a.
a. B-Lynch procedure
a. Clinical onset during labor or within 30 minutes b. Uterine artery ligation
of placental delivery. c. Vaginal artery ligation
b. Abrupt onset of cardiorespiratory arrest, or both d. Ovarian artery ligation
hypotension and respiratory compromise.
c. Documentation of overt disseminated intra- 41–38. Which hemostatic surgical procedure is
vascular coagulopathy must be detected prior demonstrated in the picture below?
to enough blood loss to cause dilutional
coagulopathy.
d. All of the above

41–34. Which of the bacteria below are associated with


consumptive coagulopathy?
a. Escherichia coli
b. Klebsiella pneumoniae
c. Group A streptococcus 2
5
d. Group B streptococcus 3
4

1 6
41–35. The use of whole blood for massive hemorrhage is
supported by which of the following?
a. Less renal failure
b. Less pulmonary edema
c. Fewer intensive care unit admissions
d. All of the above
Reproduced with permission from Cunningham FG, Leveno KJ, Bloom SL, et al (eds):
41–36. Viral infection risks from transfusion are accurately Obstetrical hemorrhage. In Williams Obstetrics, 25th ed. New York, McGraw-Hill, 2018,
depicted in which of the following statements? Figure 41-34b.

a. Hepatitis B risk is <1 per 100,000 units


a. B-Lynch procedure
b. Hepatitis C risk is 1 per 1–2 million units
b. Uterine artery ligation
c. Human immunodeficiency virus risk is 1 per
1–2 million units c. Vaginal artery ligation
d. All of the above d. Ovarian artery ligation
Obstetrical Hemorrhage 281

41–39. Ligation at which of the following labeled vessel


points in the retroperitoneum will decrease the pulse
pressure in the uterine artery?

CHAPTER 41
A

C
Reproduced with permission from Cunningham FG, Leveno KJ, Bloom SL, et al (eds):
Obstetrical hemorrhage. In Williams Obstetrics, 25th ed. New York, McGraw-Hill, 2018,
Figure 41-35.

a. Posterior division
b. Internal iliac vein
c. Internal iliac artery
d. External iliac artery
283

CHAPTER 42

Preterm Birth

42–1. What is the definition of very low birthweight? 42–6. At what gestational age does the Obstetric Care
a. <500 grams Consensus document recommend consideration of
neonatal resuscitation?
b. 500 to 1000 grams
a. 21 weeks’ gestation
c. 1000 to 1500 grams
b. 22 weeks’ gestation
d. 1500 to 2500 grams
c. 23 weeks’ gestation
42–2. What is the approximate preterm birth rate in the d. 24 weeks’ gestation
United States based on the most recently available
data (2015)? 42–7. A 25-year-old G2P1 presents at 23 weeks and 3 days’
a. 9.5% gestation with painful contractions. Her cervix is
dilated to 3 cm. Which interventions should be con-
b. 10.0% sidered based on her presentation?
c. 10.5% a. Magnesium sulfate
d. 11.0% b. Corticosteroid therapy
42–3. You deliver a 17-year-old G1 at 28 weeks’ gestation c. Cesarean delivery for fetal indications
following preterm labor. She gives birth to a female d. All of the above
infant weighing 1090 grams who is immediately taken
to the neonatal intensive care unit for evaluation. 42–8. Approximately what percentage of all preterm births
What would you tell her is the approximate survival in the United States occurs after 34 weeks’ gestation?
rate for her infant? a. 30%
a. 85% b. 50%
b. 90% c. 70%
c. 95% d. 90%
d. 99%
42–9. Of the responses listed below, which complication is
42–4. An infant born at 25 weeks’ gestation is at risk for all least commonly associated with preterm delivery?
except which of the following complications? a. Hypertension
a. Asthma b. Fetal complications
b. Blindness c. Placental abruption
c. Blood cancers d. None of the above
d. Pulmonary hypertension

42–5. What percentage of infants born at 22 weeks’


gestation is expected to survive without neuro-
developmental impairment?
a. 0.5%
b. 1.0%
c. 1.5%
d. 2.0%
284 Obstetrical Complications

42–10. A 26-year-old G3P2 presents for pregnancy 42–14. Which of the following bacteria is frequently detected
confirmation at 10 weeks’ gestation. She is noted to in the amnionic fluid of women with preterm labor?
have the ultrasound findings pictured. What is her a. Mycoplasma hominis
SECTION 11

risk for preterm birth based on the findings?


b. Gardnerella vaginalis
c. Ureaplasma urealyticum
d. All of the above

42–15. Which of the following lifestyle factors is not


associated with preterm birth?
a. Poverty
b. Short stature
c. Vitamin D deficiency
d. Advanced maternal age

42–16. A 32-year-old G2P1 presents at 16 weeks’ gestation


for prenatal care. She describes a history of preterm
premature rupture of membranes in her last pregnancy
with delivery at 31 weeks’ gestation. How significantly
increased is her preterm birth risk in her current
pregnancy based on this history?
a. 30%
a. 2-fold increase
b. 50%
b. 3-fold increase
c. 70%
c. 4-fold increase
d. 90%
d. Her recurrent preterm birth risk is not increased
42–11. Which of the following placental hormones may
42–17. A 22-year-old G2P1 at 14 weeks’ gestation complains
play a role in preterm birth caused by maternal–fetal
of malodorous vaginal discharge. A saline preparation
stress?
of the discharge is performed, and the findings are
a. Estrogen illustrated in the image below. What do you tell her
b. Human placental lactogen regarding her diagnosis?
c. Insulin-like growth hormone
d. Corticotropin-releasing hormone

42–12. Which of the following bacteria may enhance the


risk for preterm birth by secretion of hyaluronidase?
a. Escherichia coli
b. Peptostreptococcus
c. Bacterial vaginosis
d. Group B streptococcus

42–13. Which of the following enzymes is not involved in


the inflammatory cascade by which infection induces
preterm labor?
a. IL-4
b. IL-8
c. Tumor-necrosis factor alpha a. The condition is associated with frequent douching.
d. All of the above are involved b. There is an association with preeclampsia and
placental abruption.
c. Women with this diagnosis and a susceptible
genotype have a 6-fold increased risk for preterm
birth.
d. All of the above
Preterm Birth 285

42–18. You perform a routine cervical exam on a 39-year-old 42–21. The Food and Drug Administration approved
G3P2 at 30 weeks’ gestation. You find her cervix 17-hydroxyprogesterone caproate for the prevention
to be 2 to 3 cm dilated. She denies having any of recurrent preterm birth based on a 2003 Maternal-

CHAPTER 42
contractions, discharge, pelvic pain or pressure. Fetal Medicine Units (MFMU) Network study
What is her chance of a preterm delivery before performed by Meis et al. What was one of the major
34 weeks’ gestation? criticisms of this study?
a. 5% a. It was underpowered
b. 15% b. Injections were not initiated until 16 weeks’
c. 25% gestation
d. 35% c. There was an unexpectedly high preterm delivery
rate in the placebo arm
42–19. Which of the following is true regarding transvaginal d. All of the above
sonographic evaluation of the cervix as a part of the
assessment for preterm labor? 42–22. A recent study by Nelson et al showed that which of
a. It can be performed any time after 14 weeks’ the following complications might be increased in
gestation. pregnant women using 17-hydroxyprogesterone
caproate to prevent recurrent preterm birth?
b. It is not affected by maternal obesity, cervix
position, or shadowing. a. Depression
c. The American College of Obstetricians and b. Hypertension
Gynecologists recommends it for all women with c. Gestational diabetes
a history of a spontaneous preterm birth. d. Urinary tract infections
d. All of the above
42–23. A 23-year-old G1 is incidentally noted to have a
42–20. Your patient is found to have the sonographic cervical length of 18 mm at 21 weeks’ gestation.
finding shown below during her anatomy ultra- According to available research, which of the
sound at 22 weeks’ gestation. Her last pregnancy following therapies could be offered to potentially
was complicated by spontaneous preterm birth decrease her chance of preterm birth?
at 33 weeks’ gestation. Which of the following a. Cerclage
interventions could be recommended based on her
b. Vaginal progesterone
history and current findings?
c. 17-hydroxyprogesterone caproate
d. None of the above

42–24. In women who experience preterm premature


rupture of membranes between 24 and 34 weeks’
gestation, what percentage might be expected to still
be pregnant 48 hours after rupture?
a. 5–10%
b. 10–15%
c. 15–20%
d. 20–25%

42–25. A pregnant woman presents at 32 weeks and 5 days’


gestation complaining of mild contractions and
heavy discharge. You perform a speculum exam,
which demonstrates pooling in the posterior fornix.
Which of the following interventions would not be
a. Cerclage placement
considered based on the clinical presentation?
b. Aspirin 81 mg daily
a. Antimicrobials
c. Vaginal progesterone
b. Corticosteroids
d. None of the above
c. Magnesium sulfate
d. Expectant management
286 Obstetrical Complications

42–26. What appears to be the gestational age threshold 42–31. Which of the following is a potential consequence of
for lung hypoplasia in women with early membrane bed rest for suspected preterm labor?
rupture? a. Bone loss
SECTION 11

a. 19 weeks’ gestation b. Venous thromboembolism


b. 21 weeks’ gestation c. Increased risk for preterm delivery
c. 23 weeks’ gestation d. All of the above
d. 25 weeks’ gestation
42–32. An 18-year-old G1 African American female presents
42–27. Which of the following neonatal outcomes was to the emergency department at 19 weeks’ gestation
potentially improved following antimicrobial therapy complaining of pelvic pressure. She is found to be
for pregnant woman with premature rupture of 2 cm dilated with membranes prolapsing beyond
membranes before 35 weeks’ gestation? the os. No contractions are noted after extended
a. Improved survival monitoring. You perform a rescue cerclage as pictured
in the image below. Which of the following factors is
b. Decreased risk for sepsis
not associated with a decreased chance of pregnancy
c. Decreased risk for intracranial hemorrhage continuation after cerclage placement?
d. Decreased risk for respiratory distress syndrome

42–28. Which of the following drugs would not be


recommended either alone or in combination
for treatment of preterm premature rupture of
membranes?
a. Amoxicillin
b. Erythromycin
c. Ampicillin-sulbactam
d. None of the above

42–29. A pregnant patient is transferred to your facility


because of concerns for preterm labor at 35 weeks’
gestation. She received a single course of corticosteroids
prior to transfer. What neonatal complication is her
infant potentially at greater risk for?
a. Sepsis
a. Nulligravida
b. Hypothermia
b. African American race
c. Hypoglycemia
c. Membrane prolapse beyond the external os
d. Transient tachypnea of the newborn
d. Cerclage placement before 22 weeks’ gestation
42–30. Treatment with magnesium sulfate is often used
for neuroprotection for women at risk for delivery
prior to 32 weeks’ gestation. Approximately how
many women need to be treated with magnesium to
prevent one case of cerebral palsy?
a. 65
b. 70
c. 75
d. 80
Preterm Birth 287

42–33. A 28-week pregnant woman presents to labor and 42–34. Which of the following is true regarding
delivery with preterm contractions and is treated indomethacin use in pregnancy?
with terbutaline. Two days after the initiation of a. It can only be administered orally

CHAPTER 42
therapy, she begins complaining of shortness of
b. It can lead to reversible oligohydramnios
breath and cough. A chest radiograph is obtained
and shown in the image below. Which of the c. It lowers the risk for necrotizing enterocolitis
following risk factors is associated with development d. All of the above
of this complication?
42–35. Cesarean delivery would be expected to decrease
the risk for intracranial hemorrhage in which of the
following scenarios?
a. Estimated fetal weight <1000 grams
b. Estimated fetal weight <1500 grams
c. Estimated fetal weight <2000 grams
d. None of the above

a. Asthma
b. Gestational diabetes
c. Concurrent corticosteroid therapy
d. All of the above
289

CHAPTER 43

Postterm Pregnancy

43–1. A pregnancy is considered prolonged after how many 43–7. Maternal risks of postterm pregnancy include all
completed weeks? except which of the following?
a. 39 weeks a. Preeclampsia
b. 40 weeks b. Perineal lacerations
c. 41 weeks c. Postpartum hemorrhage
d. 42 weeks d. Morbidly adherent placenta

43–2. What is the most accurate way of dating a pregnancy? 43–8. What are the best estimates for the frequency of
a. Last menstrual period postmaturity syndrome in gestations that have
completed 42 weeks?
b. 1st-trimester ultrasound
a. 1%
c. 2nd-trimester ultrasound
b. 5%
d. Last menstrual period and a 2nd-trimester
ultrasound c. 10–20%
d. 25%
43–3. In the United States in 2013, what percentage of
pregnancies were considered postterm? 43–9. Which of the following is true the syndrome afflicting
a. 3% this infant?
b. 5%
c. 8%
d. 10%

43–4. Rare fetal–placental factors associated with postterm


pregnancy include which of the following?
a. Anencephaly
b. Wilms tumor
c. Adrenal hyperplasia
d. Autosomal-recessive placental sulfatase deficiency

43–5. What is a major cause of perinatal mortality associated


with pregnancy duration beyond 41 weeks?
a. Birth injuries
b. Gestational hypertension
c. Hypoxic-ischemic encephalopathy
d. All of the above

43–6. Which of the following have been found to be Reproduced with permission from Cunningham FG, Leveno KJ, Bloom SL, et al (eds):
increased in the setting of postterm pregnancies? Postterm pregnancy. In Williams Obstetrics, 25th ed. New York, McGraw-Hill, 2018,
Figure 43-2.
a. Neonatal seizures and deaths
b. Neonatal intensive care admissions a. Neurological deficits are found in 33%
c. Cesarean delivery for fetal distress b. Majority have birthweights <10th percentile
d. All of the above c. Features include simian crease and low-set ears
d. Oligohydramnios increases its likelihood at 42 weeks
290 Obstetrical Complications

43–10. Which of the following characterizes the placenta in


a postterm pregnancy?
a. Decreased rate of apoptosis
SECTION 11

b. Increased rates of placental separation


c. Downregulation of the kisspeptin gene
d. Increased levels of cord erythropoietin levels

43–11. A 33-year-old multigravida presents in labor


at 43 weeks’ gestation. She is 3 cm dilated and
approximately 2 hours after arrival undergoes an
emergent cesarean delivery due to fetal heart rate
abnormalities. Which of the following was most
likely her tracing prior to delivery?

a. c.

b. d.
Postterm Pregnancy 291

43–12. At the time of cesarean delivery in Question 43–11, 43–16. A 24-year-old primigravida presents to clinic at
which of the following was likely present? 42 weeks’ gestation after missing several clinic visits.
a. Meconium An ultrasound image of the deepest vertical pocket

CHAPTER 43
is displayed below. She is at risk for having which of
b. Polyhydramnios the following outcomes?
c. Short umbilical cord
d. Bloody amnionic fluid

43–13. When does the volume of amnionic fluid begin to


decline?
a. After 36 weeks
b. After 37 weeks
c. After 38 weeks
d. After 39 weeks

43–14. In which of the following conditions in pregnancy is


it generally recommended to be delivered at less than
42 weeks?
a. Diabetes
b. Gestational hypertension
a. Meconium
c. Previous cesarean delivery
b. Macrosomia
d. All of the above
c. Shoulder dystocia
43–15. Most stillbirths in prolonged pregnancies are associ- d. 5-minute Apgar ≤6
ated with which of the following?
43–17. Which of the following statements concerning
a. Oligohydramnios decreased amnionic fluid is false?
b. Late prenatal care a. Associated with fetal distress during labor
c. Fetal growth restriction b. Amnionic fluid index <5 cm is associated with
d. Shortened pregnancy interval more adverse outcomes than largest vertical
pocket <2 cm.
c. Amnionic fluid index is the best method
by which to estimate the occurrence of
oligohydramnios.
d. All of the above

43–18. According to the American College of Obstetricians


and Gynecologists, at what estimated fetal weight is
it reasonable to offer cesarean delivery to a nondia-
betic gravida at term?
a. 4250 grams
b. 4500 grams
c. 4750 grams
d. 5000 grams
292 Obstetrical Complications

43–19. Routine membrane sweeping on cervical exam at 43–25. With thick meconium early in the labor process,
38–40 weeks’ gestation has been shown to be which of the following is true?
associated with which of the following? a. Cesarean delivery is likely
SECTION 11

a. Increased pain b. Chances for vaginal delivery are diminished


b. Increased bleeding c. If delivery is remote, some obstetricians elect to
c. Lower rate of postterm pregnancies perform cesarean delivery
d. All of the above d. All of the above

43–20. In primigravidas undergoing induction of labor at 43–26. The American College of Obstetricians and
41 weeks’ gestation, what beginning fetal station Gynecologists recommends which of the following
was associated with the highest rate of cesarean in the setting of meconium-stained amnionic fluid?
section? a. Intubation if the baby is depressed
a. 0 b. Amnioinfusion only during active labor
b. –1 c. The pediatrician should immediately perform
c. –2 bulb suction on the warmer.
d. –4 d. The obstetrician should perform bulb suction
after delivery of the baby.
43–21. Which of the following is predictive of a successful
induction of labor? 43–27. A 44-year-old primigravida presents to clinic at
a. Cervical length <3 cm 40 weeks’ gestation. She wants to go into labor
naturally, and therefore wants to wait as long as
b. Cervical length <2.5 cm possible to be induced. Based on the American
c. Cervical dilation prior to induction College of Obstetricians and Gynecologists, when
d. All of the above should she be induced?
a. 40 weeks’ gestation
43–22. Comparing induction of labor at 41 weeks’ gestation b. 41 weeks’ gestation
to prolonging pregnancies with fetal testing, research
supports which of the following statements? c. 42 weeks’ gestation
a. Induction increases the rate of cesarean delivery. d. When the patient is ready
b. Induction increases the rate of postpartum
43–28. The patient in Question 43–27 inquires about the
hemorrhage.
risks of going past 41 weeks’ gestation. What are the
c. Induction increases the rate of anesthesia risks she is concerned about?
complications.
a. Macrosomia
d. Induction decreases the rate of meconium
b. Cesarean delivery
aspiration syndrome.
c. Postmaturity syndrome
43–23. What are considerations when considering d. Anesthesia complications
amniotomy during a postterm induction?
a. Increase risk for cord compression 43–29. The patient in Question 43–27 agrees to induction
of labor at 41 weeks’ gestation. How will you man-
b. Allows more precise fetal heart rate monitoring
age her pregnancy during this week?
c. Aids in identification of thick meconium in
amnionic fluid a. Fetal surveillance
d. All of the above b. Anesthesia consult
c. Weekly prenatal care visit only, as usual
43–24. When used during labor, amnioinfusion does which d. Cancel her clinic visit and see her on the day of
of the following? her induction
a. Prevents placental abruption
b. Decreases the occurrence of variable decelerations
c. Decreases the incidence of meconium aspiration
syndrome
d. None of the above
Postterm Pregnancy 293

43–30. The patient in Question 43–27 presents for her 43–32. The patient in Question 43–31 delivers a male
nonstress test, and her tracing is pictured below. Given infant, and the neonatologist at delivery suspects
the fetal heart rate tracing, what are you worried about? postmaturity syndrome. Her suspicion is based on

CHAPTER 43
which of the findings below?
a. Peeling skin
b. Long, thin body
c. Unusually old in appearance
d. All of the above

a. Meconium
b. Fetal acidemia
c. Oligohydramnios
d. Placental insufficiency

43–31. A 27-year-old primigravida presents to labor and


delivery reporting contractions. She reports scant
prenatal care, and based on her last menstrual period
she is 41 weeks pregnant. Bedside ultrasound is
consistent with her last menstrual period. She is not
in labor, but 1 cm dilated, and she would like to be
induced. What is the next best step?
a. Delivery
b. Follow-up in clinic in 1 week
c. Amniocentesis for fetal lung maturity
d. Schedule for induction at 42 weeks’ gestation
294 Obstetrical Complications

CHAPTER 43 ANSWER KEY

Question Letter Page


SECTION 11

number answer cited Header cited


43–1 d p. 835 Introduction
43–2 b p. 835 Estimated Gestational Age
43–3 c p. 836 Incidence
43–4 a p. 836 Incidence
43–5 d p. 836 Perinatal Morbidity and Mortality
43–6 d p. 836 Perinatal Morbidity and Mortality
43–7 d p. 836 Perinatal Morbidity and Mortality
43–8 c p. 837 Postmaturity Syndrome
43–9 d p. 837 Postmaturity Syndrome
43–10 d p. 837 Placental Dysfunction
43–11 b p. 838 Fetal Distress and Oligohydramnios
43–12 a p. 838 Fetal Distress and Oligohydramnios
43–13 c p. 838 Fetal Distress and Oligohydramnios
43–14 d p. 839 Complications
43–15 c p. 839 Fetal Growth Restriction
43–16 d p. 839 Figure 43-5
43–17 d p. 839 Oligohydramnios
43–18 d p. 839 Macrosomia
43–19 d p. 840 Induction Factors
43–20 d p. 840 Induction Factors
43–21 d p. 840 Induction Factors
43–22 d p. 840 Induction versus Fetal Testing
43–23 d p. 841 Intrapartum Management
43–24 b p. 841 Intrapartum Management
43–25 d p. 841 Intrapartum Management
43–26 a p. 842 Intrapartum Management
43–27 c p. 840 Induction versus Fetal Testing
43–28 b p. 840 Induction versus Fetal Testing
43–29 a p. 841 Figure 43-6
43–30 c p. 838 Figure 43-4
43–31 a p. 841 Management Strategies
43–32 d p. 836 Postmaturity Syndrome
324

CHAPTER 49

Cardiovascular Disorders

49–1. What percentage of pregnancy-related deaths in the 49–5. Regarding diagnostic studies, which of the following
United States were attributable to cardiovascular correctly describes a normal change that is character-
diseases between 2011 and 2013? istic of pregnancy?
a. 10–15% a. Improved systolic function on echocardiographic
b. 15–20% assessment
c. 20–25% b. A decrease in the cardiac silhouette size on chest
radiograph
d. 25–30%
c. An average 15-degree left axis deviation on
49–2. Which of the following does not contribute to the electrocardiogram
40% increase in cardiac output seen during normal d. All of the above
pregnancy?
a. Increased heart rate 49–6. A 32-year-old G2P1 presents to your office at
17 weeks’ gestation. She reports a history of tetral-
b. Increased left ventricular contractility
ogy of Fallot, which was repaired in infancy. She
c. Decreased systemic vascular resistance appears comfortable on the exam table with a heart
d. All of the above contribute to increased cardiac rate of 92 beats per minute and a blood pressure of
output in pregnancy 96/62 mm Hg. However, she does report significant
palpitations and dyspnea when climbing the two
49–3. You are caring for a 24-year-old G1 with severe flights of stairs to her apartment on the third floor.
aortic stenosis. When in pregnancy are you most Which New York Heart Association class best
concerned about cardiac decompensation? describes her functional disability?
a. Peripartum a. Class I
b. During the first trimester b. Class II
c. During the second trimester c. Class III
d. During the third trimester d. Class IV

49–4. Which of the following best describes the remodel- 49–7. You are caring for a pregnant patient with mitral
ing of the heart that occurs in normal pregnancy? stenosis. She is able to perform most activities of
a. Eccentric left ventricular mass expansion with daily living without significant limitation. On
spherical remodeling echocardiogram, her ejection fraction is 50% and
her mitral valve area measures 1.8 cm2. Which of
b. Concentric left ventricular mass expansion with
the following variables is most predictive of a
spherical remodeling
high risk for cardiac complications during this
c. Eccentric left ventricular mass expansion with pregnancy?
longitudinal remodeling
a. Her ejection fraction
d. Concentric left ventricular mass expansion with
longitudinal remodeling b. Her functional capacity
c. Her degree of left-sided obstruction
d. None of the above
Cardiovascular Disorders 325

49–8. The World Health Organization identifies which of 49–12. You obtain a chest radiograph for the patient in
the following conditions as prohibitive of pregnancy? Question 49–11 at 14 weeks’ gestation because of
a. Heart transplantation a persistent cough and shortness of breath. Her

CHAPTER 49
chest x-ray is shown. What is her mortality risk in
b. Cyanotic heart disease
pregnancy based on her history of mechanical valve
c. Pulmonary artery hypertension replacement?
d. Systemic ventricular dysfunction with left
ventricular ejection fraction of 35%

49–9. For which of the following cardiac conditions is


conduction analgesia highly recommended?
a. Severe aortic stenosis
b. Pulmonary artery hypertension
c. Repaired hypoplastic left heart syndrome
d. All of the above

49–10. You are caring for a 28-year-old G1 with a history


of moderate mitral stenosis who presented in active
labor and is now 8 cm dilated. She desires a natural
labor and declines pain medications during labor.
As her contractions get closer together, she reports
increasing pain, is visibly uncomfortable, and inter-
mittently becomes tachycardic to 130 beats per
a. 0.1–0.5%
minute. She suddenly begins complaining of short-
ness of breath and her oxygen saturation declines b. 1–2%
to 90% on room air. What is your best course of c. 3–4%
action? d. 5–6%
a. Administer metoprolol
b. Administer intravenous Lasix 49–13. In a pregnant woman who has previously
undergone a heart transplantation, what is the
c. Request immediate anesthesia assessment
approximate risk for suffering a rejection episode
d. All of the above during pregnancy?
a. 10%
49–11. A 30-year-old G2P0A1 presents at 11 weeks’ gesta-
tion to establish care. She has a history of rheumatic b. 20%
heart disease and had a mechanical valve replacement c. 30%
at age 20. She was previously on warfarin 7.5 mg d. 40%
daily, but stopped taking her medication when she
found out she was pregnant 1 week ago. What is her
risk for warfarin embryopathy based on her warfarin
dosing?
a. 1%
b. 3%
c. 5%
d. 8%
326 Medical and Surgical Complications

49–14. At 32 weeks’ gestation, an uncomplicated primi- 49–16. A graphic of pulmonary capillary wedge pressure
gravida you are caring for complains of shortness of (red line) is shown for a woman with mitral stenosis.
breath and chest pain with minimal exertion. You “A” represents the first stage of labor, “B” marks the
SECTION 12

obtain an echocardiogram, which demonstrates a second stage of labor, “C” demonstrates the immedi-
mitral valve surface area of 2.1 cm2, a dilated left ate postpartum period, and “D” shows the period
atrium, an aortic valve area of 3.7 cm2, a right 4–6 hours postdelivery. What accounts for the rise
ventricular systolic pressure of 20 mm Hg, and an in pulmonary capillary wedge pressure immediately
ejection fraction of 55%. You also obtain a chest following delivery?
x-ray, which is shown. What is the likely underlying

Pulmonary capillary wedge pressure


cause of her symptoms? 25

20

15

(mm Hg)
10
A B C
D E
5

0
A B C D E
Time-arbitrary units
Reproduced with permission from Cunningham FG, Leveno KJ, Bloom SL, et al (eds):
Cardiovascular disorders. In Williams Obstetrics, 25th ed. New York, McGraw-Hill, 2018,
Figure 49-3.

a. Aortic stenosis a. Heart rate normalization


b. Mitral stenosis b. Placental autotransfusion
c. Left ventricular heart failure c. Increase in systemic vascular resistance
d. Pulmonary artery hypertension d. Pitocin-induced increase in cardiac output

49–15. How would you manage the patient in Question


49–14?
a. Administer diuretics
b. Initiate anticoagulation
c. Recommend immediate labor induction
d. All of the above
Cardiovascular Disorders 327

49–17. The patient whose heart is shown in this image has 49–20. A patient with a term gestation and a history of
systemic lupus erythematosus and has a history of a critical aortic stenosis presents in active labor. Which
prior stroke. The left atrium (LA) and left ventricle of the following management strategies is advisable

CHAPTER 49
(LV) are identified, and the arrows point to nonin- while caring for her in her labor course?
fectious vegetations on the mitral leaflets. Which of a. Assisted second stage delivery
the following is the likely condition associated with
b. Slow-dose epidural anesthesia
her mitral insufficiency?
c. Generous fluid administration
d. All of the above
49–21. You are caring for a patient with an unrepaired
ventricular septal defect who is considering preg-
nancy. You perform an echocardiogram to assess
her future risks. Which of the following parameters
would indicate an extremely elevated maternal and
fetal risk during pregnancy?
a. A septal defect measuring 0.7 cm2
b. Presence of left ventricular hypertrophy
c. A left ventricular ejection fraction of 50%
d. A right ventricular systolic pressure of 75 mm Hg
49–22. What associated condition is the patient in
Question 49–21 at high risk for developing based
on her history?
a. Embolic stroke
b. Bacterial endocarditis
c. Venous thromboembolism
d. None of the above
a. Antiphospholipid antibodies
b. Infarction of the papillary muscle 49–23. A schematic of Eisenmenger syndrome due to a
ventricular septal defect is depicted. What is the
c. Calcification of the mitral annulus
most common cause of death in pregnant women
d. None of the above with this condition?

49–18. Which of the following conditions usually improves Initial left-to-right shunt Ultimate right-to-left shunt

during pregnancy due to pregnancy-induced


hypervolemia?
a. Aortic stenosis Pulmonary
artery
Pulmonary
artery
Pulmonary
b. Mitral stenosis Left
arteriole
atrium Left
c. Mitral valve prolapse Right
atrium
Right
d. All of the above atrium atrium

Left Left
49–19. In the United States, what is the most common ventricle
Right ventricle
ventricle
Right
cause of aortic stenosis? ventricle
hypertrophy
Narrowed pulmonary
a. Idiopathic arterioles result and
lead to pulmonary
hypertension
b. Bicuspid aortic valve
c. Infectious endocarditis Reproduced with permission from Cunningham FG, Leveno KJ, Bloom SL, et al (eds):
Cardiovascular disorders. In Williams Obstetrics, 25th ed. New York, McGraw-Hill, 2018,
d. Rheumatic heart disease Figure 49–4.

a. Sepsis
b. Cardiac tamponade
c. Left ventricular failure
d. Right ventricular failure
328 Medical and Surgical Complications

49–24. In a pregnant population, what is the most common 49–28. You are caring for a primigravida with known hyper-
cause of the condition portrayed in the chest x-ray trophic cardiomyopathy. She tells you her mother
image below? also suffered from the condition and passed away
SECTION 12

at an early age. The patient is concerned about her


child’s future risk for developing the same condition.
What do you tell her is the most likely risk for
passing the condition to her child?
a. <1%
b. 25%
c. 50%
d. 100%

49–29. You are the on-call hospitalist when you receive


an urgent call from the postpartum floor about a
patient who is complaining of chest pain and short-
ness of breath. She had an uncomplicated vaginal
delivery the previous day and has no significant past
medical or surgical history. You obtain a stat chest
x-ray, which is shown. Based on the findings, what
a. Idiopathic are you most concerned for?
b. Left-sided heart disease
c. Obstructive sleep apnea
d. Connective tissue disease

49–25. What is the gold standard for diagnosis of the condi-


tion in Question 49–24?
a. Echocardiography
b. Chest radiography
c. Electrocardiography
d. Right heart catheterization

49–26. A 32-year-old G2P1 transfers to your care at


18 weeks’ gestation. She has a history of pulmo-
nary hypertension due to chronic thromboembolic
disease. She was told by her previous obstetrician
Reproduced with permission from Cunningham FG, Leveno KJ, Bloom SL, et al (eds):
that pregnancy was contraindicated with her condi- Cardiovascular disorders. In Williams Obstetrics, 25th ed. New York, McGraw-Hill, 2018,
tion and is very concerned about her mortality risk. Figure 49-5.
What is her approximate mortality risk in pregnancy
based on the underlying cause of her pulmonary a. Pneumonia
hypertension? b. Cardiomyopathy
a. 1% c. Pulmonary edema
b. 5% d. Pulmonary hypertension
c. 10%
d. 20% 49–30. You obtain an echocardiogram on the patient in
Question 49–29. Her ejection fraction is found to be
49–27. At 24 weeks’ gestation the patient in Question 49–26 20% with no significant structural or valvular disease
begins to develop orthopnea and dyspnea on exer- noted. Which hormone has been implicated in the
tion. What treatment would you consider to decrease development of this condition?
her symptoms? a. Estrogen
a. Diuretics b. Prolactin
b. Supplemental oxygen c. Progesterone
c. Pulmonary vasodilator drugs d. Human placental lactogen
d. All of the above
Cardiovascular Disorders 329

49–31. Based on her diagnosis and assuming a recovery of 49–34. Which of the following patients do not have an indi-
systolic function by 6 months postpartum, what is cation for endocarditis prophylaxis?
the patient in Question 49–30’s risk for suffering a a. A patient with a porcine mitral valve

CHAPTER 49
relapse of the same condition in a future pregnancy?
b. A patient with a history of endocarditis
a. 5%
c. A patient with Eisenmenger syndrome due to an
b. 10% atrial septal defect
c. 20% d. A patient with a ventricular septal defect repaired
d. 30% with a synthetic patch without a current defect

49–32. What is the most common cause of heart failure in 49–35. You are caring for a patient with Marfan syndrome
pregnancy? who just found out she is 13 weeks pregnant. You
a. Idiopathic obtain an echocardiogram which demonstrates an
aortic root measurement of 4.5 cm. Based on these
b. Severe preeclampsia
findings, what would you recommend?
c. Chronic hypertension
a. Beta blocker initiation
d. Chronic hypertension with superimposed
b. Bimonthly echocardiograms
preeclampsia
c. Consideration of pregnancy termination
49–33. A pregnant woman presents to the emergency room d. All of the above
in an obtunded state. She has a fever to 102.6°F. On
exam, a 4/6 murmur is noted, she withdraws from 49–36. Which of the following therapies is safe in pregnancy
pain during her abdominal exam, and track marks to treat acute supraventricular tachycardia?
are noted on her arms and legs. Her fundal height is a. Valsalva maneuver
approximately 32 cm. An emergent echocardiogram b. Intravenous adenosine
is obtained, which demonstrates a mitral valve veg-
etation. What is the most likely organism underlying c. Synchronized cardioversion
her diagnosis? d. All of the above
a. Enterococcus
b. Staphylococcus aureus
c. Neisseria gonorrhoeae
d. Staphylococcus epidermidis
331

CHAPTER 50

Chronic Hypertension

50–1. Which of the following factors does not impact rest- 50–4. Regarding the patient in Question 50–3, accord-
ing blood pressure reading? ing to the recommendations of the Eighth Joint
a. Race National Committee, what is your goal blood pres-
sure with therapy?
b. Gender
a. <120/80 mm Hg
c. Activity level
b. <130/85 mm Hg
d. Age and weight
c. <140/90 mm Hg
50–2. The threshold of 140/90 mm Hg as the upper limit d. <150/100 mm Hg
of normal is based on which of the following?
a. Actuarial tables based on data from white adult 50–5. Ms. Thomas presents for a preconception coun-
males seling visit. She is a 37-year-old G0 with a body
mass index of 38 kg/m2, 4-year history of chronic
b. A large, widely sampled international popula-
hypertension treated with amlodipine 5 mg per day,
tion of men and women of various ages with no
and a 2-year history of diabetes which she manages
known health complications
with diet and exercise. Her blood pressure range is
c. The discrete point at which risk for myocardial 128–144/80–94 mm Hg from the week prior. What
infarction, stroke, renal failure, and peripheral workup would you like to pursue referent to her
artery disease sharply increases diagnosis of chronic hypertension and prepregnancy
d. None of the above risk stratification?
a. Serum creatinine and urine protein-to-creatinine
50–3. A 37-year-old G2P2 with no medical diagnoses and ratio
a normal body mass index saw your partner for an
b. Serum creatinine, urine protein-to-creatinine
annual well woman exam. Her blood pressure was
ratio, electrocardiogram, and maternal
142/92 mm Hg, and after checking several times
echocardiogram
over the next week at home, she calls with a blood
pressure log that ranges from 140–154/90–100 mm c. Serum creatinine, 24-hour urine collection for
Hg. Your partner is now out on maternity leave, protein and creatinine clearance, electrocardio-
so prior to calling her back you ask your nurse to gram, and cardiac stress test
confirm that patient’s ethnicity. Why is her ethnicity d. No further evaluation is indicated, as she has only
important in your recommendations to initiate an had a diagnosis of hypertension for 4 years and
antihypertensive agent? her blood pressure is well controlled on a single
a. Beta blockers are recommended as initial therapy medication.
in the Asian population.
b. Beta blockers are not recommended as initial
therapy in the Asian population.
c. Angiotensin-converting enzyme inhibitors and
angiotensin-receptor blockers are recommended
as initial therapy in the black population.
d. Angiotensin-converting enzyme inhibitors and
angiotensin-receptor blockers are not recom-
mended as initial therapy in the black population.
332 Medical and Surgical Complications

50–6. The patient in Question 50–5 has a copy of her 50–9. According to the Nationwide Patient Sample data,
records from her primary care physician. Review of which of the following is the most frequent comor-
the records shows a recent serum creatinine of bidity associated with chronic hypertension?
SECTION 12

0.9 mg/dL, urine protein-to-creatinine ratio of a. Depression


0.22, a normal electrocardiogram, and she had a
b. Hypothyroidism
normal exercise stress test performed 3 months ago.
Fasting glucose was 93 mg/dL this morning, and her c. Pregestational diabetes
hemoglobin A1C was 6.0 last month. Which of the d. Systemic lupus erythematosus
following is the most appropriate recommendation?
a. You do not yet have adequate information from 50–10. Pregnancy is associated with which of the follow-
which to make a recommendation. ing blood pressure changes in women with chronic
hypertension?
b. Risk for adverse outcome in pregnancy is unac-
ceptably high, and you recommend that she a. Blood pressure decreases from baseline in early
consider gestational surrogacy or adoption. pregnancy
c. There is no contraindication to pregnancy, but b. Blood pressure reaches its nadir at approximately
she can further reduce her risk with weight loss 20 weeks’ gestation
and regular, moderate intensity aerobic exercise c. In pregnancy, women with chronic hypertension
3–4 times per week. have persistently elevated vascular resistance
d. There is no contraindication to pregnancy, but d. All of the above
you recommend converting to an alternative
antihypertensive agent in advance of pursuing 50–11. Among women with chronic hypertension, which of
pregnancy to reduce teratogenic risk. the following is true regarding risk for adverse mater-
nal and perinatal outcome?
50–7. For a woman who does not enter pregnancy with a a. Chronic hypertension is associated with a twofold
known diagnosis, the diagnosis of chronic hyperten- increased risk for maternal death.
sion is supported when hypertension is present prior b. The risk is inversely proportional to the severity
to what gestational age threshold? and duration of hypertension prior to pregnancy.
a. 14 weeks’ gestation c. Complications are more likely with severe range
b. 20 weeks’ gestation hypertension at baseline and the presence of end-
c. 24 weeks’ gestation organ damage.
d. 28 weeks’ gestation d. All of the above

50–8. A 28-year-old G3P1102 at 9 weeks’ gestation presents 50–12. Ms. Clamp is a 30-year-old G1 presenting for ini-
for establishment of prenatal care. Her first pregnancy tiation of prenatal care at 12 weeks’ gestation. Her
was complicated by gestational hypertension at term, medical history is notable for chronic hypertension,
and her second was complicated by iatrogenic preterm and she has been on antihypertensive therapy for
delivery at 36 weeks’ gestation for preeclampsia. Her 3 years. Her blood pressure is 148/94 mm Hg. How
blood pressure was normal at her postpartum visit would you best counsel her regarding the risk for
2 years ago but has not been assessed since. Today her developing superimposed preeclampsia?
body mass index is 46 kg/m2 and her blood pressure a. The risk for developing superimposed preeclamp-
is 158/96 mm Hg. As instructed, she calls back 2 days sia is in the 7–9% range.
after her visit to report that blood pressures have been b. The risk is highest if workup reveals proteinuria
in the 146–160/94–100 mm Hg range at home. In or other end-organ impact.
addition to initiation of an antihypertensive agent,
what is the next step in management of her blood c. The risk for superimposed preeclampsia is
pressure in this pregnancy? not related to the severity of her baseline
hypertension.
a. Baseline serum creatinine
d. Maternal serum markers yield a high positive pre-
b. Renal ultrasound with Doppler dictive value in discriminating who will and will
c. Baseline 24-hour urine collection for protein, cre- not develop superimposed preeclampsia.
atinine clearance, and urinary metanephrines
d. All of the above
Chronic Hypertension 333

50–13. Which of the following is not true regarding the 50–16. Risk for the complication pictured below is increased
complication shown here, in the computerized in pregnancies complicated by chronic hypertension.
tomography scan image? How would you best characterize the risk?

CHAPTER 50
Used with permission from Dr. Patricia Santiago-Munoz.
a. This occurs during pregnancy in approximately
3 per 1000 women with chronic hypertension.
a. Incidence not impacted by the severity of blood
b. The risk is not increased among women with pressure
chronic hypertension in the absence of superim-
posed preeclampsia. b. 2-3-fold increase over the general obstetric
population
c. Systolic blood pressure ≥160 mm Hg or diastolic
blood pressure ≥110 mm Hg can rapidly result c. 5-6-fold increase over the general obstetric
in the finding shown here. population
d. None of the above d. Incidence not impacted by development of
superimposed preeclampsia
50–14. Which of the following interventions is recom-
mended by the American College of Obstetricians 50–17. For women with chronic hypertension, which modi-
and Gynecologists in their 2016 Clinical Update to fiable factor further increases the risk for the compli-
reduce risk for developing preeclampsia in women cation referenced in Question 50–16?
with chronic hypertension? a. Obesity
a. At least 3 months of preconception supplementa- b. Exercise
tion with vitamins C and E. c. Smoking
b. Initiation of low dose aspirin as early as possible d. Marijuana use
and continuation until 28 weeks’ gestation.
c. Initiation of low dose aspirin from 12–28 weeks’ 50–18. Which of the following approximates the risk for
gestation with continuation until delivery. perinatal mortality in pregnancies complicated by
d. No intervention has shown benefit over pre- chronic hypertension?
conception and early pregnancy folic acid a. Approximately 3% of births in women with mild
supplementation. hypertension
b. Approximately 10% of births in women with
50–15. In addition to chronic hypertension, which of the severe hypertension
following conditions are considered high-risk for c. 3- to 4-fold increase over pregnancies not compli-
developing preeclampsia by the American College of cated by chronic hypertension
Obstetricians and Gynecologists in their 2016
Clinical Update guidelines? d. All of the above
a. Asthma
b. Systemic lupus erythematosus
c. History of gestational diabetes
d. Elevated maternal serum alpha-fetoprotein
334 Medical and Surgical Complications

50–19. Which of the following is a major factor contribut- 50–22. A 43-year-old G5P3013 presents to labor and deliv-
ing to the incidence of perinatal mortality noted in ery at 32 weeks’ gestation with contractions. She
pregnancies complicated by chronic hypertension? only recently realized she was pregnant and has not
SECTION 12

a. Fetal growth restriction been able to establish prenatal care. She reports a
history of chronic hypertension and type 2 diabetes.
b. Iatrogenic preterm birth
Her blood pressure is 136/80 mm Hg. She is taking
c. Superimposed preeclampsia an unknown medication for her hypertension and
d. All of the above manages her diabetes with diet. Her fetal heart rate
tracing is shown below. An ultrasound is performed,
50–20. A 36-year-old multigravida with chronic hyperten- and the amnionic fluid index is 2.4 cm. You suspect
sion presents for a growth ultrasound at 36 weeks’ that this may be related to the antihypertensive agent
gestation. The ultrasound report is displayed below. she is taking. Use of which of the following classes of
Which of the following is true regarding the diagno- antihypertensive agents is contraindicated during all
sis made on the growth ultrasound? trimesters of pregnancy?

a. The risk is reduced with administration of the


antioxidants, vitamin C and vitamin E. a. Those that act centrally to reduce sympathetic
b. The risk is not modified by whether a woman outflow
with chronic hypertension requires treatment. b. Those that interfere with binding sites on
c. The risk is approximately 20% in women with voltage-dependent calcium channels.
chronic hypertension, and approximately 50% if c. Those that act to inhibit the conversion of
women with chronic hypertension develop super- angiotensin-I to angiotensin-II.
imposed preeclampsia.
d. Those that act peripherally to reduce sympathetic
d. All of the above tone and decrease cardiac output
50–21. You are seeing Ms. Blanche for prenatal care. She is a
50–23. A 24-year-old G1 with chronic hypertension has
37-year-old G1 at 8 weeks’ gestation. She has been your
required medication for 2 years. She is being induced
patient for several years, and chart review shows former
at 29 weeks’ gestation for superimposed severe
blood pressure readings have been 110–120/70–80 mm
preeclampsia. Which of the following characterizes
Hg at prior visits. The blood pressure noted at her first
the relationship between chronic hypertension and
obstetric visit is 150/96 mm Hg. Which of the follow-
superimposed preeclampsia?
ing is the most appropriate next step?
a. Approximately 30% of women with severe
a. Order maternal cardiac echocardiogram.
range hypertension will develop superimposed
b. Recommend low-sodium diet and recheck her preeclampsia.
blood pressure in 2 weeks. b. Approximately 40% of women with severe
c. Request that she have her blood pressure mea- range hypertension will develop superimposed
sured a few times over the next 2 weeks and preeclampsia.
return with her log for review. c. Approximately 80% of those who also have at
d. Counsel regarding the risks of preexisting hyper- least 300 mg/day of proteinuria at baseline will
tension in pregnancy and recommend initiation develop severe preeclampsia
of a daily baby aspirin to reduce the risk for d. None of the above
preeclampsia.
Chronic Hypertension 335

50–24. Ms. Aldo is a 30-year-old G3P2 who presents for 50–28. According to the American College of Obstetricians
establishment of prenatal care at 9 weeks’ gestation. and Gynecologists, which of the following methods
Her prior pregnancies have both been complicated of antenatal fetal surveillance has been conclusively

CHAPTER 50
by gestational hypertension near term. Her blood shown to be of benefit in the management of
pressure was 146/88 mm Hg, which she states is pregnancies complicated by chronic hypertension?
concordant with other recent measurements. Which a. Nonstress testing
of the following is the most appropriate next step in
b. Biophysical profile
her management?
c. Umbilical artery Doppler studies
a. Continued routine observation
d. Serial sonographic evaluations of fetal growth
b. Initiate antihypertensive therapy
c. Recommend that she keep a log of her blood 50–29. A 33-year-old G1 at 32 weeks’ gestation has been
pressures and bring them to her appointments diagnosed with chronic hypertension with superim-
d. None of the above posed preeclampsia via worsening hypertension. She
is asymptomatic with normal labs and normal fetal
50–25. For Ms. Aldo in Question 50–24, which of the growth. An increase in the dose of her antihyperten-
following complications is reduced with use of sive medication has resulted in nonsevere blood
antihypertensive therapy during pregnancy? pressures. She asks you about expectant management
a. Preterm birth of superimposed preeclampsia. Which of the follow-
ing is the most appropriate, evidence-based response?
b. Fetal growth restriction
a. Preeclampsia mandates immediate delivery, so
c. Development of severe hypertension
you recommend proceeding with cesarean.
d. All of the above b. Inpatient expectant management is reasonable,
and with close surveillance, adverse outcome
50–26. Guidelines provided by the American College of would not be expected.
Obstetricians and Gynecologists in 2013 and the
Society for Maternal-Fetal Medicine in 2015 state c. It is reasonable, with very close inpatient surveil-
lance, to pursue expectant management, but there
that antihypertensive therapy is mandatory for which
are risks for adverse outcomes.
of the following women during pregnancy?
a. 30-year-old G3P2 with a blood pressure of d. None of the above
160/110 mm Hg and a history of asthma
50–30. For women with chronic hypertension without
b. 42-year-old G2P1 with a blood pressure of superimposed preeclampsia, when is delivery
130/88 mm Hg and a history of myocardial recommended?
infarction
a. 37 weeks 0 days to 38 weeks 6 days
c. 28-year-old G2P1 with a blood pressure of
146/94 mm Hg and history of preeclampsia in b. 38 weeks 0 days to 39 weeks 6 days
her last pregnancy c. 39 weeks 0 days to 40 weeks 0 days
d. 38-year-old G4P3 with a blood pressure of d. There are no guidelines for a specific
148/96 mm Hg and history of transient renal recommendation
insufficiency 3 years ago when she had an
obstructive nephrolithiasis 50–31. Which intravenous antihypertensive drug commonly
used to treat intrapartum severe range hypertension
50–27. All except which of the following support the is properly matched with its most notable side effect?
diagnosis of superimposed preeclampsia? a. Hydralazine—fetal arrhythmia
a. Decreased platelet count b. Labetalol—neonatal tachycardia
b. Increased serum creatinine level c. Hydralazine—maternal tachycardia
c. Elevated serum alkaline phosphatase level d. Labetalol—neonatal rebound hypertension
d. Elevated serum aspartate aminotransferase level
336 Medical and Surgical Complications

50–32. Which of the following statements is not true 50–33. Ms. Edwards is a 26-year-old G2P2, now 3 days
regarding intrapartum management of a woman postpartum from a vaginal delivery. She has chronic
with chronic hypertension with superimposed hypertension and developed superimposed severe
SECTION 12

preeclampsia? preeclampsia at 36 weeks’ gestation. She did well


a. Trial of labor is not contraindicated. initially, but in the last 12 hours her blood pres-
sure has been trending up. Most recent blood pres-
b. Magnesium sulfate is the neuroprophylactic agent
sure was 156/90 mm Hg. The nurse has called you
of choice to prevent eclampsia.
because Ms. Edwards is feeling short of breath. A
c. These women are more sensitive to the acute representative chest radiograph is shown here. What
hypotensive effects of epidural anesthesia. is the appropriate treatment for this condition?
d. Postpartum analgesia should include scheduled
nonsteroidal antiinflammatory drugs for at least
72 hours to reduce narcotic use.

Used with permission from Dr. Barbara Hoffman.

a. Lovenox anticoagulation
b. Albuterol via metered dose inhaler
c. Incentive spirometer and ambulation
d. Intravenous furosemide administration
343

CHAPTER 52

Thromboembolic Disease

52–1. What has led to a decrease in the frequency of 52–6. A 33-year-old G4P5 presents to the hospital on
venous thromboembolism in the puerperium? POD #15 from a cesarean delivery due to a twin ges-
a. Anticoagulation tation complaining of right lower extremity swelling.
She reports the swelling started 2 days ago but has
b. Early ambulation
not improved with leg elevation. She reports she had
c. Decrease in cesarean deliveries a blood clot in her left leg 10 years ago. The only
d. Sequential compression devices medication she is taking is iron because her hemo-
globin was 8 mg/dL on discharge. Which of the fol-
52–2. What percentage of pregnancy-related deaths are due lowing risk factors for venous thromboembolism is
to pulmonary embolism? most important in pregnancy?
a. 9% a. Anemia
b. 15% b. Cesarean delivery
c. 25% c. Multifetal gestation
d. 30% d. Personal history of thrombosis

52–3. Which of the following statements is accurate 52–7. The patient in Question 52–6 reports she was never
regarding the timing of venous thrombosis? tested for a thrombophilia. What percentage of
a. Deep-vein thrombosis is more common during women with venous thrombosis during pregnancy
the antepartum period. have an underlying thrombophilia?
b. An equal number occur during the antepartum a. 10%
period and puerperium. b. 20%
c. Pulmonary embolism is more common in the c. 50%
first 6 weeks postpartum. d. 90%
d. All of the above
52–8. Which of the following is not an inherited
52–4. All except which of the following is an aspect of thrombophilia?
Virchow’s triad? a. Protein S deficiency
a. Stasis b. Antiphospholipid syndrome
b. Local trauma c. Antithrombin III deficiency
c. Immobilization d. Activated protein C resistance
d. Hypercoagulability
52–9. Which of the following statements regarding
52–5. Which of the following pregnancy-related fac- antithrombin III deficiency is accurate?
tors contributes to an increased risk for venous a. Autosomal dominant
thromboembolism? b. Inactivates thrombin and factor Xa
a. Enhanced synthesis of clotting factors c. The most thrombogenic inheritable coagulopathy
b. Endothelial cell injury during delivery d. All of the above
c. Compression of the pelvic veins and inferior vena
cava 52–10. What happens to protein S levels in pregnancy?
d. All of the above a. Increase
b. Decrease
c. Stay the same
d. Decrease in the first trimester and then increase
344 Medical and Surgical Complications

52–11. Why does the presence of a factor V Leiden 52–17. Which of the following antiphospholipid antibodies
mutation lead to increased risk for venous is associated with the lowest live birth rate?
thromboembolism? a. Lupus anticoagulant
SECTION 12

a. Increased levels of factor V are produced b. Anti-β2-glycoprotein I


b. Decreased levels of factor V are produced c. Anticardiolipin antibody
c. Factor V is resistant to inactivation by protein C d. All are equal
d. Factor V is more sensitive to inactivation by
protein C 52–18. A 32-year-old G3P0 presents for preconception
counseling after suffering three first-trimester preg-
52–12. How does the prothrombin G20210A mutation nancy losses. She reports her obstetrician performed
promote clot formation? a workup and she has a genetic blood clotting disor-
a. Leads to increased prothrombin production der. Which inherited thrombophilia has been associ-
ated with adverse pregnancy outcomes, including
b. Leads to decreased prothrombin production
first-trimester loss?
c. Accelerates prothrombin inactivation by protein S
a. Prothrombin G20210A
d. Yields prothrombin resistant to inactivation by
b. Factor V Leiden mutation
protein S
c. 5,10-Methylene-tetrahydrofolate reductase
52–13. Which of the following adverse pregnancy outcomes (MTHFR) mutation
is linked to factor V Leiden mutation? d. None of the above
a. Preeclampsia
52–19. As you discuss her results, for which thrombophilia
b. Fetal-growth restriction
do you encourage the patient in Question 52–18 to
c. First-trimester pregnancy loss undergo testing?
d. None of the above a. Protein S deficiency
b. Protein C deficiency
52–14. Which of the following is an acquired thrombophilia?
c. Antiphospholipid syndrome
a. Cancer
d. All of the above
b. Antiphospholipid syndrome
c. Heparin-induced thrombocytopenia 52–20. The American College of Obstetricians and Gyne-
d. All of the above cologists recommends screening in which of the
following situations?
52–15. Which of the following is in the clinical criteria for a. Personal history of an unprovoked deep-vein
antiphospholipid syndrome? thrombosis
a. At least one spontaneous preterm birth b. Personal history of a deep-vein thrombosis after
b. Three unexplained fetal losses before 12 weeks’ knee surgery
gestation c. Family history consisting of a sister with a pulmo-
c. At least one unexplained fetal death beyond nary embolus during pregnancy at age 25
20 weeks’ gestation d. Family history of an aunt with a deep-vein
d. At least 1 preterm birth before 34 weeks’ gesta- thrombosis while undergoing treatment for breast
tion due to severe preeclampsia, eclampsia, or cancer at the age of 62
placental insufficiency
52–21. Which one of the following thrombophilias can
52–16. A 34-year-old G1P1 presents for preconception be tested for while on treatment for a venous
counseling because her last pregnancy ended in thrombosis?
delivery at 27 weeks’ gestation due to preeclampsia a. Protein S deficiency
with severe features. She brings documentation of b. Protein C deficiency
lab results showing she is positive for anti-β2-
glycoprotein I IgM. A minimum of how many weeks c. Antithrombin III deficiency
after the initial labs are the confirmatory labs drawn? d. Prothrombin G20210A mutation
a. 8 weeks
b. 10 weeks
c. 12 weeks
d. 16 weeks
Thromboembolic Disease 345

52–22. Which of the following thrombophilias is tested for 52–25. The patient in Question 52–24 undergoes compres-
using DNA analysis? sion ultrasonography with the findings shown below.
a. Protein S deficiency What is the next best step in the management of this

CHAPTER 52
patient?
b. Protein C deficiency
c. Antithrombin III deficiency
d. Prothrombin G20210A mutation

52–23. In pregnancy, why is a left leg deep-vein thrombosis


more common than a right leg deep-vein thrombosis?
a. Natural leftward tilt of the gravid uterus
b. Compression of the left iliac vein by the left iliac
artery
c. Compression of the left iliac vein by the left
ovarian artery
d. Compression of the left iliac vein by the right
iliac artery

52–24. A 29-year-old G2P1 at 29 weeks’ gestation presents


for a routine prenatal care visit with complaints of
lower extremity swelling. On examination her feet
appear as pictured below. Which of the following is a. Serum d-dimer
true regarding this condition in pregnancy? b. Initiate anticoagulation
c. Magnetic resonance imaging
d. Ventilation-perfusion scintigraphy

52–26. Serum d-dimer is unreliable in pregnancy because it


is affected by all except which of the following?
a. Preeclampsia
b. Cesarean delivery
c. Multifetal gestation
d. All of the above

52–27. What are the advantages of low-molecular-weight


heparin in pregnancy?
a. Shorter half-life
b. Easily reversible
c. Better bioavailability
d. Increased dosing leads to more predictable
response

52–28. What is the risk of pulmonary embolism in the


setting of a treated deep-vein thrombosis?
a. 2%
b. 5%
c. 10%
d. 15%
a. A positive Homans sign is virtually diagnostic
b. Most cases are located in the iliofemoral veins
c. Associated with a pulmonary embolism in
30–60% of cases
d. Pain more than swelling correlates with the
degree of vessel involvement
346 Medical and Surgical Complications

52–29. A 33-year-old G3P2 is on prophylactic low- 52–32. Superficial venous thrombophlebitis is treated with
molecular-weight heparin due to history of venous all except which of the following?
thromboembolism and factor V Leiden mutation. a. Heat
SECTION 12

She inquires as to why she has to be switched to a


b. Analgesia
medication that requires injections twice daily at
36 weeks’ gestation. What is your response? c. Elastic support
a. Heparin has a shorter half-life and is more easily d. Anticoagulation
reversible.
52–33. What percentage of maternal deaths are caused by
b. Heparin has a lower rate of heparin-induced
pulmonary embolism?
thrombocytopenia.
a. 1%
c. Heparin does not cross the placenta as low-
molecular-weight heparin does. b. 5%
d. Fetal exposure to low-molecular-weight heparin c. 10%
close to delivery increases the risk for intraven- d. 20%
tricular hemorrhage.
52–34. What is the most common presenting symptom in
52–30. What medication is used to reverse the effects of patients with a pulmonary embolus?
heparin? a. Cough
a. Calcium b. Syncope
b. Vitamin K c. Dyspnea
c. Protamine sulfate d. Chest pain
d. Fresh frozen plasma
52–35. All except which of the following is included in
52–31. When switching from therapeutic heparin to the diagnostic algorithm of pulmonary embolus in
warfarin, how are paradoxical thrombosis and skin pregnancy?
necrosis from the anti–protein C effects of warfarin a. Chest radiograph
avoided?
b. Magnetic resonance imaging
a. Watch the patient in the hospital for the first
c. Ventilation-perfusion scintigraphy
5 days.
b. Give the patient a test dose and watch for these d. Computed-tomographic pulmonary angiography
side effects.
c. Overlap of heparin and warfarin for at least 3 days
and until the INR is therapeutic for 1 day.
d. Overlap of heparin and warfarin for at least 5 days
and until the INR is therapeutic for 2 days.
347

CHAPTER 52 ANSWER KEY

Question Letter Page

CHAPTER 52
number answer cited Header cited
52–1 b p. 1004 Introduction
52–2 a p. 1004 Introduction
52–3 d p. 1004 Introduction
52–4 c p. 1004 Pathophysiology
52–5 d p. 1004 Pathophysiology
52–6 d p. 1005 Pathophysiology
52–7 c p. 1005 Pathophysiology
52–8 b p. 1006 Table 52-2
52–9 d p. 1006 Antithrombin Deficiency
52–10 b p. 1007 Protein S Deficiency
52–11 c p. 1007 Activated Protein C Resistance-Factor V Leiden Mutation
52–12 a p. 1007 Prothrombin G20210A Mutation
52–13 d p. 1007 Activated Protein C Resistance-Factor V Leiden Mutation
52–14 d p. 1008 Acquired Thrombophilias
52–15 d p. 1008 Antiphospholipid Syndrome
52–16 c p. 1008 Antiphospholipid Syndrome
52–17 b p. 1008 Antiphospholipid Syndrome
52–18 d p. 1009 Thrombophilias and Pregnancy Complications
52–19 c p. 1010 Thrombophilia Screening
52–20 a p. 1010 Thrombophilia Screening
52–21 d p. 1010 Table 52-4
52–22 d p. 1010 Table 52-4
52–23 d p. 1010 Clinical Presentation
52–24 b p. 1010 Clinical Presentation
52–25 b p. 1011 Diagnosis
52–26 d p. 1011 D-Dimer Screening Tests
52–27 c p. 1012 Management
52–28 b p. 1012 Management
52–29 a p. 1014 Labor and Delivery
52–30 c p. 1014 Labor and Delivery
52–31 d p. 1015 Anticoagulation with Warfarin Compounds
52–32 d p. 1016 Superficial Venous Thrombophlebitis
52–33 c p. 1016 Pulmonary Embolism
52–34 c p. 1016 Clinical Presentation
52–35 b p. 1017 Figure 52-5
348

CHAPTER 53

Renal and Urinary Tract Disorders

53–1. Which of the following is not a physiological change 53–4. Which of the following is the threshold for protein-
in pregnancy that contributes to the findings in the uria in pregnancy, above which levels are considered
image below? abnormal?
a. 100 mg/d
b. 250 mg/d
c. 300 mg/d
d. 1000 mg/d

53–5. Which of the following values is not a normal serum


creatinine in pregnancy?
a. 0.4 mg/dL
b. 0.6 mg/dL
c. 0.8 mg/dL
d. 1.0 mg/dL

53–6. A pregnant patient who previously donated a kidney


and now has only one healthy kidney is at risk for
which of the following obstetric complications?
a. Vesicoureteral reflux a. Placenta previa
b. Glomerular hypertrophy b. Normocytic anemia
c. Distal ureteral compression by the uterus c. Gestational hypertension
d. Progesterone-induced relaxation of the muscularis d. Preterm premature rupture of membranes
53–2. Which of the following statements regarding 53–7. A 28-year-old G1P0 at 20 weeks’ gestation presents
physiological changes in pregnancy is true? to the emergency room complaining of frequency,
a. Glomerular filtration decreases. urgency, and dysuria. She is afebrile with no costo-
b. Effective renal plasma flow increases. vertebral angle tenderness. On review of her prenatal
records, you see that she had a urine culture at her
c. Serum creatinine concentration increases.
first prenatal visit with >100,000 colony-forming
d. The number of glomerular cells increases. units/mL gram-negative rods that was never treated.
What percentage of pregnant women with asymp-
53–3. A 35-year-old G5P3 presents at 24 weeks’ gestation. tomatic bacteriuria develop a symptomatic infection
She has a history of chronic hypertension, but she is if left untreated?
noncompliant with medication. You would like to
do a baseline 24-hour urine collection. The patient is a. 5%
of low health literacy, and you are concerned she will b. 10%
not complete the collection as an outpatient, but she c. 25%
refuses to be admitted for it. You decide to do a uri- d. 50%
nary protein-to-creatinine ratio on a spot urine sample
so you can gather additional information about the
patient’s renal status. What is the cutoff for abnormal?
a. 0.1
b. 0.2
c. 0.3
d. 0.5
Renal and Urinary Tract Disorders 349

53–8. Which of the following is an acceptable treatment 53–12. In the setting of aggressive fluid hydration, what
regimen for newly diagnosed asymptomatic bacteri- percentage of pregnant women with pyelonephritis
uria in pregnancy? develop acute kidney injury?

CHAPTER 53
a. Nitrofurantoin 100 mg by mouth at bedtime for a. 5%
10 days b. 10%
b. Ampicillin 250 mg by mouth four times per day c. 15%
for one day
d. 20%
c. Trimethoprim-sulfamethoxazole 160/800 mg by
mouth one time 53–13. An 18-year-old G1P0 presents at 30 weeks’ gestation
d. Nitrofurantoin 100 mg by mouth four times with fever, chills, vomiting, dysuria, and frequency
daily for 21 days for 3 days. She is also experiencing contractions. You
diagnose her with pyelonephritis based on her symp-
53–9. You are consulted about a 19-year-old primigravida toms, fever of 39oC, and urinalysis. She is transferred
at 18 weeks’ gestation. The patient was diagnosed at to labor and delivery for monitoring of contractions.
her first prenatal care visit with asymptomatic bacte- The patient is given intravenous (IV) antibiotics,
riuria. Her provider treated her with nitrofurantoin IV fluids, acetaminophen, and a β-agonist because
100 mg twice daily for 7 days. The patient was seen of the contractions. What complication is more
a month after treatment, and her repeat urine culture likely in the setting of β-agonist use in patients with
was negative. But now, a urine culture sent in error pyelonephritis?
is positive for >100,000 colony-forming units/mL a. Anemia
gram-negative rods again. She remains asymptom-
atic. Her provider is not sure what to do. What is b. Abruption
the best response to this consult? c. Pulmonary edema
a. Single-dose treatment is more successful, so give d. Acute kidney injury
the patient nitrofurantoin 200 mg by mouth one
time. 53–14. Once you have treated a pregnant patient for pyelo-
b. The patient likely has a highly resistant organ- nephritis as an inpatient, for how long should you
ism, so she should be admitted for intravenous continue oral therapy as an outpatient?
antibiotics. a. 1–3 days
c. The urine culture is probably a false-positive b. 3–5 days
given it was negative after treatment, so she does c. 7–14 days
not need any further antibiotics.
d. 14–21 days
d. Recurrence of asymptomatic bacteriuria is 30%
regardless of antibiotic regimen and it may indi- 53–15. What percentage of pregnant women experience a
cate covert upper tract infection, so treat the recurrent urinary tract infection after completion of
patient with nitrofurantoin 100 mg by mouth at treatment for pyelonephritis?
bedtime for 21 days.
a. 5–10%
53–10. Lower urinary tract symptoms with pyuria but a ster- b. 10–20%
ile urine culture are likely due to which pathogen? c. 30–40%
a. Escherichia coli d. 50–60%
b. Proteus mirabilis
c. Klebsiella pneumoniae
d. Chlamydia trachomatis

53–11. What is the leading cause of septic shock during


pregnancy?
a. Pneumonia
b. Breast abscess
c. Pyelonephritis
d. Chorioamnionitis
350 Medical and Surgical Complications

53–16. A 24-year-old G1P0 presents at 24 weeks’ gestation 53–18. A 25-year-old G1P0 at 16 weeks’ gestation presents
complaining of back pain that radiates forward. She for prenatal care. She has a history of a kidney trans-
reports that the pain is intense, and she looks very plant 3 years ago. She is stable on prednisone and
SECTION 12

uncomfortable. She has not been febrile. A urinalysis azathioprine. The patient has not experienced any
is significant only for red blood cells. An image from rejection, and her blood pressure is well controlled.
her renal ultrasound is provided below. You diagnose The patient is concerned about how her pregnancy
the patient with a kidney stone. Which of the fol- will be affected by her transplant and vice versa.
lowing is the best management option for the patient Which of the following statements would not be
at this time? part of your counseling?
a. Opportunistic infections are common, so there
must be proper surveillance for that.
b. It is a good prognostic indicator that she has been
stable and in good health for the past several years.
c. She would be expected to have a better outcome
than someone with end-stage renal disease on
dialysis.
d. It is unfortunate that she is on azathioprine as it
has a much higher rate of fetal malformation than
mycophenolate.

53–19. Which of the following obstetric complications is


not increased in women who have undergone a
kidney transplant?
a. Preeclampsia
b. Preterm birth
a. Analgesia c. Fetal-growth restriction
b. Analgesia and intravenous hydration d. Peripartum cardiomyopathy
c. Analgesia, intravenous hydration, and intravenous
antibiotics 53–20. Which of the following is not a recommended
d. Analgesia, intravenous hydration, intravenous requisite for renal transplant patients who want to
antibiotics, and ureteral stenting attempt pregnancy?
a. Proteinuria <300 mg/day
53–17. The patient in Question 53–16 asks how many preg- b. Serum creatinine <2 mg/dL
nant women actually pass their stone with the man-
agement plan you selected. What is your response? c. No evidence of graft rejection for 6 months
a. 50–60% d. No identifiable pyelocalyceal distention by
urography
b. 65–80%
c. 90–95% 53–21. Which of the following statements about polycystic
d. 95–100% kidney disease is true?
a. It is usually X-linked inheritance.
b. Renal complications are more common in women
than men.
c. The majority of cases are due to the PKD3 muta-
tion on chromosome 4.
d. 10% of those with this disease die from rupture
of an intracranial berry aneurysm.

53–22. Which of the following is a pulmonary-renal


syndrome?
a. Berger disease
b. Goodpasture syndrome
c. Minimal change disease
d. Focal segmental glomerulosclerosis
Renal and Urinary Tract Disorders 351

53–23. Which of the following does not characterize 53–29. What is the most common reason for acute kidney
nephrotic syndromes? injury in obstetrics?
a. Edema a. Sepsis

CHAPTER 53
b. Heavy proteinuria b. Acute blood loss
c. Hyperalbuminemia c. Use of loop diuretics
d. Hypercholesterolemia d. Ureteral injury at cesarean section

53–24. Which of the following complications is not 53–30. A pregnant woman presents with flank pain. She is
increased in pregnant patients with nephrotic afebrile but tachycardic from pain. Her serum creati-
syndrome? nine is elevated to 5 mg/dL. Images from her mag-
a. Anemia netic resonance imaging are presented below. Which
of the following is the best plan of care?
b. Preeclampsia
c. Renal insufficiency
d. Postterm pregnancy

53–25. Which of the following is most likely to lead to


end-stage renal disease?
a. Diabetes
b. Hypertension
c. Glomerulonephritis
d. Polycystic kidney disease

53–26. By what creatinine does one define moderate renal


impairment in patients with chronic renal disease?
a. 1–1.5 mg/dL
b. 1.5–3.0 mg/dL
c. 3.0–4.5 mg/dL Reproduced with permission from Cunningham FG, Leveno KJ, Bloom SL, et al (eds):
d. 5–7 mg/dL Renal and urinary tract disorders. In Williams Obstetrics, 25th ed. New York, McGraw-
Hill, 2018, Figure 53-6A and 53-6B.

53–27. Which of the following plans in a pregnant patient


on dialysis is correct?
a. Renal biopsy

a. If the patient’s creatinine is less than 7 mg/dL, b. Nephrectomy


the dialysis can be held. c. Percutaneous nephrostomy tube
b. Change from peritoneal dialysis to hemodialysis d. Intravenous antibiotics and aggressive fluid
because outcomes are worse in cases of peritoneal hydration
dialysis.
53–31. Which of the following is not commonly found in
c. Increase the frequency of dialysis to avoid the
patients with a urethral diverticulum?
abrupt volume changes which could result in
hypotension. a. Pain
d. Decrease the frequency of dialysis because the b. Palpable mass
glomerular filtration rate will increase with c. Urinary retention
increasing blood volume, making dialysis less d. Recurrent urinary infections
necessary.
53–32. Which of the following circumstances is most likely
53–28. When managing a patient with postpartum acute to lead to fistula formation?
kidney injury, which of the following medications
a. Rupture of a posterior wall fibroid
does not need to have its dose adjusted?
b. Prolonged obstructed labor in a resource-poor
a. Gentamycin
country
b. Clindamycin
c. A fetus being at +1 station for the week leading
c. Magnesium sulfate up to labor
d. Ketorolac tromethamine d. A cesarean section done for breech presentation at
term and complicated by a 2-cm extension
359

CHAPTER 55

Hepatic, Biliary, and Pancreatic Disorders

55–1. All except which of the following liver-related 55–6. A 27-year-old multigravida comes to you for her
changes are physiological in pregnancy? postpartum visit. Both of her pregnancies were com-
a. Hypolipidemia plicated by intrahepatic cholestasis of pregnancy.
She desires another child in about 2 years. Which
b. Spider angiomas
of the following methods of contraception do you
c. Palmar erythema recommend?
d. Elevated serum alkaline phosphatase levels a. Vaginal ring
b. Copper intrauterine device
55–2. The cytochrome P450 system is altered by which of
the following during pregnancy? c. Combination oral contraceptive pills
a. Estrogen levels d. None of the above
b. Progesterone levels
55–7. Which of the following have the best data for treat-
c. Placental expression ment of intrahepatic cholestasis of pregnancy?
d. All of the above a. Cholestyramine
b. Dexamethasone
55–3. Which of the following pregnancy-related compli-
cations has the capacity to demonstrate the most c. Low-cholesterol diet
prominent alterations of normal hepatic, renal, d. Ursodeoxycholic acid
hematological, and coagulation laboratory studies?
a. Preeclampsia 55–8. A 32-year-old G1 at 32 weeks’ gestation is diagnosed
with intrahepatic cholestasis of pregnancy. During
b. Hyperemesis gravidarum
the visit she inquires about whether this diagnosis
c. Acute fatty liver of pregnancy will alter her prenatal care or delivery plans. What is
d. Intrahepatic cholestasis of pregnancy the best evidence-based advice you can provide?
a. We can avoid risk for stillbirth with weekly fetal
55–4. Which of the following findings can narrow the dif- nonstress testing.
ferential of elevated liver enzymes to exclude intrahe-
b. Since your bile acid levels are <10 µmol/L,
patic cholestasis of pregnancy?
delivery will be recommended before 39 weeks’
a. Jaundice gestation.
b. AST 800 U/L c. If we pursue delivery at 38–39 weeks’ gestation,
c. Liver biopsy showing bile plugs in the hepatocytes we will avoid neonatal sequelae of intrahepatic
d. All of the above cholestasis of pregnancy.
d. None of the above
55–5. Which of the following viral infections has been
associated with a marked increase in the risk for 55–9. What is the incidence and recurrence risk of acute
intrahepatic cholestasis of pregnancy? fatty liver of pregnancy?
a. Hepatitis B a. 1 in 10,000 pregnancies, and recurrence is rare
b. Hepatitis C b. 1 in 100,000 pregnancies, and recurrence is rare
c. Cytomegalovirus c. 1 in 100,000 pregnancies, and recurrence is
d. Human immunodeficiency virus common
d. 1 in 1,000,000 pregnancies, and recurrence is rare
360 Medical and Surgical Complications

55–10. Maternal acute fatty liver of pregnancy is associ- 55–13. You know that nausea and vomiting in the third
ated with all except with of the following recessively trimester is not something to be dismissed, so you
inherited abnormalities of mitochondrial fatty acid start the work-up on the patient in Question 55–12.
SECTION 12

oxidation? Which combination of laboratory findings below


a. Carnitine palmitoyltransferase 1 would be most concerning for acute fatty liver of
pregnancy?
b. Dihydrolipoamide dehydrogenase
a. Hematocrit 34%, creatinine 1.1 mg/dL, platelets
c. Medium-chain acyl-CoA dehydrogenase
190/L, AST 60 U/L, fibrinogen 450 mg/dL, and
d. Long-chain-3-hydroxyacyl-CoA-dehydrogenase glucose 96 mg/dL
b. Hematocrit 32%, creatinine 2.0 mg/dL, platelets
55–11. From an etiopathogenesis perspective, the pregnancy-
90/L, AST 400 U/L, fibrinogen 130 mg/dL, and
related complication pictured here is analogous to
glucose 65 mg/dL
which childhood illness?
c. Hematocrit 39%, creatinine 1.2 mg/dL, platelets
90/L, AST 80 U/L, fibrinogen 450 mg/dL, and
glucose 105 mg/dL
d. Hematocrit 30%, creatinine 1.0 mg/dL, platelets
200/L, AST 600 U/L, fibrinogen 420 mg/dL,
and glucose 120 mg/dL

55–14. You send a peripheral blood smear on the patient in


Question 55–12, and a representative slide is shown
below. What is the underlying etiology of the blood
smear findings?

Reproduced with permission from Cunningham FC, Leveno KJ, Bloom SL, et al (eds):
Hepatic, biliary, and pancreatic disorders. In Williams Obstetrics, 25th ed. New York,
McGraw-Hill, 2018, Figure 55-1.

a. Biliary atresia
b. Reye-like syndromes
c. Autoimmune hepatitis
d. Epstein-Barr viral hepatitis

55–12. A 26-year-old primigravida presents at 34 weeks’


gestation with nausea and vomiting, fatigue, and
Reproduced with permission from Lichtman MA, Beutler E, Kipps TJ: Color Atlas III:
epigastric pain. All except which of the following are
Red cell morphology. In Williams Hematology, 7th ed. New York, McGraw-Hill, 2006,
clinical characteristics that increase her risk for acute Plate III-3.
fatty liver of pregnancy as the underlying cause?
a. Nulliparity a. Autoimmune antibody binding
b. Female fetus b. Decreased cholesterol production
c. Twin gestation c. Increased destruction in the spleen
d. Third trimester d. Thrombotic microangiopathy in microvessels

55–15. Which imaging modality is recommended in dis-


criminating acute fatty liver of pregnancy from other
etiologies of elevated liver enzymes?
a. Computed tomography
b. Magnetic resonance imaging
c. Ultrasound with color Doppler
d. None of the above
Hepatic, Biliary, and Pancreatic Disorders 361

55–16. Risk of maternal death with acute fatty liver is 55–22. The following viral serologies are resulted for the
related to which of the following? patient in Question 55–21. What is her diagnosis?
a. Sepsis

CHAPTER 55
b. Hemorrhage IgA Anti-Hepatitis B core antibody +
c. Renal failure Hepatitis B surface antibody –
d. All of the above
Hepatitis B e antigen +
55–17. After delivery, the liver function deterioration of
acute fatty liver of pregnancy halted. Nonethe- a. History of hepatitis B vaccination
less, you recommend continued intensive maternal b. Chronic hepatitis B infection, still active
surveillance due to the approximately 20% risk of c. History of hepatitis B infection, now latent
developing which of the following in the days after
delivery? d. It is not possible to make a diagnosis from the
serologies presented
a. Sepsis
b. Hemorrhage 55–23. How do you counsel the patient in Question 55–21
c. Diabetes mellitus regarding her lab results?
d. Acute pancreatitis a. She has a virus that can lead to cirrhosis and
death, and you will refer her to a hepatologist for
55–18. Which of the following statements regarding acute discussion of ribavirin therapy.
viral hepatitis is correct? b. She does not need to be concerned. Her results
a. Jaundice is usually the presenting symptom. represent vaccination for Hepatitis B, and she
b. Low-grade fever is more common with hepatitis A. should have lifelong antibodies.
c. Bilirubin levels typically fall as transaminase levels c. She has a virus that can lead to cirrhosis and
rise. death, but it appears dormant. You will refer her
to a hepatologist for surveillance.
d. Serum transaminase levels correspond with
disease severity. d. She has an active virus that can lead to cirrhosis
and death, and you will refer her to a hepatologist
55–19. Which of the following features are criteria for severe for long-term management recommendations.
disease that should prompt hospitalization for acute
viral hepatitis? 55–24. Lactation is contraindicated for women infected with
which of the following?
a. Hyperglycemia
a. Hepatitis A
b. AST over 500 U/L
b. Hepatitis B
c. Hyperalbuminemia
c. Hepatitis C
d. Hyperbilirubinemia
d. None of the above
55–20. During the hospital care of a patient with acute viral
hepatitis, which of the following personal protective 55–25. The Society for Maternal-Fetal Medicine recom-
equipment should be used at all times? mended which of the following agents for women
with high hepatitis B viral loads in pregnancy?
a. Gloves
a. Ribavirin
b. N95 respiratory mask
b. Tenofovir
c. Negative-pressure ventilation hospital room
c. Lamivudine
d. All of the above
d. Telbivudine
55–21. During a cesarean delivery you sustain a needle stick.
The patient’s prenatal record was not available at 55–26. Chronic hepatitis B infection is most likely to
admission, so her infectious serologies are pending at develop after acquisition by which of the following
the time of your contact. Immunization is not avail- patients?
able for which of the following? a. 9-year-old child
a. Hepatitis A b. Newborn infant
b. Hepatitis B c. 28-year-old healthy woman
c. Hepatitis C d. Risk is equivalent in all of the above
d. It is available for all three
362 Medical and Surgical Complications

55–27. Which of the following combinations represents 55–31. A 36-year-old multigravida presents for preconcep-
appropriate screening for hepatitis C in pregnancy? tion counseling. Her obstetric history includes two
a. 27-year-old woman with human immunodefi- full-term vaginal deliveries after pregnancies com-
SECTION 12

ciency virus screened via anti-hepatitis C antibody plicated by gestational diabetes and a first-trimester
pregnancy loss. She comes to you to discuss her
b. 26-year-old woman with a history of skin abscesses
recent diagnosis of nonalcoholic fatty liver disease
from heroin use screened via anti-hepatitis C
(NAFLD) by her primary care physician. Diagnosis
antibody
was based on imaging performed for an episode of
c. 37-year-old woman with a history of blood trans- abdominal pain. Her liver function tests are normal.
fusion after her last pregnancy in 2014 screened Her medical history is otherwise notable for long-
via hepatitis C RNA standing obesity (body mass index 34 kg/m2) and
d. 15-year-old woman whose mother was a chronic diabetes (hemoglobin A1c 8.2%) diagnosed 2 years
intravenous drug user and died of liver failure ago. Which of the following should be included in
when the patient was 4 years old screened via your counseling?
hepatitis C RNA a. The best interventions to reduce the risk to a
future pregnancy are weight loss and optimizing
55–28. A 29-year-old primigravida presents to you for pre- her glucose control.
natal care. She is known to have hepatitis C with no
b. Compared to women of similar weight, because
known risk factors, and your prenatal labs confirm
of the NAFLD, she has a much higher risk for
presence of hepatitis C RNA and anti-hepatitis C
liver-related adverse pregnancy outcome.
antibody. Compared to a woman with anti-hepatitis
C antibody who is RNA-negative, which of the fol- c. With normal liver enzymes, you question the
lowing is more likely to complicate her pregnancy? diagnosis of NAFLD and recommend that she
seek a second opinion from a gastroenterologist
a. Preterm delivery
d. All of the above
b. Fetal growth restriction
c. Vertical transmission of hepatitis C 55–32. This image shows the typical nodular, fibrotic
d. None of the above appearance of a cirrhotic liver. What is the most
common cause of this condition in the general
55–29. A 21-year-old nulligravida with Sjögren syndrome population?
presents at 26 weeks’ gestation with nausea, fatigue,
and vague abdominal pain. On exam she has a blood
pressure of 94/58 mmHg and you notice mild jaun-
dice. Laboratory assessment shows severely elevated
transaminases and anti-nuclear and anti-smooth
muscle antibodies. Based on your primary diagnostic
suspicion, how do you counsel her regarding compli-
cations of pregnancy?
a. She is at increased risk for preeclampsia.
b. She is at increased risk for preterm delivery.
c. She is at increased risk for cesarean delivery.
d. All of the above

55–30. A 23-year-old nulligravida and her husband present


for preconception counseling. She has Wilson disease
that is being treated with zinc sulfate. Which of the
Reproduced with permission from Geller DA, Goss JA, Tsung A: Liver. In Brunicardi FC,
following should be included in your counseling? Anderson DK, Billiar TR, et al (eds): Schwartz’s Principles of Surgery, 9th ed. New York,
a. Her husband should consider carrier testing. McGraw-Hill, 2010, Figure 31-16a.

b. Upon confirmation of pregnancy, she should dis-


continue her chelation therapy. a. Viral hepatitis
c. Available data suggests she should convert to b. Alcohol exposure
penicillamine prior to conception, as it is a better c. Autoimmune hepatitis
chelating agent. d. Nonalcoholic fatty liver disease
d. All of the above
Hepatic, Biliary, and Pancreatic Disorders 363

55–33. A 32-year-old nulligravida presents for preconcep- 55–36. Which of the following is true regarding pancreatitis
tion counseling due to cirrhosis with esophageal in pregnancy?
varices. Which of the following should be included a. Severity is correlated to the degree of enzyme

CHAPTER 55
in your counseling? elevation.
a. You do not recommend that she become pregnant. b. The primary predisposing condition in pregnancy
b. 1/3 to 1/2 of women with varices will have bleed- is cholelithiasis.
ing in pregnancy and this carries an 18% mortal- c. The Apache II scoring system is the most appro-
ity rate. priate for pancreatitis in pregnancy.
c. Without variceal rupture there is high risk for d. Total parenteral nutrition is the preferred method
liver failure, preterm delivery, fetal growth restric- of providing nutrition in severe pancreatitis.
tion, and maternal death.
d. All of the above 55–37. A 32-year-old at 26 weeks’ gestation has a 3.4-cm
liver mass noted incidentally on a right upper quad-
55–34. A 27-year-old at 32 weeks’ gestation is brought to the rant ultrasound. She undergoes magnetic resonance
emergency room by ambulance after consuming 6 grams imaging for further clarification. Which of the fol-
of acetaminophen. Which of the following is true? lowing is true?
a. The fetus is not at risk, so no monitoring is necessary. a. Due to size and bleeding risk, surgical resection is
b. N-Acetyl-p-benzoquinoneimine should be admin- indicated if it is a hepatic adenoma.
istered promptly. b. If it is a hepatic adenoma, there is a 5% risk of
c. This is the most common cause of acute liver fail- malignant transformation, so biopsy should be
ure in the United States. performed next.
d. All of the above c. If it is focal nodular hyperplasia, estrogen-
containing birth control pills are not a good
55–35. A 31-year-old at 20 weeks’ gestation presents to the contraceptive option postpartum.
emergency room with complaints of right upper d. None of the above
quadrant pain, fever, and vomiting. Exam reveals right
upper quadrant tenderness, and she has a temperature
of 38.7°C. One diagnostic image is shown here. Com-
mon bile duct obstruction is not suspected. What is
the next most appropriate step in her management?

Used with permission from Dr. Barbara Hoffman.

a. Laparoscopic cholecystectomy
b. Endoscopic retrograde cholangiopancreatography
c. Admit for serial abdominal examinations and
intravenous antibiotics
d. Discharge home with oral antibiotics, pain medi-
cation, and antiemetics
CARDIOVASCULAR PATHOLOGY

Вопрос 1
Выполнен
Баллов: 1,00 из 1,00
Отметить вопрос
Текст вопроса
A schematic of Eisenmenger syndrome due to a ventricular septal defect is
depicted. What is the most common cause of death in pregnant women with
this condition?

Выберите один ответ:


a. Left ventricular failure
b. Sepsis
c. Cardiac tamponade
d. Right ventricular failure
Вопрос 2
Выполнен
Баллов: 1,00 из 1,00
Отметить вопрос
Текст вопроса
A 32-year-old G2P1 presents to your office at 17 weeks’ gestation. She
reports a history of tetralogy of Fallot, which was repaired in infancy. She
appears comfortable on the exam table with a heart rate of 92 beats per
minute and a blood pressure of 96/62 mm Hg. However, she does report
significant palpitations and dyspnea when climbing the two flights of stairs
to her apartment on the third floor. Which New York Heart Association class
best describes her functional disability?
Выберите один ответ:
a. Class II
b. Class I
c. Class III
d. Class IV
Вопрос 3
Выполнен
Баллов: 1,00 из 1,00
Отметить вопрос
Текст вопроса _
What is the gold standard for diagnosis of the condition in Question 49–24?
Выберите один ответ:
a. Electrocardiography
b. Chest radiography
c. Right heart catheterization
d. Echocardiography
Вопрос 4
Выполнен
Баллов: 0,00 из 1,00
Отметить вопрос
Текст вопроса
Which of the following best describes the remodeling of the heart that
occurs in normal pregnancy?
Выберите один ответ:
a. Concentric left ventricular mass expansion with spherical remodeling
b. Concentric left ventricular mass expansion with longitudinal remodeling
c. Eccentric left ventricular mass expansion with spherical remodeling
d. Eccentric left ventricular mass expansion with longitudinal remodeling
Вопрос 5
Выполнен
Баллов: 1,00 из 1,00
Отметить вопрос
Текст вопроса
Which of the following therapies is safe in pregnancy to treat acute
supraventricular tachycardia?
Выберите один ответ:
a. All of the above
b. Valsalva maneuver
c. Synchronized cardioversion
d. Intravenous adenosine
Вопрос 6
Выполнен
Баллов: 0,00 из 1,00
Отметить вопрос
Текст вопроса
For which of the following cardiac conditions is conduction analgesia highly
recommended?
Выберите один ответ:
a. Pulmonary artery hypertension
b. Repaired hypoplastic left heart syndrome
c. Severe aortic stenosis
d. All of the above
Вопрос 7
Выполнен
Баллов: 1,00 из 1,00
Отметить вопрос
Текст вопроса
Which of the following conditions usually improves during pregnancy due to
pregnancy-induced hypervolemia?
Выберите один ответ:
a. Aortic stenosis
b. Mitral valve prolapse
c. All of the above
d. Mitral stenosis
Вопрос 8
Выполнен
Баллов: 0,00 из 1,00
Отметить вопрос
Текст вопроса
In a pregnant woman who has previously undergone a heart transplantation,
what is the approximate risk for suffering a rejection episode during
pregnancy?
Выберите один ответ:
a. 40%
b. 10%
c. 20%
d. 30%
Вопрос 9
Выполнен
Баллов: 0,00 из 1,00
Отметить вопрос
Текст вопроса
Which of the following does not contribute to the 40% increase in cardiac
output seen during normal pregnancy?
Выберите один ответ:
a. Decreased systemic vascular resistance
b. Increased heart rate
c. Increased left ventricular contractility
d. All of the above contribute to increased cardiac output in pregnancy
Вопрос 10
Выполнен
Баллов: 1,00 из 1,00
Отметить вопрос
Текст вопроса
In the United States, what is the most common cause of aortic stenosis?
Выберите один ответ:
a. Rheumatic heart disease
b. Idiopathic
c. Bicuspid aortic valve
d. Infectious endocarditis

Вопрос 1
Выполнен
Баллов: 0,00 из 1,00
Отметить вопрос
Текст вопроса
Regarding diagnostic studies, which of the following correctly describes a
normal change that is characteristic of pregnancy?
Выберите один ответ:
a. An average 15-degree left axis deviation on electrocardiogram
b. A decrease in the cardiac silhouette size on chest radiograph
c. All of the above
d. Improved systolic function on echocardiographic assessment
Вопрос 2
Выполнен
Баллов: 1,00 из 1,00
Отметить вопрос
Текст вопроса
You are caring for a pregnant patient with mitral stenosis. She is able to
perform most activities of daily living without significant limitation. On
echocardiogram, her ejection fraction is 50% and her mitral valve area
measures 1.8 cm2. Which of the following variables is most predictive of a
high risk for cardiac complications during this pregnancy?
Выберите один ответ:
a. None of the above
b. Her ejection fraction
c. Her degree of left-sided obstruction
d. Her functional capacity
Вопрос 3
Выполнен
Баллов: 0,00 из 1,00
Отметить вопрос
Текст вопроса
What percentage of pregnancy-related deaths in the United States were
attributable to cardiovascular diseases between 2011 and 2013?
Выберите один ответ:
a. 20–25%
b. 15–20%
c. 10–15%
d. 25–30%
Вопрос 4
Нет ответа
Балл: 1,00
Отметить вопрос
Текст вопроса
You are caring for a 24-year-old G1 with severe aortic stenosis. When in
pregnancy are you most concerned about cardiac decompensation?
Выберите один ответ:
a. During the first trimester
b. Peripartum
c. During the second trimester
d. During the third trimester
Вопрос 5
Выполнен
Баллов: 0,00 из 1,00
Отметить вопрос
Текст вопроса
The patient whose heart is shown in this image has systemic lupus
erythematosus and has a history of a prior stroke. The left atrium (LA) and
left ventricle (LV) are identified, and the arrows point to noninfectious
vegetations on the mitral leaflets. Which of the following is the likely
condition associated with her mitral insufficiency?

Выберите один ответ:


a. Calcification of the mitral annulus
b. Antiphospholipid antibodies
c. Infarction of the papillary muscle
d. None of the above
Вопрос 6
Выполнен
Баллов: 0,00 из 1,00
Отметить вопрос
Текст вопроса
The World Health Organization identifies which of the following conditions
as prohibitive of pregnancy?
Выберите один ответ:
a. Pulmonary artery hypertension
b. Cyanotic heart disease
c. Heart transplantation
d. Systemic ventricular dysfunction with left ventricular ejection fraction of
35%
Вопрос 7
Нет ответа
Балл: 1,00
Отметить вопрос
Текст вопроса
A 30-year-old G2P0A1 presents at 11 weeks’ gestation to establish care.
She has a history of rheumatic heart disease and had a mechanical valve
replacement at age 20. She was previously on warfarin 7.5 mg daily, but
stopped taking her medication when she found out she was pregnant 1
week ago. What is her risk for warfarin embryopathy based on her warfarin
dosing?
Выберите один ответ:
a. 1%
b. 8%
c. 5%
d. 3%
Вопрос 8
Выполнен
Баллов: 1,00 из 1,00
Отметить вопрос
Текст вопроса
What is the most common cause of heart failure in pregnancy?
Выберите один ответ:
a. Chronic hypertension
b. Chronic hypertension with superimposed preeclampsia
c. Severe preeclampsia
d. Idiopathic
Вопрос 9
Выполнен
Баллов: 1,00 из 1,00
Отметить вопрос
Текст вопроса
You are caring for a primigravida with known hypertrophic cardiomyopathy.
She tells you her mother also suffered from the condition and passed away
at an early age. The patient is concerned about her child’s future risk for
developing the same condition. What do you tell her is the most likely risk
for passing the condition to her child?
Выберите один ответ:
a. 100%
b. 25%
c. 50%
d. &lt;1%
Вопрос 10
Выполнен
Баллов: 0,00 из 1,00
Отметить вопрос
Текст вопроса
A graphic of pulmonary capillary wedge pressure (red line) is shown for a
woman with mitral stenosis. “A” represents the first stage of labor, “B”
marks the second stage of labor, “C” demonstrates the immediate
postpartum period, and “D” shows the period 4–6 hours postdelivery. What
accounts for the rise in pulmonary capillary wedge pressure immediately
following delivery?

Выберите один ответ:


a. Increase in systemic vascular resistance
b. Pitocin-induced increase in cardiac output
c. Placental autotransfusion
d. Heart rate normalization

Вопрос 3
Выполнен
Баллов: 0,00 из 1,00
Отметить вопрос
Текст вопроса
In a pregnant population, what is the most common cause of the condition
portrayed in the chest x-ray image below?
(enlarged pulmonary hilar arteries and attenuated peripheral markings)

Выберите один ответ:


a. Chronic renal failure
b. Left-sided heart disease
c. Metabolic disorders
d. Connective tissue disease

CASES

Question 1
Complete
Mark 0.00 out of 1.00
Flag question
Question text
You are caring for a patient with Marfan syndrome who just found out she is
13 weeks pregnant. You obtain an echocardiogram which demonstrates an
aortic root measurement of 4.5 cm. Based on these findings, what would
you recommend?
Select one:
a. Beta blocker initiation
b. Bimonthly echocardiograms
c. All of the above
d. Consideration of pregnancy termination
Question 2
Complete
Mark 1.00 out of 1.00
Flag question
Question text
At 24 weeks’ gestation the patient with a history of pulmonary hypertension
due to chronic thromboembolic disease in her past begins to develop
orthopnea and dyspnea on exertion. What treatment would you consider to
decrease her symptoms?
Select one:
a. Supplemental oxygen
b. Pulmonary vasodilator drugs
c. All of the above
d. Diuretics
Question 3
Complete
Mark 0.00 out of 1.00
Flag question
Question text
How would you manage the patient at 32 weeks’ gestation you are caring
for complains of shortness of breath and chest pain with minimal exertion
with previously uncomplicated pregnancy with a new-found mitral stenosis
and an ejection fraction of 55%?
Select one:
a. Administer diuretics
b. Initiate anticoagulation
c. All of the above
d. Recommend immediate labor induction
Question 4
Complete
Mark 1.00 out of 1.00
Flag question
Question text
What associated condition the patient with an unrepaired ventricular septal
defect, considering pregnancy, is at high risk for developing based on her
history?
Select one:
a. Bacterial endocarditis
b. Venous thromboembolism
c. None of the above
d. Embolic stroke
Question 5
Complete
Mark 1.00 out of 1.00
Flag question
Question text
You are the on-call hospitalist when you receive an urgent call from the
postpartum floor about a patient who is complaining of chest pain and
shortness of breath. She had an uncomplicated vaginal delivery the
previous day and has no significant past medical or surgical history. You
obtain a stat chest x-ray, which is shown. Based on the findings, what are
you most concerned for?
Select one:
a. Pulmonary hypertension
b. Cardiomyopathy
c. Pneumonia
d. Pulmonary edema

Вопрос 1
Выполнен
Баллов: 0,00 из 1,00
Отметить вопрос
Текст вопроса
Based on the diagnosis of a peripartum cardiomyopathy and assuming a
recovery of systolic function by 6 months postpartum, what is the patient’s
risk for suffering a relapse of the same condition in a future pregnancy?
Выберите один ответ:
a. 5%
b. 20%
c. 10%
d. 30%
Вопрос 2
Выполнен
Баллов: 1,00 из 1,00
Отметить вопрос
Текст вопроса
You obtain an echocardiogram on the patient in postdelivery period with
new-onset cardiomyopathy. Her ejection fraction is found to be 20% with
no significant structural or valvular disease noted. Which hormone has
been implicated in the development of this condition?
Выберите один ответ:
a. Prolactin
b. Estrogen
c. Human placental lactogen
d. Progesterone
Вопрос 4
Выполнен
Баллов: 1,00 из 1,00
Отметить вопрос
Текст вопроса
You are caring for a 28-year-old G1 with a history of moderate mitral
stenosis who presented in active labor and is now 8 cm dilated. She desires
a natural labor and declines pain medications during labor. As her
contractions get closer together, she reports increasing pain, is visibly
uncomfortable, and intermittently becomes tachycardic to 130 beats per
minute. She suddenly begins complaining of shortness of breath and her
oxygen saturation declines to 90% on room air. What is your best course of
action?
Выберите один ответ:
a. All of the above
b. Request immediate anesthesia assessment
c. Administer intravenous Lasix
d. Administer metoprolol

A 32-year-old G2P1 transfers to your care at 18 weeks' gestation. She has a history of NYHA class 2
heart disease. What among following are common recommendations to this patient?
Select one:
a. cigarette smoking cessation is not recommended because of potential Increase in pulmonary
hypertension X
b. for the majority of these patients prolonged hospitalization or bed rest is necessary
c. to avoid contact with persons who have respiratory infections, including the common cold, and to
report at once any evidence for Infection
d. illicit drug use may be particularly harmful, because of increased risk of liver failure
You are caring for a patient with an unrepaired ventricular septal defect who is considering pregnancy.
You perform an echocardiogram to assess her future risks. Which of the following parameters would
indicate an extremely elevated maternal and fetal risk during pregnancy?
Select one:
a. A right ventricular systolic pressure of 75 mm Hg
b. A septal defect measuring 0.7 cm2
c. A left ventricular ejection fraction of 50% X
d. Presence of left ventricular hypertrophy

A pregnant woman presents to the emergency room in an obtunded state. She has a fever to 102.6°F.
On exam, a 4/6 murmur is noted, she withdraws from pain during her abdominal exam, and track
marks are noted on her arms and legs. Her fundal height is approximately 32 cm. An emergent
echocardiogram is obtained, which demonstrates a mitrel valve vegetation. What is the most likely
organism underlying her diagnosis?
Select one:
a. Neisseria gonorrhoeae
b. Staphylococcus aureus
c. Staphylococcus epidermidis
d. Enterococcus

CHRONIC HYPERTENSION

Question 1
Complete
Mark 1.00 out of 1.00
Flag question
Question text
Why ethnicity is important in recommendations to initiate an
antihypertensive agent? Among following choose correct statements
Select one or more:
a. Angiotensin-converting enzyme inhibitors and angiotensin-
receptor blockers are recommended as initial therapy in the black
population.
b. Angiotensin-converting enzyme inhibitors and angiotensin-
receptor blockers are recommended as initial therapy in the nonblack
population.
c. Beta blockers are not recommended as initial therapy in the
nonblack population.
d. Beta blockers are recommended as initial therapy in the Asian
population.
Question 2
Complete
Mark 1.00 out of 1.00
Flag question
Question text
Among women who have persistently worrisome hypertension despite usual
therapy what antagonizing substances may be used:
Select one:
a. Smoking more than 20 cigarettes per day
b. Chronic ingestion of nonsteroidal antiinflammatory drugs
c. Everyday antacids usage
d. Marijuana abuse
Question 3
Complete
Mark 1.00 out of 1.00
Flag question
Question text
For women with chronic hypertension without superimposed preeclampsia,
when is delivery recommended by American College of Obstetricians and
Gynecologists?
Select one:
a. 37 weeks 0 days to 38 weeks 6 days
b. There are no guidelines for a specific recommendation
c. 38 weeks 0 days to 39 weeks 6 days
d. 39 weeks 0 days to 40 weeks 0 days
Question 4
Complete
Mark 1.00 out of 1.00
Flag question
Question text
Which intravenous antihypertensive drug commonly used to treat
intrapartum severe range hypertension is properly matched with its most
notable side effect?
Select one:
a. Hydralazine — neonatal tachycardia
b. Labetalol — neonatal rebound hypertension
c. Hydralazine — maternal arrhythmia
d. Labetalol — neonatal hypotension and bradycardia
Question 5
Complete
Mark 1.00 out of 1.00
Flag question
Question text
Which of the following characterizes the relationship between chronic
hypertension and superimposed preeclampsia?
Select one:
a. Approximately 80% of those who also have at least 300 mg/day of
proteinuria at baseline will develop severe preeclampsia
b. All women with severe hypertension have renal disease—as either cause
or effect
c. Women with severe chronic hypertension who did not develop
superimposed preeclampsia have the same outcomes as those, who
developed.
d. None of the mentioned
Question 6
Complete
Mark 1.00 out of 1.00
Flag question
Question text
The benefits and safety of instituting antihypertensive therapy in women
with chronic hypertension planning or during pregnancy are:
Select one:
a. Evident
b. Not clear
c. Absent
Question 7
Complete
Mark 1.00 out of 1.00
Flag question
Question text
Ms. Clamp is a 30-year-old G1 presenting for initiation of prenatal care at 12
weeks’ gestation. Her medical history is notable for chronic hypertension,
and she has been on antihypertensive therapy for 3 years. Her blood
pressure is 148/94 mm Hg. How would you best counsel her regarding the
risk for developing superimposed preeclampsia?
Select one:
a. The risk for developing superimposed preeclampsia is in the 7–9% range.
b. Maternal serum markers yield a high positive predictive value in
discriminating who will and will not develop superimposed preeclampsia.
c. The risk for superimposed preeclampsia is not related to the severity of
her baseline hypertension.
d. The risk is highest if workup reveals proteinuria or other end-organ
impact.
Question 8
Complete
Mark 0.00 out of 1.00
Flag question
Question text
The threshold of 140/90 mm Hg as the upper limit of normal is based on
which of the following?
Select one:
a. A large, widely sampled international population of men and women of
various ages with no known health complications
b. Actuarial tables based on data from white adult males
c. The discrete point at which risk for myocardial infarction, stroke, renal
failure, and peripheral artery disease sharply increases
d. None of the mentioned
Question 9
Complete
Mark 1.00 out of 1.00
Flag question
Question text
According to the Nationwide Patient Sample data, which of the following is
the most frequent comorbidity associated with chronic hypertension?
Select one:
a. Depression
b. Hypothyroidism
c. Pregestational diabetes
d. Systemic lupus erythematosus
Question 10
Complete
Mark 1.00 out of 1.00
Flag question
Question text
For women with chronic hypertension, which modifiable factor further
increases the risk for the placental abruption?
Select one:
a. Exercise
b. Smoking
c. Obesity
d. Marijuana use

Question 1
Complete
Mark 0.00 out of 1.00
Flag question
Question text
All except which of the following support the diagnosis of superimposed
preeclampsia?
Select one:
a. Decreased serum alkaline phosphatase level
b. Increased serum creatinine level
c. Elevated serum aspartate aminotransferase level
d. Decreased platelet count
Question 2
Complete
Mark 0.00 out of 1.00
Flag question
Question text
Which of the following interventions is recommended to reduce risk for
developing preeclampsia in women with chronic hypertension?
Select one:
a. Initiation of low dose aspirin from 12–28 weeks’ gestation with
continuation until delivery.
b. Initiation of low dose aspirin as early as possible and continuation until
28 weeks’ gestation.
c. No intervention has shown benefit over preconception and early
pregnancy folic acid supplementation.
d. At least 3 months of preconception supplementation with vitamins C and
E.
Question 3
Complete
Mark 0.00 out of 1.00
Flag question
Question text
Which of the following methods of antenatal fetal surveillance has been
conclusively shown to be of benefit in the management of pregnancies
complicated by chronic hypertension?
Select one:
a. Umbilical artery Doppler studies
b. Serial sonographic evaluations of fetal growth
c. Nonstress testing
d. Biophysical profile
Question 4
Complete
Mark 0.00 out of 1.00
Flag question
Question text
Risk for the placental abruption is increased in pregnancies complicated by
chronic hypertension. How would you best characterize the risk?
Select one:
a. Incidence not impacted by the severity of blood pressure
b. Incidence not impacted by development of superimposed preeclampsia
c. 2-3-fold increase over the general obstetric population
Question 5
Complete
Mark 1.00 out of 1.00
Flag question
Question text
Antihypertensive therapy is mandatory for which of the following women
during pregnancy?
Select one:
a. 28-year-old G2P1 with a blood pressure of 140/94 mm Hg and history of
preeclampsia in her last pregnancy
b. 42-year-old G2P1 with a blood pressure of 130/88 mm Hg and a history
of myocardial infarction
c. 30-year-old G3P2 with a blood pressure of 160/110 mm Hg and a
history of asthma
d. 38-year-old G4P3 with a blood pressure of 148/96 mm Hg and history of
transient renal insufficiency 3 years ago when she had an obstructive
nephrolithiasis
Question 6
Complete
Mark 1.00 out of 1.00
Flag question
Question text
Which of the following is a major factor contributing to the incidence of
perinatal mortality noted in pregnancies complicated by chronic
hypertension?
Select one:
a. All of the mentioned
b. Fetal growth restriction
c. Iatrogenic preterm birth
d. Superimposed preeclampsia
Question 7
Complete
Mark 1.00 out of 1.00
Flag question
Question text
For a woman who does not enter pregnancy with a known diagnosis, the
diagnosis of chronic hypertension is supported when hypertension is
present prior to what gestational age threshold?
Select one:
a. 20 weeks’ gestation
b. 14 weeks’ gestation
c. 28 weeks’ gestation
d. 24 weeks’ gestation
Question 8
Complete
Mark 1.00 out of 1.00
Flag question
Question text
Which of the following approximates the risk for perinatal mortality in
pregnancies complicated by chronic hypertension?
Select one:
a. 3- to 4-fold increase over pregnancies not complicated by chronic
hypertension
b. Approximately 10% of births in women with severe hypertension
c. Approximately 3% of births in women with mild hypertension
d. All of the mentioned
Question 9
Complete
Mark 1.00 out of 1.00
Flag question
Question text
Which of the following is NOT true regarding the complication shown here,
in the computerized tomography scan image?
Select one:
a. None of the mentioned
b. The risk is not increased among women with chronic hypertension
in the absence of superimposed preeclampsia.
c. Systolic blood pressure ≥160 mm Hg or diastolic blood pressure ≥110 mm
Hg can rapidly result in the finding shown here.
d. This occurs during pregnancy in approximately 3 per 1000 women with
chronic hypertension.
Question 10
Complete
Mark 0.00 out of 1.00
Flag question
Question text
Which of the following factors does not impact resting blood pressure
reading?
Select one:
a. Race
b. Gender
c. Age and weight
d. Activity level
Вопрос 1
Выполнен
Баллов: 1,00 из 1,00
Отметить вопрос
Текст вопроса
Which of the following statements is not true regarding intrapartum
management of a woman with chronic hypertension with superimposed
preeclampsia?
Выберите один ответ:
a. Postpartum analgesia should include scheduled nonsteroidal
antiinflammatory drugs for at least 72 hours to reduce narcotic use.
b. Trial of labor is not contraindicated.
c. These women are more sensitive to the acute hypotensive effects of
epidural anesthesia.
d. Magnesium sulfate is the neuroprophylactic agent of choice to prevent
eclampsia.
Вопрос 3
Выполнен
Баллов: 1,00 из 1,00
Отметить вопрос
Текст вопроса
Pregnancy is associated with which of the following blood pressure
changes in women with chronic hypertension?
Выберите один ответ:
a. All of the mentioned
b. Blood pressure decreases from baseline in early pregnancy
c. In pregnancy, women with chronic hypertension have persistently
elevated vascular resistance
d. Blood pressure reaches its nadir at approximately 20 weeks’ gestation
Вопрос 4
Выполнен
Баллов: 0,00 из 1,00
Отметить вопрос
Текст вопроса
What is main concern about antihypertensive treatment during pregnancy
for those with mild or moderate hypertension?
Выберите один ответ:
a. potentially diabetogenic
b. not improving maternal morbidity
c. potentially teratogenic
d. diminish placental perfusion

Indications for aspirin prophylaxis to reduce the risk for developing preeclampsia in pregnant women
with chronic hypertension include a history of:
Select one:
a. Preeclampsia during previous pregnancy
b. All of the mentioned
c. Renal disease
d. Multifetal gestation
e. Autoimmune disease
f. Diabetes

Among women with chronic hypertension, which of the following is true regarding risk for adverse
maternal and perinatal outcome?
Select one:
a. The risk Is inversely proportional to the severity and duration of hypertension prior to pregnancy.
b. All of the mentioned X
c. Complications are more likely with severe range hypertension at baseline and the presence of end-
organ damage.
d. Chronic hypertension is associated with a twofold Increased risk for maternal death.

CASE

Question 1
Complete
Mark 1.00 out of 1.00
Flag question
Question text
Use of which of the following classes of antihypertensive agents is
contraindicated during all trimesters of pregnancy?
Select one:
a. Those that act centrally to reduce sympathetic outflow
b. Those that interfere with binding sites on voltage-dependent calcium
channels.
c. Those that act peripherally to reduce sympathetic tone and decrease
cardiac output
d. Those that act to inhibit the conversion of angiotensin-I to
angiotensin-II.
e. Those that relaxes arterial smooth muscle
Question 2
Complete
Mark 1.00 out of 1.00
Flag question
Question text
A 37-year-old G2P2 with no medical diagnoses and a normal body mass
index on annual well woman exam found to have BP 142/92 mm Hg, and
after checking several times over the next week at home, she calls with a
blood pressure log that ranges from 140–154/90–100 mm Hg. According to
the recommendations of the Eighth Joint National Committee, what is your
goal blood pressure with therapy?
Select one:
a. &lt;120/80 mm Hg
b. &lt;130/85 mm Hg
c. &lt;150/100 mm Hg
d. &lt;140/90 mm Hg
Question 3
Complete
Mark 1.00 out of 1.00
Flag question
Question text
A 33-year-old G1 at 32 weeks’ gestation has been diagnosed with chronic
hypertension with superimposed preeclampsia via worsening hypertension.
She is asymptomatic with normal labs and normal fetal growth. An increase
in the dose of her antihypertensive medication has resulted in nonsevere
blood pressures. She asks you about expectant management of
superimposed preeclampsia. Which of the following is the most
appropriate, evidence-based response?
Select one:
a. It is reasonable, with close inpatient surveillance, to pursue
expectant management, but there are still risks for adverse
outcomes.
b. Inpatient expectant management is reasonable, and with close
surveillance, adverse outcome would not be expected.
c. None of the above
d. Preeclampsia mandates immediate delivery, so you recommend
proceeding with cesarean now.
Question 4
Complete
Mark 1.00 out of 1.00
Flag question
Question text
Ms. Edwards is a 26-year-old G2P2, now 3 days postpartum from a vaginal
delivery. She has chronic hypertension and developed superimposed severe
preeclampsia at 36 weeks’ gestation. She did well initially, but in the last 12
hours her blood pressure has been trending up. Most recent blood pressure
was 156/90 mm Hg. The nurse has called you because Ms. Edwards is
feeling short of breath. A representative chest radiograph is shown here.
What is the appropriate treatment for this condition?

Select one:
a. Intravenous furosemide
b. Immediate blood test for anemia
c. Broncholytic inhalation
d. Anticoagulation
Question 5
Complete
Mark 0.00 out of 1.00
Flag question
Question text
A 36-year-old multigravida with chronic hypertension presents for a growth
ultrasound at 36 weeks’ gestation. The ultrasound report shows you that
estimated fetal weight correspond to 33 weeks’, head size - to 34 weeks’,
while abdominal circumference and femur length – to 32 weeks’ gestation.
Which of the following is true regarding the diagnosis made on the growth
ultrasound?
Select one:
a. The risk is not modified by whether a woman with chronic hypertension
has got treatment
b. All of the mentioned
c. The risk is reduced with administration of the antioxidants, vitamin C and
vitamin E.
d. Fetal-growth dysfunction is more likely in chronically hypertensive
women who develop superimposed preeclampsia.

Question 1
Complete
Mark 1.00 out of 1.00
Flag question
Question text
You are seeing Ms. Blanche for prenatal care. She is a 37-year-old G1 at 8
weeks’ gestation. She has been your patient for several years, and chart
review shows former blood pressure readings have been 110–120/70–
80 mm Hg at prior visits. The blood pressure noted at her first obstetric
visit is 150/96 mm Hg. Which of the following is the most appropriate next
step?
Select one:
a. Counsel regarding the risks of preexisting hypertension in pregnancy and
recommend initiation of a daily baby aspirin to reduce the risk for
preeclampsia.
b. Request that she have her blood pressure measured a few times
over the next 2 weeks and return with her log for review.
c. Order maternal cardiac echocardiogram.
d. Recommend low-sodium diet and recheck her blood pressure in 2 weeks.
Question 2
Complete
Mark 0.00 out of 1.00
Flag question
Question text
Which antihypertensive drug commonly used to treat hypertension is
properly matched with its most notable side effect?
Select one:
a. Thiazide—neonatal bradycardia
b. Nifedipine — circulating volume depletion
c. Angiotensin-receptor blockers—fetal arrhythmia
d. Hydralazine—maternal tachycardia
Question 3
Complete
Mark 1.00 out of 1.00
Flag question
Question text
Ms. Thomas presents for a preconception counseling visit. She is a 37-
year-old G0 with a body mass index of 38 kg/m2, 4-year history of chronic
hypertension treated with amlodipine 5 mg per day, and a 2-year history of
diabetes which she manages with diet and exercise. Her blood pressure
range is 128–144/80–94 mm Hg from the week prior. What workup would
you like to pursue referent to her diagnosis of chronic hypertension and
prepregnancy risk stratification?
Select one:
a. Serum creatinine and urine protein-to-creatinine ratio
b. Serum creatinine, 24-hour urine collection for protein and creatinine
clearance, electrocardiogram, and cardiac stress test
c. Serum creatinine, urine protein-to-creatinine ratio,
electrocardiogram, and maternal echocardiogram
d. No further evaluation is indicated, as she has only had a diagnosis of
hypertension for 4 years and her blood pressure is well controlled on a
single medication.
Question 4
Complete
Mark 0.00 out of 1.00
Flag question
Question text
A 28-year-old G3P1102 at 9 weeks’ gestation presents for establishment of
prenatal care. Her first pregnancy was complicated by gestational
hypertension at term, and her second was complicated by iatrogenic
preterm delivery at 36 weeks’ gestation for preeclampsia. Her blood
pressure was normal at her postpartum visit 2 years ago but has not been
assessed since. Today her body mass index is 46 kg/m2 and her blood
pressure is 158/96 mm Hg. As instructed, she calls back 2 days after her
visit to report that blood pressures have been in the 146–160/94–100mm
Hg range at home. In addition to initiation of an antihypertensive agent,
what is the next step in management of her blood pressure in this
pregnancy?
Select one:
a. Baseline 24-hour urine collection for protein, creatinine clearance, and
urinary metanephrines
b. All of the mentioned
c. Baseline serum creatinine
d. Renal ultrasound with Doppler
Question 5
Complete
Mark 0.00 out of 1.00
Flag question
Question text
The 37-year-old G0 patient with 4-year history of chronic hypertension
treated with amlodipine 5 mg per day, and a 2-year history of diabetes
which she manages with diet and exercise presents for a preconception
counseling. She has a copy of her records from her primary care physician.
Serum creatinine of 0.9 mg/dL, urine protein-to-creatinine ratio of 0.22, a
normal electrocardiogram. Fasting glucose was 93 mg/dL (5.16 mmol/l) this
morning, and her hemoglobin A1C was 6.0 last month. Which of the
following is the most appropriate recommendation?
Select one:
a. There is no contraindication to pregnancy, but you recommend
converting to an alternative antihypertensive agent in advance of pursuing
pregnancy to reduce teratogenic risk.
b. You do not yet have adequate information from which to make a
recommendation.
c. There is no contraindication to pregnancy, but she can further
reduce her risk with weight loss and regular, moderate intensity
aerobic exercise 3–4 times per week.
d. Risk for adverse outcome in pregnancy is unacceptably high, and you
recommend that she consider gestational surrogacy or adoption.

KIDNEY PATHOLOGY

Question 1
Complete
Mark 0.00 out of 1.00
Flag question
Question text
Which of the following does not characterize nephrotic syndromes?
Select one:
a. Hyperalbuminemia
b. Heavy proteinuria
c. Hypercholesterolemia
d. Edema
Question 2
Complete
Mark 1.00 out of 1.00
Flag question
Question text
Which of the following statements about polycystic kidney disease is true?
Select one:
a. It is usually X-linked inheritance.
b. Asymptomatic coexisting hepatic cysts are uncommon.
c. Renal complications are more common in women than men.
d. 10% of those with this disease die from rupture of an intracranial
berry aneurysm.
Question 3
Complete
Mark 1.00 out of 1.00
Flag question
Question text
Which of the following is the threshold for proteinuria in pregnancy, above
which levels are considered abnormal?
Select one:
a. 250 mg/d
b. 300 mg/d
c. 1000 mg/d
d. 100 mg/d
Question 4
Complete
Mark 0.00 out of 1.00
Flag question
Question text
Once you have treated a pregnant patient for pyelonephritis as an inpatient,
for how long should you continue oral therapy as an outpatient?
Select one:
a. 14–21 days
b. 3–5 days
c. 1–3 days
d. 7–14 days
Question 5
Complete
Mark 1.00 out of 1.00
Flag question
Question text
Which of the following is most likely to lead to end-stage renal disease?
Select one:
a. Hypertension
b. Glomerulonephritis
c. Polycystic kidney disease
d. Diabetes
Question 6
Complete
Mark 0.00 out of 1.00
Flag question
Question text
What percentage of pregnant women with asymptomatic bacteriuria
develop a symptomatic infection if left untreated?
Select one:
a. 5%
b. 10%
c. 25%
d. 50%
Question 7
Complete
Mark 1.00 out of 1.00
Flag question
Question text
What is the leading cause of septic shock during pregnancy?
Select one:
a. Breast abscess
b. Chorioamnionitis
c. Pneumonia
d. Pyelonephritis
Question 8
Complete
Mark 1.00 out of 1.00
Flag question
Question text
In case of urinary tract obstruction what diagnostic option is the method of
a first choice?
Select one:
a. Plain abdominal radiograph
b. One-shot intravenous pyelogram
c. Renal sonography
d. MR imaging
Question 9
Complete
Mark 1.00 out of 1.00
Flag question
Question text
Which of the following is NOT a physiological change in pregnancy?
Select one:
a. Progesterone-induced relaxation of the muscularis
b. Glomerular hypertrophy
c. Vesicoureteral reflux
d. Distal ureteral compression by the uterus
Question 10
Complete
Mark 1.00 out of 1.00
Flag question
Question text
Which of the following is not commonly found in patients with a urethral
diverticulum?
Select one:
a. Urinary retention
b. Palpable mass
c. Pain
d. Recurrent urinary infections
Вопрос 1
Выполнен
Баллов: 1,00 из 1,00
Отметить вопрос
Текст вопроса
What percentage of pregnant women experience a recurrent urinary tract
infection after completion of treatment for pyelonephritis?
Выберите один ответ:
a. 30–40%
b. 50–60%
c. 5–10%
d. 10–20%
Вопрос 4
Выполнен
Баллов: 0,00 из 1,00
Отметить вопрос
Текст вопроса
Which of the following statements regarding physiological changes in
pregnancy is true?
Выберите один ответ:
a. Effective renal plasma flow increases.
b. The number of glomerular cells increases.
c. Glomerular filtration decreases.
d. Serum creatinine concentration increases.
Вопрос 5
Выполнен
Баллов: 1,00 из 1,00
Отметить вопрос
Текст вопроса
What is the cutoff for a urinary protein-to-creatinine ratio on a spot urine
sample in pregnant patient?
Выберите один ответ:
a. 0.1
b. 0.2
c. 0.3
d. 0.5
Вопрос 6
Выполнен
Баллов: 1,00 из 1,00
Отметить вопрос
Текст вопроса
What is the most common reason for acute kidney injury in obstetrics?
Выберите один ответ:
a. Acute blood loss
b. Sepsis
c. Ureteral injury at cesarean section
d. Use of loop diuretics

Вопрос 8
Выполнен
Баллов: 1,00 из 1,00
Отметить вопрос
Текст вопроса
By what creatinine does one define moderate renal impairment in patients
with chronic renal disease?
Выберите один ответ:
a. 5–7 mg/dL
b. 1.5–3.0 mg/dL
c. 3.0–4.5 mg/dL
d. 1–1.5 mg/dL
Вопрос 9
Выполнен
Баллов: 1,00 из 1,00
Отметить вопрос
Текст вопроса
Which of the following complications is not increased in pregnant patients
with nephrotic syndrome?
Выберите один ответ:
a. Anemia
b. Postterm pregnancy
c. Preeclampsia
d. Renal insufficiency
Вопрос 10
Выполнен
Баллов: 1,00 из 1,00
Отметить вопрос
Текст вопроса
Which of the following is not a recommended requisite for renal transplant
patients who want to attempt pregnancy?
Выберите один ответ:
a. Serum creatinine &lt;2 mg/dL
b. No identifiable pyelocalyceal distention by urography
c. Proteinuria &lt;300 mg/day
d. No evidence of graft rejection for 6 months

The clinical presentation of a what condition usually includes hypertension,


hematuria, red-cell casts, pyuria, proteinuria, edema, hypertension, and
circulatory congestion?
Select one:
a. Anemia
b. Polycystic kidney disease
c. Glomerulonephritis
d. Preeclampsia
e. Renal insufficiency

Young pregnant patient with pyelonephritis and labor contractions transferred to labor department for
monitoring. She is given intravenous (IV) antibiotics, IV fluids, acetaminophen, and a beta-agonist
because of the contractions. What complication is more likely in the setting of p-agonist use in patients
with pyelonephritis?
Select one:
a. Abruption
b. Acute kidney injury
c. Anemia
d. Pulmonary edema

The pregnant patient with a kidney stone asks how many pregnant women actually pass their stone
with the management plan of analgesia and intravenous hydration. What is your response?
Select one:
a. 95-100%
b. 50-60%
c. 65-80%
d. 90-95%

When managing a patient with postpartum acute kidney injury, which of the following medications
does not need to have its dose adjusted?
Select one:
a. Clindamycin
b. Ketorolac tromethamine
c. Gentamycin
d. Magnesium sulfate X

CASE

Question 1
Complete
Mark 1.00 out of 1.00
Flag question
Question text
An 18-year-old G1P0 presents at 30 weeks’ gestation with fever of 39oC,
chills, vomiting, dysuria, and frequent urination for 3 days. She is also
experiencing contractions. What is her most likely diagnosis?
Select one:
a. Labor
b. Pyelonephritis
c. Infarcted leiomyoma
d. Adnexal torsion
e. Appendicitis
Question 2
Complete
Mark 1.00 out of 1.00
Flag question
Question text
A 32-year-old G2P0 presents at 19 weeks’ gestation complaining of back
pain that radiates forward. She reports that the pain is intense, and she
looks very uncomfortable. She has not been febrile. A urinalysis is
significant only for red blood cells. An image from her renal ultrasound is
provided below. You diagnose the patient with a kidney stone. Which of the
following is the best management option for the patient at this time?

Select one:
a. Analgesia and intravenous hydration
b. Analgesia, intravenous hydration, intravenous antibiotics, and ureteral
stenting
c. Analgesia
d. Analgesia, intravenous hydration, and intravenous antibiotics
Question 3
Complete
Mark 0.00 out of 1.00
Flag question
Question text
A 25-year-old G1P0 at 16 weeks’ gestation presents for prenatal care. She
has a history of a kidney transplant 3 years ago. She is stable on prednisone
and azathioprine. The patient has not experienced any rejection, and her
blood pressure is well controlled. Which of the following obstetric
complications is not increased in her?
Select one:
a. Preeclampsia
b. Peripartum cardiomyopathy
c. Preterm birth
d. Fetal-growth restriction
Question 4
Complete
Mark 1.00 out of 1.00
Flag question
Question text
A pregnant woman presents with flank pain. She is afebrile but tachycardic
from pain. Her serum creatinine is elevated to 5 mg/dL. Images from her
magnetic resonance imaging are presented below - hydronephrosis. Which
of the following is the best plan of care?

Select one:
a. Renal biopsy
b. Nephrectomy
c. Intravenous antibiotics and aggressive fluid hydration
d. Percutaneous nephrostomy tube
Question 5
Complete
Mark 1.00 out of 1.00
Flag question
Question text
You are consulted about a 19-year-old primigravida at 18 weeks’ gestation.
The patient was diagnosed at her first prenatal care visit with asymptomatic
bacteriuria. Her provider treated her with nitrofurantoin 100 mg twice daily
for 7 days. The patient was seen a month after treatment, and her repeat
urine culture was negative. But now, a urine culture sent in error is positive
for >100,000 colony-forming units/mL gram-negative rods again. She
remains asymptomatic. Her provider is not sure what to do. What is the best
response to this consult?
Select one:
a. Single-dose treatment is more successful, so give the patient
nitrofurantoin 200 mg by mouth one time.
b. The urine culture is probably a false-positive given it was negative after
treatment, so she does not need any further antibiotics.
c. Recurrence of asymptomatic bacteriuria is 30% regardless of
antibiotic regimen and it may indicate covert upper tract infection, so
treat the patient with nitrofurantoin 100 mg by mouth at bedtime for
21 days.
d. The patient likely has a highly resistant organism, so she should be
admitted for intravenous antibiotics.
Вопрос 2
Выполнен
Баллов: 1,00 из 1,00
Отметить вопрос
Текст вопроса
A series of anterior-posterior projection chest radiographs in a second-
trimester pregnant woman with severe pyelonephritis in case of aggressive
fluid hydration showing what of the following:

Выберите один ответ:


a. Secondary pneumonia
b. Cardiac pulmonary edema
c. Streptococcal sepsis
d. Respiratory distress syndrome

PREECLAMPSIA

Question 1
Correct
Marked out of 1.00
Flag question
Question text
A 42-year-old woman presents for prenatal care. She has a demanding
professional career and reports she is unwilling to attend frequent prenatal
visits at the end of pregnancy. You discourage this, explaining these visits
are recommended for which of the following reasons?
Select one:
a. To aid in the timely detection of fetal complications such as growth
restriction or oligohydramnios
b. All of the mentioned
c. To detect possible malpresentation and discuss delivery planning
d. To aid in the early detection of preeclampsia
Question 2
Incorrect
Marked out of 1.00
Flag question
Question text
Among following choose correct statement concerning gestational
hypertension
Select one:
a. diagnosis is made in women with new-onset hypertension and proteinuria
after midpregnancy
b. 30–33% of these women subsequently develop preeclampsia syndrome
c. it is not dangerous until proteinuria has not yet developed
d. blood pressure returns to normal by 12 weeks postpartum
Question 3
Correct
Marked out of 1.00
Flag question
Question text
Among following choose correct statement concerning proteinuria in
preeclamptic patient
Select one:
a. reflects the hyperalbuminemia state
b. represent unique isolated nephphrotic syndrome
c. reflects the system-wide endothelial leak
d. is not clearly understood
Question 4
Correct
Marked out of 1.00
Flag question
Question text
Many conditions and factors are associated with an increased risk for
preeclampsia. Which of the following factors results in the greatest relative
risk for a diagnosis of preeclampsia in the current pregnancy?
Select one:
a. Primigravida
b. Systemic lupus erythematous
c. History of preeclampsia in a prior pregnancy
d. Advanced maternal age
Question 5
Correct
Marked out of 1.00
Flag question
Question text
All are features of HELLP syndrome except
Select one:
a. Increased liver enzymes
b. Eosinophillia
c. Thrombocytopenia
d. Haemolytic anemia
Question 6
Correct
Marked out of 1.00
Flag question
Question text
What percentage of eclamptic seizures occur more than 48 hours after
delivery?
Select one:
a. 10%
b. 20%
c. 2%
d. 5%
Question 7
Correct
Marked out of 1.00
Flag question
Question text
Antihypertensive contraindicated in pregnancy
Select one:
a. Hydrazaline
b. ACE inhibitors
c. Methyldopa
d. Nifedipine
Question 8
Correct
Marked out of 1.00
Flag question
Question text
DOC for hypertension in pregnancy
Select one:
a. {Methyldopa
b. Nifedipine
c. Labetalol
d. ACE inhibitors
Question 9
Incorrect
Marked out of 1.00
Flag question
Question text
What percentage of eclamptic seizures occur in women without significant
proteinuria?
Select one:
a. 5–9%
b. 10%
c. 30–33%
d. 20%
Question 10
Correct
Marked out of 1.00
Flag question
Question text
Which statement best describes renal perfusion and glomerular filtration
rates in women with preeclampsia that has not yet progressed to severe
disease?
Select one:
a. Similar compared to normal nonpregnant values
b. Increased compared to normal pregnant values
c. Similar compared to normal pregnant values
d. Decreased compared to normal nonpregnant values
Вопрос 2
Верно
Балл: 1,00
Отметить вопрос
Текст вопроса
Which of the following is thought to play a significant role in the
development of preeclampsia?
Выберите один ответ:
a. Immunological factors
b. Abnormal trophoblastic invasion
c. Genetic factors
d. All of the mentioned
Вопрос 3
Верно
Балл: 1,00
Отметить вопрос
Текст вопроса
A 39-year-old G3P2 presents at 30 weeks’ gestation with hypertension,
proteinuria, and headache. She is diagnosed with severe preeclampsia, and
induction of labor is indicated. Her pregnancy is complicated by the fetal
karyotype shown below. What is the possible explanation for her increased
risk for preeclampsia with the below fetal karyotype?
Выберите один ответ:
a. Increased frequency of placenta mosaicism
b. Higher frequency of spiral arteriole atherosis
c. Increased levels of oxidative products in the placenta
d. Increased antiangiogenic factor levels
Вопрос 6
Верно
Балл: 1,00
Отметить вопрос
Текст вопроса
What percentage of pregnancies are complicated by hypertension?
Выберите один ответ:
a. 2–3%
b. 10–20%
c. 5–10%
d. 4–5%
Вопрос 7
Верно
Балл: 1,00
Отметить вопрос
Текст вопроса
The typical blood volume of a gravida at term is 4500 mL. In patients with
preeclampsia, which of the following would be the expected blood volume?
Выберите один ответ:
a. 2500 mL
b. 4500 mL
c. 5000 mL
d. 3200 mL
Вопрос 9
Верно
Балл: 1,00
Отметить вопрос
Текст вопроса
Cause of convulsion in eclampsia
Выберите один ответ:
a. Hypokalemia
b. Shock
c. Cerebral anoxia due to arterial spasm
d. Hypovolemia
Вопрос 10
Верно
Балл: 1,00
Отметить вопрос
Текст вопроса
The following computed tomography image shows cerebral edema in a
postpartum patient with hypertension, vision changes, and confusion. What
associated morbidity is the patient at risk for?

Выберите один ответ:


a. Retinal artery occlusion
b. Hemiplegia
c. Cystic leukomalacia
d. Transtentorial herniation
CASE

Question 1
Correct
Marked out of 1.00
Flag question
Question text
Your patient is admitted to the hospital for evaluation of new-onset
hypertension at 30 weeks’ gestation. Which of the following tests or
evaluations is least appropriate?
Select one:
a. Maternal weight
b. Sonographic evaluation of fetal growth
c. Protein:creatinine ratio
d. Cell-free DNA testing for aneuploidy
Question 2
Correct
Marked out of 1.00
Flag question
Question text
Which of the following is the best management of an 18-year-old G1 P0
woman at 28 weeks’ gestation with a blood pressure of 160/110 mm Hg,
elevated liver function tests, and a platelet count of 60,000/uL?
Select one:
a. Platelet transfusion
b. Intravenous immunoglobulin therapy
c. Magnesium sulfate therapy and induction of labor
d. Oral antihypertensive therapy
Question 3
Correct
Marked out of 1.00
Flag question
Question text
A 24-year-old primigravida presents at 37 weeks’ gestation with headache,
a blood pressure of 170/102 mm Hg, and severe right upper quadrant pain.
She is diagnosed with HELLP syndrome and undergoes an uncomplicated
induction of labor. Her right upper quadrant pain persists, and a computed
tomography scan of her abdomen/pelvis is completed with the findings as
shown below. What is denoted by the asterisk (*)?

Select one:
a. Periportal hemorrhagic necrosis
b. Splenic infarction
c. Subcapsular hematoma
d. Intrahepatic infarction
Question 4
Correct
Marked out of 1.00
Flag question
Question text
A 21-year-old G1 at 36 weeks’ gestation presents for her clinic visit and is
noted to have a blood pressure of 148/88 mm Hg. A repeat blood pressure
30 minutes later is 146/92 mm Hg. Her blood pressures throughout
pregnancy have been below 140/90 mm Hg. She denies any complaints, and
urinalysis is negative for proteinuria. What is the most likely diagnosis?
Select one:
a. Chronic hypertension
b. Preeclampsia syndrome
c. Gestational hypertension
d. Delta hypertension
Question 5
Incorrect
Marked out of 1.00
Flag question
Question text
Which of the following interventions is least indicated for the postpartum
patient with hypertension, vision changes, and confusion and confirmed
cerebral edema?
Select one:
a. Intravenous immune globulin
b. Dexamethasone
c. Antihypertensive medication
d. Mannitol
Question 6
Correct
Marked out of 1.00
Flag question
Question text
A 28-year-old G1 at 38 weeks’ gestation presents with complaint of
contractions. Her blood pressure is noted to be 148/90 mm Hg and 152/96
mm Hg. She has a urine protein:creatinine ratio of 0.4, a creatinine of 1.04
mg/dL (baseline 0.48 mg/dL), normal AST and ALT, and platelet count of
110,000/µL. She denies any symptoms. What criteria for severe
preeclampsia does this patient meet?
Select one:
a. Elevated creatinine
b. She does not meet criteria for severe preeclampsia.
c. Proteinuria
d. Low platelets
Question 7
Correct
Marked out of 1.00
Flag question
Question text
A 19-year-old G1 P0 woman at 39 weeks’ gestation is diagnosed with
preeclampsia based on blood pressure of 150/90 mm Hg and 2+ proteinuria
on urine dipstick. The patient is placed on magnesium sulfate, and develops
flushing and fatigue. She asks about the need for the magnesium sulfate.
You explain that it is to prevent the seizures that may complicate
preeclampsia and may even cause death. The patient asks how seizures
associated with preeclampsia can cause mortality. Which of the following is
the most common mechanism?
Select one:
a. Electrolyte abnormalities
b. Myocardial infarction
c. Aspiration
d. Intracerebral hemorrhage
Вопрос 1
Верно
Балл: 1,00
Отметить вопрос
Текст вопроса
A 23-year-old primigravida presents with a blood pressure of 160/104 mm
Hg, 3+ proteinuria, and right upper quadrant discomfort. She has a vaginal
delivery complicated by bilateral sulcal tears and an estimated blood loss of
1500 mL. She produces 110 mL of urine in the first 4 hours postpartum and
her serum creatinine rises from 0.98 mg/dL to 1.42 mg/dL. What is the most
likely explanation for this finding?
Выберите один ответ:
a. Dehydration due to prolonged induction of labor
b. Postpartum hemorrhage
c. Subcapsular liver hematoma
d. Ureteral injury during laceration repair
Вопрос 3
Верно
Балл: 1,00
Отметить вопрос
Текст вопроса
A 29-year-old G1 P0 woman at 28 weeks’ gestation is admitted to the
hospital for preeclampsia. Her blood pressure is 150/100 mm Hg and her
protein excretion is 500 mg in 24 hours. On hospital day 7, she is diagnosed
with severe preeclampsia and the decision is made to administer
magnesium sulfate and deliver the baby. Which of the following findings is
most likely present in this patient as criteria for severe preeclampsia?
Выберите один ответ:
a. Platelet count of 115,000/uL
b. Elevated uric acid levels
c. 5 g of proteinuria excreted in a 24-hour period
d. 4+ pedal edema
Вопрос 7
Верно
Балл: 1,00
Отметить вопрос
Текст вопроса
A 24-year-old G4P2 at 26 weeks’ gestation presents via emergency
medical services after being found unconscious at home by her 6-year-old
child. She is oriented on arrival and found to have a blood pressure of
180/110 mm Hg and 4+ proteinuria. Upon questioning she reports a history
of eclampsia in her prior pregnancy at 34 weeks’ gestation. She then has a
5-minute-long tonic-clonic seizure which resolves with magnesium sulfate
administration. Once the patient is stabilized, she undergoes cesarean
delivery for malpresentation. The below magnetic resonance imaging was
performed postpartum. What is the most likely finding on imaging?

Выберите один ответ:


a. Subdural hematoma
b. Parenchymal hemorrhage
c. Subarachnoid hemorrhage
d. Posterior reversible encephalopathy syndrome

OBSTETRIC HEMORRHAGE

Question 1
Correct
Marked out of 1.00
Flag question
Question text
Couvelaire uterus is present:
Select one:
a. Placenta previa
b. Abruptio placenta
c. Vasa previa
d. All of the above
Question 2
Correct
Marked out of 1.00
Flag question
Question text
All of the following can lead to renal failure EXCEPT
Select one:
a. Eclampsia
b. Septicaemia
c. Heart disease
d. Abruption placentae
Question 3
Correct
Marked out of 1.00
Flag question
Question text
Placenta accreta is associated with:
Select one or more:
a. Uterine malformation
b. Placenta previa in present pregnancy
c. Multiple pregnancy
d. Uterine scar
e. Multipara
Question 4
Correct
Marked out of 1.00
Flag question
Question text
A pregnant woman presents with placenta previa of major degree. The fetus
is malformed. Which of the following will be the best line of management
Select one:
a. Rupture of membranes
b. Instillation of PGE2
c. Oxytocin drip
d. LSCS (lower segment Caesarean section)
Question 5
Correct
Marked out of 1.00
Flag question
Question text
A pregnant lady comes with 38 weeks with painless bleeding uterus is
nontense, non-tender, Next step is
Select one:
a. USG (ultrasonography)
b. Termination of Pregnancy
c. Per speculum examination
d. Conservative treatment
Question 6
Correct
Marked out of 1.00
Flag question
Question text
Which hemostatic surgical procedure is to be performed in the picture
below?
Select one:
a. Vaginal artery ligation
b. Uterine artery ligation
c. Ovarian artery ligation
d. B-Lynch procedure
Question 7
Correct
Marked out of 1.00
Flag question
Question text
Causes of uterine atony include which of the following?
Select one:
a. Placenta previa
b. Multiple fetuses
c. Obesity
d. Placental abruption
Question 8
Correct
Marked out of 1.00
Flag question
Question text
Viral infection risks from transfusion are accurately depicted in which of the
following statements?
Select one:
a. Hepatitis B risk is <1 per 100,000 units
b. Hepatitis C risk is 1 per 1–2 million units
c. Human immunodeficiency virus risk is 1 per 1–2 million units
d. All of the
Question 9
Incorrect
Marked out of 1.00
Flag question
Question text
What is the appropriate management of vulvovaginal hematomas?
Select one:
a. If bleeding ceases, small to moderate-sized hematomas can be treated
expectantly
b. Surgical exploration in all cases
c. In a small hematoma, if pain is severe then ice packs and analgesia are
appropriate
d. To prevent infection they should all undergo ultrasound guided drainage
Question 10
Correct
Marked out of 1.00
Flag question
Question text
During evaluation of postpartum hemorrhage following a vaginal delivery,
which of the following maneuvers or medications might be used?
Select one:
a. All of the mentioned
b. Carboprost tromethamine in patients with mild asthma
c. Bimanual uterine compression
d. Ergot alkaloids for patients with hypertension
Вопрос 1
Верно
Балл: 1,00
Отметить вопрос
Текст вопроса
Painless, causeless vaginal bleeding is seen in
Выберите один ответ:
a. Placenta previa
b. None
c. Accidental haemorrhage
d. PIH
Вопрос 2
Верно
Балл: 1,00
Отметить вопрос
Текст вопроса
The risk factors for placental abruption include the following except:
Выберите один ответ:
a. Hypertension
b. Diabetes mellitus
c. History of placental abruption in previous previous pregnancy
d. Preterm rupture of membranes
Вопрос 4
Верно
Балл: 1,00
Отметить вопрос
Текст вопроса
All of the following can predispose to coagulopathy EXCEPT
Выберите один ответ:
a. Dead fetus
b. Placenta previa
c. Placental abrupion
d. Amniotic fluid embolism
Вопрос 5
Верно
Балл: 1,00
Отметить вопрос
Текст вопроса
Placenta previa is characterized by all of the following except
Выберите один ответ:
a. Presents in 1st trimester
b. Causeless bleeding
c. Painless bleeding
d. Recurrent in nature
Вопрос 7
Верно
Балл: 1,00
Отметить вопрос
Текст вопроса
A 34-year-old G3P3 begins having brisk bright red bleeding following
completion of a vaginal delivery. You give her carboprost tromethamine and
perform the maneuver pictured below. What else should be immediately
considered?

Выберите один ответ:


a. Place large-bore intravenous lines, order blood, and begin volume
resuscitation
b. Ask for urgent help from anesthesia
c. Call for help
d. All of the mentioned
Вопрос 8
Верно
Балл: 1,00
Отметить вопрос
Текст вопроса
The slide from lung tissue pictured below is consistent with a fatal
syndrome. Which of the following proposed diagnostic criteria are required
to make the diagnosis?

Выберите один ответ:


a. Abrupt onset of cardiorespiratory arrest, or both hypotension and
respiratory compromise
b. All of the mentioned
c. Documentation of overt disseminated intravascular coagulopathy must
be detected prior to enough blood loss to cause dilutional coagulopathy
d. Clinical onset during labor or within 30 minutes of placental delivery
Вопрос 9
Верно
Балл: 1,00
Отметить вопрос
Текст вопроса
Which hemostatic surgical procedure is demonstrated in the picture below?

Выберите один ответ:


a. Vaginal artery ligation
b. B-Lynch procedure
c. Uterine artery ligation
d. Ovarian artery ligation
Вопрос 10
Неверно
Балл: 1,00
Отметить вопрос
Текст вопроса
The postpartum patient with severe haemorrhage after all necessary
measures undergoes a hysterectomy and 5 units of packed red blood cells
are given, but bleeding continues. What is the most likely etiology of the
bleeding?
Выберите один ответ:
a. Vaginal cuff bleeding
b. Placental implantation on the omentum
c. Dilutional coagulopathy
d. Lacerated internal iliac artery
CASE

Question 1
Correct
Marked out of 1.00
Flag question
Question text
An 18-year-old adolescent female is noted to have a marginal placenta
previa on an ultrasound examination at 22 weeks’ gestation. She does not
have vaginal bleeding or spotting. Which of the following is the most
appropriate management?
Select one:
a. Reassess placental position at 34 to 35 weeks’ gestation by
ultrasound.
b. Schedule cesarean delivery at 39 weeks.
c. Schedule an amniocentesis at 36 weeks and deliver by cesarean if the
fetal lungs are mature.
d. Schedule an MRI examination at 35 weeks to assess for possible percreta
involving the bladder.
e. Recommend termination of pregnancy.
Question 2
Correct
Marked out of 1.00
Flag question
Question text
An 18-year-old pregnant woman is noted to have vaginal bleeding. She is
bleeding from venipuncture sites, IV sites, and from her gums. Which of the
following is the most likely underlying diagnosis?
Select one:
a. Placenta previa
b. Placental abruption
c. Multifetal gestation
d. Gestational trophoblastic disease
e. Gestational diabetes
Question 3
Correct
Marked out of 1.00
Flag question
Question text
A 33-year-old woman G3 P2 who had two prior cesareans is currently at 38
weeks’ gestation. She is noted to have a posterior placenta.On ultrasound,
there is evidence of possible placenta accreta. The patient is counseled
about the possible risk of need for hysterectomy. Which of the following is
the most accurate statement?
Select one:
a. Having two prior cesareans is associated with a 50% risk for placenta
accreta.
b. If the patient had gestational diabetes, the risk for placenta accreta
would be even higher.
c. The posterior placenta is associated with less of a risk for accreta
than an anterior placenta.
d. Placenta accreta is associated with a defect in the myometrial layer of
the uterus.
Question 4
Correct
Marked out of 1.00
Flag question
Question text
A 33-year-old woman G3 P2 who had two prior cesareans is currently at 38
weeks’ gestation. She is noted to have a posterior placenta.On ultrasound,
there is evidence of possible placenta accreta. The patient is counseled
about the possible risk of need for hysterectomy. Which of the following is
the most accurate statement?
Select one:
a. If the patient had gestational diabetes, the risk for placenta accreta
would be even higher.
b. Placenta accreta is associated with a defect in the myometrial layer of
the uterus.
c. Having two prior cesareans is associated with a 50% risk for placenta
accreta.
d. The posterior placenta is associated with less of a risk for accreta
than an anterior placenta.
Question 5
Correct
Marked out of 1.00
Flag question
Question text
A 21-year-old patient at 28 weeks’ gestation has vaginal bleeding and is
diagnosed with placenta previa. Which of the following is a typical feature
of this condition?
Select one:
a. Painful bleeding
b. First episode of bleeding is usually profuse
c. Associated with postcoital spotting
d. Commonly associated with coagulopathy
Question 6
Correct
Marked out of 1.00
Flag question
Question text
A 26-year-old G2 P1 woman underwent a normal vaginal delivery.A viable
3.9kg male infant was delivered. The placenta delivered spontaneously. The
obstetrician noted significant blood loss from the vagina, totaling
approximately 700 mL. The uterine fundus appeared to be well contracted.
Which of the following is the most common etiology for the bleeding in this
patient?
Select one:
a. Uterine atony
b. Endometrial ulceration
c. Genital tract laceration
d. Coagulopathy
e. Retained placenta
Question 7
Correct
Marked out of 1.00
Flag question
Question text
A 34-year-old woman is noted to have significant uterine bleeding after a
vaginal delivery complicated by placenta abruption. She is noted to be
bleeding from multiple venipuncture sites. Which of the following is the
best (etiopathologycal) therapy?
Select one:
a. Hypogastric artery ligation
b. Packing of the uterus
c. Ligation of utero-ovarian ligaments
d. Immediate hysterectomy
e. Correction of coagulopathy
Question 8
Correct
Marked out of 1.00
Flag question
Question text
A 23-year-old G1 P0 woman at 38 weeks’ gestation delivered a 4.3kg baby
boy vaginally. Upon delivery of the placenta, there was noted to be an
inverted uterus, which was successfully managed including replacement of
the uterus. Which of the following placental implantation sites would most
likely predispose to an inverted uterus?
Select one:
a. Anterior
b. Lateral
c. Fundal
d. Posterior
e. Lower segment
Question 9
Incorrect
Marked out of 1.00
Flag question
Question text
A 33-year-old G5 P5 woman, who is being induced for preeclampsia
delivers a 5kg baby. Upon delivery of the placenta, uterine inversion is
noted. The physician attempts to replace the uterus, but the cervix is
tightly contracted, preventing the fundus of the uterus from being
repositioned. Which of the following is the best therapy for this patient?
Select one:
a. Infuse oxytocin intravenously.
b. Halothane anesthesia
c. Dührssen incisions of the cervix.
d. Discontinue the magnesium sulfate.
e. Vaginal hysterectomy.
Question 10
Correct
Marked out of 1.00
Flag question
Question text
A 24-year-old woman underwent a normal vaginal delivery of a term infant
female. After the delivery, the placenta does not deliver even after 30
minutes. Which of the following would be the next step for this patient?
Select one:
a. Estrogen intravaginally
b. Attempt a manual extraction of the placenta.
c. Hysterectomy.
d. Wait for an additional 30 minutes.
e. Initiate oxytocin.
Вопрос 1
Верно
Балл: 1,00
Отметить вопрос
Текст вопроса
A 25-year-old woman at 34 weeks’ gestation is noted to have a placenta
previa, after she presented with vaginal bleeding and has undergone
sonography. At 37 weeks, she has a scheduled cesarean. Upon cesarean
section, bluish tissue densely adherent between the uterus and maternal
bladder is noted. Which of the following is the most likely diagnosis?
Выберите один ответ:
a. Placental polyp
b. Placenta percreta
c. Placenta melanoma
d. Placenta accreta
Вопрос 2
Неверно
Балл: 1,00
Отметить вопрос
Текст вопроса
A 32-year-old woman is seen in the obstetrical unit at the hospital. She is at
29 weeks’ gestation, with a chief complaint of significant vaginal bleeding.
She had a stillbirth with her prior pregnancy due to placental abruption. The
patient asks the physician about the accuracy of ultrasound in the
diagnosis of abruption. Which of the following statements is most accurate?
Выберите один ответ:
a. Fetal ultrasound is more accurate in diagnosing placental abruption than
placenta previa.
b. Fetal ultrasound is quite sensitive in diagnosing placental abruption.
c. Fetal ultrasound is not sensitive in diagnosing placental abruption
d. Ultrasound is sensitive in diagnosing abruption that occurs in the lower
aspect of the uterus.
Вопрос 10
Верно
Балл: 1,00
Отметить вопрос
Текст вопроса
A 24-year-old G1 P0 woman at 39 weeks’ gestation had induction of labor
due to gestational hypertension. She was placed on magnesium sulfate for
seizure prophylaxis. She was placed on oxytocin for 15 hours and reached a
cervical dilation of 6 cm. After being at 6 cm dilation for 3 hours despite
adequate uterine contractions as judged by 240 Montevideo units, she
underwent a cesarean delivery. The baby was delivered without difficulty
through a low-transverse incision.Upon delivery of the placenta, profuse
bleeding was noted from the uterus, reaching 1500 mL. Which of the
following is the most likely cause of hemorrhage in this patient?
Выберите один ответ:
a. Retained placenta
b. Uterine laceration
c. Uterine atony
d. Uterine inversion
e. Coagulopathy
Вопрос 1
Неверно
Балл: 1,00
Отметить вопрос
Текст вопроса
A 32-year-old woman is seen in the obstetrical unit at the hospital. She is at
29 weeks’ gestation, with a chief complaint of significant vaginal bleeding.
She had a stillbirth with her prior pregnancy due to placental abruption. The
patient asks the physician about the accuracy of ultrasound in the
diagnosis of abruption. Which of the following statements is most accurate?
Выберите один ответ:
a. Fetal ultrasound is more accurate in diagnosing placental abruption than
placenta previa.
b. Ultrasound is sensitive in diagnosing abruption that occurs in the lower
aspect of the uterus.
c. Fetal ultrasound is quite sensitive in diagnosing placental abruption.
d. Fetal ultrasound is not sensitive in diagnosing placental abruption
Вопрос 2
Неверно
Балл: 1,00
Отметить вопрос
Текст вопроса
A 35-year-old woman presents with bright red vaginal bleeding at 30
weeks’ gestation. Her urine drug screen is positive. Which of the following
is most likely to be present in her drug screen?
Выберите один ответ:
a. Cocaine
b. Marijuana
c. Barbiturates
d. Benzodiazepines
e. Alcohol
Вопрос 3
Неверно
Балл: 1,00
Отметить вопрос
Текст вопроса
Which of the following statements concerning myomectomy for subserosal
(outside surface of the uterus) uterine fibroids without entrance of the
uterine cavity is most likely to be correct regarding the risk of placental
accreta?
Выберите один ответ:
a. Her risk of accreta is most likely not affected by the myomectomy.
b. Her risk of accreta is most likely to be decreased due to the
myomectomy.
c. If the myomectomy incisions are anterior, then she has an increased risk
of a placental polyp
d. Her risk of accreta is most likely to be increased due to the
myomectomy.
Вопрос 6
Неверно
Балл: 1,00
Отметить вопрос
Текст вопроса
A 32-year-old G1 P0 woman at 40 weeks’ gestation undergoes a normal
vaginal delivery. Delivery of the placenta is complicated by an inverted
uterus, with subsequent hemorrhage leading to 1500 mL of blood loss. She
is managed with a transfusion of erythrocytes. Which of the following is the
best explanation of the mechanism of hemorrhage?
Выберите один ответ:
a. Inverted uterus causes muscular abrasions and lacerations leading to
bleeding.
b. Inverted uterus leads to inability for an adequate myometrial
contraction effect.
c. Inverted uterus causes a local coagulopathic reaction to the uterus and
endometrium.
d. Inverted uterus stretches the uterus, causing trauma to blood vessels
leading to bleeding.
Вопрос 2
Верно
Балл: 1,00
Отметить вопрос
Текст вопроса
A 22-year-old G1 P0 woman at 34 weeks’ gestation presents with moderate
vaginal bleeding and no uterine contractions. Her blood pressure (BP) is
110/60 mm Hg and heart rate (HR) 103 beats per minute (bpm). The
abdomen is nontender. Which of the following sequence of examinations is
most appropriate?
Выберите один ответ:
a. Ultrasound examination, digital examination, speculum examination
b. Speculum examination, ultrasound examination, digital examination
c. Ultrasound examination, speculum examination, digital
examination
d. Digital examination, ultrasound examination, speculum examination
Вопрос 3
Верно
Балл: 1,00
Отметить вопрос
Текст вопроса
A 28-year-old woman at 32 weeks’ gestation is seen in the obstetrical
triage area for vaginal bleeding described as significant with clots. She
denies cramping or pain. An ultrasound is performed revealing that the
placenta is covering the internal os of the cervix. Which of the following is a
risk factor for this patient’s condition?
Выберите один ответ:
a. Multiple gestations
b. Polyhydramnios
c. Hypertension
d. Prior salpingitis
Вопрос 5
Верно
Балл: 1,00
Отметить вопрос
Текст вопроса
A 29-year-old G1P0 woman at 39 weeks’ gestation delivered vaginally.Her
placenta does not deliver easily. A manual extraction of the placenta is
attempted and the placenta seems to be adherent to the uterus.A
hysterectomy is contemplated, but the patient refuses due to strongly
desiring more children. The cord is ligated with suture as high as
possible.The patient is given the option of methotrexate therapy. Which of
the following is the most likely complication after this intervention?
Выберите один ответ:
a. Utero-vaginal fistula
b. Infection
c. Malignant degeneration
d. Coagulopathic bleeding

A 32-year-old woman has severe postpartum hemorrhage that does not respond
to medical therapy. The obstetrician states that surgical management is the best
therapy. The patient desires future child bearing.Which of the following is most
appropriate to achieve the therapeutic goals?
Select one:
a. Cervical cerclage X
b. Ligation of the external iliac artery
c. Utero-ovarian ligament ligation
d. Supracervical hysterectomy
e. Hypogastric artery ligation
PUBERTY

Question 1
Correct
Marked out of 1.00
Flag question
Question text
When an individual experiences difficulty identifying with their phenotypic
gender, perceiving themselves to be of the opposite gender, the condition
is termed which of the following?
Select one:
a. Homosexuality
b. Gender o rearing mismatch
c. Sexual identity conflict
d. Gender dysphoria
Question 2
Correct
Marked out of 1.00
Flag question
Question text
Which of the following organisms is most likely to cause prepubertal
vulvitis?
Select one:
a. Group A β-hemolytic streptococcus
b. Candida albicans
c. Group B streptococcus
d. Staphylococcus aureus
Question 3
Correct
Marked out of 1.00
Flag question
Question text
Delayed puberty is characterized by a lack of initial pubertal changes,
usually thelarche, by what threshold age?
Select one:
a. 16 years
b. 15 years
c. 14 years
d. 13 years
Question 4
Correct
Marked out of 1.00
Flag question
Question text
According to the U.S. FDA vaccines targeting human papillomavirus
infection are approved from what age (in years) for females or males?
Select one:
a. 15
b. 9
c. 12
d. 21
Question 5
Correct
Marked out of 1.00
Flag question
Question text
Which of the following is true regarding isolated premature thelarche?
Select one:
a. Bone age is advanced.
b. None of the mentioned
c. Gonadotropin levels are elevated.
d. It is most common in girls younger than 2 years.
Question 6
Correct
Marked out of 1.00
Flag question
Question text
What approximate percentage of malignant tumors diagnosed in childhood
are of ovarian origin?
Select one:
a. 10 percent
b. 30 percent
c. 0.1 percent
d. 1 percent
Question 7
Correct
Marked out of 1.00
Flag question
Question text
Mastitis is unusual in children and adolescents. What is the most common
pathogen isolated from breast abscesses in the pediatric population?
Select one:
a. Staphylococcus epidermidis
b. Escherichia coli
c. Streptococcus pyogenes
d. Staphylococcus aureus
Question 8
Correct
Marked out of 1.00
Flag question
Question text
Pubarche, with development of axillary and pubic hair, is stimulated by the
androgens derived primarily from which of the following?
Select one:
a. Adrenal glands
b. Physiology continues to be poorly understood.
c. Ovarian stroma
d. Peripheral aromatization of estrogens
Question 9
Incorrect
Marked out of 1.00
Flag question
Question text
A primary goal of therapy or central precocious puberty includes prevention
of which of the following consequences?
Select one:
a. Excessive breast size
b. None of the mentioned
c. Learning disabilities
d. Short stature
Question 10
Correct
Marked out of 1.00
Flag question
Question text
Accessory nipples, when present, are located along the embryonic milk line
extending from the axilla to the groin bilaterally. Their presence is termed
which of the following?
Select one:
a. Multimomium
b. Areola duplicata
c. Polymammoma
d. Polythelia
Question 1
Correct
Marked out of 1.00
Flag question
Question text
Which of the following generally occurs first among the major
developmental events of female puberty?
Select one:
a. Menarche
b. Pubarche
c. Thelarche
d. Growth spurt
Question 2
Correct
Marked out of 1.00
Flag question
Question text
All states in the United States legally allow a minor woman to consent to
medical examination and treatment on her own behalf in all of the following
situations EXCEPT:
Select one:
a. Emancipated minor status
b. Requesting a school-sponsored sports’ physical examination
c. Living permanently apart from parents
d. Pregnancy
Question 3
Incorrect
Marked out of 1.00
Flag question
Question text
With prepubertal vulvovaginitis, what approximate percentage of bacterial
cultures obtained from vaginal swabs show only normal genital flora?
Select one:
a. 25 percent
b. 90 percent
c. 50 percent
d. 75 percent
Question 4
Incorrect
Marked out of 1.00
Flag question
Question text
Which of the following is required before initiating a contraceptive method
or a healthy, sexually active adolescent?
Select one:
a. Human papillomavirus (HPV) vaccination
b. Pelvic examination
c. Cervical cancer screening
d. None of the mentioned
Question 5
Correct
Marked out of 1.00
Flag question
Question text
Which of the following statements is true regarding pelvic anatomy in the
female infant and child?
Select one:
a. All the above are true statements.
b. The ovaries have obtained their normal adult size by birth.
c. At birth, the uterus and cervix are approximately equal in size.
d. Presence of an endometrial stripe or fluid within the endometrial
cavity of the newborn uterus is a normal finding with sonography.
Question 6
Incorrect
Marked out of 1.00
Flag question
Question text
Which of the following statements is FALSE regarding the development of
the hypothalamic–pituitary-ovarian axis in the female fetus and neonate?
Select one:
a. Neonatal breast budding, minor uterine bleeding, and transient ovarian
cysts may occur as a normal response to initially high gonadotropin levels.
b. The gonadotropin-releasing hormone (GnRH) “pulse generator” remains
functionally dormant until several months after birth.
c. By 5 months’ gestation, 6 to 7 million oocytes have been created from
accelerated germ cell division.
d. At birth, follicle-stimulating hormone (FSH) and luteinizing hormone (LH)
concentrations rise and remain high during the first 3 months of life.
Question 7
Incorrect
Marked out of 1.00
Flag question
Question text
Delayed puberty in women is defined as the lack of lack of menarche by
what age?
Select one:
a. 16
b. 14
c. 15
d. 18
Question 8
Correct
Marked out of 1.00
Flag question
Question text
Abnormal breast development may be due to either fascial adherence to the
underlying muscle layer or due to high-dose exogenous hormone exposure
during puberty. In such cases, breasts have excessive forward but limited
lateral growth, which causes an abnormal shape. This condition is referred
to as which of the following?
Select one:
a. Aberrant breast hypertrophy
b. Perithelia
c. Asymmetric lactiferous hyperplasia
d. Tuberous breasts
Question 9
Correct
Marked out of 1.00
Flag question
Question text
How do BMI correlate with pubertal development?
Select one:
a. higher BMI correlate with late development
b. no correlation was observed with BMI and pubertal development
c. lower BMI correlate with early development
d. higher BMI correlate with early development
Question 10
Incorrect
Marked out of 1.00
Flag question
Question text
Compared with several decades ago, the age at which U.S. girls currently
experience thelarche and menarche has shown which of the following
trends?
Select one:
a. In general, later than in past
b. No clear trend observed
c. In general, earlier than in past
d. For girls with a higher body mass index, later than in the past
Вопрос 1
Верно
Балл: 1,00
Отметить вопрос
Текст вопроса
In the United States, adolescents can obtain medical care without parental
knowledge or consent or which of the following “medically emancipated”
conditions?
Выберите один ответ:
a. Contraception
b. Pregnancy
c. All of the mentioned
d. Substance abuse
Вопрос 3
Верно
Балл: 1,00
Отметить вопрос
Текст вопроса
Which of the following is the most common cause of central precocious
puberty?
Выберите один ответ:
a. Congenital central nervous system anomaly
b. Head trauma
c. Idiopathic
d. Hydrocephalus secondary to surgery
Вопрос 6
Верно
Балл: 1,00
Отметить вопрос
Текст вопроса
Which of the following physiologic or anatomic characteristics predispose
to prepubertal vulvovaginitis?
Выберите один ответ:
a. All of the mentioned
b. Nonestrogenized vulvovaginal epithelium
c. Lack of labial fat pads and labial hair
d. Poor vulvar hygiene

Which of the following organs is correctly paired with its embryologic origin?
Select one:
a. Bladder—mesonephric duct
b. Uterus—mesonephric duct
c. Testes and ovaries—genital ridge
d. Kidney—paramesonephric duct

During a first-trimester ultrasound or pregnancy dating, you discover that your patient has a subseptate
uterus. While counseling her, you explain that this mullerian anomaly developed in utero secondary to
failure of the midline uterine septum to reabsorb completely. Normally, reabsorption of the uterine
septum in a fetus should occur by how many weeks of gestation?
Select one:
a. 28
b. 36
c. 32
d. 20

Septate uterus, as shown in the sonogram below, has a significantly higher spontaneous abortion rate
and early pregnancy loss rate than bicornuate uterus. What is the primary mechanism thought to be
responsible for this extraordinarily high pregnancy wastage?

Select one:
a. Associated cervical anomalies
b. Distortion of the uterine cavity
c. Partial or complete Implantation on the largely avascular septum
d. Distortion of the uterine serosal contour

Which of the following are seen in offspring of women who took diethylstilbestrol (DES) during the
associated pregnancy?
Select one:
a. Impaired conception rates in female offspring
b. T-shaped uterus, "cockscomb" cervix, and increased rates of clear cell adenocarcinoma in female
offspring
c. Cryptorchidism, testicular hypoplasia, and malformations of the penis in male offspring
d. All of the mentioned

During a first-trimester ultrasound or pregnancy dating, you discover that your patient has a subseptate
uterus. While counseling her, you explain that this miillerian anomaly developed in utero secondary to
failure of the midline uterine septum to reabsorb completely. Normally, reabsorption of the uterine
septum in a fetus should occur by how many weeks of gestation?
Select one:
a. 28
b. 36
c. 32 X
d. 20

Which of the following anatomic structures is correctly paired with its embryologic origin?
Select one:
a. All of the mentioned
b. Genital tubercle—clitoris
c. Urethral folds—labia minora
d. Labioscrotal folds—labia majora

Congenital anatomic disorders of the female reproductive tract may result from which of the following
mechanisms?
Select one:
a. Developmental arrest
b. Genetic mutation
c. All of the mentioned
d. Epigenetic factors
Which of the following are seen in offspring of women who took diethylstilbestrol (DES) during the
associated pregnancy?
Select one:
a. T-shaped uterus, "cockscomb" cervix, and increased rates of clear cell adenocarcinoma in female
offspring
b. Cryptorchidism, testicular hypoplasia, and malformations of the penis in male offspring
c. All of the mentioned
d. Impaired conception rates in female offspring

Compared with other müllerian duct defects,a transverse vaginal septum is associated with a lower rate
of which of the following?Select one:a. Urologic abnormalitiesb. Need or surgical correctionc. None of
the mentionedd. Endometriosis

CASE

Вопрос 1
Верно
Балл: 1,00
Отметить вопрос
Текст вопроса
A 5-year-old female child is brought into the physician’s office for breast
development and menses. On examination, the child is found to have Tanner
IV breast development and Tanner IV pubic and axillary hair. Which of the
following is the most likely therapy for this patient?
Выберите один ответ:
a. Aromatase inhibitor
b. Gonadotropin-releasing hormone (GnRH) agonist
c. Surgical therapy and removal of an ovarian tumor
d. Oral contraceptive use
e. Reassurance and expectant management
Вопрос 2
Неверно
Балл: 1,00
Отметить вопрос
Текст вопроса
A 6-year-old girl is noted to have breast development and vaginal spotting.
No abnormal hair growth is noted. A 10-cm ovarian mass is palpated on
rectal examination. Which of the following is the most likely diagnosis?
Выберите один ответ:
a. Granulosa-theca cell tumor
b. Congenital adrenal hyperplasia
c. Idiopathic precocious puberty
d. Benign cystic tumor (dermoid)
e. Sertoli– Leydig cell tumor
Вопрос 3
Верно
Балл: 1,00
Отметить вопрос
Текст вопроса
A smooth, firm, mobile, nontender right-breast mass is palpable during
examination of a 12-year-old girl. It is 1 cm in its greatest dimension and
appears solid sonographically, as shown here. She has had no major health
problems to date. What is the most likely diagnosis?

Выберите один ответ:


a. Ductal ectasia
b. Fibroadenoma
c. Malignancy
d. Asymmetric breast budding
Вопрос 4
Верно
Балл: 1,00
Отметить вопрос
Текст вопроса
An 8-year-old girl presents with a mucopurulent and bloody vaginal
discharge. Her parents report that, while traveling recently, the girl was
briefly hospitalized for treatment of severe diarrhea. Examinations of the
vagina in the office and under anesthesia reveal no tumor or foreign body.
What is the most likely organism involved?
Выберите один ответ:
a. Shigella spp.
b. Salmonella spp.
c. Candidia glabrata
d. Enterobius vermicularis
Вопрос 5
Верно
Балл: 1,00
Отметить вопрос
Текст вопроса
Medication used in the treatment of idiopathic central precocious puberty
include:
Выберите один ответ:
a. Exogenous gonadotropins
b. Clomiphene citrate
c. Ethinyl estradiol
d. GnRH agonists

ANATOMIC DISORDERS

Вопрос 1
Неверно
Балл: 1,00
Отметить вопрос
Текст вопроса
Müllerian anomalies are associated with anomalies of all of the following
systems EXCEPT:
Выберите один ответ:
a. Hepatic
b. Skeletal
c. Renal
d. Neurological
Вопрос 2
Верно
Балл: 1,00
Отметить вопрос
Текст вопроса
Which of the following can cause virilization of a 46,XX infant?
Выберите один ответ:
a. Placental aromatase deficiency
b. Maternal virilizing ovarian tumors, such as Sertoli–Leydig tumor, or
maternal use of androgenic medication
c. All of the mentioned
d. Fetal congenital adrenal hyperplasia due to deficiency of 21-hydroxylase
Вопрос 3
Верно
Балл: 1,00
Отметить вопрос
Текст вопроса
Appropriate techniques or the surgical correction of the imperforate hymen
involve all of the following EXCEPT:
Выберите один ответ:
a. Needle aspiration of the hematocolpos
b. Repair in infancy or after thelarche
c. Hymenectomy
d. Laparoscopy to exclude endometriosis
Вопрос 4
Верно
Балл: 1,00
Отметить вопрос
Текст вопроса
Which of the following is NOT true of Klinefelter syndrome?
Выберите один ответ:
a. Affected men are tall and excessively virilized.
b. It confers an increased risk for germ cell tumors, osteoporosis, and
breast cancer.
c. The syndrome occurs in 1 in 500-600 births of all males.
d. Affected men have significantly reduced fertility.
Вопрос 5
Верно
Балл: 1,00
Отметить вопрос
Текст вопроса
Which of the following statements regarding bladder exstrophy is NOT
true?
Выберите один ответ:
a. It is characterized by an exposed bladder lying outside the abdomen.
b. External genitalia are typically normal except for occasional
defects.
c. Surgical closure is performed early in life and as a staged procedure.
d. It occurs from failure of the cloacal membrane to be reinforced by an
ingrowth of mesoderm.
Вопрос 6
Верно
Балл: 1,00
Отметить вопрос
Текст вопроса
Which of the following disorders of sex development is correctly paired with
its category of abnormality?
Выберите один ответ:
a. All of the mentioned
b. Hypospadias—ambiguous genitalia
c. True hermaphroditism–ovotesticular disorder
d. Streak gonad—gonadal dysgenesis
Вопрос 7
Верно
Балл: 1,00
Отметить вопрос
Текст вопроса
Both urinary and genital systems develop from which of the following cell
types?
Выберите один ответ:
a. Ectoderm
b. None of the mentioned
c. Mesoderm
d. Endoderm
Вопрос 8
Верно
Балл: 1,00
Отметить вопрос
Текст вопроса
The vagina forms in part from both the müllerian ducts and which other
structure?
Выберите один ответ:
a. Genital ridge
b. Paramesonephric duct
c. Urogenital sinus
d. Mesonephric duct
Вопрос 9
Верно
Балл: 1,00
Отметить вопрос
Текст вопроса
In the male fetus, dihydrotestosterone (DHT) is involved in which of the
following?
Выберите один ответ:
a. Fusion of the labioscrotal folds to form the scrotum
b. Enlargement of the phallus
c. Lengthening of the anogenital distance
d. All of the mentioned
Вопрос 10
Неверно
Балл: 1,00
Отметить вопрос
Текст вопроса
One of your patients brings her 13-year-old daughter for consultation
regarding the girl’s complaints of pelvic pain. The daughter has
experienced normal pubertal development to date but has not yet
experienced menarche. She began having monthly pelvic pain about 1 year
ago, and the pain has been progressively worsening. During physical
examination, her external genitalia, including hymeneal ring, appear normal.
However, proximal to the hymen, only a vaginal dimple is found.
Rectovaginal examination confirms the presence of a uterus and cervix.
What is your diagnosis?
Выберите один ответ:
a. Cervical atresia
b. Müllerian agenesis
c. Vaginal atresia
d. Imperforate hymen

CASE

Вопрос 1
Неверно
Балл: 1,00
Отметить вопрос
Текст вопроса
Müllerian anomalies are associated with anomalies of all of the following
systems EXCEPT:
Выберите один ответ:
a. Hepatic
b. Renal
c. Neurological
d. Skeletal
Вопрос 2
Неверно
Балл: 1,00
Отметить вопрос
Текст вопроса
Why is careful preoperative planning warranted with congenital vaginal
cysts, as shown in sonogram below?

Выберите один ответ:


a. The recurrence rate after excision is high.
b. Their typical location is in the posterior-lateral wall of the vagina.
c. Some may extend up to the broad ligament and anatomically
approximate the distal course of the ureter.
d. They are frequently large, measuring greater than 8 cm in size.
Вопрос 3
Неверно
Балл: 1,00
Отметить вопрос
Текст вопроса
A 14-year-old nulligravida presents to the emergency department with
complaints of worsening lower abdominal pain over the past few days. She
states that she has had a similar pain in the past, usually for a few days
each month, but then it subsides. She is afebrile with stable vital signs.
Although she has breasts and axillary and pubic hair, she has never had a
period. Examination reveals a tender midline lower abdominal mass and a
bluish bulging vaginal mass, as shown here. Based on her history and your
physical examination, which of the following conditions is your most likely
diagnosis?

Выберите один ответ:


a. Gartner duct cyst
b. Longitudinal vaginal septum
c. Bartholin cyst
d. Imperforate hymen
Вопрос 4
Верно
Балл: 1,00
Отметить вопрос
Текст вопроса
When should surgical reconstruction of a bicornuate uterus (Strassman
metroplasty) be performed?
Выберите один ответ:
a. Prior to conception
b. With recurrent pregnancy loss and no other identifiable cause
except the uterine anomaly
c. In all women with the diagnosis of bicornuate uterus
d. Procedure no longer performed for repair of bicornuate uterus
Вопрос 5
Верно
Балл: 1,00
Отметить вопрос
Текст вопроса
The pathogenesis of poor pregnancy outcomes with a unicornuate uterus is
thought to be related to which of the following actors?
Выберите один ответ:
a. Cervical incompetence
b. Anomalous distribution of the uterine artery
c. All of the mentioned
d. Reduced uterine capacity
Вопрос 1
Верно
Балл: 1,00
Отметить вопрос
Текст вопроса
Appropriate techniques for surgical correction of the müIIerian aplasia
include which of the following?
Выберите один ответ:
a. Uterine transplantation
b. Hysterectomy
c. Hymenectomy
d. Surgical creation of a neovagina
Вопрос 2
Неверно
Балл: 1,00
Отметить вопрос
Текст вопроса
A 16-year-old nulligravida presents to the emergency department with
complaints of abdominal and vaginal pain, worsening during the past
several months. She describes the pain as being mostly on her right side
and much worse during menstruation. During your physical examination, a
patent vagina and cervix are noted, but a lateral vaginal and pelvic mass is
palpated. Transvaginal sonography demonstrates a single uterus and cervix
but also a large pelvic mass filled with complex fluid, as shown below, and
the absence of the right kidney. What is the most likely diagnosis?

Выберите один ответ:


a. Transverse vaginal septum
b. OHVIRA syndrome
c. Imperforate hymen
d. Ovarian neoplasm
Вопрос 3
Верно
Балл: 1,00
Отметить вопрос
Текст вопроса
Which müllerian anomaly results from complete failure of the paired
müllerian ducts to use? This anomaly is shown in this sonogram, and a
gestational sac is seen on the image’s left side.
Выберите один ответ:
a. Uterine didelphys
b. Unicornuate uterus
c. Septate uterus
d. Bicornuate uterus
Вопрос 4
Верно
Балл: 1,00
Отметить вопрос
Текст вопроса
Why is careful preoperative planning warranted with congenital vaginal
cysts, as shown in sonogram below?
Выберите один ответ:
a. Their typical location is in the posterior-lateral wall of the vagina.
b. Some may extend up to the broad ligament and anatomically
approximate the distal course of the ureter.
c. The recurrence rate after excision is high.
d. They are frequently large, measuring greater than 8 cm in size.
Вопрос 5
Верно
Балл: 1,00
Отметить вопрос
Текст вопроса
While staffing labor and delivery, you are asked to evaluate a pelvic mass
discovered during the performance of a postpartum tubal ligation. The
mass is a thin-walled, translucent cystic structure measuring less than 1 cm
in diameter. It is attached by a thin pedicle to the distal end of the right
fallopian tube. The resident physician is concerned that the mass may be
neoplastic, but you assure her that it is not. What is the likely diagnosis?
Выберите один ответ:
a. Accessory ovary
b. Paratubal cyst (hydatid of Morgagni)
c. Hydrosalpinx
d. Supernumerary ovary
AUB (abnormal uterine bleeding)

Question 1
Correct
Marked out of 1.00
Flag question
Question text
Which of the following is a limitation of Pipelle samplers used for
endometrial biopsy to evaluate abnormal uterine bleeding?
Select one:
a. Low sensitivity and high false-negative rate or focal endometrial
pathology
b. Greater patient discomfort compared with a stif metal curette
c. Rate of inadequate sampling that exceeds 50 percent
d. Inability to be performed in an office setting
Question 2
Correct
Marked out of 1.00
Flag question
Question text
When should irregular spotting or bleeding be evaluated in a
postmenopausal patient using hormone replacement therapy (HRT)?
Select one:
a. Continued bleeding after 6 months o HRT use
b. All of the mentioned
c. History of endometrial polyps
d. Abnormal bleeding that develops after initial amenorrhea
Question 3
Incorrect
Marked out of 1.00
Flag question
Question text
A patient using the progestin-only implant (Nexplanon) complains of
unscheduled light bleeding since her implant was placed 4 months ago.
Which of the following treatment options would be LEAST likely to correct
the bleeding?
Select one:
a. Addition of combination oral contraceptives for 1 month
b. Use of a nonsteroidal anti-inflammatory medication for 1 week
c. Use of a daily estrogen-only supplement such as ethinyl estradiol or
conjugated equine estrogen for 1 month
d. Single dose of depot medroxyprogesterone acetate (DMPA)
Question 4
Correct
Marked out of 1.00
Flag question
Question text
All of the following diagnostic tests are typically obtained during the initial
evaluation of abnormal uterine bleeding EXCEPT:
Select one:
a. Complete blood count with platelets
b. Pap test
c. Liver function tests
d. Pregnancy test
Question 5
Incorrect
Marked out of 1.00
Flag question
Question text
Which cause of abnormal uterine bleeding is NOT represented in the
International Federation of Gynecology and Obstetrics (FIGO) classification
acronym PALM-COEIN?
Select one:
a. Coagulopathy
b. Pregnancy
c. Leiomyoma
d. Iatrogenic
Question 6
Correct
Marked out of 1.00
Flag question
Question text
Control of blood loss during menses involves which of the following
mechanisms?
Select one:
a. Platelet aggregation
b. All of the mentioned
c. Vasoconstriction of endometrial arteries
d. Thrombus formation
Question 7
Correct
Marked out of 1.00
Flag question
Question text
Which is an effective first-line treatment or women with heavy menstrual
bleeding and von Willebrand disease?
Select one:
a. Endometrial ablation
b. Nonsteroidal anti-inflammatory drugs
c. Combination oral contraceptive pills
d. Dilatation and curettage
Question 8
Correct
Marked out of 1.00
Flag question
Question text
A patient with heavy menstrual bleeding reveals a personal history of
requent gingival bleeding and excessive bleeding during a recent tooth
extraction. You screen her for a coagulation disorder. Which of the following
laboratory tests is LEAST likely to be informative?
Select one:
a. Bleeding time
b. Partial thromboplastin time (PTT)
c. Prothrombin time (PT)
d. Complete blood count (CBC) with platelets
Question 9
Correct
Marked out of 1.00
Flag question
Question text
According to the American College of Obstetricians and Gynecologists
(2013), endometrial sampling to assess abnormal uterine bleeding is NOT
recommended for a woman with which of the following characteristics?
Select one:
a. Has persistent abnormal uterine bleeding
b. Is 35 years old
c. Has ailed medical management
d. Has history o unopposed estrogen exposure
Question 10
Correct
Marked out of 1.00
Flag question
Question text
Which layer of the endometrium sloughs and therefore is responsible or
observed menstrual discharge?
Select one:
a. Functionalis
b. Radial
c. Spiral
d. Basalis
Вопрос 1
Верно
Балл: 1,00
Отметить вопрос
Текст вопроса
Which of the following defnitions of abnormal uterine bleeding is incorrect?
Выберите один ответ:
a. Hypomenorrhea refers to menses with diminished flow or shortened
interval.
b. Heavy menstrual bleeding is defined as prolonged or heavy cyclic
menstruation, with menses lasting longer than 8 days or exceeding 80 mL
of blood loss.
c. Oligomenorrhea refers to cycles with intervals shorter than 35
days.
d. Intermenstrual bleeding is bleeding that occurs between cycles.
Вопрос 3
Верно
Балл: 1,00
Отметить вопрос
Текст вопроса
Which of the following statements regarding tranexamic acid is true?
Выберите один ответ:
a. It is an antifibrinolytic drug that permanently blocks lysine binding sites
on plasminogen, thereby increasing plasmin levels and fibrinolytic activity.
b. Contraindications to its use include a history or intrinsic risk of
thromboembolic disease.
c. To be most effective, it requires administration for 2 weeks before and
during menses.
d. It affects blood coagulation parameters, such as platelet count,
prothrombin time (PT), and partial thromboplastin time (PTT).
Вопрос 4
Верно
Балл: 1,00
Отметить вопрос
Текст вопроса
For what age group anovulation and coagulation defects as a cause of AUB
are at disproportionately higher rates compared with other?
Выберите один ответ:
a. Adolescence
b. Perimenopause
c. Menopause
d. Childhood
Вопрос 5
Верно
Балл: 1,00
Отметить вопрос
Текст вопроса
Suggested etiologies or abnormal uterine bleeding associated with use of
the device shown below in this three-dimensional (3-D) sonogram include
which of the following?

Выберите один ответ:


a. All of the mentioned
b. Malpositioned device
c. Increased endometrial vascularity, congestion, and degeneration
d. Unbalanced ratio of prostaglandin to thromboxane levels
Вопрос 6
Неверно
Балл: 1,00
Отметить вопрос
Текст вопроса
A 60-year-old postmenopausal woman presents for evaluation of genital
tract bleeding, which is confirmed as uterine in origin by physical
examination. Which diagnostic procedure is a logical first step in her
evaluation?
Выберите один ответ:
a. Diagnostic hysteroscopy
b. Colposcopy
c. Transvaginal sonography
d. Saline infusion sonography
Вопрос 7
Верно
Балл: 1,00
Отметить вопрос
Текст вопроса
An advantage of transvaginal sonography for the evaluation of abnormal
uterine bleeding includes which of the following?
Выберите один ответ:
a. Greater patient com ort compared with endometrial biopsy or
hysteroscopy
b. Simultaneous assessment of myometrium and endometrium
c. Reduced use of endometrial biopsy
d. All of the mentioned
Вопрос 8
Неверно
Балл: 1,00
Отметить вопрос
Текст вопроса
A 27-year-old woman who previously underwent dilatation and curettage
for an incomplete abortion presents with new-onset heavy menstrual
bleeding. Transvaginal sonography first reveals a hypoechoic tubular
structure within the myometrium. With application of color Doppler, large-
caliber vessels are seen in the second image. What is the most appropriate
next step in her evaluation?

Выберите один ответ:


a. Pelvic computed tomography (CT) with contrast
b. Saline infusion sonography
c. Hysteroscopy
d. Angiography
Вопрос 9
Неверно
Балл: 1,00
Отметить вопрос
Текст вопроса
Pelvic sonography with applied color Doppler reveals an intrauterine growth
with a single feeder vessel. Which statement is true regarding this
abnormality?

Выберите один ответ:


a. The main diagnostic tool is the Pap test.
b. Infertility has been linked directly to this condition.
c. It is an uncommon cause of abnormal uterine bleeding.
d. Use of oral contraceptive pills appears to be protective.
A 18-year-old girl has been bleeding heavily for the past 2 weeks. She experienced menarche about 3
years ago, and since that time her periods have been extremely irregular and heavy. She appears very
pale and fatigued. Her blood pressure is 110/60mmHg and the pulse is 70/min. All of the following are
appropriate tests to be done,except:Select one:a. Estradiol levelb. CBCc. Beta-HCG Xd. Bleeding time

In the evaluation of abnormal uterine bleeding, what is the primary advantage of hysteroscopy
compared with saline infusion sonography?Select one:a. Hysteroscopy permits simultaneous
identification and removal of focal endometrial lesions.b. Hysteroscopy is more accurate in
identifying global endometrial pathology.c. Hysteroscopy is less expensive.d. Hysteroscopy is less
painful.

A 60-year-old patient with breast cancer taking oral anticoagulants for recent deep-vein thrombosis is
admitted for acute heavy menstrual bleeding. Which of the following management options is LEAST
suitable for this patient?Select one:a. High-dose estrogen (intravenous Premarin)b. Intrauterine cavity
insertion of a Foley catheter Xc. Reversal of anticoagulationd. All are suitable for this woman.

What is the primary advantage of saline in fusion sonography compared with


transvaginal sonography?
Select one:
a. Superior detection of intracavitary masses
b. Less patient discomfort
c. Simultaneous assessment of myometrium and endometrium
d. Ability to perform at any time of the cycle

CASE

Question 1
Incorrect
Marked out of 1.00
Flag question
Question text
A 28-year-old G1 P1 woman complains of painful menses and pain with
intercourse. She has menses every month and denies a history of sexually
transmitted diseases. Which of the following tests would most likely
identify the etiology of the patient's abdominal pain?
Select one:
a. pelvic and abdominal MRI
b. hysteroscopy
c. transvaginal ultrasound
d. endometrial biopsy
e. laparoscopy
Question 2
Incorrect
Marked out of 1.00
Flag question
Question text
A 30-year-old nulliparous hypertensive woman has menorrhagia. The best
treatment for her is:
Select one:
a. LNG-IUS Mirena
b. Hysterectomy
c. OCP
d. Transcervical resection of endometrium
Question 3
Correct
Marked out of 1.00
Flag question
Question text
All of the following assist in predicting ovulation except:
Select one:
a. endometrial biopsy
b. cervical mucus test
c. basal body temperature
d. serum progesterone in follicular phase
Question 4
Correct
Marked out of 1.00
Flag question
Question text
A 32-year-old patient complains of bleeding between her periods and
increasingly heavy menses. Over the past 9 months, she has had two
dilation and curettages (D&Cs), which have failed to resolve her symptoms,
and contraceptives and antiprostaglandins have not decreased the
abnormal bleeding. Of the following options, which is the most appropriate
at this time?
Select one:
a. Perform hysteroscopy
b. Treat with a GnRH agonist
c. Perform hysterectomy
d. Perform endometrial ablation
Question 5
Correct
Marked out of 1.00
Flag question
Question text
All are evidence-based treatments for menorrhagia, except:
Select one:
a. Progesterone 5–25 days cyclically
b. Tranexamic acid
c. OC pills
d. Ethamsylate
Вопрос 1
Верно
Балл: 1,00
Отметить вопрос
Текст вопроса
The continuous regimen of the combined oral contraceptives use means:
Выберите один ответ:
a. weeks of daily use, and non-hormonal break after
b. 12-16 weeks of daily use, and non-hormonal break after
c. 6-8 weeks of daily use, and non-hormonal break after
d. 365 days of daily use, and non-hormonal break after
Вопрос 3
Верно
Балл: 1,00
Отметить вопрос
Текст вопроса
Ritu, 15 years old, complains of heavy periods since 2 months. On
examination: weight 40 kg and BP 120/80 mmHg. All of the following
investigations are indicated, except:
Выберите один ответ:
a. TSH
b. Bleeding and clotting time
c. Platelet count
d. None of the above
Вопрос 4
Верно
Балл: 1,00
Отметить вопрос
Текст вопроса
A 46-year-old G3P3 complains of menorrhagia since 3 months. Next line of
management is:
Выберите один ответ:
a. OC pills × 6 months
b. Progesterone × 6 months
c. Hysterectomy
d. D&amp;amp;C
Вопрос 5
Верно
Балл: 1,00
Отметить вопрос
Текст вопроса
What definitive treatment is the most preferable in a 48-year-old parous,
significantly obese, diabetic and hypertonic patient with recurrent DUB?
Выберите один ответ:
a. combined oral contraceptives
b. hysterectomy
c. endometrium ablation
d. medroxiprogesterone acetate

INFERTILITY

Question 1
Incorrect
Mark 0.00 out of 1.00
Flag question
Question text
An infertile woman has bilateral tubal block at cornua diagnosed on
hysterosalpingography. Next treatment of choice is:
Select one:
a. Laparoscopy and hysteroscopy
b. Hydrotubation
c. Tuboplasty
d. IVF
Question 2
Correct
Mark 1.00 out of 1.00
Flag question
Question text
A 34-year-old infertile woman is noted to have evidence of blocked
fallopian tubes by hysterosalpingogram. Which of the following is the best
next step for this patient?
Select one:
a. Laparoscopy
b. Clomiphene citrate therapy
c. Intrauterine insemination
d. FSH therapy
Question 3
Correct
Mark 1.00 out of 1.00
Flag question
Question text
Artificial inseminationwithhusband’ssemenisindicatedinall thefollowing
situations, except:
Select one:
a. Oligospermia
b. Antisperm antibodies in the cervical mucous
c. Impotency
d. Azoospermia
Question 4
Correct
Mark 1.00 out of 1.00
Flag question
Question text
A 27-year-old married nulligravid resident physician has been married or 3
years. She is worried about delaying childbearing or another few years
while she pursues fellowship training. She can be reassured that a
significant decline in the fecundability rate does not begin until what age
(years)?
Select one:
a. 26
b. 32
c. 44
d. 38
Question 5
Correct
Mark 1.00 out of 1.00
Flag question
Question text
Which of the following Y-chromosome deletions carries the best prognosis
or recovering sperm from the testes in an azoospermic patient?
Select one:
a. AZFd
b. AZFa
c. AZFc
d. AZFb
Question 6
Correct
Mark 1.00 out of 1.00
Flag question
Question text
Of infertility causes, which of the following is most likely encountered?
Select one:
a. Tubal disease
b. Unexplained
c. Ovulatory dysfunction
d. Male actors
Question 7
Incorrect
Mark 0.00 out of 1.00
Flag question
Question text
Regarding commercially available urinary luteinizing hormone kits, when
does ovulation take place in relation to a positive result?
Select one:
a. Day before
b. 48 hours after
c. Same day
d. Day after
Question 8
Incorrect
Mark 0.00 out of 1.00
Flag question
Question text
Which of the following is true regarding letrozole, an aromatase inhibitor, in
the context of ovulation induction?
Select one:
a. It is typically dosed daily or 4 to 6 weeks.
b. Data regarding its teratogenicity are contradictory.
c. It has become an accepted alternative to gonadotropins and clomiphene
citrate.
d. It is Food and Drug Administration (FDA) approved for this indication.
Question 9
Correct
Mark 1.00 out of 1.00
Flag question
Question text
Which of these statements regarding infertility is FALSE?
Select one:
a. Infertility generally is defined as the inability to achieve successful
pregnancy after at least 12 months of regular, unprotected intercourse.
b. Women are affected more than twice as often as men.
c. Around 10 to 15 percent of the reproductive-aged population is infertile.
d. Early evaluation after only 6 months may be justied for certain medical
conditions or for women aged 35 years or older.
Question 10
Incorrect
Mark 0.00 out of 1.00
Flag question
Question text
Compared with in vitro fertilization, advantages of bilateral tubal
reanastomosis to reverse tubal occlusion from a prior sterilization
procedure do NOT include which of the following?
Select one:
a. Decreased risk of multifetal gestation X
b. Avoidance of ovarian stimulation with exogenous hormonal therapy
c. Ability to conceive naturally
d. Negligible risk of ectopic pregnancy
Вопрос 2
Неверно
Баллов: 0,00 из 1,00
Отметить вопрос
Текст вопроса
Gold standard treatment for female infertility is:
Выберите один ответ:
a. Hysteroscopy
b. Laparoscopy and hysteroscopy
c. Laparoscopy X
d. Transvaginal USG X
Вопрос 3
Верно
Баллов: 1,00 из 1,00
Отметить вопрос
Текст вопроса
A 26-year-old G0P0 woman has regular menses every 28 days. The semen
analysis is normal. The patient had a postcoital test revealing motile sperm
and stretchy watery cervical mucus. She has been treated for chlamydial
infection in the past. Which of the following is the most likely etiology of her
infertility?
Выберите один ответ:
a. Cervical factor
b. Uterine factor
c. Male factor
d. Tubal factor
e. Peritoneal factor
f. Ovulatory factor
Вопрос 4
Неверно
Баллов: 0,00 из 1,00
Отметить вопрос
Текст вопроса
When treating hypertension with β-blocking agents, which of the following
semen abnormalities may be seen due to retrograde ejaculation?
Выберите один ответ:
a. Low semen volume
b. Oligospermia
c. Azoospermia X
d. Teratospermia
Вопрос 5
Верно
Баллов: 1,00 из 1,00
Отметить вопрос
Текст вопроса
As shown here, sonography is clinically most useful for the diagnosis of
which of the following conditions that contributes to a significant
percentage of infertility cases?
Выберите один ответ:
a. Anovulation
b. Periovarian adhesive disease
c. Polycystic ovarian syndrome
d. Sex cord-stromal tumors
Вопрос 6
Верно
Баллов: 1,00 из 1,00
Отметить вопрос
Текст вопроса
A 30-year-old nulligravida has been trying to conceive for the last 2 years.
She has no medical problems. She consumes five alcoholic drinks weekly,
smokes half a pack of cigarettes daily. She describes herself as a heavy
coffee drinker. She works in a dry-cleaning facility. Which of the following
exposures is LEAST likely affecting her fertility?
Выберите один ответ:
a. Dry cleaning fluid exposure
b. Cigarettes
c. Caffeine
d. Alcohol
Вопрос 7
Неверно
Баллов: 0,00 из 1,00
Отметить вопрос
Текст вопроса
A woman complaining of infertility reports that her husband has already
undergone evaluation by a urologist to exclude male actors. This leads to
the surgical repair of a unilateral varicocele. With regard to varicoceles and
infertility, which of the following is true?
Выберите один ответ:
a. Repair o subclinical varicoceles leads to correction of semen
abnormalities.
b. All should be repaired.
c. A varicocele is present in nearly 20 percent of the adult male general
population.
d. They are an established cause of male infertility.
Вопрос 8
Неверно
Баллов: 0,00 из 1,00
Отметить вопрос
Текст вопроса
Which of the following assisted reproductive technologies, typically is used
to treat male actor infertility, is shown here?
Выберите один ответ:
a. Assisted hatching
b. Embryo biopsy X
c. Oocyte in vitro maturation
d. Intracytoplasmic sperm injection (ICSI)
Вопрос 9
Неверно
Баллов: 0,00 из 1,00
Отметить вопрос
Текст вопроса
Which of the following is the best treatment or a woman with a significant
decline in ovarian reserve?
Выберите один ответ:
a. Clomiphene citrate ovulation induction X
b. Use of donor eggs
c. In vitro fertilization with intracytoplasmic sperm injection
d. Exogenous gonadotropin ovulation induction
Вопрос 10
Неверно
Баллов: 0,00 из 1,00
Отметить вопрос
Текст вопроса
Which of the following treatment options offers the best chance or
intrauterine pregnancy in a patient with the following problem?

Выберите один ответ:


a. In vitro fertilization after bilateral salpingectomy
b. Fimbriectomy X
c. Neosalpingostomy
d. Tubal reanastomosis
Вопрос 1
Верно
Баллов: 1,00 из 1,00
Отметить вопрос
Текст вопроса
A 22-year-old nulliparous woman with irregular menses of 7 years’ duration
complains of primary infertility. She has a family history of diabetes. She
has mild hirsutism on examination. Which of the following is the most likely
therapy?
Выберите один ответ:
a. Surgical excision of an ovarian tumor
b. Oral clomiphene citrate or letrozole
c. Excision of an adrenal tumor
d. Cortisol and mineralocorticoid replacement
e. Intrauterine insemination
Вопрос 3
Верно
Баллов: 1,00 из 1,00
Отметить вопрос
Текст вопроса
Postcoital test is used to assess:
Выберите один ответ:
a. Tubal factor
b. Uterine factor
c. Any of the above
d. Cervical factor
Вопрос 4
Верно
Баллов: 1,00 из 1,00
Отметить вопрос
Текст вопроса
Which of the following tests are currently recommended components of a
basic male factor infertility evaluation?
Выберите один ответ:
a. Antisperm antibody assay
b. DNA integrity analysis
c. None of the mentioned
d. Sperm penetration assay
Вопрос 5
Верно
Баллов: 1,00 из 1,00
Отметить вопрос
Текст вопроса
Hysterosalpingography is LEAST helpful or detecting which of the
following?
Выберите один ответ:
a. Pelvic adhesions
b. Asherman syndrome
c. Tubal patency
d. Congenital uterine anomalies
Вопрос 6
Верно
Баллов: 1,00 из 1,00
Отметить вопрос
Текст вопроса
Congenital bilateral absence of the vas deferens is genetically related to
which of the following?
Выберите один ответ:
a. Noonan syndrome
b. Cystic fibrosis
c. Red–green color blindness
d. Klinefelter syndrome
Вопрос 7
Верно
Баллов: 1,00 из 1,00
Отметить вопрос
Текст вопроса
A 32-year-old woman undergoes infertility evaluation after trying to
conceive for 5 years. She has cyclic but heavy menses. Prior to this, she
experienced one pregnancy that ended in a first-trimester spontaneous
abortion. What uterine abnormality is suggested by her
hysterosalpingogram?

Выберите один ответ:


a. Normal
b. Submucous leiomyoma
c. Bilateral hydrosalpinges
d. Asherman syndrome
Вопрос 9
Верно
Баллов: 1,00 из 1,00
Отметить вопрос
Текст вопроса
A 28-year-old woman with polycystic ovarian syndrome (PCOS) and
infertility asks about the potential use of metformin to help achieve
pregnancy. She can be counseled that current evidence LEAST supports
which of the following benefits of metformin therapy in women with PCOS?
Выберите один ответ:
a. Higher frequency of spontaneous ovulation
b. Improved menstrual cyclicity
c. Improved live-birth rate
d. Increased ovulatory response to clomiphene citrate therapy
Вопрос 10
Верно
Баллов: 1,00 из 1,00
Отметить вопрос
Текст вопроса
Which of the following tubal obstruction locations is LEAST amenable to
surgical repair?
Выберите один ответ:
a. Isthmic
b. Ampullary
c. Fimbrial
d. Interstitial

Which of the following statements regarding antimullerian hormone level (AMH) is true?
a AMH expression is strong in preantral follicles and weak in larger follicles
b AMH levels are increased in women with polycystic ovanan syndrome
c All of the mentioned statements are true
d Compared with estradiol or follicle-stimulating hormone (FSH) levels, AMH levels correlate better
with the number of ovarian primordial follicles (seen In photomicrograph above)

Reduced secretion of which of the following hormones is most likely responsible or the rising serum
follicle-stimulating hormone (FSH) level seen as a woman ages?
a Inhibin
b Follistatm X
c Actwin
d Estradiol

Gynecomastia in a male patient may suggest the presence of which of the following conditions?
a Klmefelter syndrome
b Pituitary prolactmoma X
c 17 beta-hydroxysteroid dehydrogenase defficiency
d Noonan syndrome

Which of the following is present in some gonadotropin preparations and provides the needed
luteinizing hormone (LH) activity by virtue of its shared receptor with LH?
a Human chononic gonadotropin
b Follistatm
c Activin
d inhibin X

As shown here, sonography is clinically most useful for the diagnosis of which of the following
conditions that contributes to a significant percentage of infertility cases?
a. Anovulation
b. Periovarian adhesive disease
c. Polycystic ovarian syndrome
d. Sex cord-stromal tumors

A 30-year-old nulligravida has been trying to conceive for the last 2 years. She has no medical
problems. She consumes five alcoholic drinks weekly, smokes half a pack of cigarettes daily. She
describes herself as a heavy coffee drinker. She works in a dry-cleaning facility. Which of the
following exposures is LEAST likely affecting her fertility?
a. Dry cleaning fluid exposure
b. Cigarettes
c. Caffeine
d. Alcohol

A woman complaining of infertility reports that her husband has already undergone evaluation by a
urologist to exclude male actors. This leads to the surgical repair of a unilateral varicocele. With regard
to varicoceles and infertility, which of the following is true?
a. RePair o subclinical varicoceles leads to correction of semen abnormalities.
b. All should be repaired. X
c. A varicocele is present in nearly 20 percent of the adult male general population.
d. They are an established cause of male infertility.

Artificial insemination with husband’s semen is indicated in all of the following situations, except:
a. Azoospermia
b. Antisperm antibodies in the cervical mucous
c. Impotency
d. Oligospemia

When treating hypertension with beta-blocking agents, which of the following semen abnormalities
may be seen due to retrograde ejaculation?
a. Low semen volume
b. Oligospermia
c. Azoospermia X
d. Teratospermia

Oligospermiais defined as less than which of the following sperm count thresholds per milliliter of
semen?
a. 15 million/mL
b. 50 million/mL
c. 150 million/mL
d. 100 million/mL

What is expected per cycle fecundability rate?


Select one:
a. 0.5 percent
b. 5 percent X
c. 85 percent
d. 25 percent

When providing treatment to increase sperm counts, it should be kept in mind that the process of
spermatogenesis takes approximately how long?
Select one:
a. 30 days
b. 100 days
c. 6 months
d. 9 months

Which of the following is NOT true of basal body temperature testing to identify ovulation in adult
women?
Select one:
a. Once the temperature rises, a patient should expect ovulation during the next 48 hours.
b. With ovulation, the temperature rises from 0.4°F to 0.8°F.
c. It is an inexpensive and easy test or ovulation monitoring.
d. It can be an insensitive test in many women.

Use of which of these is most likely to damage testicular function permanently,


Select one:
a. Anabolic steroids
b. Gentamycin
c. Alcohol consumption
d. Spironolactone

Which of the following is true of the process of sperm capitation?


Select one:
a. Involves sperm hyperactivation.
b. Aids sperm penetration of the ovum's zona pellucida.
c. Results in the sperm’s ability to release across all enzymes important to fertilization of the ovum.
d. All of the mentioned

Most adverse maternal or perinatal outcomes of higher-order muttifetal pregnancies are the result of
which of the following?
a. Advanced maternal age
b. Venous thromboembolic events
c. Fetal growth restriction and discordance X
d. Premature delivery

Which of the following gonadotropins is a recombinant product?


Select one:
a. Repronex X
b. Bravelle
c. FollIstim
d. Menopur

In cases of male hypogonadotropic hypogonadusm, which of the following semen analysis


abnormalities is most typically seen?
Select one:
a. Oligospermia
b. Asthenospermia
c. Teratozoospermia
d. Aspermia

Preconception also carrier screening or cystic fibrosis is currently recommended or which group?
Select one:
a. All individuals considering pregnancy
b. Non-Hispanic white race
c. Family or personal history of cystic fibrosis
d. Ashkenazi Jewish descent

An in infertile couple presents or evaluation. Neither partner has ever conceived a pregnancy. Both
believe that the female partner alone is the source of their failure to achieve a pregnancy. It should be
explained that infertility can be attributed to the male partner in approximately one third of cases and to
both partners in approximately what percentage of cases,
Select one:
a. 1 percent
b. 66 percent
c. 33 percent
d. 10 percent X

A 28-year-old nullipara who has been seeking pregnancy is diagnosed with polycystic ovarian
syndrome (PCOS). She is generally healthy, but her body mass index (BMI) is in the obese range.
Which of the following should be recommended as first-line management of her anovulation? Select
one:
a. Gonadotropins
b. Weight loss and exercise
c. Insulin-sensitizing agents
d. Clomiphene citrate

Which of the following complication is more common in pregnancies resulting from assisted
reproductive technologies compared with spontaneously conceived pregnancies?
Select one: -
a. All of the mentioned
b. Gestational diabetes
c. Cesarean delivery
d. Placental abruption

Infertility thought secondary to which of the following scenarios warrants intracytoplasmic sperm
injection (ICSI)?
Select one:
a. Bilateral distal fallopian tube occlusion with hydrosalpinxes X
b. Chronic anovulation
c. Diminished ovarian reserve in an older woman
d. Severe oligospermia

Which of the following midluteal progesterone level values signifies ovulation?


Select one:
a. 0.05 ng/mL
b. 0.1 ng/mL
c. 1.0 ng/mL X
d. 5.0 ng/mL

A couple seeking pregnancy has undergone a thorough infertility evaluation. Both partners are
generally healthy. The Correct female partner is without apparent problems that would contribute to
infertility. Semen analysis is performed. The sperm count is 4 million/mL, and semen volume is less
than 1 mL. In addition to a serum testosterone level, which of of 1.00 the following should be
rneasured during the hormonal evaluation of the male partner if an endocrinopathy is suspected?
Select one:
a. Follicle-stimulating hormone (FSH)
b. Prolactin
c. All of the mentioned
d. Thyroid-stimulating hormone

Which of the following is NOT a typical clinical manifestation of ovarian hyperstimulation syndrome,
Select one:
a. Seizures
b. Hypercoagulability
c. Hemoconcentration
d. Ascites

In women with inadequate cervical mucus quantity or quality as a suspected contributor to infertility,
what is the most reasonable approach to achieving pregnancy?
Select one:
a. Cryotherapy
b. Intrauterine insemination
c. Ovulation Induction
d. Flax seed oll supplementation

LIVER PATHOLOGY

Question 1
Correct
Mark 1.00 out of 1.00
Flag question
Question text
After delivery, the liver function deterioration of acute fatty liver of
pregnancy halted. Nonetheless, you recommend continued intensive
maternal surveillance due to the approximately 20% risk of developing
which of the following in the days after delivery?
Select one:
a. Hemorrhage
b. Sepsis
c. Acute pancreatitis
d. Diabetes mellitus
Question 2
Incorrect
Mark 0.00 out of 1.00
Flag question
Question text
Which of the following combinations represents appropriate screening for
hepatitis C in pregnancy?
Select one:
a. 37-year-old woman with a history of blood transfusion after her last
pregnancy in 2014 screened via hepatitis C RNA
b. 15-year-old woman whose mother was a chronic intravenous drug user
and died of liver failure when the patient was 4 years old screened via
hepatitis C RNA
c. 27-year-old woman with human immunodeficiency virus screened via
anti-hepatitis C antibody
d. 26-year-old woman with a history of skin abscesses from heroin use
screened via anti-hepatitis C antibody
Question 3
Correct
Mark 1.00 out of 1.00
Flag question
Question text
The cytochrome P450 system is altered by which of the following during
pregnancy?
Select one:
a. Placental expression
b. Progesterone levels
c. All of the above
d. Estrogen levels
Question 4
Incorrect
Mark 0.00 out of 1.00
Flag question
Question text
Which of the following viral infections has been associated with a marked
increase in the risk for intrahepatic cholestasis of pregnancy?
Select one:
a. Hepatitis B
b. Human immunodeficiency virus
c. Cytomegalovirus
d. Hepatitis C
Question 5
Incorrect
Mark 0.00 out of 1.00
Flag question
Question text
What is the incidence and recurrence risk of acute fatty liver of pregnancy?
Select one:
a. 1 in 100,000 pregnancies, and recurrence is rare
b. 1 in 1,000,000 pregnancies, and recurrence is rare
c. 1 in 10,000 pregnancies, and recurrence is rare
d. 1 in 100,000 pregnancies, and recurrence is common
Question 6
Correct
Mark 1.00 out of 1.00
Flag question
Question text
Chronic hepatitis B infection is most likely to develop after acquisition by
which of the following patients?
Select one:
a. 28-year-old healthy woman
b. Newborn infant
c. 9-year-old child
d. Risk is equivalent in all of the above
Question 7
Correct
Mark 1.00 out of 1.00
Flag question
Question text
Immunization is not available for which of the following?
Select one:
a. It is available for all three
b. Hepatitis A
c. Hepatitis C
d. Hepatitis B
Question 8
Not answered
Marked out of 1.00
Flag question
Question text
A 27-year-old at 32 weeks’ gestation is brought to the emergency room by
ambulance after consuming 6 grams of acetaminophen. Which of the
following is true?
Select one:
a. All of the above
b. N-Acetyl-p-benzoquinoneimine should be administered promptly.
c. The fetus is not at risk, so no monitoring is necessary.
d. This is the most common cause of acute liver failure in the United States.
Question 9
Not answered
Marked out of 1.00
Flag question
Question text
Which of the following have the best data for treatment of intrahepatic
cholestasis of pregnancy?
Select one or more:
a. Dexamethasone
b. Ursodeoxycholic acid
c. Low-cholesterol diet
d. Cholestyramine
Question 10
Not answered
Marked out of 1.00
Flag question
Question text
Risk of maternal death with acute fatty liver is related to which of the
following?
Select one:
a. All of the above
b. Renal failure
c. Hemorrhage
d. Sepsis
Question 1
Incorrect
Mark 0.00 out of 1.00
Flag question
Question text
Concerning the gastrointestinal tract during pregnancy, which of the
following is correct?
Select one:
a. None of the above
b. Intraesophageal pressures are lower during pregnancy.
c. Gastric emptying time increases during pregnancy.
d. Lower esophageal sphincter tone is increased during pregnancy.
Question 2
Incorrect
Mark 0.00 out of 1.00
Flag question
Question text
During the hospital care of a patient with acute viral hepatitis, which of the
following personal protective equipment should be used at all times?
Select one:
a. All of the above
b. Gloves
c. Negative-pressure ventilation hospital room
d. N95 respiratory mask
Question 3
Correct
Mark 1.00 out of 1.00
Flag question
Question text
Which of the following findings make in case of elevated liver enzymes
exclude intrahepatic cholestasis of pregnancy?
Select one:
a. Jaundice
b. All of the above
c. Liver biopsy showing bile plugs in the hepatocytes
d. AST 800 U/L
Question 4
Incorrect
Mark 0.00 out of 1.00
Flag question
Question text
Maternal acute fatty liver of pregnancy is associated with all except with of
the following recessively inherited abnormalities of mitochondrial fatty acid
oxidation?
Select one:
a. Long-chain-3-hydroxyacyl-CoA-dehydrogenase
b. Medium-chain acyl-CoA dehydrogenase
c. Carnitine palmitoyltransferase 1
d. Dihydrolipoamide dehydrogenase
Question 5
Correct
Mark 1.00 out of 1.00
Flag question
Question text
Which of the following pregnancy-related complications has the capacity to
demonstrate the most prominent alterations of normal hepatic, renal,
hematological, and coagulation laboratory studies?
Select one:
a. Intrahepatic cholestasis of pregnancy
b. Hyperemesis gravidarum
c. Preeclampsia
d. Acute fatty liver of pregnancy
Question 6
Correct
Mark 1.00 out of 1.00
Flag question
Question text
Lactation is contraindicated for women infected with which of the
following?
Select one:
a. HIV
b. Hepatitis C
c. Hepatitis B
d. Hepatitis A
Question 7
Correct
Mark 1.00 out of 1.00
Flag question
Question text
All EXCEPT which of the following liver-related changes are physiological in
pregnancy?
Select one:
a. Elevated serum alkaline phosphatase levels
b. Hypolipidemia
c. Palmar erythema
d. Spider angiomas
Question 8
Incorrect
Mark 0.00 out of 1.00
Flag question
Question text
This image shows the typical nodular, fibrotic appearance of a cirrhotic
liver. What is the most common cause of this condition in the general
population?

Select one:
a. Nonalcoholic fatty liver disease
b. Autoimmune hepatitis
c. Viral hepatitis
d. Alcohol exposure
Question 9
Correct
Mark 1.00 out of 1.00
Flag question
Question text
Which of the following statements regarding acute viral hepatitis is correct?
Select one:
a. Bilirubin levels typically fall as transaminase levels rise.
b. Low-grade fever is more common with hepatitis A.
c. Serum transaminase levels correspond with disease severity.
d. Jaundice is usually the presenting symptom.
Question 10
Incorrect
Mark 0.00 out of 1.00
Flag question
Question text
Which imaging modality is recommended in discriminating acute fatty liver
of pregnancy from other etiologies of elevated liver enzymes?
Select one:
a. Ultrasound with color Doppler
b. Magnetic resonance imaging
c. Computed tomography
d. None of the above
From an etiopathogenesis perspective, the pregnancy fatty liver pictured
here is analogous to which childhood illness?
Выберите один ответ:
a. Epstein-Barr viral hepatitis
b. Biliary atresia
c. Reye-like syndromes
d. Autoimmune hepatitis

CASE

Вопрос 1
Неверно
Баллов: 0,00 из 1,00
Отметить вопрос
Текст вопроса
A 32-year-old G1 at 32 weeks’ gestation is diagnosed with intrahepatic
cholestasis of pregnancy. During the visit she inquires about whether this
diagnosis will alter her prenatal care or delivery plans. What is the best
evidence-based advice you can provide?
Выберите один ответ:
a. Since your bile acid levels are less 10 µmol/L, delivery will be
recommended before 39 weeks’ gestation.
b. None of the above
c. We can avoid risk for stillbirth with weekly fetal nonstress testing.
d. If we pursue delivery at 38–39 weeks’ gestation, we will avoid neonatal
sequelae of intrahepatic cholestasis of pregnancy.
Вопрос 2
Верно
Баллов: 1,00 из 1,00
Отметить вопрос
Текст вопроса
A 28-year-old at 19 weeks’ gestation presents to the emergency room with
complaints of right upper quadrant pain, fever, and vomiting. Exam reveals
right upper quadrant tenderness, and she has a temperature of 38.7°C. One
diagnostic image is shown here. Common bile duct obstruction is not
suspected. What is the next most appropriate step in her management?
Выберите один ответ:
a. Laparoscopic cholecystectomy
b. Admit for serial abdominal examinations and intravenous antibiotics
c. Discharge home with oral antibiotics, pain medication, and antiemetics
d. Endoscopic retrograde cholangiopancreatography
Вопрос 3
Неверно
Баллов: 0,00 из 1,00
Отметить вопрос
Текст вопроса
A 36-year-old multigravida presents for preconception counseling. Her
obstetric history includes two full-term vaginal deliveries after pregnancies
complicated by gestational diabetes and a first-trimester pregnancy loss.
She comes to you to discuss her recent diagnosis of nonalcoholic fatty liver
disease (NAFLD) by her primary care physician. Diagnosis was based on
imaging performed for an episode of abdominal pain. Her liver function
tests are normal. Her medical history is otherwise notable for longstanding
obesity (body mass index 34 kg/m2) and diabetes (hemoglobin A1c 8.2%)
diagnosed 2 years ago. Which of the following should be included in your
counseling?
Выберите один ответ:
a. Compared to women of similar weight, because of the NAFLD, she has a
much higher risk for liver-related adverse pregnancy outcome.
b. With normal liver enzymes, you question the diagnosis of NAFLD and
recommend that she seek a second opinion from a gastroenterologist
c. The best interventions to reduce the risk to a future pregnancy are
weight loss and optimizing her glucose control.
d. All of the above
Вопрос 4
Неверно
Баллов: 0,00 из 1,00
Отметить вопрос
Текст вопроса
A 29-year-old primigravida presents to you for prenatal care. She is known
to have hepatitis C with no known risk factors, and your prenatal labs
confirm presence of hepatitis C RNA and anti-hepatitis C antibody.
Compared to a woman with anti-hepatitis C antibody who is RNA-negative,
which of the following is more likely to complicate her pregnancy?
Выберите один ответ:
a. Fetal growth restriction
b. None of the above
c. Preterm delivery
d. Vertical transmission of hepatitis C
Вопрос 5

A 27-year-old nulligravida and her husband present for preconception counseling. She has Wilson
disease that is being treated with zinc sulfate. Which of the following should be included in your
counseling?
Select one:
a. All of the above
b. Upon confirmation of pregnancy, she should discontinue her chelation therapy.
c. Her husband should consider carrier testing.
d. Available data suggests she should convert to penicillamine prior to conception, as it is a better
chelating agent

A 27-year-old multigravida comes to you for her postpartum visit. Both of her pregnancies were
complicated by intrahepatic cholestasis of pregnancy. She desires another child in about 2 years. Which
of the following methods of contraception do you recommend?
Select one:
O a. Combination oral contraceptive pills
O b. None of the above
O c. Copper intrauterine device
d. Vaginal ring

Which combination of laboratory findings below would be most concerning for acute fatty liver of
pregnancy
a. Hematocrit 30%. creatinine 1.0 mg/dL platelets 200/L AST 600 U/L fibrinogen 420 mg/dL and
glucose 120- mg/d1_
b. Hematocrit 39%. creatinine 1.2 mg/dL platelets 90/L. AST 80 U/L fibrinogen 450 mg/dL and
glucose 105 mg/dL
Hematocrit 32%. creatinine 2.0 mg/dL platelets 90/L AST 400 U/L fibrinogen 130 mg/dL and
glucose 65 mg/dL.
d. Hematocrit 34%. creatinine 11 mg/dL platelets 190/L AST 60 U/L fibrinogen 450 mg/dL and
glucose 96 mg/dL

A 26-year-old primigravida presents at 34 weeks' gestation with nausea and vomiting, fatigue, and
epigastric pain. All except which of the following are clinical characteristics that increase her risk for
acute fatty liver of pregnancy as the underlying cause?
Select one:
a. Nulliparity
b. Female fetus
c. Twin gestation
d. Third trimester

A 32-year-old nulligravida presents for preconception counseling due to cirrhosis with esophageal
varies. Which of the following should be included in your counseling?
a. All of the above
b. Without variceal rupture there is high risk for liver failure, preterm delivery, fetal growth
restriction,maternal death.
c. You do not recommend that she become pregnant.
d. 1/3 to 1/2 of women with varices will have bleeding in pregnancy and this carries an 18% mortality

The following are viral serologies of the pregnant patient. What is her diagnosis?
1. IgA Anti-Hepatitis Bcore antibody +
2. Hepatitis B surface antibody -
3. Hepatitis Be antigen +
a. Chronic hepatitis B infection, still active
b. It is not possible to make a diagnosis from the serologies presented
c. History of hepatitis B vaccination
d. History of hepatitis B infection. now latent

How do you counsel the early post delivery patient regarding her following viral serologies,
• IgA Anti-Hepatitis B core antibody
• Hepatitis B surface antibody --
• Hepatitis Li e antigen +

a. She has an active virus that can lead to cirrhosis and death. and you will refer her to a
hepatologtst tor long-term management recomrnendations.
b. She has a virus that can lead to cirrhosis and death, but it appears dormant. You will reter her to a
hepatologIst for surveillance.
c She has a virus that can lead to cirrhosis a. death. and you will refer her to a hepatologist dIscussion
ot ribavirin therapy.
d. She does not need to be concerned, Her results represent vaccination for Hepatitis B. and she should
have lifelong antibodies.

Maternal acute fatty liver of pregnancy is associated with all except with of the following recessively
inherited abnormalities of mitochondrial fatty acid oxidation?
Select one:
a. Long-chain-3-hydroxyacyl-CoA-dehydrogenase
b. Medium-chain acyl-CoA dehydrogenase
c. Carnitine palmitoyltransferase 1
d. Dihydrolipoamide dehydrogenase

Which imaging modality is recommended in discriminating acute fatty liver of pregnancy from other
etiologies of elevated liver enzymes?
Select one:
a. Ultrasound with color Doppler
b. Magnetic resonance imaging
c. Computed tomography
d. None of the above

You send a peripheral blood smear of the patient at 34 weeks’ gestation


with an acute fatty liver, and a representative slide is shown below. What is
the underlying etiology of the blood smear findings?
Выберите один ответ:
a. Autoimmune antibody binding
b. Decreased cholesterol production
c. Increased destruction in the spleen
d. Thrombotic microangiopathy in microvessels

OVARIAN CANCER

A 55-year-old woman is noted to have an abdominal mass and increased abdominal girth. On
examination, there is shifting dullness and a fluid wave. Which of the following malignancies is most
likely to be found in this patient?
Select one:
a. Ovarian cancer
b. Endometrial cancer
c. Cervical cancer
d. Vulvar cancer
e. Colon cancer

The cut surface of ovary has 4 layers. Which layer is the next from the very superficial?
Select one:
a. connective tissue
b. cortex
c. medulla
d. cuboidal epithelium

What type of ovarian cysts is associated with pregnancy (particularly multiple) and gestational
trophoblastic disease?
Select one:
a. follicular cyst
b. corpus luteal cyst
c. para-ovarian cyst
d. dermoid cyst
e. theca luteal cyst

From listed below what does not prevent ovarian cancer?


Select one:
a. bilateral salpingectomy
b. nulliparity
c. tubal ligation
d. combined oral contraceptive pills use
e. bilateral salpingo-oophorectomy

Tumor maker related to both pancreatic carcinoma and mucinous ovarian carcinoma:
Select one:
a. CA-19-9
b. CA-125
c. All of the above
d. CD-15-3

Meigs' syndrome is diagnosed based on a triad of the following signs, except:


Select one:
a. oedema
b. ascites
c. pleural effusion
d. ovarian tumour

What biochemical tumour marker is used in diagnostics of ovarian mucinous tumours?


Select one:
a. alpha-fetoprotein
b. inhibin
c. CA 125
d. Ca 19-9
e. Beta-hCG

What is incorrect regarding germ cell tumours?


Select one:
a. hair, teeth, fat, skin, muscle, cartilage, bone and endocrine tissue are frequently present
b. these are the most common ovarian tumours in young women aged 20-40 years
c. a peak incidence in the early 20s
d. up to 10% of dermoid cysts are bilateral X
e. the risk of malignant transformation is high (&gt; 20 %), usually occurring in women over 40 years

Risk factors for ovarian cancer are the following, except:


Select one:
a. multiparity
b. hereditary factor
c. increasing age
d. personal history of breast cancer
e. postmenopausal hormone replacement therapy

Kruti, 56 years old, complained of pain in abdomen, with USG showing 4 cm bilateral ovarian mass
with increased vascularity. Next line of management is:
Select one:
a. Surgery
b. Wait and watch X
c. USG-guided ovarian tapping
d. OC pills x three cycles X

Indications for surgery in patients with functional ovarian cysts are the following, except:
a arnenorrhea induced by functional ovarian cyst
b. large ovarian cyst (&got; 7 cm)
c. rupture or torsion of ovarian cyst
d. if any cyst petsists more than 3 months

What is the most common clinical manifestation of granulosa cell tumor in pre-puberty girls?
a. Maids' syndrome
b. delayed puberty
c. isosexual precocious puberty
d. ascites
e. hirsutism

What ovarian mass may be found in any female age period (from infancy to menopause)?
a. benign ovarian tumor
b. paratubal cyst
c. follicular cyst
d. endometrioma

A 25-year-old woman is noted to have a 4-cm simple cyst of the right ovary. She denies any abdominal
pain, nausea, or vomiting. Which of the following is the next best step?
a Chemotherapy
b. Expectant management
c. Exploratory laparotomy
d. LaparoScopy

Conditions diagnosed as a pelvic mass in women of reproductive age are the following except
a. Prolactinoma
b. Ovarian or adnexal cysts and tumors
c. Full urinary bladder
d. Tuboavarian abscess

What is a surgery of choice in a young, nulliparous patient with ovarian carcinoma stage la?
a. unilateral oophorectomy
b. total hysterectomy with unilateral salpingo- oophorectomy
c. total hysterectomy with bilateral salpingo-oophorectomy, omentectomy X
d. bilateral salpingo-oophorectomy X
e. unilateral salpingoroophorectorny

All conditions listed below are refered to functional ovarian cysts, except:
a. paraovarian cyst
b. corpus luteal cyst
c. follicular cyst
d. theca-lutein cyst

These tumours may produce estrogens, except:


a. juvenile granulosa cell tumour
b. Sertoli-Leydig tumour
c. thecoma
d. adult granulosa cell tumour
e. Brenner tumour

The presentation with the different types of benign ovarian tumours varies with age. Mark the incorrect
statement.
a. the peak age for malignarn epithelial tumours is postmenopause
b. germ cell tumours occur more commonly in older women
c. functional cysts are common in young girls, adolescents and women in their reproductive years
d. benign epithelial tumours are more prevalent in older and postmenopausal women

A 44-year-old woman is noted to have a 30-cm tumor of the ovary. Which of the following is the most
likely cell type?
a. serous tumor
b. Mucinous tumor
c. Granulosa cell tumor
d. Dermoid cyst

What statement is incorrect regarding epithelial ovarian cancer?


a. a combination of surgery and chemotherapy is almost always recommended for treatment X
b. epithelial ovarian cancer is more common for nulliparous women X
c. elderly women are more likely to develop ovarian cancer
d. ovarian cancer is the most lethal of all gynecologic cancers
e. oral contraceptives use increases the incidence of ovarian cancer

What is a standard surgical approach In patient with benign epithelial tumour who already Completed
childbearing?
a unilateral cystectomy X
b. total hysterectomy and bilateral salpingo-oophorectomy
c. unilateral salpingo-oophorectomy
d. bilateral salpingo-oophorectomy
e. total hysterectomy

Which biochemical tumour marker is used during treatment and follow-up of ovarian dysgerminoma?
a. HE4
b. alpha-fetoprotein
c. lactate dehydrogenase
d. beta-hCG X
e. CA 125

Theca-lutein cysts are most often assosiated with:


a. ovulation induction
b. Rh-sensitization
c. hydatidiform moles and choriocarcinoma
d. pregnancy

What clinical sign of Sertoli-Leydig cell tumour in patients of reproductive age will not resolve after
tumour removal?
a. hirsutism
b.amenorrhea
c. acne vulgaris
d. temporal balding
e. clitoromegaly

A lady has ovarian mass, pelvic X-ray shows a radio plaque shadow. The most likely diagnosis is:
Select one:
a. Serous cyst Aden’s a
b. Mucinous cyst adenoma X
c. Dysgerminoma
d. Mature tera tona
e. Follicular cyst

What is a standard surgical approach in patient with benign epithelial tumour who already completed
childbearing?
ENDOMETRIUM CANCER
Выберите один ответ:

a. bilateral salpingo-oophorectomy
What gestational age is known as a predisposing factor for developement of corpus luteal cysts?
b.Select one:
total hysterectomy
a. Corpus luteal cysts may be found within all the pregnancy duration
c. unilateral cystectomy

d. total hysterectomy and bilateral salpingo-oophorectomy

e. unilateral salpingo-oophorectomy
.1 A5-year-oldgirlisnotedtohavebreastenlargement,vaginalbleeding,andan 8-cm pelvic mass.
Which of the following is the most likely etiology?
A. Benign cystic teratoma (dermoid)
B. Endodermal sinus tumor
C. Brenner tumor
D. Choriocarcinoma
E. Granulosa-theca cell tumor
59.2 A25-year-oldwomanisnotedtohavea4-cmsimplecystoftherightovary. She denies any
abdominal pain, nausea, or vomiting. Which of the following is the next best step?
A. Expectant management
B. Laparoscopy
C. Exploratory laparotomy
D. Chemotherapy
59.3 Which of the following is the best treatment for a suspected dermoid cyst found in an
18-year-old nulliparous woman?
A. Total abdominal hysterectomy
B. Unilateral salpingo-oophorectomy
C. Ovarian cystectomy
D. Observation

59.7 A44-year-oldwomanisnotedtohavea30-cmtumoroftheovary.Whichof the following is the most likely cell type?


A. Dermoid cyst
B. Granulosa cell tumor
C. Serous tumor
D. Mucinous tumor

What is incorrect regarding the characteristics of ovarian normal anatomy?


Выберите один ответ:
a. laterally, each ovary is attached to the suspensory ligament of the ovary with folds of peritoneum that becomes continuous
with that of the overlying psoas major
b. each ovary is attached to the cornu of the uterus by the broad ligament and at the hilum to the ovarian ligament by the
mesovarium, which contains its supply of nerves and blood vessels

All the following are true about Krukenberg’s tumor, except:


Выберите один ответ:
a. Enlarged ovaries
b. Stomach is the most common site of primary tumor
c. None of the above
d. Bilateral

Which ovarian tumour is known to be a risk factor for endometrial carcinoma?

Выберите один ответ:

a. ovarian endometrioma

b. mucinous tumour

c. thecoma

d. serous tumour

e. epithelial ovarian carcinoma Which biochemical tumour marker is used during treatment and follow-up of ovarian
dysgerminoma?
Indications for surgery in patients with functional ovarian cysts are the following, except:

Выберите один ответ: Выберите один ответ:


a. large ovarian cyst (&gt; 7 cm)
a. beta-hCG
b. amenorrhea induced by functional ovarian cyst

c. rupture or torsion of ovarian cyst b. lactate dehydrogenase


d. if any cyst persists more than 3 months
c. HE4

The Risk of Malignancy Index (RMI) is calculated using the following criteria, except:
d. CA 125

Выберите один ответ: e. alpha-fetoprotein


a. serum CA-125 level

b. serum HE4 level

c. pelvic ultrasound features


What type of ovarian cysts is traditionally associated with increased risk of rupture and subsequent
d. menopausal status intraabdominal bleeding?

Выберите один ответ:

a. corpus luteal cyst

b. theca luteal cyst

c. follicular cyst

d. dermoid cyst

e. para-ovarian cyst
ENDOMETRIUM CANCER
What gestational age is known as a predisposing factor for developement of corpus luteal
b. 3rd trimester cysts? Select one:
c. 2ndt trimester a. Corpus luteal cysts may be found within all the pregnancy duration
d. 1st trimester

CERVICAL CANCER

A 35-year-old lady with postcoital bleeding management is:


Выберите один ответ:
a. Visual examination with Lugol iodine
b. Visual examination with acetic acid
c. Colposcopy
d. Clinical examination and PAP smear
Вопрос 2
Верно
Балл: 1,00
Отметить вопрос
Текст вопроса
Most common cause of postmenopausal bleeding in India is:
Выберите один ответ:
a. Endometrial cancer
b. Endometrial atrophy
c. Endometrial hyperplasia
d. Carcinoma cervix
Вопрос 3
Верно
Балл: 1,00
Отметить вопрос
Текст вопроса
A 50-year-old P4L4 has PAP smear showing dysplasia. She undergoes colposcopic-directed cervical biopsy, the report of which is normal.
Next line of management is:
Выберите один ответ:
a. Therapeutic cone biopsy
b. Diagnostic cone biopsy
c. Wait and watch
d. Hysterectomy
Вопрос 4
Верно
Балл: 1,00
Отметить вопрос
Текст вопроса
A 39-year-old woman is diagnosed with advanced cervical cancer that appears to have spread to her right pelvic sidewall. She has right
hydronephrosis as evidenced by the intravenous pyelogram (IVP). The biopsy specimen confirms that it is a poorly differentiated carcinoma.
Which of the following statements regarding this patient’s condition is most accurate?
Выберите один ответ:
a. Radical hysterectomy is an option in the therapy of this patient
b. The best therapy for her is surgical excision
c. The majority of cervical cancers are of adenomatous cell type
d. Both brachytherapy and teletherapy are important in the treatment of this patient
Вопрос 5
Верно
Балл: 1,00
Отметить вопрос
Текст вопроса
A 35-year-old lady has undergone radical hysterectomy for Ca cervix. Histopathology shows stage IBI with outer one-third of cervix and lower
uterine segment involvement. Next line of management is:
Выберите один ответ:
a. Chemotherapy
b. Chemoradiation
c. Radiation
d. Follow-up
Вопрос 6
Неверно
Балл: 1,00
Отметить вопрос
Текст вопроса
Stage lb cervical cancer is diagnosed in a young woman. Assuming that the cancer is confirmed to the cervix and that intraoperative biopsies
are negative, which of the following structure would not be removed during the radical hysterectomy?
Выберите один ответ:
a. Pelvic nodes
b. The entire parametrium on both sides of the cervix
c. Uterosacral and uterovesical ligaments
d. Both ovaries
Вопрос 7
Верно
Балл: 1,00
Отметить вопрос
Текст вопроса
A 40-year-old woman is referred for a Pap smear showing high-grade squamous intraepithelial lesions. Which of the following statements is
most accurate?
Выберите один ответ:
a. If an endocervical curetting shows cervical dysplasia, then an excisional procedure of the cervix is appropriate
b. If HPV subtyping reveals no high-risk virus present, then routine cytology is recommended
c. If colposcopy demonstrates the entire transformation zone, then no further analysis is needed
d. Cervical cancer is highly unlikely due to the Pap smear revealing only HSIL
A 33-year-old woman has a Pap smear showing moderately severe cervical dysplasia (high-grade squamous intraepithelial neoplasia). She
denies a smoking history and does not recall having any sexually transmitted infections. Which of the following is the best next step?
Выберите один ответ:
a. Colposcopy-directed biopsies
b. Computed tomography scan of the abdomen and pelvis
c. Conization of the cervix
d. Radical hysterectomy
e. Repeat Pap smear in 3 months
Вопрос 9
Неверно
Балл: 1,00
Отметить вопрос
Текст вопроса
A 22-year-old nulliparous woman is seen by her gynecologist and given a routine Pap smear. The Pap smear revealed LSIL, and a colposcopy is
performed. A biopsy at 3:00 shows CIN 2. Which of the following is the best therapy for this patient?
Выберите один ответ:
a. Offer hysterectomy
b. Repeat Pap smear in 6 months
c. LEEP excision of the cervix
d. Conization of the cervix with top hat to address any endocervical involvement
Вопрос 10
Неверно
Балл: 1,00
Отметить вопрос
Текст вопроса
A 45-year-old woman is diagnosed with an early cervical cancer, noted to be confined to the cervix and about 3 cm in diameter. Which of the
following is a risk factor for cervical cancer?
Выберите один ответ:
a. Obesity
b. Early age of coitus
c. Family history of cervical cancer
d. Nulliparity
e. Late menopause
A 35-year-old lady has undergone radical hysterectomy for Ca cervix. Histopathology shows stage IBI with outer one-third of cervix and lower
uterine segment involvement. Next line of management is:
Выберите один ответ:
a. Follow-up
b. Chemotherapy
c. Radiation
d. Chemoradiation
Вопрос 2
Верно
Балл: 1,00
Отметить вопрос
Текст вопроса
Pap smear is useful in the diagnosis of all, except:
Выберите один ответ:
a. Gonorrhea
b. Human papilloma virus
c. Trichomonas vaginalis
d. Inflammatory changes
Вопрос 3
Верно
Балл: 1,00
Отметить вопрос
Текст вопроса
Advantages of surgery over radiotherapy in cervical cancer treatment are all, except:
Выберите один ответ:
a. Preservation of vaginal function
b. Conservation of ovaries
c. Lesser surgical mortality
d. None of the above
Вопрос 4
Неверно
Балл: 1,00
Отметить вопрос
Текст вопроса
A 48-year-old healthy postmenopausal woman has a Pap smear performed, which reveals atypical glandular cells. She does not have a history of
abnormal Pap smears. Which of the following is the best next step?
Выберите один ответ:
a. Hormone-replacement therapy
b. Repeat Pap smear in 3 months
c. Colposcopy, endocervical curettage, endometrial sampling
d. Vaginal sampling
Вопрос 5
Верно
Балл: 1,00
Отметить вопрос
Текст вопроса
A 39-year-old woman is diagnosed with advanced cervical cancer that appears to have spread to her right pelvic sidewall. She has right
hydronephrosis as evidenced by the intravenous pyelogram (IVP). The biopsy specimen confirms that it is a poorly differentiated carcinoma.
Which of the following statements regarding this patient’s condition is most accurate?
Выберите один ответ:
a. Both brachytherapy and teletherapy are important in the treatment of this patient
b. The best therapy for her is surgical excision
c. The majority of cervical cancers are of adenomatous cell type
A 48-year-old woman who presents with postcoital vaginal bleeding is noted to have a cervical exophytic mass. A biopsy of the mass
confirms squamous cell carcinoma. If molecular analysis of the cancer is performed, which of the following H PV subtypes is most likely to
be found in the specimen?
Выберите один ответ:
a. 6 and 11
b. 89 and 92
c. 16 and 18
d. 55 and 57
Вопрос 7
Верно
Балл: 1,00
Отметить вопрос
Текст вопроса
A 50-year-old P4L4 has PAP smear showing dysplasia. She undergoes colposcopic-directed cervical biopsy, the report of which is normal.
Next line of management is:
Выберите один ответ:
a. Diagnostic cone biopsy
b. Therapeutic cone biopsy
c. Hysterectomy
d. Wait and watch
Вопрос 8
Верно
Балл: 1,00
Отметить вопрос
Текст вопроса
Cytogenetics is difficult in solid tumors, that too especially in carcinoma cervix, due to:
Выберите один ответ:
a. High mitotic activity
b. Specimen often not adequate
c. Good-quality metaphase
d. Often contaminated and infeсted with microorganisms
Вопрос 9
Верно
Балл: 1,00
Отметить вопрос
Текст вопроса
All are causes of postmenopausal bleeding except:
Выберите один ответ:
a. Carcinoma in situ cervix
b. Ca ovary
c. Ca fallopian tube
d. Ca endometrium
Вопрос 10
Верно
Балл: 1,00
Отметить вопрос
Текст вопроса
A patient presents with Ca cervix with stage IIIb. Treatment of choice is:
Выберите один ответ:
a. Wertheim’s hysterectomy
b. Intracavitatory brachytherapy followed by external beam radiotherapy
c. Schauta’s operation
d. Chemotherapy

58.6 A 47-year-old woman G4P4 has a Pap smear which shows H SIL. Colpos- copy is performed
which is adequate, and reveals CIN III. An endocervical curettage is negative. The patient also has
menorrhagia caused by uterine fibroids. Thus, the patient undergoes a total abdominal
hysterectomy, includ- ing removal of the cervix. The patient asks whether Pap smears need to be
performed now that her cervix has been surgically removed. Which of the following is the most
accurate statement?
A. The patient should continue to have annual Pap smears of the vaginal cuff.
B. The patient should have Pap smears every 2 to 3 years, which may be discontinued if negative
after 10 years.
C. The patient does not need Pap smears any longer.
D. The patient should have the H PV vaccine.

All of the following are indications for postoperative radiotherapy in a case of carcinoma
endometrium, except:
Выберите один ответ:
a. Myometrial invasion &amp;gt; ½ thickness
b. Endocervical involvement
c. Positive lymph nodes
d. Tumor positive for estrogen receptors
A patient is receiving external beam radiation for the treatment of metastatic endometrial cancer. The treatment field
includes the entire pelvis. Which of the following tissues within this radiation field is the most radiosensitive?
Выберите один ответ:
a. Vagina
b. Rectovaginal septum
c. Ovary
d. Bladder

A 35-year-old woman is diagnosed with endometrial cancer. Which of the following is most likely to be present?
Выберите один ответ:
a. BRCA-1 mutation
b. Ascites
c. Pelvic irradiation
d. Galactorrhea
e. Polycystic ovarian syndrome

A 45-year-old postmenopausal woman with AUB has 8 mm thickness of endometrium. Next line of management is:
Выберите один ответ:
a. Hysterectomy
b. Endometrial histopathological examination
c. Progesterone therapy
d. Follow-up and USG

Choice of adjuvant treatment for endometrial carcinoma stage IA, grade I is:
Выберите один ответ:
a. Chemotherapy plus radiotherapy
b. Chemotherapy
c. No treatment
d. Radiotherapy
Choriocarcinoma metastasizes most commonly to:
Выберите один ответ:
a. Vagina
b. Liver
c. Lungs
d. Brain
The investigation of choice in a 55-year-old postmenopausal woman who has presented with postmenopausal bleeding is:
Выберите один ответ:
a. CA-125
b. Pap smear
c. Transvaginal ultrasound
d. Fractional curettage
A 60-year-old woman presents to her physician’s office with postmenopausal bleeding. She undergoes endometrial sampling, and is diagnosed with endometrial cancer. Which of the following is a risk factor
for endometrial cancer?

Выберите один ответ:

a. Oral contraceptive use

b. Smoking

c. Multiparity

d. Diabetes mellitus

e. Herpes simplex infection

57-year-old postmenopausal woman with hypertension, diabetes, and a history of polycystic ovarian syndrome complains of vaginal bleeding for 2 weeks. The endometrial sampling shows a few fragments of atrophic endometrium. Estrogen-replacement therapy is begun.
The patient continues to have several episodes of vaginal bleeding 3 months later. Which of the following is the best next step?

Выберите один ответ:

a. Unopposed estrogen-replacement therapy

b. Endometrial ablation

c. Continued observation and reassurance

d. Serum CA-125 testing

e. Hysteroscopic examination

All of the following are indications for postoperative radiotherapy in a case of carcinoma endometrium, except:

Выберите один ответ:

a. Endocervical involvement

b. Positive lymph nodes

c. Myometrial invasion &amp;gt; ½ thickness -

d. Tumor positive for estrogen receptors

Chemotherapy is recommended in postevacuation phase of molar pregnancy in all, except:

Выберите один ответ:

a. Plateau of hCG for 6 weeks

b. Theca-lutein cysts &amp;gt;6 cm size

c. Persistent vaginal bleeding

d. Regression of uterine size

Incidence of choriocarcinoma is seen more after:

Выберите один ответ:

a. Cesarean section

b. Normal delivery

c. Ectopic pregnancy

d. Spontaneous abortion

A 35-year-old G0P0 presents to her doctor for infertility. She also has heavy vaginal bleeding that persisted for 3 weeks. She has a
long history of oligomenorrhea. An endometrial biopsy shows Grade 1 endometrial carcinoma. Which of the following is the best
treatment for this patient?

Выберите один ответ:

a. Surgical staging

b. Radiation therapy

c. Clomiphene citrate

d. Combination chemotherapy

e. Progestin therapy
A52-year-oldwoman,whohashypertensionanddiabetes,isdiagnosedwith endometrial cancer. H er diseases are well
controlled. H er physician has diag- nosed the condition as tentatively stage I disease (confined to the uterus). Which of the
following is the most important therapeutic measure in the treatment of this patient?
A. Radiation therapy
B. Chemotherapy
C. Immunostimulation therapy
D. Progestin therapy
E. Surgical therapy

Approximately what percentage of postpartum women require readmission to the hospital within 8 weeks of delivery?

Выберите один ответ:

a. 10%

b. 3%

c. 7%

d. &lt;1%

Отметить вопрос

Текст вопроса

What is the mean time to ovulation resumption in a postpartum woman who elects not to breastfeed?

Выберите один ответ:

a. 5 weeks

b. 7 weeks

c. 4 weeks

d. 6 weeks

Вопрос 3

Верно

Баллов: 1,00 из 1,00

Отметить вопрос

Текст вопроса

What duration of time is encompassed in the puerperium?

Выберите один ответ:

a. 10–12 weeks

b. 6–8 weeks

c. 4–6 weeks

d. 2–4 weeks

Вопрос 4

Верно

Баллов: 1,00 из 1,00

Отметить вопрос

Текст вопроса

Following an uncomplicated vaginal delivery, when can women be advised to resume coitus based on desire and comfort?

Выберите один ответ:

a. 2 weeks postpartum

b. 1 week postpartum

c. 4 weeks postpartum

d. 3 weeks postpartum

Вопрос 5

Верно

Баллов: 1,00 из 1,00

Отметить вопрос

Текст вопроса

Which of the following has not been shown to lower the risk for infection after cesarean delivery?

Выберите один ответ:

a. Chlorhexidine-alcohol skin preparation

b. Single-dose antibiotics prior to skin incision

c. Spontaneous separation of the placenta

d. Surgeons changing gloves after delivery of the placenta


Which of the following is the best treatment for toxic shock syndrome?

Выберите один ответ:

a. There is no necessary treatment as it will resolve on its own over time

b. Supportive care and antibiotics

c. Supportive care

d. Supportive care, antibiotics, and wound debridement if necessary

Вопрос 7

Неверно

Баллов: 0,00 из 1,00

Отметить вопрос

Текст вопроса

When is mastitis most likely to occur?

Выберите один ответ:

a. Postpartum day 1

b. Postpartum day 5

c. 3–4 weeks postpartum

d. 6–9 months postpartum

Вопрос 8

Верно

Баллов: 1,00 из 1,00

Отметить вопрос

Текст вопроса

Which of the following is a risk factor for episiotomy dehiscence?

Выберите один ответ:

a. Genital warts

b. Smoking

c. All of the mentioned

d. Infection

Вопрос 9

Неверно

Баллов: 0,00 из 1,00

Отметить вопрос

Текст вопроса

Which of the following is least likely to occur during surgery for uterine incisional necrosis?

Выберите один ответ:

a. Bilateral salpingo-oophorectomy

b. Surgical debridement

c. Hysterectomy

d. Transfusion

Вопрос 10

Верно

Баллов: 1,00 из 1,00

Отметить вопрос

Текст вопроса

A 35-year-old woman is noted to have a blood pressure of 80/40 mm Hg, fever, and abdominal pain. Which of the following is the likely mechanism of the patient’s hypotension?

Выберите один ответ:

a. Cardiac bradycardia

b. Vasodilation

c. Third spacing of fluid

d. Cardiac contractility dysfunction


A 26-year-old G3P3 is postpartum day 1 following an uncomplicated vaginal delivery. She reports sharp, intermittent lower abdominal pain, which is more severe than in her prior deliveries. Her
heart rate is 84 beats per minute, blood pressure 110/60 mmHg, and her temperature 99.3 °F. Her abdomen is soft on exam and there is no uterine tenderness. A scant amount of lochia is
appreciated on bimanual exam. What is the most likely diagnosis?

Выберите один ответ:

a. Bladder flap hematoma

b. Endometritis

c. Septic pelvic thrombophlebitis

d. Afterpains

Вопрос 12

Верно

Баллов: 1,00 из 1,00

Отметить вопрос

Текст вопроса

A 29-year-old G1P1 presents for a follow-up visit 4 weeks postpartum following an uncomplicated spontaneous vaginal delivery. She is overall doing well and has returned to most of her usual
activities. However, she complains of low energy levels despite the fact that her baby is sleeping for long intervals at night. What should you offer her?

Выберите один ответ:

a. Reassurance only

b. Psychiatry referral

c. All of the mentioned

d. Check her thyroid hormone levels

Вопрос 13

Верно

Баллов: 1,00 из 1,00

Отметить вопрос

Текст вопроса

A 27-year-old woman G1 P0 at 39 weeks’ gestation is noted to be in labor. She underwent artificial rupture of membranes, and experiences fetal bradycardia. Palpation of the vagina reveals a
rope-like structure prolapsing through the cervix. She is diagnosed with a cord prolapse and underwent stat cesarean delivery. On postoperative day 2, the patient has a temperature of 102°F
(38.8°C), and is diagnosed with endometritis. The patient who works in the microbiology laboratory asks which of the following is the most commonly isolated bacteria in her infection?

Выберите один ответ:

a. Group B streptococcus

b. Staphylococcus aureus

c. Bacteroides species

d. Escherichia coli

Вопрос 14

Верно

Баллов: 1,00 из 1,00

Отметить вопрос

Текст вопроса

A 34-year-old G3P3 with a history of migraine headaches calls your office 1 week following an uncomplicated vaginal delivery complaining of severe headaches. She did receive epidural
anesthesia during her labor course, but review of her chart confirms that the epidural placement was easy and uncomplicated. The patient reports that the headaches began a few days
following delivery and are associated with nausea, but no vomiting, and are minimally improved with either rest or over-the-counter analgesics. What is the most likely trigger of this patient’s
postpartum headaches?

Выберите один ответ:

a. Progesterone withdrawal

b. Intracerebral hemorrhage

c. Estrogen withdrawal

d. Dural puncture

Вопрос 15

Верно

Баллов: 1,00 из 1,00

Отметить вопрос

Текст вопроса

A 32-year-old woman has just delivered a 40-week baby vaginally. She desires to breast-feed. Her physician recommends that she not breast feed. Which of the following conditions is most
likely to be present?

Выберите один ответ:

a. Ampicillin therapy for cystitis

b. Maternal HIV infection

c. Maternal dilantin therapy for seizure disorder


All except which of the following conditions are contraindications to breastfeeding?
Выберите один ответ:
a. Infant with galactosemia
b. Maternal active and untreated tuberculosis
c. Maternal hepatitis B infection
d. Human immunodeficiency virus infection

What percentage of women experience regression of high-grade dysplasia following delivery?


Выберите один ответ:
a. 35%
b. 10%
c. 50%
d. 33%
What vitamin is virtually absent in human breast milk?
Выберите один ответ:
a. All vitamins are equally represented in breast milk
b. Vitamin C
c. Vitamin A
d. Vitamin K

Which of the following regimens is the gold standard for treatment of a pelvic infection following a cesarean delivery?
Выберите один ответ:
a. Clindamycin and gentamicin
b. Clindamycin and aztreonam
c. Vancomycin
d. Meropenem

Which of the following has not been shown to lower the risk for infection after cesarean delivery?
Выберите один ответ:
a. Surgeons changing gloves after delivery of the placenta
b. Single-dose antibiotics prior to skin incision
c. Spontaneous separation of the placenta
d. Chlorhexidine-alcohol skin preparation

Which of the following is the best treatment for toxic shock syndrome?
Выберите один ответ:
a. Supportive care
b. There is no necessary treatment as it will resolve on its own over time
c. Supportive care and antibiotics
d. Supportive care, antibiotics, and wound debridement if necessary

Which of the following is least likely to occur during surgery for uterine incisional
necrosis?
Выберите один ответ:
a. Bilateral salpingo-oophorectomy
b. Surgical debridement
c. Transfusion
d. Hysterectomy

Which of the following is frequently the first sign/symptom of peritonitis in a


postpartum woman?
Выберите один ответ:
a. Skin erythema
b. Adynamic ileus
c. Abdominal rigidity
d. Diarrhea

A 24-year-old G1P1 complains of abdominal pain and subjective fever 1 day following an uncomplicated vaginal delivery.
Which of the following physical exam signs or laboratory values would be most helpful to make a diagnosis of
endometritis in this postpartum patient?
Выберите один ответ:
a. White blood cell count of 25,000/µL
b. An absolute neutrophilia
c. Fundal tenderness
d. All of the mentioned

A 26-year-old G3P3 is postpartum day 1 following an uncomplicated vaginal delivery. She reports sharp, intermittent lower abdominal pain, which is
more severe than in her prior deliveries. Her heart rate is 84 beats per minute, blood pressure 110/60 mmHg, and her temperature 99.3 °F. Her
abdomen is soft on exam and there is no uterine tenderness. A scant amount of lochia is appreciated on bimanual exam. What is the most likely
diagnosis?
Выберите один ответ:
a. Bladder flap hematoma
b. Endometritis
c. Afterpains
d. Septic pelvic thrombophlebitis

Which of the following is an evidence-based statement about the use of vacuum-assisted wound closure devices in
obstetrics?
Выберите один ответ:
a. It is significantly more cost effective than standard dressings.
b. It is superior to standard dressings.
c. It prevents wound infection.
d. Provider time is decreased substantially.
Your patient presents 5 days postpartum with an axillary mass. She noted it during pregnancy although reports it was much
smaller. Yesterday, she noticed that the mass became larger and more tender. Her face is shielded and a sagittal photograph
of her breast, axilla, and arm is show below. She denies fever and has no additional complaints. Management of the patient
should primarily include which of the following?

Выберите один ответ:


a. Observation and reassurance
b. Axillary lymph node excision
c. Needle aspiration
d. Antibiotic therapy with gram-positive coverage

Which of the following is true regarding the condition shown in the pelvic radiograph?

Выберите один ответ:


a. All of the mentioned
b. Symphyseal separation &gt;1.5 cm is diagnostic.
c. Surgery is usually necessary for separation &gt;3 cm.
d. Treatment is generally conservative and consists of rest and placement of a pelvic binder.

29-year-old G2P2 with an 18-hour labor course is unable to void 5 hours following a vaginal delivery. What is the best management option?
Выберите один ответ:
a. A single in-and-out catheterization
b. Place a Foley catheter for 24 hours
c. Continue close observation without intervention
d. Encourage increased fluid intake since the bladder is likely empty following delivery

25.1 A 30-year-old G1P1 who underwent a cesarean section 3 days previously has a fever of 101°F (38.3°C).
The skin incision is indurated, tender, and erythematous. Which of the following is the best management?
A. Initiation of intravenous ampicillin
B. Initiation of intravenous heparin
C. Placement of a warm compress on the wound
D. Opening of the wound

25.2 A 29-year-old woman is diagnosed with postpartum endometritis based on fever, abdominal pain, fundal
tenderness, and elimination of other etiologies. Which of the following is the most significant risk factor for
postpartum endomyometritis?
A. Numerous vaginal examinations
B. Bacterial vaginosis
C. Cesarean delivery
D. Internal uterine pressure monitors
E. Prolonged rupture of membranes
25.3 A27-year-oldG1P0womanat39weeks’gestationisnotedtobeinlabor.She underwent artificial rupture of
membranes, and experiences fetal bradycar- dia. Palpation of the vagina reveals a rope-like structure
prolapsing through the cervix. She is diagnosed with a cord prolapse and underwent stat cesar- ean delivery.
On postoperative day 2, the patient has a temperature of 102°F (38.8°C), and is diagnosed with endometritis.
The patient who works in the microbiology laboratory asks which of the following is the most commonly
isolated bacteria in her infection?
A. Peptostreptococcus species
B. Staphylococcus aureus
C. Group B Streptococcus
D. Escherichia coli
25.4 A 22-year-old woman who underwent cesarean delivery has persistent fever of 102°F (38.8°C), despite
the use of triple antibiotic therapy (ampicillin, gentamicin, and clindamycin). The urinalysis, wound, breasts,
and uterine fundus are normal on examination. A CT scan of the pelvis is suggestive of septic pelvic
thrombophlebitis. Which of the following is the best therapy for
this condition?
A. Hysterectomy
B. Discontinue antibiotic therapy and initiate intravenous heparin
C. Continue antibiotic therapy and begin intravenous heparin
D. Surgical embolectomy
What are myrtiform caruncles?
Выберите один ответ:
a. A sexually transmitted infection
b. Scarred tags of hymenal tissue
c. Microscopic tears in the vaginal epithelium
d. Vaginal rugae

Which of the following statements about necrotizing fasciitis is false?


Выберите один ответ:
a. Early diagnosis, surgical debridement, antimicrobials, and intensive care are paramount to successful treatment.
b. Three risk factors are diabetes, obesity, and hypertension.
c. It is common with low mortality rates.
d. Surgical debridement of infected tissue should leave wide margins of healthy bleeding tissue.

Which of the following statements about an ovarian abscess in the puerperium is true?
Выберите один ответ:
a. Women present 4–6 weeks after delivery
b. Usually affects both ovaries
c. It is thought to be caused by bacterial invasion of the ovary through a rent in the capsule.
d. Rupture is rare
In more than 90% of women, metritis responds to treatment with antibiotics within what period of time?
Выберите один ответ:
a. 12–24 hours
b. 24–36 hours
c. 72–96 hours
d. 48–72 hours

A 22-year-old nulliparous woman is noted to have a tender, red, right breast and enlarged, tender axillary lymph nodes that have persisted
despite antibiotics for 3 weeks. She denies manipulation of her breasts and is not lactating. Which of the following is the most appropriate
next step?
Выберите один ответ:
a. Mammographic examination of the breasts
b. Check the serum prolactin level
c. Biopsy of the breast
d. Sonographic examination of the breasts
e. Course of oral antibiotic therapy

A 28-year-old G1 P1 woman has delivered vaginally 3 weeks ago. She is breast-feeding, and notes that the baby prefers to breast-feed from the right
breast. On the left breast, she notes a 3-day history of a tender mass on the upper outer quadrant. On examination, she is afebrile. The left breast
has a fluctuant mass of 4 × 8 cm of the upper outer quadrant without redness. It is somewhat tender. Which of the following is the best treatment for
this condition?
Выберите один ответ:
a. Mastectomy
b. Aspiration
c. Bromocriptine therapy
d. Oral antifungal therapy
e. Oral antibiotic therapy

A 30-year-old G1 P1 who underwent a cesarean section 3 days previously has a fever of 101°F (38.3°C). The skin incision is indurated, tender,
and erythematous. Which of the following is the best management?
Выберите один ответ:
a. Opening of the wound
b. Initiation of intravenous ampicillin
c. Placement of a warm compress on the wound
d. Initiation of intravenous heparin
What is the mechanism of action by which surfactant prevents respiratory distress syndrome in neonatal lungs? (W25th)
Выберите один ответ:
a. Dilates the pulmonary vasculature
b. Aids in maturation of the terminal bronchioles
c. Lowers surface tension in the alveoli
d. Clears fluid from the lungs
Respiratory distress syndrome can be caused by which of the following? (W25th)
All are tocolytics except: Выберите один ответ:
Выберите один ответ: a. Infection
a. Ritodrine b. Meconium aspiration
b. Isoxsuprine c. Heart failure
c. Salbutamol d. All of the mentioned
d. Misoprostol

Risk of preterm delivery is increased if cervical length is


Выберите один ответ:
a. 3.0 cm
b. 3.5 cm
c. 2.5 cm
d. 4.0 cm

True about premature rupture of membrane (PROM) before 32 wks:


Выберите один ответ:
a. Amnioinfusion is done
b. Steroid is used
c. Aseptic manual cervical examination indicated
d. Amoxiclav antiobiotic should be given

An infant born at 25 weeks gestation is at risk for all except which of the following complications? (W25th)
Выберите один ответ:
a. Blood cancers
b. Blindness
c. Pulmonary hypertension
d. Asthma

Which of the following lifestyle factors is not associated with preterm birth? (W25th)
Выберите один ответ:
a. Advanced maternal age
b. Short stature
c. fold increase
d. Poverty
e. Vitamin D deficiency
Which of the following is true regarding transvaginal sonographic evaluation of the cervix as a part of the assessment for preterm labor? (W25th)
Выберите один ответ:
a. The WHO recommends it for all women with a history of a preterm birth
b. It is not affected by maternal obesity, cervix position, or shadowing
c. It can be performed any time after 14 weeks’ gestation
d. All of the mentioned

Compared to term infants, neonates born prematurely (all) have higher associated rates of which of the following? (W25th)
Выберите один ответ:
a. All of the mentioned
b. Sleep apnea
c. Congenital malformations
d. Developmental delay

Risk factors for poor progress of labour include the following except:
Выберите один ответ:
a. Big baby
b. Meconium stained amniotic fluid
c. Malpresentation
d. Premature rupture of membrane

Tocolytics are beneficial in preterm labour because:


Выберите один ответ:
a. They arrest preterm labour
b. They prolong pregnancy to term
c. They decrease prenatal mortality
d. They provide time for antenatal steroids

When evaluating a pregnant woman for rupture of membranes, which of the following has been
associated with a false-positive nitrazine test result?
Выберите один ответ:
a. All of the mentioned
b. Semen
c. Blood
d. Bacterial vaginosis
On TVS(transvaginal sonography) which of the following shape of cervix indicates preterm labour:
Выберите один ответ:
a. Y
b. O
c. U
d. T

You perform a routine cervical exam on a 39-year-old G3P2 at 30 weeks gestation. You find her cervix to be 2 to 3 cm dilated. She denies having
any contractions, discharge, pelvic pain or pressure. What is her chance of a preterm delivery before 34 weeks gestation? (W25th)
Выберите один ответ:
a. 25%
b. 35%
c. 5%
d. 15%

Of the responses listed below, which complication is most commonly associated with preterm delivery?
Выберите один ответ:
a. Placental abruption
b. Fetal complications
c. None of the above
d. Hypertension

Which of the following bacteria is frequently detected in the amnionic fluid of women with preterm labor? (W25th)
Выберите один ответ:
a. All of the mentioned
b. Mycoplasma hominis
c. Gardnerella vaginalis
d. Ureaplasma urealyticum

Respiratory distress syndrome can develop in the term infant as surfactant can be inactivated by which of the following
substances? (W25th)
Выберите один ответ:
a. Vernix
b. Blood
c. Meconium
d. All of the mentioned

Drug given to reduce uterine contractions during preterm labour with least side effects:
Выберите один ответ:
a. Nifidipine
b. Ritodrine
c. Atosiban
d. Magnessium sulphate

In primi and in preterm labour, which of the following can be used as tocolytic:
Выберите один ответ:
a. Dexamethasone
b. Misoprostol
c. Ritodrine
d. Propranolol

All are risk factors for preterm delivery except:


Выберите один ответ:
a. Previous history of preterm baby
b. Asymptomatic cervical dilatation
c. Absence of fetal fibronectin at < 37 weeks
d. Chylamydial infection of genital tract
MEDICAL COMPLICATIONS

Renal and Urinary Tract Disorders


» Which of the following is the most commonly isolated
etiologic agent causing pyelonephritis in pregnancy?
A. Proteus species
B. Candida species
C. Escherichia coli
D. Klebsiella species
» If patient does not improve on adequate antibiotic therapy for
48 hours and experiences continued severe flank tenderness and
fever, which of the following should be next considered?
A. Obstruction of the urinary tract
B. Anaerobic organisms
C. Hemolytic uremic syndrome
D. Factitious fever
» What severe condition is the most likely?
A. Acute respiratory distress syndrome
B. Septic shock
C. Hemolytic uremic syndrome
D. Factitious fever
» Which of the following is the most common cause of this
complication in pregnancy?
A. Pelvic inflammatory disease
B. Pyelonephritis
C. Wound infection
D. Mastitis
________________ From Willam’s Obstetrics 25th (Ch.53)_______________
Which of the following is not a physiological change in
pregnancy that contributes to the findings in the image below?
a. Vesicoureteral reflux
b. Glomerular hypertrophy
c. Distal ureteral compression by the uterus
d. Progesterone-induced relaxation of the muscularis

Which of the following statements regarding physiological


changes in pregnancy is true?
a. Glomerular filtration decreases.
b. Effective renal plasma flow increases.
c. Serum creatinine concentration increases.
d. The number of glomerular cells increases.
Which of the following is the threshold for proteinuria in
pregnancy, above which levels are considered
abnormal?
a. 100 mg/d
b. 250 mg/d
c. 300 mg/d
d. 1000 mg/d

Which of the following values is not a normal serum creatinine


in pregnancy?
a. 0.4 mg/dL
b. 0.6 mg/dL
c. 0.8 mg/dL
d. 1.0 mg/dL
A pregnant patient who previously donated a kidney and now
has only one healthy kidney is at risk for
which of the following obstetric complications?
a. Placenta previa
b. Normocytic anemia
c. Gestational hypertension
d. Preterm premature rupture of membranes

Which of the following is an acceptable treatment regimen for


newly diagnosed asymptomatic bacteriuria in pregnancy?
a. Nitrofurantoin 100 mg by mouth at bedtime for 10 days
b. Ampicillin 250 mg by mouth four times per day for one day
c. Trimethoprim-sulfamethoxazole 160/800 mg by mouth one
time
d. Nitrofurantoin 100 mg by mouth four times daily for 21 days

Lower urinary tract symptoms with pyuria but a sterile urine


culture are likely due to which pathogen?
a. Escherichia coli
b. Proteus mirabilis
c. Klebsiella pneumoniae
d. Chlamydia trachomatis

What is the leading cause of septic shock during pregnancy?


a. Pneumonia
b. Breast abscess
c. Pyelonephritis
d. Chorioamnionitis

In the setting of aggressive fluid hydration, what percentage of


pregnant women with pyelonephritis develop acute kidney
injury?
a. 5%
b. 10%
c. 15%
d. 20%

Once you have treated a pregnant patient for pyelonephritis as


an inpatient, for how long should you
continue oral therapy as an outpatient?
a. 1–3 days
b. 3–5 days
c. 7–14 days
d. 14–21 days

What percentage of pregnant women experience a recurrent


urinary tract infection after completion of
treatment for pyelonephritis?
a. 5–10%
b. 10–20%
c. 30–40%
d. 50–60%

Which of the following does not characterize nephrotic


syndromes?
a. Edema
b. Heavy proteinuria
c. Hyperalbuminemia
d. Hypercholesterolemia

Which of the following complications is not increased in


pregnant patients with nephrotic
syndrome?
a. Anemia
b. Preeclampsia
c. Renal insufficiency
d. Postterm pregnancy

Which of the following is most likely to lead to end-stage renal


disease?
a. Diabetes
b. Hypertension
c. Glomerulonephritis
d. Polycystic kidney disease

___________________________________________________
________________________________
Hematological Disorders

» Whichof the following is the most likely etiology of the anemia?


A. Iron deficiency
B. Folate deficiency
C. Vitamin B12 deficiency
D. Physiologic anemia of pregnancy
E. Acute blood loss anemia
F. Anemia of chronic disease
» Which of the following is the most likely diagnosis?
A. Iron deficiency anemia
B. Thalassemia
C. Hemolysis
D. Folate deficiency
E. Vitamin B12 deficiency
» Which of the following would you advise based on her clinical
status?
A. Transfusion of 2 units of blood
B. Repeat a complete blood count emergently
C. Three months of iron supplementation at discharge
D. All of the above
________________ From Willam’s Obstetrics 25th (Ch.56)_______________

What hemoglobin value is used to define anemia in the second


trimester of pregnancy?
a. 9.5 g/dL
b. 10.0 g/dL
c. 10.5 g/dL
d. 11.0 g/dL

Which of the following adverse pregnancy outcomes is


associated with anemia in pregnancy?
a. Stillbirth
b. Preeclampsia
c. Preterm birth
d. All of the above

What is the maternal iron requirement in pregnancy for a typical


singleton gestation?
a. 500 mg
b. 800 mg
c. 1000 mg
d. 1200 mg

If you repeat a complete blood count one week after starting


iron therapy, what would you expect to
see if she is compliant with taking her iron?
A. A 20% rise in her hematocrit
B. An elevated reticulocyte count
C. An increase in red cell distribution width
D. An increase in the mean corpuscular volume
Based on the picture below, what additional symptom is she
most likely to complain of?
a. Dry skin
b. Anorexia
c. Hair loss
d. Excessive perspiration

Which of the following is not a cause of vitamin B12 deficiency


encountered in pregnancy?
a. Crohn disease
b. Ulcerative colitis
c. History of Roux-en-Y gastric bypass
d. Prior ileal resection following trauma

Which of the following is true regarding paroxysmal nocturnal


hemoglobinuria?
a. Hemoglobinuria develops at irregular intervals
b. Almost half of patients suffer venous thromboses
c. Often precipitated by transfusions, infection or surgery
d. All of the above

Which of the following obstetric complications is not increased


in the context of sickle-cell disease in
pregnancy?
a. Stillbirth
b. Preeclampsia
c. Gestational diabetes
d. Fetal-growth restriction

Thrombocytopenia, defined as a platelet count <150,000/µL,


complicates what percentage of
pregnancies?
a. 1%
b. 3%
c. 5%
d. 10%

Which of the following treatments would you consider as first-


line therapy for the 29 weeks pregnant patient with idiopathic
thrombocytopenic purpura and platelet count 21,000/µL?
a. Splenectomy
b. Azathioprine
c. Corticosteroids
d. Intravenous immune globulin

What determines the factor VIII activity level in women affected


with hemophilia A?
a. Mosaicism
b. Lyonization
c. Co-dominance
d. None of the above

What is the incidence of postpartum hemorrhage in women with


von Willebrand disease?
a. 10%
b. 30%
c. 50%
d. 70%
Diabetes Mellitus

» What is the diagnosis?


A. Gestational diabetes mellitus
B. Type 1 diabetes
C. Type 2 diabetes
D. Transient glycosuria

» Which of the following is a risk factor in pregnant women for impaired carbohydrate
metabolism?
A. Family history of diabetes
B. Previous infant with polycystic kidney disease
C. High serum levels of antiphospholipid antibodies
D. All of the above
» What further investigations need to be arranged?
A. Retinal assessment if not performed in last 12 months
B. 50-g oral glucose tolerance test
C. Serum total cholesterol level
D. Quad screen to assess fetal risk of aneuploidy and neural-tube defect

» Outline the principles of management of the pregnancy.


A. Reduction of maternal insulin regime
B. Aim to keep fasting blood glucose between 3.5 and 5.9 mmol/L
C. Folate supplementation
D. Labor induction at 38 weeks’ gestation

________________ From Willam’s Obstetrics 25th (Ch.57)_______________


What does in utero exposure to hyperglycemia lead to?
a. Increase fetal fat cells
b. Fetal hyperinsulinemia
c. Insulin resistance in adolescence
d. All of the above

How is diabetes classified during pregnancy?


a. Type 1 or type 2
b. Pregestational or gestational
c. Type 1, type 2, or gestational
d. Using the White classification
Which of the following findings are considered diagnostic for overt diabetes in pregnancy?
a. Hemoglobin A1C >6%
b. Glucosuria on urine dip
c. Fasting plasma glucose ≥120 mg/dL
d. Random plasma glucose ≥200 mg/dL

Fetuses of overtly diabetic mothers have an increased risk for which of the following?
a. Preterm delivery
b. Spontaneous abortion
c. Congenital malformation
d. All of the above
How does diminished fetal growth occur in pregestational diabetes?
a. Substrate deprivation
b. Fetal hyperinsulinemia
c. Altered lipid metabolism
d. All of the above

Compared to women without diabetes, which fetal condition occurs more often in the setting of
pregestational diabetes?
a. Stillbirth
b. Perinatal death
c. Postpartum hemorrhage
d. Gestational hypertension
What might be said of the pregnancy yielding 5000-g newborn?
a. The mother likely had poor glycemic control.
b. The baby was at risk for neonatal hypoglycemia.
c. The mother had an increased risk for shoulder dystocia.
d. All of the above

Which of the following are considered reasons for unexplained fetal demise in women requiring
insulin during pregnancy?
a. Nonketotic acidosis
b. Elevated lactic acid levels
c. Decreased fetal hematocrit
d. None of the above

Which of the following is a reasonable explanation for hydramnios in a pregnancy complicated


by diabetes?
a. Maternal endothelial leak caused by hyperglycemia
b. Glucose reabsorption by the fetal glomerular collecting system
c. Osmotic gradient created by high glucose concentrations in the amnionic fluid
d. All of the above

What is the most likely cause for the increased incidence of respiratory distress syndrome in the
neonates of diabetic mothers?
a. Indicated preterm delivery
b. Delayed maturation of type II pneumocytes
c. Decreased production of surfactant in a hyperglycemic environment
d. All of the above
Which of the following statements regarding cardiomyopathy in infants of diabetic mothers is
true?
a. Is reversible after birth
b. Ventricular hypertrophy is due to insulin excess
c. In severe cases may lead to pulmonary hypertension
d. In the first trimester systolic dysfunction is already present

Which of the following is true concerning management of overt


maternal diabetes in the second trimester?
a. Should undergo amniocentesis
b. Should undergo a fetal echocardiogram
c. Have higher rates of chromosomal abnormalities
d. Should be offered genetic screening if >35 years of age

All except which of the following statements regarding hypocalcemia in the newborn is
accurate?
a. Defined as <9 mg/dL
b. Etiology is unexplained
c. May be related to preterm birth
d. Seen more often with strict glucose control

Maternal mortality in women with type 1 diabetes results from which of the following?
a. Infection
b. Hypertension
c. Diabetic ketoacidosis
d. All of the above
The first and most common visible retinal lesions in diabetes are small microaneurysms followed
by blot hemorrhages. This describes which of the following
conditions?
a. Proliferative retinopathy
b. Nonproliferative retinopathy
c. Is irreversible with improved control
d. The etiology of cotton wool exudates

What is the most important component of diabetic ketoacidosis treatment in pregnancy?


a. Restore euglycemia
b. Provide intravenous hydration
c. Provide intravenous potassium repletion
d. Provide intravenous bicarbonate to correct acidosis

Which of the following infections is increased in pregnant women with overt diabetes?
a. Pyelonephritis
b. Respiratory infections
c. Wound infection after cesarean delivery
d. All of the above

The most difficult aspect of preconceptional control in women with diabetes is which of the
following?
a. Resistance to insulin therapy
b. Unpredictable insulin requirements
c. Half of pregnancies in the United States are unplanned
d. All of the above

Women with type 1 diabetes should achieve glycemic control with which of the following during
pregnancy?
a. Insulin
b. Diet alone
c. Insulin and diet
d. Oral hypoglycemic agents

Which of the following is associated with fasting blood glucose levels >120 mg/dL (6.7
mmol/l)?
a. Preeclampsia
b. Cesarean delivery
c. Birthweight >90th percentile
d. All of the above
Preeclampsia

» What is the most likely diagnosis?


A. HELLP syndrome
B. Cardiac insufficiency
C. Electrolyte abnormalities
D. Aspiration
E. Severe Preeclampsia
» What is your immediate next step?
A. Improve oxygenation
B. Intravenous antihypertensive agents
C. Platelet transfusion
D. Intravenous immunoglobulin therapy
E. Intravenous Mannitol

» Which of the following is the best management?


A. Oral antihypertensive therapy
B. Platelet transfusion
C. Magnesium sulfate therapy and induction of labor
D. Intravenous immunoglobulin therapy

» Which of the following will be the most common mechanism if seizures appear in this patient?
A. Electrolyte abnormalities
B. Intracerebral hemorrhage
C. Myocardial infarction
D. Aspiration

» Which of the following is the best management for this patient?


A. Induction of labor
B. Cesarean section
C. Antihypertensive therapy
D. Expectant management
» Which of the following findings is most likely present in this patient to necessitate delivery?
A. Blood pressures persist in the range of 150/ 95 mm Hg
B. The patient reaches 32 weeks’ gestation
C. Patient develops pulmonary edema
D. Repeat platelet count is 95 000/mm3

» Which of the following is the best next step?


A. Antihypertensive agent
B. IV Mannitol
C. MRI of the brain
D. CT imaging of the brain
E. Ophthalmic eye drops to both eyes

» Which of the following is the best next step?


A. Corticosteroids
B. Antihypertensive agent
C. Biophysical profile
D. Magnesium sulfate and delivery
E. Continued observation

» Which of the following is the best next step?


A. Corticosteroids
B. Antihypertensive agent
C. Biophysical profile
D. Magnesium sulfate and delivery
E. Continued observation

» Which of the following is the best next step?


A. Corticosteroids
B. Antihypertensive agent
C. Biophysical profile
D. Magnesium sulfate and delivery
E. Continued observation

» Which of the following is the best next step?


A. Corticosteroids
B. Antihypertensive agent
C. Biophysical profile
D. Magnesium sulfate and delivery
E. Continued observation

What percentage of pregnancies are complicated by hypertension?


A. 2–3%
B. 4–5%
C. 5–10%
D. 10–20%
What percentage of eclamptic seizures occur in women without significant proteinuria?
A. 5–9%
B. 10–17%
C. 20–25%
D. 30–33%
What percentage of eclamptic seizures occur more than 48 hours after delivery?
A. 2%
B. 5%
C. 10%
D. 20%

Many conditions and factors are associated with an increased risk for preeclampsia. Which of the
following factors results in the greatest relative risk for a diagnosis of preeclampsia in the current
pregnancy?
A. Primigravida
B. Advanced maternal age
C. Systemic lupus erythematous
D. History of preeclampsia in a prior pregnancy

Which of the following is thought to play a significant role in the development of preeclampsia?
A. Genetic factors
B. Immunological factors
C. Abnormal trophoblastic invasion
D. All of the above

Which of the following is not a pathophysiological change to the cardiovascular system seen in
the setting of preeclampsia?
A. Increased preload
B. Decreased preload
C. Increased afterload
D. Endothelial activation
Which statement best describes renal perfusion and glomerular filtration rates in women with
preeclampsia that has not yet progressed to severe disease?
A. Similar compared to normal pregnant values
B. Increased compared to normal pregnant values
C. Similar compared to normal nonpregnant values
D. Decreased compared to normal nonpregnant values

What is the underlying etiology of the proteinuria seen with preeclampsia?


A. Increased capillary permeability
B. Increased renal artery resistance
C. Increased glomerular filtration rate
D. Increased systemic vascular resistance
Which of the following nutritional supplements has been shown to reduce the incidence of
preeclampsia?
A. Calcium
B. Vitamin E
C. Ascorbic acid
D. None of the above

Which of the following physiological responses is typically seen in preeclamptic patients?


A. Increased production of nitric acid
B. Increased sensitivity to angiotensin II
C. Decreased reactivity to norepinephrine
D. All of the above

The typical blood volume of a gravida at term is 4500 mL. In patients with preeclampsia, which
of the following would be the expected blood volume?
A. 2500 mL
B. 3200 mL
C. 4500 mL
D. 5000 mL

Which of the following medical therapies has been associated with a modest decrease in the
incidence of preeclampsia in women at increased risk?
A. Aspirin
B. Metformin
C. Enoxaparin
D. Heparin sulfate
_________________From William’s Obstetric
25th_____________________________
Many conditions and factors are associated with an increased risk for preeclampsia. Which of the
following factors results in the greatest relative risk for a diagnosis of preeclampsia in the current
pregnancy?
a. Primigravida
b. Advanced maternal age
c. Systemic lupus erythematous
d. History of preeclampsia in a prior pregnancy

Which of the following is thought to play a significant role in the development of preeclampsia?
a. Genetic factors
b. Immunological factors
c. Abnormal trophoblastic invasion
d. All of the above

Which of the following angiogenic factors are elevated in women who proceed to develop
preeclampsia?
a. sFlt-1
b. PIGF
c. VEGF
d. TGF-β

A 21-year-old primigravida presents at 36 weeks’ gestation with new-onset headache. Her blood
pressure is 150/90 mm Hg, her serum creatinine is 0.8 mg/mL, AST is 32 U/L, and platelet count
is 28,000/µL. Which of the following criteria for severe preeclampsia is met?
a. Hypertension
b. Liver dysfunction
c. Thrombocytopenia
d. Elevated serum creatinine

Which of the following is an indication for primary cesarean delivery without an attempt at
induction?
a. Malpresentation
b. Unfavorable cervix
c. Possible fetal thrombocytopenia
d. None of the above

Which statement best describes renal perfusion and glomerular filtration rates in women with
preeclampsia that has not yet progressed to severe disease?
a. Similar compared to normal pregnant values
b. Increased compared to normal pregnant values
c. Similar compared to normal nonpregnant values
d. Decreased compared to normal nonpregnant values

Patient is diagnosed with HELLP syndrome and undergoes an uncomplicated induction of labor.
Her right upper quadrant pain persists, and a computed tomography scan of her abdomen/pelvis
is completed with the findings as shown below. What is denoted by the asterisk (*)?

a. Splenic infarction
b. Intrahepatic infarction
c. Subcapsular hematoma
d. Periportal hemorrhagic necrosis

A G3P2 at 16 weeks’ gestation presents to the emergency department complaining of vaginal


bleeding. Her blood pressure is 148/96 mm Hg and she has 3+ proteinuria on a specimen
obtained during bladder catheterization. Which of the following may explain the development of
preeclampsia in this patient?

a. Increased volume of chorionic villi


b. Extensive remodeling of the spiral arterioles
c. Increased invasion of extravillous trophoblastic tissue
d. None of the above
The following computed tomography image shows cerebral edema in a postpartum patient with
hypertension, vision changes, and confusion. What associated morbidity is the patient at risk for?

a. Hemiplegia
b. Cystic leukomalacia
c. Retinal artery occlusion
d. Transtentorial herniation
The below magnetic resonance imaging was performed postpartum. What is the most likely
finding on imaging?

a. Subdural hematoma
b. Parenchymal hemorrhage
c. Subarachnoid hemorrhage
d. Posterior reversible encephalopathy syndrome
Antenatal bleeding

» Which of the following sequence of examinations is most appropriate?


A. Speculum examination, ultrasound examination
B. Ultrasound examination, digital examination
C. Digital examination, ultrasound examination, speculum examination
D. Ultrasound examination, speculum examination, digital examination

» What is the most likely diagnosis?


A. Placental abruption
B. Placenta previa
C. Gestational diabetes
D. Multifetal gestation
E. Gestational trophoblastic disease

» What will be the long-term management of this patient?


A. Induction of labor
B. Tocolysis if fetal condition is stable
C. Immediate cesarean delivery
D. Expectant management as long as the bleeding is not excessive
E. Intrauterine transfusion
» Which of the following is a risk factor for this patient’s condition?
A. Prior salpingitis
B. Hypertension
C. Multiple gestations
D. Polyhydramnios
» Which of the following is a typical feature of this condition?
A. Painful bleeding
B. Commonly associated with coagulopathy
C. First episode of bleeding is usually profuse
D. Associated with postcoital spotting
» Which of the following is the best management for this patient?
A. Induction of labor
B. Tocolysis of labor
C. Cesarean delivery
D. Expectant management
E. Intrauterine transfusion

» What is uncommon cause of antepartum haemorrhage (APH) at term


A. Blood-stained show
B. Bleeding placenta praevia
C. Placental abruption sure
D. Cervical ectropion Not
E. Bacterial vaginosis
F. Vasa praevia

» Which of the following sequence of examinations is most appropriate?


A. Speculum examination, ultrasound examination, digital examination
B. Ultrasound examination, digital examination, speculum examination
C. Digital examination, ultrasound examination, speculum examination
D. Ultrasound examination, speculum examination, digital examination

» Which of the following is the most appropriate management?


A. Schedule cesarean delivery at 39 weeks
B. Schedule an amniocentesis at 34 weeks and deliver by cesarean if the fetal lungs are mature
C. Schedule an MRI examination at 35 weeks to assess for possible percreta involving the
bladder
D. Reassess placental position at 32 weeks’ gestation by ultrasound
E. Recommend termination of pregnancy

How do you interpret the examination and blood test findings?


A. DIC
B. Liver failure sure
C. Haemorrhagic shock not
D. Kidney failure
E. Electrolyte disorder
Which of the following statements is most accurate?
A. Fetal ultrasound is more accurate in diagnosing placental abruption than placenta previa.
B. Fetal ultrasound is quite sensitive in diagnosing placental abruption.
C. Ultrasound is sensitive in diagnosing abruption that occurs in the lower aspect of the uterus.
D. Fetal ultrasound is not sensitive in diagnosing placental abruption.

Which of the following is the most significant risk factor for abruption placentae?
A. Prior cesarean delivery
B. Breech presentation
C. Trauma
D. Marijuana use
E. Placenta accreta

Which of the following is most likely to be present in drug screen of the pregnant patient with
bright vaginal discharges?
A. Marijuana
B. Alcohol
C. Barbiturates
D. Cocaine
E. Benzodiazepines

Which of the following is the best management for this patient?


A. Admission and careful observation in the ICU
B. Induction of labor with plan for vaginal delivery
C. Partial exchange transfusion
D. Urgent cesarean delivery
E. Intravenous terbutaline

What is the most appropriate next step in management?


A. Admit to antenatal unit for bed rest and betamethasone. !

B. Admit to antenatal unit for bed rest and blood transfusion.


C. Induction of labour
D. Perform emergency cesarean section.

What is the best course of action?


A. Emergent cesarean section
B. Fetal umbilical blood transfusion
C. Expectant management
D. Induction of labour with prost aglandins
Which of the following is most likely a complication of this patient’s present condition?
A. DIC
B. IUGR
C. Subarachnoid hemorrhage 26
D. Vasa previa

Which of the following is the most likely cause of this patients abnormal lab tests (International
Normalized Ratio of 2.3, partial thromboplastin time of 48 seconds, D-dimer elevated,
fibrinogen decreased)?
A. Disruption of placenta and release of fetal tissue into circulation
B. Liver failure
C. Haemorrhagic shock
D. Release of thromboplastin by damaged placenta
E. Ruptured ectopic

Which of the following is the most appropriate first step in management?


A. Administer a tocolytic
B. Administer a corticosteroid.
C. Administer fresh frozen plasma.
D. Deliver the fetus immediately by LSCS
It
E. Observe closely.

Which of the following statements is accurate concerning postpartum hemorrhage?


A. The blood loss at delivery approaches the volume of blood added during pregnancy.
B. Studies show that estimated blood loss reported is often less than the actual blood loss.
C. The WHO defines postpartum hemorrhage as cumulative blood loss of >100 mL
accompanied by symptoms and signs of hypovolemia.
D. None of the above

Which of the following is true concerning placental abruption?


A. It may lead to a dilutional coagulopathy.
B. Concealed abruption forces thromboplastin into large veins draining the implantation site.
C. In most women with abruption severe enough to kill the fetus, the plasma fibrinogen level
will be less than 150 mg/dL.
D. All of the above

How is a low-lying placenta defined?


A. The placenta is implanted in the lower uterine segment.
B. The placental edge covers the internal os, but not completely.
C. The placental edge does not cover the internal os but lies within a 1-cm wide perimeter.
D. The placental edge does not cover the internal os but lies within a 2-cm wide perimeter.

What percentage of women who have a primary cesarean with a placenta previa have
hysterectomies
performed?
A. 1%
B. 2%
C. 6%
D. 10%

Which of the statements below accurately depicts the difference between consumptive
coagulopathy and disseminated intravascular coagulation?
A. Abruption best represents a consumptive coagulopathy.
B. Loss of procoagulants with massive hemorrhage is the basis of dilutional coagulopathy.
C. A concealed abruption forces thromboplastin into the circulation and leads to the
consumption of procoagulants.
D. All of the above

Which of the bacteria below are associated with consumptive coagulopathy?


A

A. Escherichia coli
B. Klebsiella pneumoniae
C. Group A streptococcus
D. Group B streptococcus
The use of whole blood for massive hemorrhage is supported by which of the following?
A. Less renal failure
B. Less pulmonary edema
C. Fewer intensive care unit admissions
D. All of the above
PICTURES!!!
1. Placenta previa
2. Placenta accreta
3. Couvelaire uterus
Postpartum Haemorrhage

» What is the most likely diagnosis?


A. Retained placenta
B. Genital tract laceration
C. Uterine atony
D. Uterine inversion
» What is the most likely complication to occur in this patient?
A. Postpartum hemorrhage
B. Uterine laceration
C. Coagulopathy
D. Hysterectomy
» Which of the following placental implantation sites would most likely predispose to an
inverted uterus?
A. Fundal
B. Anterior
C. Posterior
D. Lateral
E. Lower segment
» What is the most likely complication to occur in this patient?
A. Postpartum hemorrhage
B. Uterine laceration
C. Coagulopathy
D. Hysterectomy
» Which of the following is patient possible diagnosis?
A. Placenta accreta
B. Placental abruption
C. Placenta previa
D. Placental polyp
» Which of the following would be the next step for this patient?
A. Initiate oxytocin
B. Wait for an additional 30 minutes
C. Hysterectomy
D. Attempt a manual extraction of the placenta
E. Misoprostol estrogen intravaginally
» Which of the following is the best explanation of the mechanism of hemorrhage?
A. Inverted uterus stretches the uterus, causing trauma to blood vessels leading to bleeding.
B. Inverted uterus leads to inability for an adequate myometrial contraction effect.
C. Inverted uterus causes a local coagulopathy reaction to the uterus and endometrium.
D. Inverted uterus causes muscular abrasions and lacerations leading to bleeding.
» Which of the following is the most likely further complication?
A. Uterine atony
B. Cervical infection
C. Vaginal haematoma
D. Coagulopathic bleeding
» What is the most likely diagnosis?
A. Retained placenta
B. Genital tract laceration
C. Uterine atony
D. Uterine inversion
» What is the next step in therapy?
A. Uterotonics, uterine massage
B. Hypogastric artery ligation
C. Supracervical hysterectomy
D. Ligation of the external iliac artery
E. Cervical cerclage
» Which of the following is the most common etiology for the bleeding in this patient?
A. Retained placenta
B. Genital tract laceration
C. Uterine atony
D. Coagulopathy
E. Endometrial ulceration
» Which of the following is the best therapy?
A. Immediate hysterectomy
B. Packing of the uterus
C. Hypogastric artery ligation
D. Ligation of utero-ovarian ligaments
E. Fresh frozen plasma transfusion
Which of the following is the most likely cause of hemorrhage in this patient?
A. Uterine atony
B. Uterine laceration
C. Coagulopathy
D. Uterine inversion
E. Retained placenta

Which of the following is most appropriate to achieve the therapeutic goals?


A. Utero-ovarian ligament ligation
B. Hypogastric artery ligation
C. Supracervical hysterectomy
D. Ligation of the external iliac artery
E. Cervical cerclage
Which of the following is the best therapy?
A. Immediate hysterectomy
B. Packing of the uterus
C. Hypogastric artery ligation
D. Ligation of utero-ovarian ligaments
E. Correction of coagulopathy
Which of the following is the most likely diagnosis?
A. Uterine atony
B. Cervical laceration
C. Vaginal laceration
D. Subinvolution of the uterus
E. Normal menses
Which of the following is the most accurate statement?
A. Having two prior cesareans is associated with a 50% risk for placenta accreta.
B. Placenta accreta is associated with a defect in the myometrial layer of the uterus.
C. If the patient had gestational diabetes, the risk for placenta accreta would be even higher.
D. The posterior placenta may be associated with less of a risk for accrete than an anterior
placenta.
Which of the following is the most likely diagnosis?
A. Placenta accreta
B. Placenta melanoma
C. Placenta percreta
D. Placental polyp
Which of the following is the most likely complication after this intervention?
A. Coagulopathy
B. Utero-vaginal fistula
C. Infection
D. Malignant degeneration

Which of the following statements is most likely to be correct regarding the risk of placental
accreta?
A. Her risk of accreta is most likely to be increased due to the myomectomy.
B. Her risk of accreta is most likely to be decreased due to the myomectomy.
C. Her risk of accreta is most likely not affected by the myomectomy.
D. If the myomectomy incisions are anterior, then she has an increased risk of a placental
polyp.
________________ From Willam’s Obstetrics 25th (Ch.41)_______________

Which of the following statements is accurate concerning postpartum hemorrhage?


a. The blood loss at delivery approaches the volume of blood added during pregnancy.
b. Studies show that estimated blood loss reported is often less than the actual blood loss.
-
-

c. The American College of Obstetricians and Gynecologists defines postpartum hemorrhage as


cumulative blood loss of >100 mL accompanied by symptoms and signs of hypovolemia.
d. None of the above

Given the diagram below, which of the following statements is true?

a. Very few vaginal deliveries have blood loss less than 500 mL.
b. All cesarean hysterectomies have blood loss greater than 1000 mL.
c. A smaller percentage of vaginal deliveries have blood loss greater than 1000 mL than cesarean
hysterectomies have blood loss less than 500 mL.
d. None of the above

For a woman measuring 5′0″ and 120 lb, what is her expected pregravid blood volume?
a. 3000 mL
b. 3250 mL
c. 3500 mL
d. 3800 mL
Assuming a 50% increase in the blood volume of a woman during pregnancy, what would the
blood volume of a 5′2″ woman who weighed 140 lb pregravid be at term?
a. 4000 mL
b. 4340 mL
c. 4700 mL
d. 4930 mL

Causes of uterine atony include which of the following?


a. Obesity
b. Placenta previa
c. Multiple fetuses
d. Placental abruption
e. Previous urine infection

Which of the following maneuvers should be performed in the setting of postpartum hemorrhage
following a vaginal delivery?
a. Evaluate birth canal for lacerations
b. Evaluate the placenta for possible retained fragments
c. The uterus should be manually explored, and placental fragments removed
d. All of the above

@Choose the correct use of the uterotonics:


a. Ergot alkaloids for patients with hypertension
b. Carboprost tromethamine in patients with mild asthma
c. Dinoprostone for patients with diarrhea
d. Oxytocin in patient with HIV infection

What is represented in the following image?


a. Bakri balloon
b. Foley catheter
c. Blakemore tube
d. Jackson-Pratt drain

÷
Which of the following maneuvers might have avoided to be performed for this patient after
uterotonics are given?
a. Evaluate birth canal for lacerations
b. Evaluate the placenta for possible retained fragments
c. The uterus should be manually explored, and placental fragments removed
d. Hysterectomy

The patient with uterine atony continues bleeding after the interventions mentioned above. Which
of the following maneuvers might be employed while you are preparing for surgery?
a. Cervical cerclage
b. Uterine balloon placement (tamponade)
c. Wait for an additional 30 minutes
d. Transfusion of platelets
e.
What percentage of women have lacerations at the time of vaginal delivery?
a. 50%
b. 90%
c. 80%
d. 65%

What is the appropriate management of vulvovaginal hematomas?


a. Surgical exploration in all cases
b. To prevent infection they should all undergo ultrasound guided drainage.
c. In a small hematoma, if pain is severe then ice packs and analgesia are appropriate.
d. If bleeding ceases, small to moderate-sized hematomas can be treated expectantly.

The hematoma in the following picture is in which location?


a. Ischiorectal hematoma
b. Periurethral hematoma
c. Supralevator hematoma
d. Left-sided anterior perineal triangle

What percentage of women who have a primary cesarean with a placenta previa have
hysterectomies performed?
a. 1%
b. 2%
c. 6%
d. 10%

What is depicted in the following picture?


a. Placenta previa
b. Placenta accreta
c. Couvelaire uterus
d. None of the above

What is represented in the following image?


The picture below represents which of the following placental
pathologies?

a. Placenta increta
b. Placenta accreta
c. Placenta percreta
d. Placental abruption

Which hemostatic surgical procedure is to be performed in the picture below?

a. B-Lynch procedure
b. Uterine artery ligation
c. Vaginal artery ligation
d. Ovarian artery ligation

Which hemostatic surgical procedure is demonstrated in the picture below?


a. B-Lynch procedure
b. Uterine artery ligation
c. Vaginal artery ligation
d. Ovarian artery ligation
Ligation at which of the following labeled vessel points in the retroperitoneum will decrease the
pulse pressure in the uterine artery?

a. Posterior division
b. Internal iliac vein
c. Internal iliac artery
d. External iliac artery
Which of the following is a potential indication for a classical hysterotomy, as shown
below?

a. Maternal morbid obesity


b. Densely adherent bladder
c. Back-down transverse fetal lie
d. All of the above

Which of the following is a disadvantage of uterine exteriorization for repair of the


hysterotomy, as show
below?

a. Increased blood loss


b. Increased febrile morbidity
c. Increased nausea and vomiting
d. Increased postoperative infection

During cesarean delivery a hysterotomy is made in the


lower uterine segment, as shown here. In which of the
following settings should the incision be made relatively
higher on the uterus to avoid uterine vessel laceration or
unintended entry into the vagina?

a. Maternal anemia
b. Fetal malpresentation
c. A completely dilated cervix
d. Cesarean performed prior to onset of labor

Compared with manual extraction, spontaneous delivery


of the placenta with fundal massage, as shown below, has
been shown to reduce the risk of which complication?

a. Retained placenta
b. Postpartum infection
c. Deep-vein thrombosis
d. Amniotic fluid embolism
Preterm delivery

» What is the most likely diagnosis?


A. Preterm labor
B. Suspected placental abruption
C. Chorioamnionitis
D. Intrauterine growth retardation
E. Cervical incompetence
» What is your next step in management?
A. Expectant management
B. Intramuscular corticosteroids
C. Induction of labor
D. Ultrasound-guided amniocentesis
E. Tocolysis

» What test of the vaginal fluid prior to digital examination may indicate risk for preterm
delivery?
A. Fetal fibronectin assay
B. HPV positive status
C. Group B streptococcus of the vagina
D. Positive amniotest

» What medication can be given to decrease the risk of neurological impairment in the baby?
A. Magnesium sulfate may be given for pregnancies of <32 weeks when there is imminent
delivery
B. Intravenous antibiotics (ampicillin and gentamicin) and induction of labor
C. Antenatal corticosteroids (usually betamethasone intramuscular) prior to delivery
D. Intravenous heparin therapy for suspected placental insufficiency
E. Indomethacin for closure of ductus arteriosus

» What screening test need to be provided to estimate the risk for preterm delivery?
A. Fetal fibronectin assay
B. Cervical length <25 mm on ultrasound
C. HPV positive status
D. Recent laparotomy
E. Uterine fibroids

» In screening for various types of infection, which of the following is most likely to be
associated with preterm delivery?
A. Herpes simplex virus
B. Candida vaginitis
C. Chlamydia cervicitis
D. Gonococcal cervicitis
E. Group B streptococcus of the vagina

» Which one of the following is a contraindication for tocolysis?


A. Suspected placental abruption
B. Group B streptococcal bacteriuria
C. Recent laparotomy
D. Uterine fibroids

» What medication can be given to decrease the risk of neurological impairment in the baby?
A. Magnesium sulfate
B. Broad-spectrum antibiotic
C. IV heparin therapy
D. IV furosemide
E. Indomethacin.

»Which of the following is the most likely tocolytic agent used in patient with tachypnea treated
with terbutaline?
A. Nifedipine
B. Indomethacin
C. Magnesium sulfate
D. Terbutaline

» What are reasons for tocolysis in case of preterm contractions?


A. Prolong time to delivery
B. Delay labour in order for the steroids to be maximally effective
C. To settle contractions for in utero transfer of the mother to a tertiary unit
D. Increase the chance for natural birth

» What is the most likely diagnosis (PPROM and fever)?


A. Intra-Amniotic Infection
B. Thrombophlebitis
C. Preterm labor
D. Cervicitis

» What is the best management for this patient (PPROM and fever)?
A. Intravenous antibiotics
B. Expectant management
C. Intramuscular corticosteroids
D. Induction of labor
E. Ultrasound-guided amniocentesis
F. Tocolysis

» What is the most likely etiology of PPROM?


A. Deep venous thrombosis
B. Uterine fibroids
C. Ascending infection from vagina
D. Gestational hypertension

» Which of the following statements is correct?


A. Magnesium sulfate should be given for neuroprotectionin all cases.
B. Broad-spectrum antibiotic therapy is indicated only with maternal fever.
C. Labor is the most common acute complication to be expected.
D. Vaginal candidiasis is a risk factor for preterm premature rupture of membranes.

» Which of the following is the most accurate method to confirm the intra-amniotic infection?
A. Serum maternal leukocyte count
B. Speculum examination of the vaginal discharge
C. Amniotic fluid Gram stain by amniocentesis
D. Palpation of the maternal uterus
E. Height of oral temperature

»Which of the following organisms is most likely to be the underlying etiology of a preterm
delivery in case of clinical presentation of intraamniotic infection without PPROM?
A. Group B streptococci
B. Listeria monocytogenes
C. Clostridia difficile
D. Chlamydia trachomatis
E. Escherichia coli

»Which of the following is the next step?


A. Expectant management
B. Intramuscular corticosteroids
C. Induction of labor
D. Ultrasound-guided amniocentesis

»Which of the following is most likely to be seen?


A. Early decelerations
B. Late decelerations
C. Variable decelerations
D. Sinusoidal heart rate pattern

What is the definition of very low birthweight? (W25th)


A. <500 grams
B. 500 to 1000 grams
C. 1000 to 1500 grams
D. 1500 to 2500 grams

What is the approximate survival rate for infant weighing 1090 grams and immediately taken to
the neonatal intensive care unit for evaluation? (W25th)
A. 80%
B. 70%
C. 95%
D. 99%

An infant born at 25 weeks’ gestation is at risk for all except which of the following
complications?
A. Asthma
B. Blindness
C. Blood cancers
D. Pulmonary hypertension

Of the responses listed below, which complication is most commonly associated with preterm
delivery?
A. Hypertension
B. Fetal complications
C. Placental abruption
D. None of the above
Which of the following placental hormones may play a role in preterm birth caused by maternal–
fetal stress?
A. Estrogen
B. Human placental lactogen
C. Insulin-like growth hormone
D. Corticotropin-releasing hormone

Which of the following bacteria may enhance the risk for preterm birth by secretion of
hyaluronidase?
A. Escherichia coli
B. Peptostreptococcus
C. Bacterial vaginosis
D. Group B streptococcus

Which of the following bacteria is frequently detected in the amnionic fluid of women with
preterm labor?
A. Mycoplasma hominis
B. Gardnerella vaginalis
C. Ureaplasma urealyticum
D. All of the above

Which of the following is true regarding transvaginal sonographic evaluation of the cervix as a
part of the assessment for preterm labor?
A. It can be performed any time after 14 weeks’ gestation.
B. It is not affected by maternal obesity, cervix position, or shadowing.
C. The FIGO recommends it for all women with a history of a spontaneous preterm birth.
D. All of the above

Which of the following neonatal outcomes was potentially improved following antimicrobial
therapy for pregnant woman with premature rupture of membranes before 35 weeks’ gestation?
A. Improved survival
B. Decreased risk for sepsis
C. Decreased risk for intracranial hemorrhage
D. Decreased risk for respiratory distress syndrome

Which of the following is a potential consequence of bed rest for suspected preterm labor?
A. Bone loss
B. Venous thromboembolism
C. Increased risk for preterm delivery
D. All of the above

Which of the following is true regarding indomethacin use in pregnancy?


A. It can only be administered orally
B. It can lead to reversible oligohydramnios
C. It lowers the risk for necrotizing enterocolitis
D. All of the above

Cesarean delivery would be expected to decrease the risk for intracranial hemorrhage in which
of the following scenarios?
A. Estimated fetal weight <1000 grams
B. Estimated fetal weight <1500 grams
C. Estimated fetal weight <2000 grams
D. None of the above
Puerperium.
» Which of the following is the most significant risk factor in this patient?
A. Numerous vaginal examinations
B. Bacterial vaginosis
C. Cesarean delivery
D. Internal uterine pressure monitors
E. Prolonged rupture of membranes
» What is the best therapy for the condition?
A. Initiation of intravenous antibiotics with anaerobic coverage
B. Initiation of intravenous heparin
C. Placement of a warm compress on the wound
D. Opening of the wound
» What is the best therapy for the condition?
A. Initiation of intravenous antibiotics with anaerobic coverage
B. Initiation of intravenous heparin
C. Placement of a warm compress on the wound
D. Opening of the wound
» Which of the following is the most significant risk factor for postpartum endomyometritis?
A. Numerous vaginal examinations
B. Bacterial vaginosis
C. Cesarean delivery
D. Internal uterine pressure monitors
E. Prolonged rupture of membranes
» What is the best therapy for the condition?
A. Initiation of intravenous antibiotics with anaerobic coverage
B. Initiation of intravenous heparin
C. Placement of a warm compress on the wound
D. Opening of the wound
» The patient who works in the microbiology laboratory asks which of the following is the most
commonly isolated bacteria in her infection?
A. Peptostreptococcus species
B. Staphylococcus aureus
C. Group B Streptococcus
D. Escherichia coli
» Which of the following is the most significant risk factor in this patient?
A. Numerous vaginal examinations
B. Bacterial vaginosis
C. Cesarean delivery
D. Internal uterine pressure monitors
E. Prolonged rupture of membranes
» Which of the following is the best therapy for this condition?
A. Hysterectomy
B. Discontinue antibiotic therapy and initiate intravenous heparin
C. Continue antibiotic therapy and begin intravenous heparin
D. Surgical embolectomy
E. Streptokinase therapy
» What is the most likely diagnosis?
A. Abscess of the right breast
B. Galactocele
C. Inflammatory breast carcinoma
D. Mastitis
» What is your next step in therapy?
A. Incision and drainage of the abscess
B. Oral antibiotic therapy
C. Oral antifungal therapy
D. Bromocriptine therapy
E. Mastectomy
» What is the most likely diagnosis?
A. Abscess of the left breast
B. Galactocele
C. Inflammatory breast carcinoma
D. Mastitis
» Which of the following is the best treatment for this condition?
A. Oral antibiotic therapy
B. Oral antifungal therapy
C. Bromocriptine therapy
D. Aspiration
E. Mastectomy

What duration of time is encompassed in the puerperium?


a. 2–4 weeks
b. 4–6 weeks
c. 6–8 weeks
d. 10–12 weeks

What are myrtiform caruncles?


a. Vaginal rugae
b. Scarred tags of hymenal tissue
c. A sexually transmitted infection
d. Microscopic tears in the vaginal epithelium

At what point in the puerperium does the endocervical canal reform?


a. 1 week postpartum
b. 2 weeks postpartum
c. 3 weeks postpartum
d. 4 weeks postpartum

What percentage of women experience regression of high-grade dysplasia following delivery?


a. 10%
b. 33%
c. 35%
d. 50%

How long does complete uterine involution take following delivery?


a. 1 week
b. 2 weeks
c. 3 weeks
d. 4 weeks
Secondary postpartum hemorrhage is defined as uterine hemorrhage occurring during what
timeframe after delivery?
a. 24 hours to 6 weeks
b. 48 hours to 6 weeks
c. 24 hours to 12 weeks
d. 48 hours to 12 weeks

What are common features of the urinary bladder in the postpartum period?
a. Increased capacity
b. Incomplete emptying
c. Insensitivity to intravesical pressures
d. All of the above

Which of the following physical exam signs or laboratory values would be most helpful to make
a diagnosis of endometritis in one-day postpartum patient?
a. Fundal tenderness
b. An absolute neutrophilia
c. White blood cell count of 25,000/µL
d. All of the above
After delivery, how long do cardiovascular parameters including cardiac output, heart rate, and
blood pressure take to return to nonpregnant levels?
a. 48 hours
b. 72 hours
c. 7 days
d. 10 days

By which time interval following delivery most women first approach their prepregnancy
weight?
a. 3 months
b. 6 months
c. 9 months
d. 12 months

How long would postoperative (primary cesarean delivery) patient will expect colostrum
production before beginning her conversion to a more mature milk?
a. 2 to 5 days
b. 2 to 10 days
c. 5 to 14 days
d. 5 to 21 days

What vitamin is virtually absent in human breast milk?


a. Vitamin C
b. Vitamin A
c. Vitamin K
d. All vitamins are equally represented in breast milk

Which hormone is most responsible for milk expression during lactation?


a. Prolactin
b. Oxytocin
c. Dopamine
d. Progesterone

How would you advise a patient who is exclusively breastfeeding 4 weeks after delivery and
develops nipple fissures?
a. Wash the area with mild soap and water daily.
b. Apply a steroid cream and use a nipple shield temporarily.
c. Continue exclusive breastfeeding without interruption.
d. Do not allow infant to feed on the affected side and empty the breast regularly with a pump.

All except which of the following conditions are contraindications to breastfeeding?


a. Infant with galactosemia
b. Maternal hepatitis B infection
c. Human immunodeficiency virus infection
d. Maternal active and untreated tuberculosis
Which of the following is true regarding the postpartum blues?
a. May be affected by body image concerns
b. Usually lasts for no more than 10 days after delivery
c. Effective treatment involves recognition and reassurance
d. All of the above

Injury to what nerve is the most likely cause of the patient’s bilateral foot drop on day 1
following a labor with 3 hours of pushing in stirrups?
a. Femoral nerve
b. Ilioinguinal nerve
c. Common peroneal nerve
d. Lateral femoral cutaneous nerve
What is the mean time to ovulation resumption in a postpartum woman who elects not to
breastfeed?
a. 4 weeks
b. 5 weeks
c. 6 weeks
d. 7 weeks
What contraceptive option would you not recommend for this postpartum patient?
a. Progestin-only pills
b. Estrogen-progestin patch
c. Depot medroxyprogesterone
d. A levonorgestrel intrauterine device

Following an uncomplicated vaginal delivery, when can women be advised to resume coitus
based on desire and comfort?
a. 1 week postpartum
b. 2 weeks postpartum
c. 3 weeks postpartum
d. 4 weeks postpartum

What treatment can be offered to the postpartum woman who is exclusively breastfeeding and
complains of vaginal dryness during intercourse 7 weeks following delivery?
a. Topical estrogen
b. Reassurance only
c. Topical testosterone
d. Intercourse is not recommended within the first
8 weeks of delivery.

Approximately what percentage of postpartum women require readmission to the hospital within
8 weeks of delivery?
a. <1%
b. 3%
c. 7%
d. 10%

4 weeks postpartum patient is overall doing well but complains of low energy levels despite
the fact that her baby is sleeping for long intervals at night. What should you offer her?
a. Reassurance only
b. Psychiatry referral
c. Check her thyroid hormone levels
d. All of the above
MEDICAL COMPLICATIONS

Renal and Urinary Tract Disorders


CASE MP- 1
A 36-year-old G1P0 woman at 27 weeks’ gestation is noted to have fever, right flank
tenderness, and pyuria. She is diagnosed with pyelonephritis. A urine culture is performed.
» Which of the following is the most commonly isolated etiologic agent causing pyelonephritis in
pregnancy?
A. Proteus species
B. Candida species
C. Escherichia coli
D. Klebsiella species
» If patient does not improve on adequate antibiotic therapy for 48 hours and experiences continued
severe flank tenderness and fever, which of the following should be next considered?
A. Obstruction of the urinary tract
B. Anaerobic organisms
C. Hemolytic uremic syndrome
D. Factitious fever
» What severe condition is the most likely?
A. Acute respiratory distress syndrome
B. Septic shock
C. Hemolytic uremic syndrome
D. Factitious fever
» Which of the following is the most common cause of this complication in pregnancy?
A. Pelvic inflammatory disease
B. Pyelonephritis
C. Wound infection
D. Mastitis

________________ From Willam’s Obstetrics 25th (Ch.53)_______________


Which of the following is not a physiological change in pregnancy that contributes to the findings
in the image below?

a. Vesicoureteral reflux
b. Glomerular hypertrophy
c. Distal ureteral compression by the uterus
d. Progesterone-induced relaxation of the muscularis

Which of the following statements regarding physiological changes in pregnancy is true?


a. Glomerular filtration decreases.
b. Effective renal plasma flow increases.
c. Serum creatinine concentration increases.
d. The number of glomerular cells increases.

Which of the following is the threshold for proteinuria in pregnancy, above which levels are
considered
abnormal?
a. 100 mg/d
b. 250 mg/d
c. 300 mg/d
d. 1000 mg/d

Which of the following values is not a normal serum creatinine in pregnancy?


a. 0.4 mg/dL
b. 0.6 mg/dL
c. 0.8 mg/dL
d. 1.0 mg/dL

A pregnant patient who previously donated a kidney and now has only one healthy kidney is at risk
for
which of the following obstetric complications?
a. Placenta previa
b. Normocytic anemia
c. Gestational hypertension
d. Preterm premature rupture of membranes

Which of the following is an acceptable treatment regimen for newly diagnosed asymptomatic
bacteriuria in pregnancy?
a. Nitrofurantoin 100 mg by mouth at bedtime for 10 days
b. Ampicillin 250 mg by mouth four times per day for one day
c. Trimethoprim-sulfamethoxazole 160/800 mg by mouth one time
d. Nitrofurantoin 100 mg by mouth four times daily for 21 days

Lower urinary tract symptoms with pyuria but a sterile urine culture are likely due to which
pathogen?
a. Escherichia coli
b. Proteus mirabilis
c. Klebsiella pneumoniae
d. Chlamydia trachomatis

What is the leading cause of septic shock during pregnancy?


a. Pneumonia
b. Breast abscess
c. Pyelonephritis
d. Chorioamnionitis

In the setting of aggressive fluid hydration, what percentage of pregnant women with pyelonephritis
develop acute kidney injury?
a. 5%
b. 10%
c. 15%
d. 20%

Once you have treated a pregnant patient for pyelonephritis as an inpatient, for how long should
you
continue oral therapy as an outpatient?
a. 1–3 days
b. 3–5 days
c. 7–14 days
d. 14–21 days

What percentage of pregnant women experience a recurrent urinary tract infection after completion
of
treatment for pyelonephritis?
a. 5–10%
b. 10–20%
c. 30–40%
d. 50–60%

Which of the following does not characterize nephrotic syndromes?


a. Edema
b. Heavy proteinuria
c. Hyperalbuminemia
d. Hypercholesterolemia

Which of the following complications is not increased in pregnant patients with nephrotic
syndrome?
a. Anemia
b. Preeclampsia
c. Renal insufficiency
d. Postterm pregnancy

Which of the following is most likely to lead to end-stage renal disease?


a. Diabetes
b. Hypertension
c. Glomerulonephritis
d. Polycystic kidney disease
________________________________________________________________________________
___
Hematological Disorders

CASE MP-3
A 30-year-old G1P0 woman complains of nausea and vomiting for the first 3 months of her
pregnancy. On examination, her blood pressure (BP) is 110/70 mm Hg, heart rate (HR) 83 beats per
minute (bpm), and she is afebrile. The thyroid gland appears normal on palpation. The heart and
lung examinations are unremarkable. The fundus is at the symphysis level. The evaluation of the
anemia includes: hemoglobin level of 9.0 g/ dL and a mean corpuscular volume of 110 fL (normal
90-105 fL).

» Which of the following is the most likely etiology of the anemia?


A. Iron deficiency
B. Folate deficiency
C. Vitamin B12 deficiency
D. Physiologic anemia of pregnancy
E. Acute blood loss anemia
F. Anemia of chronic disease

CASE MP-4
A 29-year-old G2P1 woman at 28 weeks’ gestation, who had normal hemoglobin level 4 weeks ago
at her first prenatal visit, complains of 1 week of fatigue. Her antenatal history is unremarkable
except for a urinary tract infection treated with an antibiotic 1 weeks ago. On examination, her
blood pressure (BP) is 100/60 mm Hg, heart rate (HR) 90 beats per minute (bpm), and she is
afebrile. The thyroid gland appears normal on palpation. The heart and lung examinations are
unremarkable. The fundal height is 26 cm. The fetal heart tones are in the 140- to 150-bpm range.
The evaluation of the anemia includes: hemoglobin level of 7.0 g/ dL and a mean corpuscular
volume of 100 fL (normal 90-105 fL). She noted dark-colored urine after taking an antibiotic for a
urinary tract infection.

» Which of the following is the most likely diagnosis?


A. Iron deficiency anemia
B. Thalassemia
C. Hemolysis
D. Folate deficiency
E. Vitamin B12 deficiency
» Which of the following would you advise based on her clinical status?
A. Transfusion of 2 units of blood (NOT SURE!!!)
B. Repeat a complete blood count emergently
C. Three months of iron supplementation at discharge
D. All of the above

________________ From Willam’s Obstetrics 25th (Ch.56)_______________

What hemoglobin value is used to define anemia in the second trimester of pregnancy?
a. 9.5 g/dL
b. 10.0 g/dL
c. 10.5 g/dL
d. 11.0 g/dL
Which of the following adverse pregnancy outcomes is associated with anemia in pregnancy?
a. Stillbirth
b. Preeclampsia
c. Preterm birth
d. All of the above

What is the maternal iron requirement in pregnancy for a typical singleton gestation?
a. 500 mg
b. 800 mg
c. 1000 mg
d. 1200 mg

If you repeat a complete blood count one week after starting iron therapy, what would you expect to
see if she is compliant with taking her iron?
A. A 20% rise in her hematocrit
B. An elevated reticulocyte count
C. An increase in red cell distribution width
D. An increase in the mean corpuscular volume

Based on the picture below, what additional symptom is she most likely to complain of?

a. Dry skin
b. Anorexia
c. Hair loss
d. Excessive perspiration

Which of the following is not a cause of vitamin B12 deficiency encountered in pregnancy?
a. Crohn disease
b. Ulcerative colitis
c. History of Roux-en-Y gastric bypass
d. Prior ileal resection following trauma

Which of the following is true regarding paroxysmal nocturnal hemoglobinuria?


a. Hemoglobinuria develops at irregular intervals
b. Almost half of patients suffer venous thromboses
c. Often precipitated by transfusions, infection or surgery
d. All of the above

Which of the following obstetric complications is not increased in the context of sickle-cell disease
in pregnancy?
a. Stillbirth
b. Preeclampsia
c. Gestational diabetes
d. Fetal-growth restriction

Thrombocytopenia, defined as a platelet count <150,000/µL, complicates what percentage of


pregnancies?
a. 1%
b. 3%
c. 5%
d. 10%

Which of the following treatments would you consider as first-line therapy for the 29 weeks
pregnant patient with idiopathic thrombocytopenic purpura and platelet count 21,000/µL?
a. Splenectomy
b. Azathioprine
c. Corticosteroids
d. Intravenous immune globulin

What determines the factor VIII activity level in women affected with hemophilia A?
a. Mosaicism
b. Lyonization
c. Co-dominance
d. None of the above

What is the incidence of postpartum hemorrhage in women with von Willebrand disease?
a. 10%
b. 30%
c. 50%
d. 70%

Diabetes Mellitus

CASE MP - 5
A woman attends the antenatal day assessment unit to discuss the result of her glucose tolerance
test. She is 42 years old and this is her sixth pregnancy. She has previously had three caesarean
sections, one early miscarriage and a termination of pregnancy. All booking tests were normal as
were her 11–14-week and anomaly ultrasound scans. She is now 26 weeks’ gestation and her
midwife arranged a glucose tolerance test because of a family history of type 2 diabetes (her father
and paternal aunt). Her body mass index (BMI) is 31 kg/m2. Blood pressure is 146/87 mmHg. The
symphysiofundal height is 29 cm and the fetal heart rate is normal on auscultation.
Her urinalysis shows glycosuria ++. Glucose tolerance test (75 g glucose drink): Pretest fasting
blood glucose: 6.4 mmol/, 2 h blood glucose following glucose load: 11.2 mmol/L

» What is the diagnosis?


A. Gestational diabetes mellitus
B. Type 1 diabetes
C. Type 2 diabetes
D. Transient glycosuria

» Which of the following is a risk factor in pregnant women for impaired carbohydrate metabolism?
A. Family history of diabetes
B. Previous infant with polycystic kidney disease
C. High serum levels of antiphospholipid antibodies
D. All of the above
» What further investigations need to be arranged?
A. Retinal assessment if not performed in last 12 months
B. 50-g oral glucose tolerance test
C. Serum total cholesterol level
D. Quad screen to assess fetal risk of aneuploidy and neural-tube defect

» Outline the principles of management of the pregnancy.


A. Reduction of maternal insulin regime
B. Aim to keep fasting blood glucose between 3.5 and 5.9 mmol/L
C. Folate supplementation
D. Labor induction at 38 weeks’ gestation

________________ From Willam’s Obstetrics 25th (Ch.57)_______________


What does in utero exposure to hyperglycemia lead to?
a. Increase fetal fat cells
b. Fetal hyperinsulinemia
c. Insulin resistance in adolescence
d. All of the above

How is diabetes classified during pregnancy?


a. Type 1 or type 2
b. Pregestational or gestational
c. Type 1, type 2, or gestational
d. Using the White classification

Which of the following findings are considered diagnostic for overt diabetes in pregnancy?
a. Hemoglobin A1C >6%
b. Glucosuria on urine dip
c. Fasting plasma glucose ≥120 mg/dL
d. Random plasma glucose ≥200 mg/dL

Fetuses of overtly diabetic mothers have an increased risk for which of the following?
a. Preterm delivery
b. Spontaneous abortion
c. Congenital malformation
d. All of the above

How does diminished fetal growth occur in pregestational diabetes?


a. Substrate deprivation
b. Fetal hyperinsulinemia
c. Altered lipid metabolism
d. All of the above

Compared to women without diabetes, which fetal condition occurs more often in the setting of
pregestational diabetes?
a. Stillbirth
b. Perinatal death
c. Postpartum hemorrhage
d. Gestational hypertension

What might be said of the pregnancy yielding 5000-g newborn?


a. The mother likely had poor glycemic control.
b. The baby was at risk for neonatal hypoglycemia.
c. The mother had an increased risk for shoulder dystocia.
d. All of the above

Which of the following are considered reasons for unexplained fetal demise in women requiring
insulin during pregnancy?
a. Nonketotic acidosis
b. Elevated lactic acid levels
c. Decreased fetal hematocrit
d. None of the above

Which of the following is a reasonable explanation for hydramnios in a pregnancy complicated by


diabetes?
a. Maternal endothelial leak caused by hyperglycemia
b. Glucose reabsorption by the fetal glomerular collecting system
c. Osmotic gradient created by high glucose concentrations in the amnionic fluid
d. All of the above

What is the most likely cause for the increased incidence of respiratory distress syndrome in the
neonates of diabetic mothers?
a. Indicated preterm delivery
b. Delayed maturation of type II pneumocytes
c. Decreased production of surfactant in a hyperglycemic environment
d. All of the above

Which of the following statements regarding cardiomyopathy in infants of diabetic mothers is true?
a. Is reversible after birth
b. Ventricular hypertrophy is due to insulin excess
c. In severe cases may lead to pulmonary hypertension
d. In the first trimester systolic dysfunction is already present

Which of the following is true concerning management of overt maternal diabetes in the second
trimester?
a. Should undergo amniocentesis
b. Should undergo a fetal echocardiogram
c. Have higher rates of chromosomal abnormalities
d. Should be offered genetic screening if >35 years of age

All except which of the following statements regarding hypocalcemia in the newborn is accurate?
a. Defined as <9 mg/dL
b. Etiology is unexplained
c. May be related to preterm birth
d. Seen more often with strict glucose control

Maternal mortality in women with type 1 diabetes results from which of the following?
a. Infection
b. Hypertension
c. Diabetic ketoacidosis
d. All of the above

The first and most common visible retinal lesions in diabetes are small microaneurysms followed
by blot hemorrhages. This describes which of the following
conditions?
a. Proliferative retinopathy
b. Nonproliferative retinopathy
c. Is irreversible with improved control
d. The etiology of cotton wool exudates

What is the most important component of diabetic ketoacidosis treatment in pregnancy?


a. Restore euglycemia
b. Provide intravenous hydration
c. Provide intravenous potassium repletion
d. Provide intravenous bicarbonate to correct acidosis

Which of the following infections is increased in pregnant women with overt diabetes?
a. Pyelonephritis
b. Respiratory infections
c. Wound infection after cesarean delivery
d. All of the above

The most difficult aspect of preconceptional control in women with diabetes is which of the
following?
a. Resistance to insulin therapy
b. Unpredictable insulin requirements
c. Half of pregnancies in the United States are unplanned
d. All of the above

Women with type 1 diabetes should achieve glycemic control with which of the following during
pregnancy?
a. Insulin
b. Diet alone
c. Insulin and diet
d. Oral hypoglycemic agents

Which of the following is associated with fasting blood glucose levels >120 mg/dL (6.7 mmol/l)?
a. Preeclampsia
b. Cesarean delivery
c. Birthweight >90th percentile
d. All of the above
Which of the following is a potential indication for a classical hysterotomy, as shown
below?

a. Maternal morbid obesity


b. Densely adherent bladder
c. Back-down transverse fetal lie
d. All of the above

Which of the following is a disadvantage of uterine exteriorization for repair of the


hysterotomy, as show below?

a. Increased blood loss


b. Increased febrile morbidity
c. Increased nausea and vomiting
d. Increased postoperative infection

During cesarean delivery a hysterotomy is made in the lower uterine segment, as shown
here. In which of the following settings should the incision be made relatively higher on the
uterus to avoid uterine vessel laceration or unintended entry into the vagina?

a. Maternal anemia
b. Fetal malpresentation
c. A completely dilated cervix
d. Cesarean performed prior to onset of labor

Compared with manual extraction, spontaneous delivery of the placenta with fundal
massage, as shown below, has been shown to reduce the risk of which complication?
a. Retained placenta
b. Postpartum infection
c. Deep-vein thrombosis
d. Amniotic fluid embolism

Postpartum Haemorrhage

CASE PPH-1:After a 4-hour labor, a 31-year-old G4P3 woman undergoes an uneventful vaginal
delivery of a 7lb 8oz (3.4 kg) infant over an intact perineum. During her labor, she is noted to have
mild variable decelerations and accelerations that increase 20 beats per minute (bpm) above the
baseline heart rate. At delivery, the male baby has Apgar scores of 8 at 1 minute, and 9 at 5 minute.
Slight lengthening of the cord occurs after 28 minutes along with a small gush of blood per vagina.
As the placenta is being delivered, a shaggy, reddish, bulging mass is noted at the introitus around
the placenta.

» What is the most likely diagnosis?


E. Retained placenta
F. Genital tract laceration
G. Uterine atony
H. Uterine inversion
» What is the most likely complication to occur in this patient?
A. Postpartum hemorrhage
B. Uterine laceration
C. Coagulopathy
D. Hysterectomy

CASE PPH-2: A 23-year-old G1P0 woman at 38 weeks’ gestation delivered a 7lb 4oz (3.170 kg)
baby boy vaginally. Upon delivery of the placenta, there was noted to be an inverted uterus, which
was successfully managed including replacement of the uterus.

» Which of the following placental implantation sites would most likely predispose to an inverted
uterus?
E. Fundal
F. Anterior
G. Posterior
H. Lateral
I. Lower segment
» What is the most likely complication to occur in this patient?
A. Postpartum hemorrhage
B. Uterine laceration
C. Coagulopathy
D. Hysterectomy

CASE PPH-3: A 24-year-old woman underwent a normal vaginal delivery of a term infant female.
After the delivery, the placenta does not deliver even after 30 minutes.

» Which of the following is patient possible diagnosis?


A. Placenta accreta
B. Placental abruption
C. Placenta previa
D. Placental polyp
» Which of the following would be the next step for this patient?
A. Initiate oxytocin
B. Wait for an additional 30 minutes
C. Hysterectomy
D. Attempt a manual extraction of the placenta
E. Misoprostol estrogen intravaginally

CASE PPH-4:A 32-year-old G1P0 woman at 40 weeks’ gestation undergoes a normal vaginal
delivery. Delivery of the placenta is complicated by an inverted uterus, with subsequent hemorrhage
leading to 1500 mL of blood loss. She is managed with a transfusion of erythrocytes.

» Which of the following is the best explanation of the mechanism of hemorrhage?


A. Inverted uterus stretches the uterus, causing trauma to blood vessels leading to bleeding.
B. Inverted uterus leads to inability for an adequate myometrial contraction effect.
C. Inverted uterus causes a local coagulopathy reaction to the uterus and endometrium.
D. Inverted uterus causes muscular abrasions and lacerations leading to bleeding.
» Which of the following is the most likely further complication?
E. Uterine atony
F. Cervical infection
G. Vaginal haematoma
H. Coagulopathic bleeding

CASE PPH-5:A 29-year-old parous (G5P4) woman at 39 weeks’ gestation with preeclampsia
delivers vaginally. Her prenatal course has been uncomplicated except for asymptomatic bacteriuria
caused by Escherichia coli in the first trimester treated with oral cephalexin. She denies a family
history of bleeding diathesis. After the placenta is delivered, there is appreciable vaginal bleeding
estimated at 1000 ml. Under palpation uterus appears to be soft and on the level of umbilicus.

» What is the most likely diagnosis?


A. Retained placenta
B. Genital tract laceration
C. Uterine atony
D. Uterine inversion
» What is the next step in therapy?
A. Uterotonics, uterine massage
B. Hypogastric artery ligation
C. Supracervical hysterectomy
D. Ligation of the external iliac artery
E. Cervical cerclage

CASE PPH-6: A 26-year-old G2P1001 woman underwent a normal vaginal delivery. A viable 7 lb 4
oz (3.4 kg) male infant was delivered. The placenta delivered spontaneously. The obstetrician noted
significant blood loss from the vagina, totaling approximately 700 mL. The uterine fundus appeared
to be well contracted.

» Which of the following is the most common etiology for the bleeding in this patient?
A. Retained placenta
B. Genital tract laceration
C. Uterine atony
D. Coagulopathy
E. Endometrial ulceration

CASE PPH-7
A 34-year-old woman is noted to have significant uterine bleeding after a vaginal delivery
complicated by placenta abruption. She is noted to be bleeding from multiple venipuncture sites.

» Which of the following is the best therapy?


A. Immediate hysterectomy
B. Packing of the uterus
C. Hypogastric artery ligation
D. Ligation of utero-ovarian ligaments
E. Fresh frozen plasma transfusion
Which of the following is the most likely cause of hemorrhage in this patient?
A. Uterine atony
B. Uterine laceration
C. Coagulopathy
D. Uterine inversion
E. Retained placenta

Which of the following is most appropriate to achieve the therapeutic goals?


E. Utero-ovarian ligament ligation
F. Hypogastric artery ligation
G. Supracervical hysterectomy
H. Ligation of the external iliac artery
I. Cervical cerclage

A 34-year-old woman is noted to have significant uterine bleeding after a vaginal


delivery complicated by placenta abruption. She is noted to be bleeding from
multiple venipuncture sites.Which of the following is the best therapy?
A. Immediate hysterectomy
B. Packing of the uterus
C. Hypogastric artery ligation
D. Ligation of utero-ovarian ligaments
E. Correction of coagulopathy

A 26-year-old G2P2 woman underwent a normal vaginal delivery 10 days previously. She
comes into the doctor’s clinic complaining of a large amount of bright red bleeding
beginning since 5 PM the previous day.
Which of the following is the most likely diagnosis?
A. Uterine atony
B. Cervical laceration
C. Vaginal laceration
D. Subinvolution of the uterus
E. Normal menses

A 33-year-old G3P2002 woman who had two prior cesareans is currently at 38 weeks’
gestation. She is noted to have a posterior placenta. On ultrasound, there is evidence of
possible placenta accreta. The patient is counseled about the possible risk of need for
hysterectomy.
Which of the following is the most accurate statement?
A. Having two prior cesareans is associated with a 50% risk for placenta accreta.
B. Placenta accreta is associated with a defect in the myometrial layer of the uterus.
C. If the patient had gestational diabetes, the risk for placenta accreta would be even higher.
D. The posterior placenta may be associated with less of a risk for accrete than an anterior
placenta.

A 25-year-old woman at 34 weeks’ gestation is noted to have a placenta previa,


after she presented with vaginal bleeding and has undergone sonography. At 37
weeks, she has a scheduled cesarean. Upon cesarean section, bluish tissue
densely adherent between the uterus and maternal bladder is noted.
Which of the following is the most likely diagnosis?
A. Placenta accreta
B. Placenta melanoma
C. Placenta percreta
D. Placental polyp

A 29-year-old G1P0 woman at 39 weeks’ gestation delivered vaginally. Her placenta does
not deliver easily. A manual extraction of the placenta is attempted and the placenta
seems to be adherent to the uterus. A hysterectomy is contemplated, but the patient
refuses due to strongly desiring more children. The cord is ligated with suture as high as
possible. The patient is given the option of methotrexate therapy.
Which of the following is the most likely complication after this intervention?
E. Coagulopathy
F. Utero-vaginal fistula
G. Infection
H. Malignant degeneration

A 32-year-old woman undergoes myomectomy for symptomatic uterine fibroids, all of


which are subserosal. The endometrial cavity was not entered during the procedure.Which
of the following statements is most likely to be correct regarding the risk of placental
accreta?
A. Her risk of accreta is most likely to be increased due to the myomectomy.
B. Her risk of accreta is most likely to be decreased due to the myomectomy.
C. Her risk of accreta is most likely not affected by the myomectomy.
D. If the myomectomy incisions are anterior, then she has an increased risk of a placental polyp.
________________ From Willam’s Obstetrics 25th (Ch.41)_______________

Which of the following statements is accurate concerning postpartum hemorrhage?


a. The blood loss at delivery approaches the volume of blood added during pregnancy.
b. Studies show that estimated blood loss reported is often less than the actual blood loss.
c. The American College of Obstetricians and Gynecologists defines postpartum hemorrhage as
cumulative blood loss of >100 mL accompanied by symptoms and signs of hypovolemia.
d. None of the above

Given the diagram below, which of the following statements is true?

a. Very few vaginal deliveries have blood loss less than 500 mL.
b. All cesarean hysterectomies have blood loss greater than 1000 mL.
c. A smaller percentage of vaginal deliveries have blood loss greater than 1000 mL than cesarean
hysterectomies have blood loss less than 500 mL.
d. None of the above

For a woman measuring 5′0″ and 120 lb, what is her expected pregravid blood volume?
a. 3000 mL
b. 3250 mL
c. 3500 mL
d. 3800 mL

Assuming a 50% increase in the blood volume of a woman during pregnancy, what would the blood
volume of a 5′2″ woman who weighed 140 lb pregravid be at term?
a. 4000 mL
b. 4340 mL
c. 4700 mL
d. 4930 mL

Causes of uterine atony include which of the following?


E. Obesity
F. Placenta previa
G. Multiple fetuses
H. Placental abruption
I. Previous urine infection

Which of the following maneuvers should be performed in the setting of postpartum hemorrhage
following a vaginal delivery?
a. Evaluate birth canal for lacerations
b. Evaluate the placenta for possible retained fragments
c. The uterus should be manually explored, and placental fragments removed
d. All of the above
Choose the correct use of the uterotonics:
a. Ergot alkaloids for patients with hypertension
b. Carboprost tromethamine in patients with mild asthma
c. Dinoprostone for patients with diarrhea
d. Oxytocin in patient with HIV infection

What is represented in the following image?

a. Bakri balloon
b. Foley catheter
c. Blakemore tube
d. Jackson-Pratt drain

Which of the following maneuvers might have avoided to be performed for this patient after
uterotonics are given?
a. Evaluate birth canal for lacerations
b. Evaluate the placenta for possible retained fragments
c. The uterus should be manually explored, and placental fragments removed
d. Hysterectomy

The patient with uterine atony continues bleeding after the interventions mentioned above. Which of
the following maneuvers might be employed while you are preparing for surgery?
a. Cervical cerclage
b. Uterine balloon placement (tamponade)
c. Wait for an additional 30 minutes
d. Transfusion of platelets (NOT SURE!!!!)

What percentage of women have lacerations at the time of vaginal delivery?


a. 50%
b. 90%
c. 80%
d. 65%

What is the appropriate management of vulvovaginal hematomas?


a. Surgical exploration in all cases
b. To prevent infection they should all undergo ultrasound guided drainage.
c. In a small hematoma, if pain is severe then ice packs and analgesia are appropriate.
d. If bleeding ceases, small to moderate-sized hematomas can be treated expectantly.

The hematoma in the following picture is in which location?

a. Ischiorectal hematoma b. Periurethral hematoma


c. Supralevator hematoma
d. Left-sided anterior perineal triangle

What percentage of women who have a primary cesarean with a placenta previa have
hysterectomies performed?
a. 1%
b. 2%
c. 6%
d. 10%

What is depicted in the following picture?

a. Placenta previa
b. Placenta accreta
c. Couvelaire uterus
d. None of the above

What is represented in the following image?


The picture below represents which of the following placental pathologies?

a. Placenta increta
b. Placenta accreta
c. Placenta percreta
d. Placental abruption

Which hemostatic surgical procedure is to be performed in the picture below?

a. B-Lynch procedure
b. Uterine artery ligation
c. Vaginal artery ligation
d. Ovarian artery ligation

Which hemostatic surgical procedure is demonstrated in the picture below?


a. B-Lynch procedure
b. Uterine artery ligation
c. Vaginal artery ligation
d. Ovarian artery ligation

Ligation at which of the following labeled vessel points in the retroperitoneum will decrease the
pulse pressure in the uterine artery?

a. Posterior division
b. Internal iliac vein
c. Internal iliac artery
d. External iliac artery

Preeclampsia

CASE P-1
A 19-year-old G1P0 woman at 29 weeks’ gestation arrives to the hospital because of severe
dyspnea of 6 hours’ duration. Her prenatal course has been unremarkable, and she denies any
medical problems. Her blood pressure (BP) is 160/114 mm Hg, heart rate (HR) is 105 beats per
minute (bpm), respiratory rate (RR) is 40 breaths per minute and labored, and oxygen saturation is
90%. The fetal heart tones are in the range of 140 bpm. A urine protein to creatinine ratio is 0.6.
The serum alanine transaminase (ALT) is 84 IU/L (normal < 35) and aspartate transaminase (AST)
is 90 IU/L (normal < 35). The prenatal records show the following:
Gestational Age BP (mm Hg) Urine Protein FHT (bpm) Fundal Height (cm)
8 weeks 100/60 0 140
12 weeks 110/70 0 148
16 weeks 100/76 0 150
20 weeks 105/58 0 138 20
26 weeks 130/89 1+ 142 25

» What is the most likely diagnosis?


E. HELLP syndrome
F. Cardiac insufficiency
G. Electrolyte abnormalities
H. Aspiration
I. Severe Preeclampsia

» What is your immediate next step?


F. Improve oxygenation
G. Intravenous antihypertensive agents
H. Platelet transfusion
I. Intravenous immunoglobulin therapy
J. Intravenous Mannitol

CASE P-2
A 18-year-old G1P0 woman at 28 weeks’ gestation is admitted to the hospital with a blood pressure
of 160/ 110 mm Hg, elevated liver function tests, and a platelet count of 60 000/ µL. She complains
of a severe headache.

» Which of the following is the best management?


E. Oral antihypertensive therapy
F. Platelet transfusion
G. Magnesium sulfate therapy and induction of labor
H. Intravenous immunoglobulin therapy

» Which of the following will be the most common mechanism if seizures appear in this patient?
A. Electrolyte abnormalities
B. Intracerebral hemorrhage
C. Myocardial infarction
D. Aspiration

CASE P-3
A 33-year-old woman at 29 weeks’ gestation is noted to have a blood pressure of 150/ 90 mm Hg
and a protein/creatinine ratio of 0.6 (protein excretion is 300 mg in 24 hours). The platelet count,
liver function tests, and creatinine are normal.

» Which of the following is the best management for this patient?


A. Induction of labor
B. Cesarean section
C. Antihypertensive therapy
D. Expectant management

» Which of the following findings is most likely present in this patient to necessitate delivery?
E. Blood pressures persist in the range of 150/ 95 mm Hg
F. The patient reaches 32 weeks’ gestation
G. Patient develops pulmonary edema
H. Repeat platelet count is 95 000/mm3

CASE P-4
On postpartum day 1, a 28-year-old G1P1 woman reports some headache and problems with her
vision bilaterally. Her BP is 150/ 95 mm Hg and P/C ratio is 0.5. Her neurological examination is
normal but her vision is impaired in both eyes.

» Which of the following is the best next step?


A. Antihypertensive agent
B. IV Mannitol
C. MRI of the brain
D. CT imaging of the brain
E. Ophthalmic eye drops to both eyes
CASE P-5
A 32-year-old G2P1 woman is at 35 weeks with chronic hypertension. The BP is in the 140/ 95
range.
» Which of the following is the best next step?
A. Corticosteroids
B. Antihypertensive agent
C. Biophysical profile
D. Magnesium sulfate and delivery
E. Continued observation

CASE P-6
A 28-year-old G1P0 woman is at 30 weeks’ gestation with superimposed preeclampsia. The BP is
150/ 100. The platelet count is 95 000 and LFT is 2× normal. BPP is 10/ 10.
» Which of the following is the best next step?
A. Corticosteroids
B. Antihypertensive agent
C. Biophysical profile
D. Magnesium sulfate and delivery
E. Continued observation

CASE P-7
A 30-year-old G2P1 woman is at 31 weeks with chronic hypertension, using oral labetolol. Her BP
in the office is 160/95 and 162/90. The urine protein is negative.
» Which of the following is the best next step?
A. Corticosteroids
B. Antihypertensive agent
C. Biophysical profile
D. Magnesium sulfate and delivery
E. Continued observation

CASE P-8
A 24-year-old G3P2 woman at 34 weeks’ gestation is noted to have preeclampsia. The BP is 150/
90 and P/ C ratio is 0.5. A fetal ultrasound shows the estimated fetal weight is at the 8th percentile.
» Which of the following is the best next step?
A. Corticosteroids
B. Antihypertensive agent
C. Biophysical profile

What percentage of pregnancies are complicated by hypertension?


e. 2–3%
f. 4–5%
g. 5–10%
h. 10–20%

What percentage of eclamptic seizures occur in women without significant proteinuria?


J. 5–9%
K. 10–17%
L. 20–25%
M. 30–33%

What percentage of eclamptic seizures occur more than 48 hours after delivery?
E. 2%
F. 5%
G. 10%
H. 20%

Many conditions and factors are associated with an increased risk for preeclampsia. Which of the
following factors results in the greatest relative risk for a diagnosis of preeclampsia in the current
pregnancy?
A. Primigravida
B. Advanced maternal age
C. Systemic lupus erythematous
D. History of preeclampsia in a prior pregnancy

Which of the following is thought to play a significant role in the development of preeclampsia?
e. Genetic factors
f. Immunological factors
g. Abnormal trophoblastic invasion
h. All of the above

Which of the following is not a pathophysiological change to the cardiovascular system seen in the
setting of preeclampsia?
E. Increased preload
F. Decreased preload
G. Increased afterload
H. Endothelial activation

Which statement best describes renal perfusion and glomerular filtration rates in women with
preeclampsia that has not yet progressed to severe disease?
J. Similar compared to normal pregnant values
K. Increased compared to normal pregnant values
L. Similar compared to normal nonpregnant values
M. Decreased compared to normal nonpregnant values

What is the underlying etiology of the proteinuria seen with preeclampsia?


F. Increased capillary permeability
G. Increased renal artery resistance
H. Increased glomerular filtration rate
I. Increased systemic vascular resistance

Which of the following nutritional supplements has been shown to reduce the incidence of
preeclampsia?
F. Calcium
G. Vitamin E
H. Ascorbic acid
I. None of the above

Which of the following physiological responses is typically seen in preeclamptic patients?


E. Increased production of nitric acid
F. Increased sensitivity to angiotensin II
G. Decreased reactivity to norepinephrine
H. All of the above

The typical blood volume of a gravida at term is 4500 mL. In patients with preeclampsia, which of
the following would be the expected blood volume?
E. 2500 mL
F. 3200 mL
G. 4500 mL
H. 5000 mL

Which of the following medical therapies has been associated with a modest decrease in the
incidence of preeclampsia in women at increased risk?
I. Aspirin
J. Metformin
K. Enoxaparin
L. Heparin sulfate

_________________From William’s Obstetric


th
25 _____________________________

Many conditions and factors are associated with an increased risk for preeclampsia. Which of the
following factors results in the greatest relative risk for a diagnosis of preeclampsia in the current
pregnancy?
a. Primigravida
b. Advanced maternal age
c. Systemic lupus erythematous
d. History of preeclampsia in a prior pregnancy

Which of the following is thought to play a significant role in the development of preeclampsia?
a. Genetic factors
b. Immunological factors
c. Abnormal trophoblastic invasion
d. All of the above

Which of the following angiogenic factors are elevated in women who proceed to develop
preeclampsia?
a. sFlt-1
b. PIGF
c. VEGF
d. TGF-β

A 21-year-old primigravida presents at 36 weeks’ gestation with new-onset headache. Her blood
pressure is 150/90 mm Hg, her serum creatinine is 0.8 mg/mL, AST is 32 U/L, and platelet count is
28,000/µL. Which of the following criteria for severe preeclampsia is met?
a. Hypertension
b. Liver dysfunction
c. Thrombocytopenia
d. Elevated serum creatinine

Which of the following is an indication for primary cesarean delivery without an attempt at
induction?
a. Malpresentation
b. Unfavorable cervix
c. Possible fetal thrombocytopenia
d. None of the above

Which statement best describes renal perfusion and glomerular filtration rates in women with
preeclampsia that has not yet progressed to severe disease?
a. Similar compared to normal pregnant values
b. Increased compared to normal pregnant values
c. Similar compared to normal nonpregnant values
d. Decreased compared to normal nonpregnant values

Patient is diagnosed with HELLP syndrome and undergoes an uncomplicated induction of labor.
Her right upper quadrant pain persists, and a computed tomography scan of her abdomen/pelvis is
completed with the findings as shown below. What is denoted by the asterisk (*)?

a. Splenic infarction
b. Intrahepatic infarction
c. Subcapsular hematoma
d. Periportal hemorrhagic necrosis

A G3P2 at 16 weeks’ gestation presents to the emergency department complaining of vaginal


bleeding. Her blood pressure is 148/96 mm Hg and she has 3+ proteinuria on a specimen obtained
during bladder catheterization. Which of the following may explain the development of
preeclampsia in this patient?
a. Increased volume of chorionic villi
b. Extensive remodeling of the spiral arterioles
c. Increased invasion of extravillous trophoblastic tissue
d. None of the above
The following computed tomography image shows cerebral edema in a postpartum patient with
hypertension, vision changes, and confusion. What associated morbidity is the patient at risk for?

a. Hemiplegia
b. Cystic leukomalacia
c. Retinal artery occlusion
d. Transtentorial herniation
The below magnetic resonance imaging was performed postpartum. What is the most likely finding
on imaging?

a. Subdural hematoma
b. Parenchymal hemorrhage
c. Subarachnoid hemorrhage
d. Posterior reversible encephalopathy syndrome

Preterm delivery

CASE PD-1
A healthy 19-year-old G1P0 woman at 29 weeks’ gestation presents to the labor and delivery area
complaining of intermittent abdominal pain. She denies leakage of fluid or bleeding per vagina. Her
antenatal history has been unremarkable.
She has been eating and drinking normally. On examination, her blood pressure (BP) is 110/70 mm
Hg, heart rate (HR) is 90 beats per minute (bpm), and temperature is 99°F (37.2°C). The fetal heart
rate tracing reveals a baseline heart rate of 120 bpm and a reactive pattern. Uterine contractions are
occurring every 3 to 5 minutes. On pelvic examination, her cervix is 3 cm dilated, 90% effaced, and
the fetal vertex is presenting at –1 station.

» What is the most likely diagnosis?


I. Preterm labor
J. Suspected placental abruption
K. Chorioamnionitis
L. Intrauterine growth retardation
M. Cervical incompetence

» What is your next step in management?


A. Expectant management
B. Intramuscular corticosteroids
C. Induction of labor
D. Ultrasound-guided amniocentesis
E. Tocolysis

» What test of the vaginal fluid prior to digital examination may indicate risk for preterm delivery?
e. Fetal fibronectin assay
f. HPV positive status
g. Group B streptococcus of the vagina
h. Positive amniotest

» What medication can be given to decrease the risk of neurological impairment in the baby?
E. Magnesium sulfate may be given for pregnancies of <32 weeks when there is imminent
delivery
F. Intravenous antibiotics (ampicillin and gentamicin) and induction of labor
G. Antenatal corticosteroids (usually betamethasone intramuscular) prior to delivery
H. Intravenous heparin therapy for suspected placental insufficiency
I. Indomethacin for closure of ductus arteriosus

CASE PD-2
A 26-year-old woman is noted to be at 29 weeks’ gestation. Her last pregnancy ended in delivery at
30 weeks’ gestation. On examination, her temperature is 96.8°F (36.6°C), blood pressure (BP) is
100/60 mm Hg, and heart rate (HR) is 80 beats per minute (bpm). Her lungs are clear to
auscultation. No costovertebral angle tenderness is found. The uterine fundal height is 30 cm, and
the uterus is slightly tender to palpation. No lower extremity cords are palpated. The fetal heart
tones are persistently in the range of 150 to 155 bpm without decelerations. There are no uterine
contractions.

» What screening test need to be provided to estimate the risk for preterm delivery?
G. Fetal fibronectin assay
H. Cervical length <25 mm on ultrasound
I. HPV positive status
J. Recent laparotomy
K. Uterine fibroids

» In screening for various types of infection, which of the following is most likely to be associated
with preterm delivery?
A. Herpes simplex virus
B. Candida vaginitis
C. Chlamydia cervicitis
D. Gonococcal cervicitis
E. Group B streptococcus of the vagina

CASE PD-3
A 25-year-old G1P0 woman is at 28 weeks’ gestation. She is noted to have regular uterine
contractions, and her cervix is dilated at 2 cm and 80% effaced.
Preterm labor is diagnosed. The physician reviews the record and notes that the patient should not
have tocolytic therapy.
» Which one of the following is a contraindication for tocolysis?
A. Suspected placental abruption
B. Group B streptococcal bacteriuria
C. Recent laparotomy
D. Uterine fibroids

» What medication can be given to decrease the risk of neurological impairment in the baby?
I. Magnesium sulfate
J. Broad-spectrum antibiotic
K. IV heparin therapy
L. IV furosemide
M. Indomethacin.

CASE PD-4
A 35-year-old G1P0 woman at 32 weeks’ gestation was seen in the obstetric (OB) triage unit the
previous day with uterine contractions. On admission, the fetal heart rate is 140 bpm with
accelerations and no decelerations. A fetal fibronectin assay is performed, which was positive. Over
the course of the next 24 hours, the patient was examined and noted to have cervical dilation from 1
to 2 cm and effacement from 30% to 90%. A tocolytic agent is used. A repeat fetal heart rate pattern
reveals a baseline of 140 bpm with moderate repetitive variable decelerations.

»Which of the following is the most likely tocolytic agent used in patient with tachypnea treated
with terbutaline?
A. Nifedipine
B. Indomethacin
C. Magnesium sulfate
D. Terbutaline

» What are reasons for tocolysis in case of preterm contractions?


N. Prolong time to delivery
O. Delay labour in order for the steroids to be maximally effective
P. To settle contractions for in utero transfer of the mother to a tertiary unit
Q. Increase the chance for natural birth

» What is the most likely diagnosis (PPROM and fever)?


A. Intra-Amniotic Infection
B. Thrombophlebitis
C. Preterm labor
D. Cervicitis

» What is the best management for this patient (PPROM and fever)?
F. Intravenous antibiotics (NOT SURE)
G. Expectant management
H. Intramuscular corticosteroids
I. Induction of labor
J. Ultrasound-guided amniocentesis
K. Tocolysis

» What is the most likely etiology of PPROM?


E. Deep venous thrombosis
F. Uterine fibroids
G. Ascending infection from vagina
H. Gestational hypertension

» Which of the following statements is correct?


A. Magnesium sulfate should be given for neuroprotectionin all cases.
B. Broad-spectrum antibiotic therapy is indicated only with maternal fever.
C. Labor is the most common acute complication to be expected.
D. Vaginal candidiasis is a risk factor for preterm premature rupture of membranes.

» Which of the following is the most accurate method to confirm the intra-amniotic infection?
A. Serum maternal leukocyte count
B. Speculum examination of the vaginal discharge
C. Amniotic fluid Gram stain by amniocentesis
D. Palpation of the maternal uterus
E. Height of oral temperature

»Which of the following organisms is most likely to be the underlying etiology of a preterm
delivery in case of clinical presentation of intraamniotic infection without PPROM?
A. Group B streptococci
B. Listeria monocytogenes
C. Clostridia difficile
D. Chlamydia trachomatis
E. Escherichia coli

»Which of the following is the next step?


A. Expectant management
B. Intramuscular corticosteroids
C. Induction of labor
D. Ultrasound-guided amniocentesis

»Which of the following is most likely to be seen?


I. Early decelerations
J. Late decelerations
K. Variable decelerations
L. Sinusoidal heart rate pattern

What is the definition of very low birthweight? (W25th)


i. <500 grams
j. 500 to 1000 grams
k. 1000 to 1500 grams
l. 1500 to 2500 grams

What is the approximate survival rate for infant weighing 1090 grams and immediately taken to the
neonatal intensive care unit for evaluation? (W25th)
E. 80%
F. 70%
G. 95%
H. 99%

An infant born at 25 weeks’ gestation is at risk for all except which of the following complications?
J. Asthma
K. Blindness
L. Blood cancers
M. Pulmonary hypertension

Of the responses listed below, which complication is most commonly associated with preterm
delivery?
A. Hypertension
B. Fetal complications
C. Placental abruption
D. None of the above

Which of the following placental hormones may play a role in preterm birth caused by
maternal–fetal stress?
A. Estrogen
B. Human placental lactogen
C. Insulin-like growth hormone
D. Corticotropin-releasing hormone

Which of the following bacteria may enhance the risk for preterm birth by secretion of
hyaluronidase?
A. Escherichia coli
B. Peptostreptococcus
C. Bacterial vaginosis
D. Group B streptococcus
Which of the following bacteria is frequently detected in the amnionic fluid of women with preterm
labor?
J. Mycoplasma hominis
K. Gardnerella vaginalis
L. Ureaplasma urealyticum
M. All of the above

Which of the following is true regarding transvaginal sonographic evaluation of the cervix as a part
of the assessment for preterm labor?
M. It can be performed any time after 14 weeks’ gestation.
N. It is not affected by maternal obesity, cervix position, or shadowing.
O. The FIGO recommends it for all women with a history of a spontaneous preterm birth.
P. All of the above

Which of the following neonatal outcomes was potentially improved following antimicrobial
therapy for pregnant woman with premature rupture of membranes before 35 weeks’ gestation?
A. Improved survival
B. Decreased risk for sepsis
C. Decreased risk for intracranial hemorrhage
D. Decreased risk for respiratory distress syndrome

Which of the following is a potential consequence of bed rest for suspected preterm labor?
I. Bone loss
J. Venous thromboembolism
K. Increased risk for preterm delivery
L. All of the above

Which of the following is true regarding indomethacin use in pregnancy?


A. It can only be administered orally
B. It can lead to reversible oligohydramnios
C. It lowers the risk for necrotizing enterocolitis
D. All of the above

Cesarean delivery would be expected to decrease the risk for intracranial hemorrhage in which of
the following scenarios?
I. Estimated fetal weight <1000 grams
J. Estimated fetal weight <1500 grams
K. Estimated fetal weight <2000 grams
L. None of the above

Puerperium.
CASE Pu-1
A 24-year-old G1P1 woman underwent a low-transverse cesarean section 2 days ago for arrest of
active phase of labor. The past medical and surgical histories were unremarkable. She denies a
cough or dysuria. On examination, the temperature is 102°F (38.8°C), heart rate is 80 beats per
minute, blood pressure is 120/70 mm Hg, and respiratory rate is 12 breaths per minute. The breasts
are nontender. The lungs are clear to auscultation. The abdomen reveals that the skin incision is
without erythema or tenderness. The uterine fundus is firm, at the level of the umbilicus, and
somewhat tender. No lower extremity cords are palpated.

» Which of the following is the most significant risk factor in this patient?
I. Numerous vaginal examinations
J. Bacterial vaginosis
K. Cesarean delivery
L. Internal uterine pressure monitors
M. Prolonged rupture of membranes
» What is the best therapy for the condition?
m. Initiation of intravenous antibiotics with anaerobic coverage
n. Initiation of intravenous heparin
o. Placement of a warm compress on the wound
p. Opening of the wound

CASE Pu-2
A 30-year-old G1P1 who underwent a cesarean section 3 days previously has a fever of 101°F
(38.3°C). The skin incision is indurated, tender, and erythematous. Which of the following is the
best management?

» What is the best therapy for the condition?


J. Initiation of intravenous antibiotics with anaerobic coverage
K. Initiation of intravenous heparin
L. Placement of a warm compress on the wound
M. Opening of the wound

CASE Pu-3
A 29-year-old woman is diagnosed with postpartum endometritis based on fever, abdominal pain,
fundal tenderness, and elimination of other etiologies.

» Which of the following is the most significant risk factor for postpartum endomyometritis?
Q. Numerous vaginal examinations
R. Bacterial vaginosis
S. Cesarean delivery
T. Internal uterine pressure monitors
U. Prolonged rupture of membranes
» What is the best therapy for the condition?
A. Initiation of intravenous antibiotics with anaerobic coverage
B. Initiation of intravenous heparin
C. Placement of a warm compress on the wound
D. Opening of the wound

CASE Pu-4
A 27-year-old G1P0 woman at 39 weeks’ gestation is noted to be in labor. She underwent artificial
rupture of membranes, and experiences fetal bradycardia. Palpation of the vagina reveals a rope-
like structure prolapsing through the cervix. She is diagnosed with a cord prolapse and underwent
stat cesarean delivery. On postoperative day 2, the patient has a temperature of 102°F (38.8°C), and
is diagnosed with endometritis.

» The patient who works in the microbiology laboratory asks which of the following is the most
commonly isolated bacteria in her infection?
A. Peptostreptococcus species
B. Staphylococcus aureus
C. Group B Streptococcus
D. Escherichia coli
» Which of the following is the most significant risk factor in this patient?
N. Numerous vaginal examinations
O. Bacterial vaginosis
P. Cesarean delivery
Q. Internal uterine pressure monitors
R. Prolonged rupture of membranes

CASE Pu-5
A 22-year-old woman who underwent cesarean delivery has persistent fever of 102°F (38.8°C),
despite the use of triple antibiotic therapy (ampicillin, gentamicin, and clindamycin). The
urinalysis, wound, breasts, and uterine fundus are normal on examination. A CT scan of the pelvis
is suggestive of septic pelvic thrombophlebitis.

» Which of the following is the best therapy for this condition?


A. Hysterectomy
B. Discontinue antibiotic therapy and initiate intravenous heparin
C. Continue antibiotic therapy and begin intravenous heparin
D. Surgical embolectomy
E. Streptokinase therapy

CASE Pu-6
A 20-year-old parous woman complains of right breast pain and fever. She states that 3 weeks
previously, she underwent a normal spontaneous vaginal delivery. She had been breastfeeding
without difficulty until 2 days ago, when she noted progressive pain, induration, and redness to the
right breast. On examination, her temperature is 102°F (38.8°C), blood pressure is 100/70 mm Hg,
and heart rate is 110 beats per minute. Her neck is supple. Her right breast has induration on the
upper outer region with redness and tenderness. There is also significant fluctuance noted in the
breast tissue. The abdomen is nontender and there is no costovertebral
angle tenderness. The pelvic examination is unremarkable.

» What is the most likely diagnosis?


N. Abscess of the right breast
O. Galactocele
P. Inflammatory breast carcinoma
Q. Mastitis
» What is your next step in therapy?
L. Incision and drainage of the abscess
M. Oral antibiotic therapy
N. Oral antifungal therapy
O. Bromocriptine therapy
P. Mastectomy

CASE Pu-7
A 28-year-old G1P1 woman has delivered vaginally 3 weeks ago. She is breastfeeding and notes
that the baby prefers to breastfeed from the right breast. On the left breast, she notes a 3-day history
of a tender mass on the upper outer quadrant. On examination, she is afebrile. The left breast has a
fluctuant mass of 4 × 8 cm of the upper outer quadrant without redness. It is somewhat tender.

» What is the most likely diagnosis?


I. Abscess of the left breast
J. Galactocele
K. Inflammatory breast carcinoma
L. Mastitis
» Which of the following is the best treatment for this condition?
N. Oral antibiotic therapy
O. Oral antifungal therapy
P. Bromocriptine therapy
Q. Aspiration
R. Mastectomy

What duration of time is encompassed in the puerperium?


a. 2–4 weeks
b. 4–6 weeks
c. 6–8 weeks
d. 10–12 weeks

What are myrtiform caruncles?


a. Vaginal rugae
b. Scarred tags of hymenal tissue
c. A sexually transmitted infection
d. Microscopic tears in the vaginal epithelium

At what point in the puerperium does the endocervical canal reform?


a. 1 week postpartum
b. 2 weeks postpartum
c. 3 weeks postpartum
d. 4 weeks postpartum

What percentage of women experience regression of high-grade dysplasia following delivery?


a. 10%
b. 33%
c. 35%
d. 50%

How long does complete uterine involution take following delivery?


a. 1 week
b. 2 weeks
c. 3 weeks
d. 4 weeks
Secondary postpartum hemorrhage is defined as uterine hemorrhage occurring during what
timeframe after delivery?
a. 24 hours to 6 weeks
b. 48 hours to 6 weeks
c. 24 hours to 12 weeks
d. 48 hours to 12 weeks

What are common features of the urinary bladder in the postpartum period?
a. Increased capacity
b. Incomplete emptying
c. Insensitivity to intravesical pressures
d. All of the above

Which of the following physical exam signs or laboratory values would be most helpful to make a
diagnosis of endometritis in one-day postpartum patient?
a. Fundal tenderness
b. An absolute neutrophilia
c. White blood cell count of 25,000/µL
d. All of the above

After delivery, how long do cardiovascular parameters including cardiac output, heart rate, and
blood pressure take to return to nonpregnant levels?
a. 48 hours
b. 72 hours
c. 7 days
d. 10 days

By which time interval following delivery most women first approach their prepregnancy weight?
a. 3 months
b. 6 months
c. 9 months
d. 12 months

How long would postoperative (primary cesarean delivery) patient will expect colostrum
production before beginning her conversion to a more mature milk?
a. 2 to 5 days
b. 2 to 10 days
c. 5 to 14 days
d. 5 to 21 days

What vitamin is virtually absent in human breast milk?


a. Vitamin C
b. Vitamin A
c. Vitamin K
d. All vitamins are equally represented in breast milk

Which hormone is most responsible for milk expression during lactation?


a. Prolactin
b. Oxytocin
c. Dopamine
d. Progesterone

How would you advise a patient who is exclusively breastfeeding 4 weeks after delivery and
develops nipple fissures?
a. Wash the area with mild soap and water daily.
b. Apply a steroid cream and use a nipple shield temporarily.
c. Continue exclusive breastfeeding without interruption.
d. Do not allow infant to feed on the affected side and empty the breast regularly with a pump.

All except which of the following conditions are contraindications to breastfeeding?


a. Infant with galactosemia
b. Maternal hepatitis B infection
c. Human immunodeficiency virus infection
d. Maternal active and untreated tuberculosis
Which of the following is true regarding the postpartum blues?
a. May be affected by body image concerns
b. Usually lasts for no more than 10 days after delivery
c. Effective treatment involves recognition and reassurance
d. All of the above

Injury to what nerve is the most likely cause of the patient’s bilateral foot drop on day 1 following a
labor with 3 hours of pushing in stirrups?
a. Femoral nerve
b. Ilioinguinal nerve
c. Common peroneal nerve
d. Lateral femoral cutaneous nerve

What is the mean time to ovulation resumption in a postpartum woman who elects not to
breastfeed?
a. 4 weeks
b. 5 weeks
c. 6 weeks
d. 7 weeks
What contraceptive option would you not recommend for this postpartum patient?
a. Progestin-only pills
b. Estrogen-progestin patch
c. Depot medroxyprogesterone
d. A levonorgestrel intrauterine device

Following an uncomplicated vaginal delivery, when can women be advised to resume coitus based
on desire and comfort?
a. 1 week postpartum
b. 2 weeks postpartum
c. 3 weeks postpartum
d. 4 weeks postpartum

What treatment can be offered to the postpartum woman who is exclusively breastfeeding and
complains of vaginal dryness during intercourse 7 weeks following delivery?
a. Topical estrogen
b. Reassurance only
c. Topical testosterone
d. Intercourse is not recommended within the first
8 weeks of delivery.

Approximately what percentage of postpartum women require readmission to the hospital within 8
weeks of delivery?
a. <1%
b. 3%
c. 7%
d. 10%

4 weeks postpartum patient is overall doing well but complains of low energy levels despite
the fact that her baby is sleeping for long intervals at night. What should you offer her?
a. Reassurance only
b. Psychiatry referral
c. Check her thyroid hormone levels
d. All of the above

Antenatal bleeding

CASE AB-5:A 22-year-old G1P0 woman at 34 weeks’ gestation presents with moderate vaginal
bleeding and no uterine contractions. Her blood pressure (BP) is 110/ 60 mm Hg and heart rate
(HR) 103 beats per minute (bpm). The abdomen is nontender.

» Which of the following sequence of examinations is most appropriate?


Q. Speculum examination, ultrasound examination
R. Ultrasound examination, digital examination
S. Digital examination, ultrasound examination, speculum examination
T. Ultrasound examination, speculum examination, digital examination

CASE AB-1: A 30-year-old G5P4 woman at 32 weeks’ gestation complains of bright red
vaginal bleeding. She denies uterine contractions, leakage of fluid, or trauma. The patient states that
4 weeks previously, after she had engaged in sexual intercourse, she experienced some vaginal
spotting. On examination, her blood pressure is 110/60 mm Hg, heart rate (HR) is 80 beats per
minute (bpm), and temperature is 99°F (37.2°C). The heart and lung examinations are normal. The
abdomen is soft and uterus nontender. Fetal heart tones are in the range of 140 to 150 bpm.

» What is the most likely diagnosis?


N. Placental abruption
O. Placenta previa
P. Gestational diabetes
Q. Multifetal gestation
R. Gestational trophoblastic disease

» What will be the long-term management of this patient?


R. Induction of labor
S. Tocolysis if fetal condition is stable
T. Immediate cesarean delivery
U. Expectant management as long as the bleeding is not excessive
V. Intrauterine transfusion
CASE AB-2: A 28-year-old woman at 32 weeks’ gestation is seen in the obstetrical (OB) triage
area for vaginal bleeding described as significant with clots. She denies cramping or pain. An
ultrasound is performed revealing that the placenta is covering the internal os of the cervix.

» Which of the following is a risk factor for this patient’s condition?


N. Prior salpingitis
O. Hypertension
P. Multiple gestations
Q. Polyhydramnios
CASE AB-3: A 21-year-old patient at 28 weeks’ gestation has vaginal bleeding and is diagnosed
with placenta previa.
» Which of the following is a typical feature of this condition?
V. Painful bleeding
W. Commonly associated with coagulopathy
X. First episode of bleeding is usually profuse
Y. Associated with postcoital spotting

CASE AB-4: A 33-year-old woman at 37 weeks’ gestation, confirmed by first-trimester


sonography, presents with moderately severe vaginal bleeding. She is noted on sonography to have
a placenta previa.

» Which of the following is the best management for this patient?


q. Induction of labor
r. Tocolysis of labor
s. Cesarean delivery
t. Expectant management
u. Intrauterine transfusion

» What is uncommon cause of antepartum haemorrhage (APH) at term


F. Blood-stained show
G. Bleeding placenta praevia
H. Placental abruption
I. Cervical ectropion
J. Bacterial vaginosis
K. Vasa praevia

» Which of the following sequence of examinations is most appropriate?


I. Speculum examination, ultrasound examination, digital examination
J. Ultrasound examination, digital examination, speculum examination
K. Digital examination, ultrasound examination, speculum examination
L. Ultrasound examination, speculum examination, digital examination
CASE AB-6: An 18-year-old adolescent is noted to have a marginal placenta previa on an
ultrasound examination at 22 weeks’ gestation. She does not have vaginal bleeding or spotting.

» Which of the following is the most appropriate management?


A. Schedule cesarean delivery at 39 weeks
B. Schedule an amniocentesis at 34 weeks and deliver by cesarean if the fetal lungs are mature
C. Schedule an MRI examination at 35 weeks to assess for possible percreta involving the bladder
D. Reassess placental position at 32 weeks’ gestation by ultrasound
E. Recommend termination of pregnancy

How do you interpret the examination and blood test findings?


M. DIC
N. Liver failure
O. Haemorrhagic shock
P. Kidney failure
Q. Electrolyte disorder

Which of the following statements is most accurate?


A. Fetal ultrasound is more accurate in diagnosing placental abruption than placenta previa.
B. Fetal ultrasound is quite sensitive in diagnosing placental abruption.
C. Ultrasound is sensitive in diagnosing abruption that occurs in the lower aspect of the uterus.
D. Fetal ultrasound is not sensitive in diagnosing placental abruption.

Which of the following is the most significant risk factor for abruption placentae?
A. Prior cesarean delivery
B. Breech presentation
C. Trauma
D. Marijuana use
E. Placenta accreta

Which of the following is most likely to be present in drug screen of the pregnant patient with
bright vaginal discharges?
A. Marijuana
B. Alcohol
C. Barbiturates
D. Cocaine
E. Benzodiazepines

Which of the following is the best management for this patient?


A. Admission and careful observation in the ICU
B. Induction of labor with plan for vaginal delivery
C. Partial exchange transfusion
D. Urgent cesarean delivery
E. Intravenous terbutaline

What is the most appropriate next step in management?


M. Admit to antenatal unit for bed rest and betamethasone.
N. Admit to antenatal unit for bed rest and blood transfusion.
O. Induction of labour
P. Perform emergency cesarean section.

What is the best course of action?


i. Emergent cesarean section
j. Fetal umbilical blood transfusion
k. Expectant management
l. Induction of labour with prostaglandins

Which of the following is most likely a complication of this patient’s present condition?
L. DIC
M. IUGR
N. Subarachnoid hemorrhage
O. Vasa previa
Which of the following is the most likely cause of this patients abnormal lab tests (International
Normalized Ratio of 2.3, partial thromboplastin time of 48 seconds, D-dimer elevated, fibrinogen
decreased)?
M. Disruption of placenta and release of fetal tissue into circulation
N. Liver failure
O. Haemorrhagic shock
P. Release of thromboplastin by damaged placenta
Q. Ruptured ectopic

Which of the following is the most appropriate first step in management?


K. Administer a tocolytic
L. Administer a corticosteroid.
M. Administer fresh frozen plasma.
N. Deliver the fetus immediately by LSCS
O. Observe closely.

Which of the following statements is accurate concerning postpartum hemorrhage?


N. The blood loss at delivery approaches the volume of blood added during pregnancy.
O. Studies show that estimated blood loss reported is often less than the actual blood loss.
P. The WHO defines postpartum hemorrhage as cumulative blood loss of >100 mL
accompanied by symptoms and signs of hypovolemia.
Q. None of the above

Which of the following is true concerning placental abruption?


E. It may lead to a dilutional coagulopathy.
F. Concealed abruption forces thromboplastin into large veins draining the implantation site.
G. In most women with abruption severe enough to kill the fetus, the plasma fibrinogen
level will be less than 150 mg/dL.
H. All of the above

How is a low-lying placenta defined?


J. The placenta is implanted in the lower uterine segment.
K. The placental edge covers the internal os, but not completely.
L. The placental edge does not cover the internal os but lies within a 1-cm wide perimeter.
M. The placental edge does not cover the internal os but lies within a 2-cm wide perimeter.

What percentage of women who have a primary cesarean with a placenta previa have
hysterectomies
performed?
M. 1%
N. 2%
O. 6%
P. 10%

Which of the statements below accurately depicts the difference between consumptive
coagulopathy and disseminated intravascular coagulation?
S. Abruption best represents a consumptive coagulopathy.
T. Loss of procoagulants with massive hemorrhage is the basis of dilutional coagulopathy.
U. A concealed abruption forces thromboplastin into the circulation and leads to the
consumption of procoagulants.
V. All of the above

Which of the bacteria below are associated with consumptive coagulopathy?


E. Escherichia coli
F. Klebsiella pneumoniae
G. Group A streptococcus
H. Group B streptococcus

The use of whole blood for massive hemorrhage is supported by which of the following?
S. Less renal failure
T. Less pulmonary edema
U. Fewer intensive care unit admissions
V. All of the above

PICTURES!!!
E. Placenta previa

Culdocentesis Step-by-Step
Explain the risks, benefits, potential complications, and alternatives (e.g., US or immediate
laparoscopy) to culdocentesis to the patient and/or their representative. Provide emotional
support and encouragement. Obtain a signed consent for the procedure.
Perform bimanual pelvic examination to rule out a fixed pelvic mass and to assess the position
of the uterus prior to culdocentesis. A bulging of the cul-de-sac into the posterior fornix suggests
pooling of intraperitoneal fluid.
Place the patient in the lithotomy position with the head of the table slightly elevated in reverse
Trendelenburg position so that the intraperitoneal fluid gravitates into the posterior cul-de-sac.
Insert speculum to visualize cervix. Cleanse vagina and cervix with antiseptic (povidone iodine).
Apply 2 percent lidocaine gel to the posterior vagina and posterior cervical lip (optional).
Grasp the posterior lip of the cervix with the tenaculum and lift the cervix slightly to expose the
posterior fornix, which is bounded laterally by the uterosacral ligaments.
Insert an 18-gauge needle attached to a 20 mL syringe containing 5 mL of normal saline (or air)
through the posterior vaginal wall into the posterior cul-de-sac using a rapid, deliberate thrust.
The insertion point is in the midline, about 1 to 2 cm below the insertion of the uterosacral
ligaments onto the uterus and below where the vaginal mucosa sweeps off the cervix and
crosses the posterior cul-de-sac. The needle should not extend more than 3 to 4 cm through the
vaginal mucosa and is directed slightly caudad, away from the uterus and toward the sacrum.
Inject 5 mL of normal saline (or air) to confirm placement and clear tissue from the needle point.
Reposition the needle if there is resistance.
Aspirate peritoneal fluid, and then remove the needle slowly. If no fluid is obtained, repeat the
procedure at a different angle with a new needle. Discontinue the procedure if unsuccessful
after two or three attempts.
10 Apply pressure for any bleeding at the puncture or tenaculum site. Interpret obtained fluid (pus,
blood, etc.)

Endometrial Biopsy Using Pipelle Single-use Curette


Any women of reproductive age or with the potential for pregnancy should have a documented
negative pregnancy test prior to the procedure. The patient can take an NSAID 30 to 60 minutes
prior to the procedure to reduce the pain associated with cramping. Prophylactic antibiotics are
not necessary during endometrial sampling.
The patient is first placed in the lithotomy position, and a bimanual examination is done to
determine the uterine size and position of the uterus. The speculum can now be inserted to
allow for cervical visualization.
Once visualized the cervix can be anesthetized and cleansed by spraying a 20 percent
benzocaine spray for 5 seconds then applying an iodine solution. At this time it is appropriate to
wash hands and don sterile gloves.
Stabilize the cervix with a tenaculum placed on the anterior lip of the cervix, it allows the
provider to straighten the uterocervical angle. The uterine sound is then inserted to an average
depth of 6 to 10 cm within the uterus. If the uterine sound will not pass through the internal
cervical os, this can be overcome by using cervical dilators.
Once achieving adequate os dilation, and determining the depth of the uterus, the sampling
pipelle can be inserted. The same as the uterine sound the pipelle should be advanced until
encountering resistance. This resistance should be at the same depth as the sounding of the
uterus.
Once the pipelle is in the uterine cavity, the internal piston on the catheter is fully withdrawn
which creates suction at the catheter tip. This suction accompanied by moving the tip with an in
and out motion allows for sample collection. This motion should be completed with a 360-
degree twisting motion to reach all four quadrants of the endometrium.
The pipelle is now removed. Expel the specimen into the formalin container by pushing the
piston into the sheath, thereby discharging the specimen into the fixative. If there appears to be
insufficient tissue for diagnosis, perform a second pass of the catheter. The same catheter may
be used if it has not touched the formalin.
Remove the instruments. Most bleeding can be controlled with pressure via cotton swabs or a
sponge stick.
Decontaminate all used instruments, gloves and blood soiled linen.

Insertion Copper-Releasing IUD step-by-Step


1
Take into consideration possible contraindications, results of investigation (STD?) and
menstrual cycle phase (IUDs may be inserted anytime during the menstrual cycle).
2
Discuss with the patient the risks and benefits of the IUD and, if necessary, other forms
of contraception. Informed consent may be obtained after these steps are completed.
3
Administration of a nonsteroidal anti-inflammatory drug (e.g., 600 to 800 mg of
ibuprofen) one hour before insertion may alleviate discomfort.
4
Prepare IUD prior to starting procedure (use sterile technique, review insertion
technique in package insert).
5
Place vaginal speculum (short speculum may prevent pushing cervix posteriorly).
6
Prepare vaginal wall with antiseptic solution (povidone-iodine or chlorhexidine
gluconate).
7
Apply tenaculum at anterior (12:00) cervical position to stabilize the uterus while
inserting uterine sound and IUD. Mild traction to the tenaculum may also be used to
reorient an anteflexed or retroflexed uterus.
8
Insert and remove uterine sound (confirm depth of uterus at least 6 cm, clears path for
IUD insertion).
9
Place IUD according to package insert. If insertion is difficult dilate endocervical canal
with both 3 mm and 4 mm sound. Consider paracervical block if discomfort occurs.
10 Cut IUD threads 2-3 cm from cervical os.
11 After insertion observe the patient's condition (weakness, pallor, tachycardia, etc.) and
give her recommendations regarding the use of the IUD.

Key Initial Management for Suspected Ruptured Ectopic Pregnancy.


1
Facial 100% oxygen by mask.
2
Lie flat with head down.
3
Two large-bore (a 16-18 gauge) cannulae with 2 L of intravenous fluid (Ringer's
lactate or normal saline) given immediately. Ensure infusion is running well.
4
Give fluids at rapid rate if shock, systolic BP <90 mm Hg, pulse>110/minute, or heavy
vaginal bleeding: infuse 1 litre in 15-20 minutes (as rapid as possible), infuse 1 litre in 30
minutes at 30 ml/minute. Repeat if necessary.
5
Monitor every 15 minutes for: blood pressure (BP) and pulse, shortness of breath or
puffiness. Monitor urine output. Record time and amount of fluids given.
6
Reduce the infusion rate to 3 ml/minute (1 litre in 6-8 hours) when pulse slows to less
than 100/minute, systolic BP increases to 100 mmHg or higher.
7
Crossmatch 4 units (and alert haematologist to the haemorrhage).
8
Consent for laparoscopy/laparotomy and salpingectomy.
9
Transfer to theatre for salpingectomy

Cervical cytology
Procedure Steps
Step 1 Introduce yourself to the patient and clarify her identity. Explain what you would like to do
and obtain
consent. Explain she should feel little, if any, discomfort and that the examination should be over
fairly quickly.
Step 2 Ensure you have all of the necessary equipment ready. This includes: Gloves, Speculum,
Lubricant, A
cervical brush, A pot of cytology preservative solution, A light source.
Step 3 The patient should be exposed from the waist down. Ask her to lie on the bed with her
ankles
together and allow her knees to spread apart as much as possible.
Step 4 Wash you hands, put on your gloves and examine the outer vagina checking for any obvious
abnormalities.
Step 5 Warm the blades of the speculum with warm water. The water also acts to lubricate the
speculum)
but you may also like to apply some lubricant. Ensure the lubricant is not placed at the end of the
speculum as
this may alter the result.
Step 6 Inform the patient that you are about to start the procedure. Use your left hand to part the
labia
minora and insert the speculum with the screw facing sideways.
Step 7 As you advance the speculum, turn it so that the screw faces upwards. Open the blades and
fix them
open with the screw. Ensure that you can see well by adjusting the light source. Check for any gross
pathology
and identify the transition zone.
Step 8 Place the tip of the cervical brush into the external cervical os and rotate it three times
through 360
degrees ensuring that it is always in contact with the cervix.
Step 9 Remove the brush ensuring it does not wipe against anything. Tap the brush 10 times on the
edge of
the pot of cytology medium. Tighten the cap, record the patient's name and history number on the
vial or
attach a printed patient identification label. Complete a cytopathology test request form.
Step 10 Release the screw on the speculum and carefully remove it from the vagina, completing the
examination.
Step 11 Offer the patient some tissue, cover the patient, and thank her. You should explain to the
patient that
her smear results will be sent to them in approximately six weeks, thus ensuring appropriate follow-
u

pap test collection procedure


THINPREP PAP TEST PROCUREMENT
1. Assemble collection devices and ThinPrep Pap test vial.

2. Obtain an adequate sampling of the ectocervix using a plastic spatula. (A wooden spatula is not
suitable as the cellular material is more difficult to rinse into the collection fluid.)

3. Rinse the spatula into the solution vial by swirling the spatula vigorously in the vial 10 times.
Discard the spatula.

4. Obtain an adequate sampling from the endocervix using an endocervical brush device. Insert the
brush into the endocervical canal until only the bottom-most fibers are exposed. Slowly rotate ¼ or
½ turn in one direction. DO NOT OVER ROTATE.

5. Rinse the brush in the solution vial by rotating the device in the solution 10 times while pushing
against the vial wall. Swirl the brush vigorously to further release material. Discard the brush.

6. Tighten the cap so that the torque line on the cap passes the torque line on the vial.

7. Record the patient’s name and history number on the vial or attach a printed patient
identification label. Complete a cytopathology test request form (in preprinted paper form or test
order printed from Epic).

8. Send labeled specimen vial and completed request form to the laboratory for processing. No
refrigeration of the specimen is necess

Dilatation and curettage


Procedure steps
When you arrive, a nurse or a technician will ask you to remove your clothing and put on a hospital
gown. If you’re receiving general anesthetic or IV sedation, a nurse will insert a small plastic
catheter into a vein. They’ll also hook you up to monitors that painlessly measure your blood
pressure, breathing, and heartbeat.

When your doctor is ready to start the procedure, they’ll ask you to lie back on an examination table
as you would if you were having a Pap test. You’ll rest your feet in stirrups, and a sheet or blanket
will cover your knees. Usually, one nurse is present to help the doctor and another is available to
monitor your vital signs and provide support and reassurance.

The operation will proceed as follows:

Your doctor inserts a device called a speculum to spread your vaginal walls so that they can see the
cervix.
Your doctor dilates the cervix by inserting a series of rods into your cervical opening. Each rod is a
little thicker than the one before it.
After dilating the cervix, your doctor inserts a spoon-shaped device called a curette, and draws the
sides of the device along the lining of the uterus.
If the curette can’t loosen all the tissue, your doctor may use a suction device as well. If you’ve had
a local anesthetic, you’ll probably notice some cramping.
After removing the material from your uterus, your doctor removes the instruments from your body.
Your doctor then sends the material removed from the uterus to a laboratory for analysis.
Ant
enat
albl
eedi
ng

CASEAB-
1

A30-year-
oldG5P4womanat32weeks’ gestationcompl ai
nsofbrightredvaginal bleedi
ng.
Shedeniesuteri
necontracti
ons, l
eakageoff luid,ortrauma.Thepat ientstat
est hat4weeks
previ
ously
,aft
ershehadengagedi nsex ualintercourse, sheexperi
encedsomev aginal
spott
ing.Onexaminati
on, herbloodpr essurei s110/ 60mm Hg, heartrate(HR)is80beat s
perminute(bpm),andtemper atureis99° F(37. 2°C).Thehear tandlungex aminat i
onsar e
normal.Theabdomenissof tandut er
usnont ender.Fet alheartt
onesar einther angeof140
to150bpm.

»Whatwi
l
lbet
hel
ong-
ter
m managementoft
hispat
ient
?

Ans-Long-t
erm management:
Expect
antmanagementasl
ongast
hebl
eedi
ngi
snot
excessi
ve.Cesar
eandeli
ver
yat34weeks’gest
ati
on

22.
1A21-year
-ol
dG1P0womani sseenforherfi
rstprenat
alvisi
tatt
heobst
etri
cian’
soff
ice.
Basedont
heLMP, thepati
enti
s36weeks’ gest
ation.Onult
rasound,
themeasurements
i
ndicat
e32weeksforall
parameter
sincl
udingtheHC, AC,
andFL.

Whi
choft
hef
oll
owi
ngi
sthebestmanagementf
ort
hispat
ient
?

A.Ant
enat
alst
eroi
dsf
orpr
obabl
yIUGR

B.Recommendamni
ocent
esi
sforkar
yot
ype

C.Del
i
ver
yin1week(
att
erm)

D.Cont
inuedmoni
tor
ingandr
epeatul
tr
asound+

22.2A27- y
ear-ol
dG2P1womani sat37weeks’ gestat
ionsupportedbyLMPanda10- week
ul
trasound.Theesti
matedf et
al weightis2000g,whichislessthanthe3rdpercenti
l
efor
gestati
onalage.TheDopplerstudiesindicat
ethepresenceofforwardend-di
astol
icfl
ow.
Your ecommenddel i
ver
yforthispat i
ent.

Whi
choft
hef
oll
owi
ngi
sthebestr
easonf
ory
ourr
ecommendat
ion?

A.Af
etal
wei
ghtof2000gcor
rel
ateswi
thahi
ghsur
viv
ali
nthenur
ser
y

B.I
UGRcar
ri
esasi
gni
fi
cantr
iskoff
etal
deat
h+

C.TheDoppl
erst
udi
esi
ndi
cat
eaconcer
nforcont
inui
ngt
hepr
egnancy

D.Withdel
i
ver
y,f
urt
herdi
agnost
icst
udi
essuchaskar
yot
ypeandv
iral
studi
escanbe
conduct
ed

22.3An18- y
ear-
oldG1P0womanat38weeks’ gestat
ionconfi
rmedbya12-weekultr
asound
hasaf undalhei
ghtof34cm.Thepatienthasgained20lbsduri
ngthepr
egnancy.She
deniessmokingoralcohol
ori
ll
i
citsubstanceuse.HerBPi s110/70mm Hg.
Whi
choft
hef
oll
owi
ngi
sthebestmanagementoft
hispat
ient
?

A.Per
for
m abasi
cul
tr
asoundst
udy+

B.Schedul
efordel
i
ver
ysi
ncet
hepat
ienthasr
eachedat
erm gest
ati
onal
age

C.Schedul
ebi
ophy
sical
prof
il
eandDoppl
erst
udi
esf
ort
hispat
ient

D.Sendherur
inef
oradr
ugscr
eenandconsi
deror
der
ingser
um TORCHt
it
ers

Ant
enat
alscr
eeni
ng

CASEAS-
1

A20- year-
oldG1P0womanat16weeks’ gestat
ionhasr eceivedaserum mat er
nalα-
fetoprotei
ntestthatretur
nedas2. 8mul
t i
plesofthemedi an( MOM) .Shedeniesaf amil
y
historyofcongenital
anomal i
esorchromosomal abnormaliti
es.Onex aminati
on,sheis
afebril
e,herbl
oodpr essure(BP)is100/
70mm Hg, andherhear trat
e( HR)is70beatsper
mi nute(bpm).Thehear tandlungexaminationsarenormal .Thefundalheighti
smi dway
bet weenthesymphy sispubisandtheumbi li
cus.Fetal
hear ttonesareintherangeof140
bpm.

»Whati
syournextdi
agnost
icst
ep?

Ans-
Nextdi
agnost
icst
ep:
Basi
cobst
etr
icul
tr
asoundexami
nat
iont
oassessf
ordat
esand
mult
ipl
egest
ati
ons.

»Whati
sthepur
poseoft
hemat
ernal
ser
um α-
fet
opr
otei
n(msAFP)t
est
?

Ans-Pur
poseofmsAFP:Thepur
poseist
oassesstheriskforafet
alopenneur
alt
ubedefect
,
andcanalsobeusedt
oassessfort
heri
skofaneupl
oidy,suchasfet
alDownsyndromeor
tr
isomy18

7.1A23- y
ear-ol
dG1P0womanat20weeks’ gestati
onunder goesanultrasoundexami nati
on
forsi
zegr eat
erthanherdates.Theult
rasoundr eveal
shy dramnioswithanamni oti
cf l
uid
i
ndexof30cm.Thef etalabdomenr ev
ealsacy sticmassi ntherightabdominalregi
onanda
cysti
cmassi nthelef
tabdomi nalar
ea.Whichoft hef ol
l
owi ngisthemostl i
kelyassociated
condit
ion?

A.Gest
ati
onal
diabet
es

B.Congeni
tal
ovar
iant
umor
s

C.Downsy
ndr
ome+

D.Rhi
soi
mmuni
sat
ion

7.
3A22-
year
-ol
dG2P1womanat25weeks’
gest
ati
onwi
thasur
elastmenst
rual
per
iodasks
f
orserum scr
eening.Thepatient
’ssist
erhasonechi
ldwit
hDownsyndromeand,ot
her
wise,
t
herei
snof amil
yhi st
oryofanomaliesorgenet
icdi
sorder
s.Whi
chofthefol
l
owingi
sthe
mostappropr
iat
er esponse?

A.Amni
ocent
esi
sist
heappr
opr
iat
etest
.B.Ser
um scr
eeni
ngshoul
dbeper
for
med.

C.Expl
aint
othepat
ientt
hatitist
oolateforser
um screeni
ng,
butt
hatherr
iskf
orDown
syndr
omeisnotmuchhigherthanherage-r
elat
edr
isk.+

D.Thepat
ientbei
ngonl
y22y
ear
sofagedoesnotneedser
um scr
eeni
ng.

E.Thepat
ienthasa25%chanceofherbabyhav
ingDownsy
ndr
ome.

7.4A28- y
ear-ol
dG1P0womanat16weeks’ gestat
ionisnotedtohaveanelevatedmsAFP
at2.9MOM.Sheunder wentatar
getedultrasoundex aminati
onwhichdidnotreveal
aneural
tubedefect.Herphysi
cianal
soundertakesadi l
igentsearchforaneti
ologyf
ortheelev
ated
msAFPwi t
houtidenti
fyi
nganeti
ology
.Whi choft hefoll
owingcondit
ionsist
hispati
entat
i
ncreasedrisk?

A.I
ncr
easedi
nci
denceofst
il
lbi
rt
h+

B.Gest
ati
onal
diabet
es

C.Pl
acent
apr
evi
a

D.Mol
arpr
egnancy

E.Downsy
ndr
ome

7.5A22-year-
oldwomanisseenforherfir
stprenatalvisi
tat16weeks’gestat
ionwi
tha
famil
yhist
oryofcongeni
tal
deafnessandneonat alrenaldi
sease.Thepati
ent’
shear
ingi
s
normal.Whichofthef
oll
owingi
st hebestnextstep?

A.Amni
ocent
esi
sforkar
yot
ype

B.Amni
ocent
esi
sforr
ubel
l
aPCR

C.Genet
iccounsel
i
ng+

D.Gl
ucosechal
l
enget
est
ing

CASEAS-
6

Awomanaged23y earsi
sreferredbyhergener alpr
acti
ti
onert otheantenatal
cli
nicat12
weeksinherfir
stpregnancy.Shehasnosi gnif
icantmedicalhistoryandisoneoffour
si
bli
ngs.Ondirectquesti
oni
nghermot herapparentl
yhadast i
llbir
thattr
ibut
edtosomef orm
ofcongenit
alabnormali
ty28y earsago.Otherwisethepregnancyi sassessedtobelowrisk.
Sheisoffer
edascr eeni
ngtestforDown’ ssyndrome.
•Pr
egnancy
-associ
atedpl
asmapr
otei
n-A(
PAPP-
A):
0.4mul
ti
plesoft
hemean(
MoM)

•Fr
eebet
ahumanchor
ioni
cgonadot
ropi
n(hCG)
:1.
7mul
ti
plesoft
hemean(
MoM)

•Nuchal
transl
ucency
:2.
9mm (
nor
mal
)

Ov
eral
lDown’
ssy
ndr
omer
iskcal
cul
atedas1i
n118

»Whatt
est
sar
eav
ail
abl
eforhert
oconf
ir
m/ex
cludeDown’
ssy
ndr
ome?

Ans-
Amni
ocent
esi
s,cel
lfr
eef
etal
DNA,chor
ioni
cvi
l
lussampl
i
ng

»Whi
choft
hef
oll
owi
ngi
sanaccur
atest
atementr
egar
dingDown’
ssy
ndr
ome?

Ans-
Downsy
ndr
omescr
eeni
ng-decr
easedPAPP-A,
incr
easedNTandf
reebet
aHCG.

CASEAS-
7

Awomanaged34y earsis12weeks’ gestati


oninherthir
dpregnancy.Herfi
rstpregnancy
endedi
naf i
rst
-t
rimestersuctionter
mi nati
onat18y earsofageandshehadami scarr
iage8
monthsagorequir
ingsur gi
calmanagement .Shei
sgener al
l
ywellexceptformildasthma.
Shehasnofamilyhistoryofcongenit
al abnormali
ti
es.Heronlymedicati
onisfoli
cacid400
mcgdail
y.Herpartneris31y earsol
dandwasadopt ed.Hehasnoknownmedi calproblems.

Ther
out
inebooki
ngbl
oodandur
inet
est
sar
enor
mal
.

Thef
ir
st-
tr
imest
erul
tr
asoundf
indi
ngsar
eshownbel
ow

Ul
tr
asoundr
epor
t:

Si
ngl
efet
us.Fet
alhear
tact
ionnor
mal

Cr
own–r
umpl
engt
h:62.
4m (
cor
respondst
o12weeks3day
s’gest
ati
on)

Nuchal
transl
ucency(
NT)
:3.
2mm

Ri
skoft
ri
somybymat
ernal
age(
34y
ear
s):
1:276

Adj
ust
edr
iskoft
ri
somyaf
terNT:
1:30

»Wi
thr
egar
dtot
her
out
ineanomal
yscani
npr
egnancy
,whatt
hreshol
dofnuchal
-f
old

measur
ementshoul
dtr
iggerar
efer
ral
toaf
etal
medi
ci
nespeci
ali
st?

»Thecouplechosetohaveafurt
hert
estandtheresul
tsar
enormal(
46XYnor
mal
kar
yot
ype).Whatfur
therdi
agnost
icopt
ionsshoul
dbeconsider
ed?

Ans-
Nuchal
transl
ucencyt
esti
sscr
eeni
ngt
est

Fal
seort
rue
Pl
easemat
cht
rueamongt
hef
oll
owi
ngst
atement
s

Phy
siol
ogyofobst
etr
ics:

1Obst
etr
ichi
stor
y:

I
tisr
ecommendedt
hatwomenar
eseenont
hei
rownatl
eastoncedur
ingant
enat
alcar
e.T

Af
ami
l
yhi
stor
yofpr
e-ecl
ampsi
ashoul
dtr
iggeri
ncr
easedant
enat
alsur
vei
l
lance.
T

Dur
ingpr
egnancy3percentofwomenusei
l
li
citdr
ugs.

Ahi
stor
yofsub-
fer
ti
li
tyi
simpor
tantev
eni
fthepat
ienti
scur
rent
lypr
egnant
.T

Awomanwhohashadapr
evi
ousect
opi
cpr
egnancyshoul
dbeof
fer
edanear
lypr
egnancy
ul
tr
asoundscanT

2Thef
oll
owi
ngt
ermsar
eappr
opr
iat
e:

Li
e:cephal
i
c(v
ari
ant
s:def
lex
ed,
breech,
shoul
der
,longi
tudi
nal
,tr
ansv
erseet
c.)
.

Posi
ti
on:
flexed(
var
iant
s:def
lex
ed,
breech,
shoul
der
,longi
tudi
nal
,tr
ansv
erseet
c.)
.

St
ati
on:
att
hel
evel
oft
hespi
nes(
var
iant
s:-
2,+3,
brow,
bregmat
ic,
2/5,
etc.
).
T

Engagement
:two-
fi
ft
hs(
var
iant
s:-
2,+3,
brow,
bregmat
ic,
att
hel
evel
oft
hespi
nes,
etc.
).
T

Present
ingpar
t:shoul
der(
var
iant
s:def
lexed,
breech,
shoul
der
,longi
tudi
nal
,tr
ansv
erse
et
c.).
T

3Wi
thr
egar
dtor
out
ineant
enat
alcar
e:

Al
lwomenshoul
dbeof
fer
edscr
eeni
ngf
orhaemogl
obi
nopat
hies.
T

Ahi
ghv
agi
nal
swabshoul
dbesentr
out
inel
yatbooki
ng.

Sy
phi
l
ist
est
ingf
ormspar
toft
her
out
inebooki
ngv
isi
t.
T

Anul
tr
asoundscanf
oranomal
i
esshoul
dbeper
for
medat24weeksgest
ati
on.

Ev
erypat
ientshoul
dhav
eanamedconsul
tant
.

4Theai
msoft
he18–22weeksanomal
yscani
ncl
ude:

Tol
ocat
ethepl
acent
a.T
Todet
ermi
net
hechor
ioni
cit
yofat
winpr
egnancy
.

Assessmentofamni
oti
cfl
uidv
olume.

Pr
omot
ingpar
ent
albondi
ngwi
tht
hef
etus.
T

Toi
dent
if
yfet
alst
ruct
ural
def
ect
s.

5Humanmi
l
khast
hef
oll
owi
ngadv
ant
agesov
erf
ormul
ami
l
k:

Humanmi
l
kcont
ainsmor
epr
otei
n.

Humanmi
l
kcont
ainsmor
elact
ose.
T

Humanmi
l
kisassoci
atedwi
thar
educt
ioni
nat
opi
cil
l
ness.
T

Humanmi
l
kcont
ainsmor
eir
on.

Humanmi
l
kisagoodsour
ceofv
itami
nK.

6Thef
oll
owi
ngf
act
orsi
nfl
uencef
etal
bir
thwei
ght
:

Thepar
it
yoft
hemot
her
.T

Theexer
cisehabi
tsoft
hemot
her
.

Theet
hni
ci
tyoft
hemot
her
.T

Thesexoft
hef
etus.
T

Mat
ernal
fol
atesuppl
ement
ati
on.

Fet
alassessment

7Dur
ingl
ungdev
elopment
:

Sur
fact
antpr
oduct
ionoccur
sfr
om about20weeks.

Thepr
edomi
nantphosphol
i
pidi
sphosphat
idy
lchol
i
ne.
T

Themajori
tyofi
nfant
sbor
nat27/
40exper
iencesomedegr
eeofr
espi
rat
orydi
str
ess
syndr
ome.T

Fet
albr
eat
hingmov
ementoccur
sleastdur
ingREM sl
eep.

Thepr
oduct
ionofl
eci
thi
nisenhancedbycor
ti
sol
anddi
abet
es.
8Regar
dingt
hef
etal
li
ver
:

Gl
ycogeni
sst
oredi
nlar
gequant
it
iesi
nthet
hir
dtr
imest
er.
T

Theenz
ymesr
equi
redt
oconj
ugat
ebi
l
irubi
nar
enotpr
esent
.T

Redbl
oodcel
lmanuf
act
urebegi
nsat20/
40.

Der
ivesf
rom t
hemesoder
m.

Hast
hesameembr
yol
ogi
cal
ori
ginast
hegal
lbl
adder
.T

9Consi
der
ingDoppl
erul
tr
asound:

Abnor
mal
uter
inear
ter
yDoppl
erf
lowi
ndi
cat
esf
etal
hypoxaemi
a.

Abnor
mal
umbi
l
ical
art
eryf
lowi
ndi
cat
espoorpl
acent
alper
fusi
on.
T

Fet
alhy
poxaemi
aisassoci
atedwi
thr
edi
str
ibut
ionofbl
oodf
low.
T

Fet
alanaemi
aisbestassessedusi
ngmeasur
ement
sfr
om t
hemi
ddl
ecer
ebr
alar
ter
y.T

Abnor
mal
duct
usv
enosusbl
oodf
lowoccur
spr
iort
oar
ter
ial
changes.

10Thef
oll
owi
ngar
eev
aluat
edwhenper
for
mingaf
etal
biophy
sical
prof
il
e:

Est
imat
edf
etal
wei
ght
.

Fet
alt
one.
T

Mat
ernal
bloodpr
essur
e.

Amni
oti
cfl
uidv
olume.
T

Pl
acent
albl
oodf
low.

11Thef
oll
owi
ngst
atement
sar
etr
uef
orpr
enat
alt
est
s:

Ser
um bi
ochemi
str
yissuper
iort
omat
ernal
ageasascr
eeni
ngt
estf
orDown’
ssy
ndr
ome.
T

Mat
ernal
ser
um al
pha-
fet
opr
otei
nisadi
agnost
ict
estf
orneur
alt
ubedef
ect
s.

Amni
ocent
esi
shasahi
gherpr
egnancyl
ossr
atet
hanchor
ioni
cvi
l
lussampl
i
ng.

Test
susi
ngDNAt
echnol
ogycanbeper
for
medonamni
ocent
esi
sspeci
mens.
T

Chor
ioni
cvi
l
lussampl
i
ngcanbeper
for
medonl
ybef
ore12weeks’
gest
ati
on.
12Consi
der
ingneur
alt
ubedef
ect
s:

Theseoccurasar
esul
tofapoorper
i-
concept
ual
mat
ernal
diet
.

Themaj
ori
tyoft
hesedef
ect
soccuratt
heendoft
hespi
ne.
T

Thepr
ognosi
sforspi
nabi
fi
dadependsont
hel
evel
oft
hel
esi
on.
T

Wi
thapr
evi
ousaf
fect
edsi
bli
ngt
her
iskofr
ecur
rencei
s1percent
.

Asuppl
ementof5mgf
oli
caci
dsi
gni
fi
cant
lyr
educest
her
iskofr
ecur
rence.
T

13Chor
ioni
cvi
l
lussampl
i
ng:

Car
ri
esa2percentr
iskofcausi
ngmi
scar
ri
age.
T

Mayshowapl
acent
almosai
cphenot
ype.
T

Maybeunsuccessf
uli
nobt
aini
ngasampl
e.T

Themostcommonappr
oachi
str
anscer
vical
.

Maybecar
ri
edoutat<11wksgest
ati
on.

14Ol
i
gohy
dramni
osi
sassoci
atedwi
tht
hef
oll
owi
ng:

Tr
acheo-
oesophageal
fist
ula.

Tal
i
pes.
T

I
ntr
aut
eri
negr
owt
hrest
ri
cti
on.
T

Anencephal
y.

Pr
emat
urer
upt
ureoft
hef
etal
membr
anes.
T

15Pol
yhy
dramni
osi
sassoci
atedwi
tht
hef
oll
owi
ng:

Mat
ernal
diabet
es.
T

Neur
omuscul
arf
etal
condi
ti
ons.
T

Mat
ernal
non-
ster
oidal
ant
i-
inf
lammat
orydr
ugs(
NSAI
Ds)
.

Post
-mat
uri
ty.

Chor
ioangi
omaoft
hepl
acent
a.T
Pat
hol
ogyofobst
etr
ics

16Dur
inganassi
stedbr
eechdel
i
ver
y:

Anepi
si
otomymaybecutoncet
heanusi
sseenatt
hef
our
chet
te.
T

Pi
nar
d’smanoeuv
recanbeusedt
odel
i
vert
hel
egsi
nanex
tendedposi
ti
on.
T

Maur
iceau–Smel
l
ie–Vei
tisusedt
odel
i
verex
tendedar
ms.

For
cepsshoul
dnotbeappl
i
edt
othef
etal
head.

Epi
dur
alanal
gesi
aismandat
ory
.T

17Thef
oll
owi
ngar
econt
rai
ndi
cat
ionst
oext
ernal
cephal
i
cver
sion:

Pol
yhy
dramni
os.
T

Compl
etebr
eechposi
ti
on.

Pr
eecl
ampsi
a.T

Sy
mphy
sispubi
sdy
sfunct
ion.
T

Twi
npr
egnancy
.T

18Wi
thr
egar
dtot
hepl
acent
a:

I
trecei
vest
hehi
ghestbl
oodf
lowofanyf
etal
organ.
T

Ther
esi
stanceoft
hespi
ral
art
eri
olesi
ncr
easessi
gni
fi
cant
lyi
nthesecondt
ri
mest
er.

Abnor
mallyi
ncr
easedbor
eoft
hespi
ral
art
eri
escont
ri
but
est
opat
hogenesi
sinpr
e-
ecl
ampsia.

I
tisamaj
orendocr
ineor
gan.
T

Eachcot
ely
doncont
ainsapr
imar
yst
em v
il
lus.
T

19Pr
eecl
ampsi
aismor
ecommoni
n:

Mul
ti
grav
idwomen.

Womenwi
thcongeni
tal
car
diacdi
sease.
Mul
ti
plepr
egnancy
.T

Womenwi
thdi
abet
esi
nsi
pidus.

Womenwi
thpr
e-ex
ist
ingr
enal
disease.
T

20Themanagementofpr
eecl
ampsi
aincl
udes:

Hospi
tal
assessment
.T

Labet
alol
.T

Ear
lydel
i
ver
y.T

Fur
osemi
de.

Magnesi
um sul
phat
e.T

21Thef
oll
owi
ngar
eri
skf
act
orsf
ori
ntr
aut
eri
negr
owt
hrest
ri
cti
on:

Mul
ti
plepr
egnancy
.T

Aspi
ri
nusei
npr
egnancy
.

Ant
iphosphol
i
pidsy
ndr
ome.
T

Fet
alkar
yot
ypeanomal
i
es.
T

Mat
ernal
age>35.

22Consequencesofpl
acent
alabr
upt
ioni
ncl
ude:

Hy
pov
olaemi
cshock.
T

Fet
alanemi
a.

Amni
oti
cfl
uidembol
us.

I
ncr
easedper
inat
almor
tal
i
ty.
T

I
ntr
aut
eri
nedeat
h.T

23Wi
thr
egar
dtoshoul
derdy
stoci
a:al
ltr
ue

I
toccur
sinappr
oxi
mat
ely1percentofl
abour
s.

I
tismor
ecommoni
nassi
stedv
agi
nal
del
i
ver
y.
McRober
t’
smanoeuv
rewi
l
lbeef
fect
ivei
n90percentofcases.

Fundal
pressur
eshoul
dbeav
oided.

Av
oidl
ater
alf
lex
ionoft
heheadont
heneck.

24Thesy
mpt
omsofpr
eecl
ampsi
aincl
ude:

Rest
lessness.
T

Sl
eepi
ness

Fl
ashi
ngl
i
ght
sinv
isi
on.
T

Rash

Epi
gast
ri
cpai
n.T

25I
nit
ial
managementofsev
erepost
par
tum haemor
rhager
equi
res:

Faci
aloxy
gen.
T

Admi
nist
rat
ionofI
Vfl
uidsv
iaacent
ral
li
ne

Seni
orhaemat
ologi
stpr
esent

Seni
oranaest
het
istpr
esent
.T

Ul
tr
asoundscanoft
heut
erus

26Ext
raf
act
orst
oconsi
deri
nmat
ernal
resusci
tat
ioni
ncl
ude:

Ti
l
tingoft
hegr
avi
dabdomen.
T

Lar
gebr
east
scr
eat
ingdi
ff
icul
ti
esi
nvent
il
ati
on.
T

Needf
orper
imor
tem Caesar
eansect
ion.
T

I
ncr
easedf
unct
ional
lungcapaci
ty.

Lowerr
iskofaspi
rat
ion.

27Thef
oll
owi
ngcompl
i
cat
ionsar
emor
ecommonaf
teri
nst
rument
aldel
i
ver
y:

Anaemi
a.T

Pr
olongedper
ineal
discomf
ort
.T
Mast
it
is.

Obst
etr
icpal
sy.
T

Puer
per
ali
nfect
ion.
T

28I
nfor
medconsentr
equi
rest
hatapat
ientshoul
d:

Beabl
etounder
standt
hebenef
it
soft
hepr
oposedt
reat
ment
.T

Si
gnaf
ormal
wri
tt
enconsentf
orm.

Under
standt
heconsequencesofr
efusi
ngt
hepr
oposedt
reat
ment
.T

Beabl
etor
etai
nandwei
ghupt
hei
nfor
mat
ionpr
ovi
ded.
T

Begi
vensev
eral
hour
stoconsi
derandev
aluat
etheopt
ions.

I
ugr

A32-y ear-ol
dpr i
mi gravidaisseeni ny ourof f
iceat33weeks’ gestat
ionforar outinepr enatal
vi
sit
.Hergest ati
onal age( GA)wascal culatedbyherl astnormal menstrualperiod, which
wasconsi st
entwi thanul tr
asoundper formedat8weeks’ gestat
ion.Herpregnancyhasbeen
unevent f
ul t
odat e,althoughshehascont inuedtosmokeonepackormor eofci garett
es
dai
ly.Shest at
est hatshehasbeenf eelingnor malfetalmov ementandnout eri
ne
cont
r actions.Onex ami nation,herheightis5f t6in.
,herwei ghti
s118l b(53.5kg) , andher
bl
oodpr essure(BP)i s90/ 60mm Hg.Herf undalheightis26cm.Onul t
rasound, younot ea
si
nglepr egnancywi t
hanest imatedf etalweightof900g, whichisatthethirdper centil
ef or
gest
at ionalage.

»Whati
sthemostl
i
kel
ydi
agnosi
s?

Ans-
Mostli
kel
ydi
agnosi
s:I
ntr
aut
eri
negr
owt
hrest
ri
cti
on(
IUGR)
,li
kel
yduet
oci
gar
ett
e
smoking

»Whatot
heri
mpor
tanti
temsshoul
dbenot
edont
heul
tr
asound

Ans-
.Otherult
rasoundi
tems:(a)Det
ermi
newhet
hert
hisi
ssy
mmet
ri
corasy
mmet
ri
cIUGR,
and(b)assesstheamni
oticf
luid

22.1A21-y
ear-ol
dG1P0womani sseenforherfi
rstprenat
alvisi
tatt
heobstetr
ician’soff
ice.
BasedontheLMP, thepati
enti
s36weeks’ gest
ation.Onult
rasound,
themeasur ement s
i
ndicat
e32weeksf orall
parametersi
ncl
udingtheHC, AC,
andFL.Whi chofthefollowingis
thebestmanagementforthi
spatient
?

A.Ant
enat
alst
eroi
dsf
orpr
obabl
yIUGR

B.Recommendamni
ocent
esi
sforkar
yot
ype

C.Del
i
ver
yin1week(
att
erm)
D.Cont
inuedmoni
tor
ingandr
epeat
edul
tr
asound+

Whati
sthef
ir
str
easont
obeex
cluded?

Ans–Ther
eisl
i
kehoodofwr
ongdat
es

22.2A27- y
ear-ol
dG2P1womani sat37weeks’ gestat
ionsupportedbyLMPanda10- week
ult
rasound.Theesti
matedf et
al weightis2000g,whichislessthanthe3rdpercenti
lefor
gestati
onalage.TheDopplerstudiesindicat
ethepresenceofforwardend-di
astol
icflow.
Your ecommenddel i
ver
yforthispat i
ent.Whichofthefoll
owingisthebestreasonforyour
recommendat i
on?

A.Af
etal
wei
ghtof2000gcor
rel
ateswi
thahi
ghsur
viv
ali
nthenur
ser
y

B.I
UGRcar
ri
esasi
gni
fi
cantr
iskoff
etal
deat
h+

C.TheDoppl
erst
udi
esi
ndi
cat
eaconcer
nforcont
inui
ngt
hepr
egnancy

D.Withdel
i
ver
y,f
urt
herdi
agnost
icst
udi
essuchaskar
yot
ypeandv
iral
studi
escanbe
conduct
ed

Whatar
ethepot
ent
ial
compl
i
cat
ionofpat
ientdi
sor
der
?

Ans-
fet
aldeat
h

22.3An18- y
ear-
oldG1P0womanat38weeks’ gestat
ionconfi
rmedbya12-weekultr
asound
hasaf undalhei
ghtof34cm.Thepatienthasgained20lbsduri
ngthepr
egnancy.She
deniessmokingoralcohol
ori
ll
i
citsubstanceuse.HerBPi s110/70mm Hg.

Whi
choft
hef
oll
owi
ngi
sthebestmanagementoft
hispat
ient
?

A.Per
for
m abasi
cul
tr
asoundst
udy+

B.Schedul
efordel
i
ver
ysi
ncet
hepat
ienthasr
eachedat
erm gest
ati
onal
age

C.Schedul
ebi
ophy
sical
prof
il
eandDoppl
erst
udi
esf
ort
hispat
ient

D.Sendherur
inef
oradr
ugscr
eenandconsi
deror
der
ingser
um TORCHt
it
ers

Whatar
ethepossi
blecasesf
ort
hiscondi
ti
onexcept
?

Ans–chr
omosomal
anamol
i
es

CASEGR-
5

A31- year-ol
dwomanat t
endsar outi
neant enatalappointmentat28weeks’ gestati
on.Shei s
G3P2.Herchi l
drenareaged4and2y earsandwei ghed3.7and3. 6kgrespectivel
yatbi rt
h.
Sher eportsnoant enatalconcerns,t
herei snoabdomi nalpain,nov agi
nalbleedingandgood
fetalmov ement sarereported.I
niti
alantenatalbookingbloodt est
sandf ir
st-t
rimesterand
20-weekanomal yscanswer enormal.Shei sanon- smokerandhasabst ai
nedf rom alcohol
duringthepr egnancy.Thewomanappear swellwi t
hnosi gnsofoedema.HerBMIi s24.
Bloodpr essureis115/ 74mmHg.Thesy mphy siofundalheightis24cm.Thef etusi
sf eltto
beacephalicpr esent
ation.Auscult
ati
onwi t
hhand-hel
dDoppl erconfi
rmst hefet
alhear
tbeat
tobe150/mi n.Onul t
rasound:bipari
etal
diameterandfemurl engthareonthe10thcenti
le.
Abdominalcircumf er
enceandest imatedfetal
weightarebelowt hefi
fthcenti
l
e.Theli
quor
vol
umei snormal .Umbilical
arter
yresist
anceindexiswithi
nnor malrange.

»Whatar
ethepossi
blecausesf
ort
hiscondi
ti
on,
EXCEPT?

Ans-
Mal
nut
ri
ti
on,congeni
tal
inf
ect
ion

»Whi
choft
hef
oll
owi
ngi
sthebestmanagementf
ort
hispat
ient
?

Ans-
Managementbasedont
hei
nvest
igat
ionr
esul
ts

I
fthechromosomalanal
ysi
sisabnor
mal
thenwomenshoul
dbeabl
etodi
scusst
heopt
ion
ofcont
inui
ngthepr
egnancy

I
freducedamnioti
cfl
uidt
henshouldbeadmini
sterMater
nalcor
ti
coster
oidi
nject
ionsi
t
r
educestheinci
denceofRespi
rat
orydi
str
esssyndromeinNeonates

LAbour

A26-year-
oldG1P0womanat39weeks’ gest
ationi sadmi t
tedtot hehospitalinl
abor.Sheis
notedtohaveuteri
necont ractionsevery2t o3mi nutes.Herant epart
um historyi
s
si
gnifi
cantforanonimmuner ubell
astatus.Onexami nati
on, herbloodpressure(BP)is
110/70mm Hgandhear tr ate(HR)is80beat spermi nut
e(bpm) .Theesti
mat edfet
al wei
ght
i
s7l bs.Onpelvi
cex aminat i
on, shehasbeennot edt ohaveachangei ncervical
examinati
onsfrom 4-cm dilat
iont o5-cm overthel ast2hour s.Thepelvisisassessedt obe
adequateondigit
alexami nation.

»Whati
syournextst
epi
nthemanagementoft
hispat
ient
?

Ans-
Nextst
epi
nmanagement
:Cont
inuet
oobser
vet
hel
abor
.

Whi
choft
hef
oll
i
ngbestdescr
ibet
hel
abour?

Ans-l
atentphase

CASEL-
2

A32- year-ol
dwomani nhersecondpr egnancyhasar ri
vedont helabourwardat38weeks3
days.Shehadanor mal deli
very18monthsago.Thi spregnancyhasbeencompl i
catedby
persi
st entvomi t
inguntil 20weeksandmor erecentl
ybyanaemi a.Shereportscontracti
ons
commenci ngappr oximat el
y4hago.Shet ookparacetamolathomeandt ri
edtor eli
evethe
pai
nwi thabat h,butnowf eelsshecannotcopewi ththepain.Twohour sagoshef eltagush
ofclearf l
uidfr
om t hev aginaandsincethenpai nshavebecomemuchmor esever enow
occurringev er
y4mi n,l
ast i
ngf or45s.Shehasf eltt
hebabymov i
ngnormal l
yallday.

Thebl
oodpr
essur
eis110/
58mmHgandhear
trat
eis98/
min.Thepr
esent
ati
oni
scephal
i
c
wit
h2/5pal pabl
eabdomi nal
l
y .Uter
inecontr
acti
onsarepalpabl
eandtheuter
usis
noni
rr
it
able.Speculum examinati
onshowscl earf
lui
dpooledinthepost
eri
orvagi
nalforni
x.
Onvaginalexaminati
onthecer v
ixis5cm dil
atedandtheheadis1cm abovetheischial
spi
nes.Thef et
alposi
tionisr
ightoccipi
tot
ransver
sewithmi l
dcaputandmouldi
ng.

»
Whi
choft
hef
oll
owi
ngbestdescr
ibest
hel
abor
?ans-
cgti
nter
per
tat
ion

»Whatshoul
dbey
oursubsequentmanagement
?ans-
rul
eof3’
s

A33- year-oldwomani nlaboris41weeks2day s’


gest ationofherfirstpregnancy.The
pregnancywasuncompl i
cateduntil2daysagowhenshedev el
opedmi l
dhypertensi
on,
withoutpr oteinuri
a.Invi
ewoft hegest at
ionalageadeci sionwasmadef ori
nductionof
l
aboury esterday.Shehad2mgpr ostagl
andingel admi nister
edintot hevaginaat6pm last
ni
ghtandagai nat6am t hismorning.Spontaneousr upt ureofmembr anesoccur r
edat10
am todayaf t
erwhi chcontract
ionscommenced.Bl oodpr essureis135/ 68mmHg, hear
trat
e
90/min, andt emper atur
eis37.1°C.Onabdomi nalpalpat i
onthef et
usi scephali
c,1/5
palpable,andst r
ongcont r
acti
onsar efel
t.

Vagi
nal
l
ythecer
vixisf
ull
yeff
acedand6cm di
l
ated.Thef
etusiscephal
i
cati
schi
alspi
nes
wit
hmil
dcaputbutnomouldi
ng.Gr
ade1meconium i
snoted.

TheCTG,
asshownbel
ow,
hasdemonst
rat
edasi
mil
arpat
ter
nforappr
oxi
mat
ely50mi
n.

Adeci
si
onismadef
orf
etal
bloodsampl
i
ngandt
her
esul
tisasf
oll
ows:
pH:
7.10(
nor
mal
7.
25-7.35)
.

»Howwoul
dyoui
nter
prett
heCTGandf
etal
bloodsampl
eresul
t?ct
gshowsbasel
i
ne
155/
min

»Howwoul
dyoumanaget
hepat
ientdel
i
ver
ybyv
ent
ousf
orcep

CASEL-
4

Ami dwi f
eisconcernedaboutacar diotocogr aph(CTG)ont helaborwar d.Thewomani s42
yearsold;sheisnowatagest ati
onalageof38weeksand1dayandpr esent edt othel abor
wardanhourago.Att het i
meofar ri
val
, exami nati
onrev ealedasy mphy siofundal heightof
39cm, cephal
icpr
esent ati
onand3/ 5pal pableabdomi nall
yandt hreecont r
actionsi n10mi n
l
asting50seach.Vagi nalexaminati
onr evealedintactmembr aneswi t
ht hehead1cm abov e
theischialspi
nes,occipi
toanteri
orposi
tionandt hecer vi
x5cm di l
ated.Shewas
commencedoncont i
nuousCTGmoni t
or i
ng( becauseoft heprev i
ouscaesar eansect ion),
whichshowedani nit
ialbaseli
nerateof135/ mi n,goodv ari
abil
i
t y
,accelerationsandno
decelerati
ons.Twentymi nutesago,spont aneousr uptureofmembr anesoccur redwi thclear
l
iquorleaking.

TheCTGt
race
»Descr
ibet
heCTG

»Whati
sthemostpossi
blecausef
ort
hisCTGpat
ter
n?

»Whatisyournextst
epint
hemanagementoft
hispat
ient
?immegi
atev
agi
nal
assesment
,
babydel
iv
eryinst
rument
al

CASEL-
5

A22-year
-oldG3P0womanpr esent
stothelabourwardwit
habdominalpai
n.Sheisnow40
weeksand6day s.Al
lpregnancybloodtest
sandultr
asoundscanshavebeennormal.The
babywasbr eechat34weeksbutcephal icat37weeks.Shehadashow2day sagobuthas
hadnobleedingsincethismor ni
ngshehadamucus- l
ikedarkr
eddischar
gefol
lowedbythe
onsetofi
rregularper
iod-
typepains.Thebabyhasmov ednormall
yduri
ngtheday.

Onexaminat
ionsheiscomfortabl
ebetweenpai
ns.Herbl
oodpr
essur
eis129/76mmHgand
pul
se101/min.Symphysi
ofundalhei
ghti
s37cm andthefet
usi
scephal
icwit
h2/5pal
pabl
e.

Vaginalexaminat
ionreveal
sthecerv
ixtobeful
lyef
facedand4cm dil
ated.Theposi
tionis
ri
ghtoccipit
opost
eriorandtheheadis2cm abovet
heischi
alspi
nes.Thereisnofet
al caput
moulding.

Themembr anesar
eint
actbutr
upt
urespont
aneousl
ydur
ingexami
nat
ion,
wit
hcl
earl
i
quor
drai
ning.

Thewomanrequest
sanepidur
alforpai
nreli
efandisther
efor
ecommencedoncont i
nuous
car
diot
ocogr
aphmonit
ori
ng.Aft
er20miny ouarecall
edintorev
iewt
hesi
tuat
ion.TheCTG
asyouwalki
nisbel
ow

»Descr
ibet
heCTG

»Whatar
epossi
blecausesf
ort
hisCTGpat
ter
n,EXCEPT?

»Whati
syournextst
epint
hemanagementoft
hispat
ient
?noev
idenceoff
etal
ormat
eranal
di
sor
dert
hatrequi
recar
e

CASEL-
6

A22- year-ol
dwomani sadmitt
edt othelabourwardforinductionoflabourat40weeks’
gestation.Thisi
sherfir
stongoingpregnancy,hav
inghadaf i
rst-
tri
mestermiscarri
age13
mont hspr evi
ousl
y.Shebookedat9weeksandhadnor mal bookingbloodtests.The11–14-
weekscanand21- weekanomal yscandidnotshowanyobv i
ousf et
alabnormalit
y.Bl
ood
pressur eanduri
nalysi
shavealwaysbeennor mal.
Ather32-
weekmidwif
eappointmentsherepor
tedfeeli
ngveryuncomf
ort
ableabdomi nal
l
y,
andthemidwi
femeasuredthesymphysi
ofundalhei
ghttobe36cm.Af ur
therul
trasound
scanwasther
efor
erequest
edwhichshowednor malfetal
growthbuti
ncr
easedli
quor
vol
ume.

Shehadbeenr evi
ewedintheant enatalcli
nicandwast est
edforgestati
onaldiabet eswith
glucosetol
erancetestbutthi
swasnor mal .Adecisionhadbeenmadef orinductionof
l
abourat40weeksbecauset hewomanhadbecomesouncomf ort
ableandbr eathless.On
palpati
onthefetuswascephal i
cwi t
ht hehead4/ 5pal pabl
eabdomi nall
y.Cardiotocograph
(CTG)wasr eassuri
ng;2mgofpr ostaglandingelhadbeeni nsert
edintothepost eri
orf or
nix
ofthev agi
naandCTGmoni tori
ngcont inuedforaf urt
her20mi n.

Thewomant henmobi li
zedandcont
ract
ionsstar
tedwit
hinanhour
.Sherequest
edan
epidural
foranalgesiaandwhil
ethi
swasbeingpreparedCTGmonitor
ingwascommenced.
Atthisstage,
spont aneousr
uptur
eofmembr anesoccurr
edwit
hav er
ylar
gev ol
umeofcl
ear
l
iquorsoakingthebedsheets.

TheCTGi
sshownbel
ow

»Descr
ibet
heCTGshowesdeepat
ypi
cal
var
iabl
edecel
erat
ion,
prol
aapseofcor
d

»Whati
sthemostpossi
blecausef
ort
hisCTGpat
ter
n?suddenct
gdet
eri
orat
ionaf
ter
r
apur
eofmemberane

»Whati
syournextst
epi
nthemanagementoft
hispat
ient
?ex
ami
nat
ionofv
agi
nabyf
inger

1.1A31-year
-ol
dG2P1womanat39weeks’ gest
ationcompl
ainsofpai
nfulut
eri
ne
cont
ract
ionsthatar
eoccurr
ingev
ery3t
o4mi nutes.Hercer
vixhasnotchangedfr
om 6-cm
di
lat
ionover3hours.Whi
choneofthef
oll
owingmanagementplansismostappropri
ate?

A.Cesar
eandel
i
ver
y

B.I
ntr
avenousoxy
toci
n

C.Obser
vat
ion+

D.Fet
alscal
ppHmoni
tor
ing

E.I
ntr
anasal
gonadot
ropi
nther
apy

1.2A26-year
-ol
dG2P1womanat41weeks’ gestati
onhasbeenpushingfor3hour swithout
progr
ess.Thr
oughoutt
histi
me,hervagi
nalexami nati
onhasremainedcomplet
elydil
ated,
complet
elyeff
aced,
and0stati
on,wi
ththeheadper si
stent
lyi
ntheocci
putpost
eriorpositi
on.
Whichofthefol
lowi
ngstat
ementsaccurat
elydescribesthesi
tuat
ion?

A.Theocci
putpost
eri
orposi
ti
oni
sfr
equent
lyassoci
atedwi
thagy
necoi
dpel
vi
s.
B.Thelaborprogr
essi
snor
malift
hepati
entdoesnothaveanepi
dur
alcat
het
erf
or
anal
gesia,buti
sabnor
mali
fepi
dural
anal
gesiai
sbeingused.

C.Thepat
ienti
sbestdescr
ibedashav
inganar
restofdescent
.+

D.Thebonypar
toft
hef
etal
headi
sli
kel
ytobeatt
hepl
aneoft
hepel
vi
cinl
et.

E.Mi
sopr
ost
olf
orcer
vical
ripeni
ng.

1.
3A31- year-
oldG2P1womanat40weeks’gest
ati
onhasprogressedinlaborfr
om 5t
o6
cm cer
vicaldi
lat
ionov
er3hour
s.Whi
chofthefoll
owingbestdescr
ibesthelabor?

A.Pr
olongedl
atentphase

B.Pr
olongedact
ivephase

C.Ar
restofact
ivephase

D.Pr
otr
act
edact
ivephase

E.Nor
mal
labor+

1.4A24-year
-ol
dG2P1womanat39weeks’ gest
ati
onpresentswit
hpai
nful
uter
ine
cont
ract
ions.Sheal
socompl
ainsofdark,
vaginalbl
oodmi x
edwi t
hsomemucus.Whichof
thef
oll
owingdescri
best
hemostli
kelyeti
ologyofherbl
eeding?

A.Pl
acent
apr
evi
a

B.Pl
acent
aabr
upt
ion

C.Bl
oodyshow+

D.Vasapr
evi
a

E.Cer
vical
lacer
ati
on

1.6A32- year-
oldG1P0womanat40weeks’ gestationalagear r
ivest othelaborf l
oorwit
h
frequentandstrongcont ract
ions.Thepatienthadbeenseen24hour spreviouslyand
thoughtt obeinlatentl
aborat2cm di lat
ed, 70%ef faced,fetalvert
exat–1st ati
on.Shewas
admi tt
edat3-cm di l
ati
on/80%ef facement ,
–1st ation.Sheunder goesamni otomy,andis
startedonoxy t
ocinforprolongedlatentphase.Af ter4hour sofoxy toci
n, sheisstil
lat3-
cm
dilat
ion,90%effacement ,–1st at
ion.Whichoft hef oll
owingi sthebestmanagementoft hi
s
pat i
ent’
slaboratthispoint?

A.Cesar
eandel
i
ver
y

B.Cont
inuedobser
vat
iononoxy
toci
n+

C.Di
schar
gehomewi
thf
oll
ow-
upi
n3day
s

D.Fol
eybul
bdi
l
ati
onoft
hecer
vix
1.5A24- y
ear-ol
dG2P1001womanat38weeksbyLMPandsuppor tedbya9-week
ul
trasoundstatesthathermotheri
sintownfort
henext4day sandwillbeav
ailabl
eto
assisti
ntakingcareofherbaby.Sherequest
saninduct
ionoflabor
.Whichofthefoll
owi
ng
i
st hebestresponsetothisr
equest?

A.Si
ncethepat
ienti
ster
m, t
her
eisnoincr
easedneonat
alcompl
i
cat
ions,
butani
ncr
eased
ri
skofcesar
eanascomparedtospont
aneouslabor
.

B.Ifthecervi
xisunf
avor
abl
e,t
henpr
ost
agl
andi
nri
peni
ngwoul
dincr
easet
hechangesf
or
vaginaldel
i
very.

C.I
nducti
onat38weeksi
ncr
easesneonat
alcompl
i
cat
ionsascompar
edt
odel
i
ver
yto39
weeks.

D.Thepati
ent
’srequesti
sreasonabl
e,andi
nduct
ioncanbeper
for
medwi
thl
i
ttl
eneonat
alor
mater
nalcompli
cati
ons.+

Medi
cal
pat
hol
ogyi
npr
egnancy

23.1A36- year-
oldG1P0womanat27weeks’ gestati
onisnotedt
ohav efev
er,r
ightfl
ank
tenderness,andpyuri
a.Sheisdi
agnosedwi
thpyelonephri
ti
s.Auri
necult
ureisperfor
med.
Whichoft hefoll
owingisthemostcommonlyi
solatedeti
ologi
cagentcausi
ngpy el
onephr
it
is
i
npr egnancy?

A.Pr
oteusspeci
es

B.Candi
daspeci
es

C.Escher
ichi
acol
i+

D.Kl
ebsi
ell
aspeci
es

»Ifpati
entdoesnotimproveonadequat
eantibi
oti
ctherapyf
or48hoursandexper
iences
conti
nuedseveref
lanktender
nessandfever
,whichofthefol
lowi
ngshoul
dbenext
consi
dered?

Ans-
USGandCTt
oassesshy
dronephr
osi
s.St
one,
uter
ineobst
ruct
ion

23.2A21-y
ear
-ol
dG1P0womanat15weeks’ gestat
ionisnotedtohav
efeverof101°
F
(38.
3°C)
,BPof80/40mm Hg,anddecreasedurineoutput.Whichoft
hefol
lowingi
sthe
mostcommoncauseofsept
icshocki
npr egnancy?

A.Pel
vi
cinf
lammat
orydi
sease

B.Py
elonephr
it
is+

C.Woundi
nfect
ion

D.Mast
it
is

2.
1A30-
year
-ol
dG1P0womancompl
ainsofnauseaandv
omi
ti
ngf
ort
hef
ir
st3mont
hsof
herpr
egnancy
.Sheisnot
edtohav
eahemoglobinl
evelof9.
0g/dLandameancorpuscul
ar
vol
umeof110f L(
normal90-
105f
L).Whi
chofthef
oll
owingist
hemostl
ikel
yet
iol
ogyofthe
anemia?

Or

A30- y
ear -
oldG1P0womancompl ai
nsofnauseaandv omi t
ingforthefi
rst3monthsofher
pregnancy.Onex aminat
ion,
herbloodpressur
e(BP)is110/ 70mm Hg, heartr
ate(HR)83
beatspermi nute(bpm),
andsheisafebril
e.Thethyr
oidglandappearsnormal onpal
pati
on.
Thehear tandlungexaminati
onsareunremarkabl
e.Thefundusi satthesymphysisl
evel
.
Theev al
uationoftheanemiaincl
udes:hemoglobinl
evelof9.0g/dLandamean
corpuscularvol
umeof110f L(normal90-105fL).

A.I
rondef
ici
ency

B.Fol
atedef
ici
ency+

C.Vi
tami
nB12def
ici
ency

D.Phy
siol
ogi
canemi
aofpr
egnancy

2.2A29-year
-ol
dG2P1womanat28weeks’ gest
ati
on,whohadnormalhemoglobi
nlevel4
weeksagoatherf i
rstpr
enatal
visi
t,complai
nsof1weekoff at
igueandnowhasa
hemoglobinl
evelof7.0g/dL.Shenot eddark-
col
oreduri
neaft
ertaki
nganant
ibiot
icfora
uri
nar
yt r
acti
nfecti
on.Whichofthefollowi
ngisthemostli
kel
ydiagnosi
s?

Or

A29-year-
oldG2P1womanat28weeks’ gestati
on,whohadnor malhemogl obinlevel4
weeksagoatherf irstprenatalv i
sit,compl ainsof1weekoff ati
gue.Herant enatalhist
oryis
unremarkableex ceptf oraurinaryt r
actinf ectiontr
eatedwi t
hanant ibi
oti
c1weeksago.On
examinati
on, herbloodpr essure( BP)i s100/ 60mm Hg, heartrate(HR)90beat spermi nute
(bpm),andshei saf ebril
e.Thet hyroidgl andappear snor malonpalpation.Thehear tand
l
ungex aminat i
onsar eunr emarkabl e.Thef undalheighti
s26cm.Thef et
alhearttonesarein
the140-to150- bpm r ange.Theev aluati
onoft heanemi aincl
udes:hemogl obinlevelof7.0
g/dLandameancor puscularv olumeof100f L(normal 90-
105f L)
.Shenot eddar k-
color
ed
uri
neaftertakinganant i
bioti
cf oraur inaryt r
actinfecti
on.

A.I
rondef
ici
encyanemi
a

B.Thal
assemi
a

C.Hemol
ysi
s+

D.Fol
atedef
ici
ency

E.Vi
tami
nB12def
ici
ency

CASEMP-5
Awomanat tendstheant enat
al dayassessmentuni ttodi scusst her esul
tofherglucose
tolerancet est.Sheis42y earsoldandt hi
sishersixthpr egnancy .Shehaspr evi
ouslyhad
threecaesar eansecti
ons, oneear l
ymi scar
ri
ageandat er minati
onofpr egnancy.Al
lbooking
testswer enor malaswer eher11–14- weekandanomal yul t
rasoundscans.Shei snow26
weeks’ gestationandhermi dwifear r
angedagl ucoset olerancet estbecauseofaf amily
historyoft ype2di abetes(herfatherandpat er
nalaunt ).Herbodymassi ndex(BMI)is31
kg/ m2.Bl oodpr essureis146/87mmHg.Thesy mphy siofundal heightis29cm andt hefetal
hear trateisnor malonauscul t
at i
on.

Herur
inal
ysi
sshowsglycosur
ia++.Gl
ucoset
olerancet
est(
75gglucosedri
nk)
:Pret
est
f
asti
ngbloodgl
ucose:
6.4mmol /,2hbl
oodglucosefol
lowi
nggl
ucoseload:
11.2mmol/L

»Whati
sthedi
agnosi
s?AnsGest
ati
onal
diabet
es

»Whi
choft
hef
oll
owi
ngi
sar
iskf
act
ori
npr
egnantwomenf
ori
mpai
redcar
bohy
drat
e
met
abol
i
sm?

AnsPresidentex
ist
ing-obesi
ty,
fai
mlyhi
stor
y,mat
ernal
ageandi
nthi
scaseGl
ycosur
iaand
pol
yhy
dr oamnios

CASEMP-6

A20- year-
oldwomani spregnantforthefi
rstti
me.Thepregnancyisunplannedandt he
partnerhasleftbutshei
ssuppor t
edbyhermot herandhasdecidedt
ocont inue.Shewas
diagnosedwi t
ht y
pe1diabetesatage15y ears.Shehasbeent aki
nglong-actingandshort
-
actinginsul
inunderthecareofhergeneralpracti
ti
oner(
GP),butther
eferrallett
ersuggest
s
thatshehasnotal waysbeencompl iant
.

Shehadapositi
vepregnancyt
est2weeksagoandherGPhasref
erredherur
gent
lyt
othe
ant
enatal
cli
nicf
orreviewinvi
ewofthedi
abet
es.Byherdat
essheisnow7weeksand5
days’

gest
ati
on.Shehasnoothersi
gni
fi
cantgynaecol
ogicalormedi
cal
hist
ory.Thewomanhasa
bodymassindexof29kg/m2.Bl
oodpressureis131/68mmHgandpul seis81/
min.

Haemogl
obi
n(Hb)
A1c7.
8%(
nor
mal
range<7.
0%)Ur
inal
ysi
s:gl
ucose++

»Whatf
urt
heri
nvest
igat
ionsneedt
obear
ranged?Mat
ernal
wel
lbei
ngandal
sof
orf
etal
wel
l
bei
ng

»Out
li
net
hepr
inci
plesofmanagementoft
hepr
egnancy
.

Ant
enat
al,I
nlabor,
postnat
al

Obst
ret
ri
cemer
gency

CASEOE-
1
Awomanaged28y ears,G4P3isinlabourwhenshesuddenl ycollapses.Thi spr egnancyhas
beenuncompl icatedandshehasbeenadmi t
tedwi thcont racti
onsat37weeksand6day s.
Thedel i
ver
ypr ocesshasbeenuncompl i
catedunt il
spont aneousr uptureofmembr anes
occurredduringacont r
acti
on,wit
hal ar gegushofcl earf luidfrom thev agina.Thewoman
report
edanur get opushatt hatstageandt henbecameconf usedanddi sori
entatedsay i
ng
thatshecouldnotbr eatheandwasgoi ngt odie.Immedi atelyfollowi ngt hi
sshecol lapsed,i
s
unrouseabl
et opai nfulsti
muli
.Thebl oodpr essur eis98/ 40mmHgandhear trat
e120/ min.
Theox ygensat urat i
onis86percentonai randr espiratoryr ate20/ mi n.Thehear tsounds
arenor malbutonchestex aminat
iont herear einspiratorycr acklest hroughoutt hechest .

Theabdomenissoftwi
thint
ermi
tt
entcont
racti
onsconti
nui
ng,
andi
nfactt
hef
etal
headi
s
nowvisi
bleatt
heperi
neum.Ther
eisnovaginalbl
eedi
ng.

»Whati
sthel
i
kel
ydiagnosisanddif
fer
enti
aldi
agnosi
s?ans-
amni
oti
cfl
uidembol
i
sm,
DD-
pul
monar
yembol
ism ,
my ocar
dial
inf
arct
ion

»Howwoul
dyoumanaget
hiswoman?
ans-babydel
i
ver
dimmedi
atel
y

A31- y
ear-
oldwomani sadmit
tedwi
thcontr
acti
onsat40weeks’
gest
ati
on.Thi
sisherf
our
th
pregnancy,hav
inghadtwoter
minat
ionsappr
oximat
ely10y
earsagoandanelect
ive
caesarean

sect
ionf
orbr
eechpr
esent
ati
on3y
ear
sago.

Shewasadmi t
tedwi t
hspont aneousr uptureofmembr anesaf ternex t2hcont racti
ons
becamest rongerandsher equestedanepi dural.Anepiduralwassi tedandani ndwelli
ng
uri
narycatheteri nserted.Threehoursl aterthewomanr eportedmor esev erepainwhi chdi
d
notdisappearbet weencont ract
ions.Att hattimeapprox i
mat ely200mLoff reshbloodwas
seencomi ngf rom thev agina.Thehear tratei s105/minandbl oodpr essure105/ 58mmHg.
Thewomanf eelswar m andwel lperfused.Theabdomeni ssof tandt heut erusisalsosoft
butverytender, witheasypal pati
onoff etal part
s.Onv aginalexami nationthecer vi
xis6cm
dil
atedandt hef etalheadf eelshighint hepelv i
sandpoor lyappliedtot hecer v
ix.The
cathetercontainsbl ood-stai
nedur i
ne.

»Whati
sthel
i
kel
ydi
agnosi
s?ans-ut
ri
ner
upt
ure

»Howwoul
dyoumanaget
hispat
ient
?ans-gener
alr
esusci
tat
iongi
veni
mmedi
atel
y

CASEOE-
3

A19- year-
oldG1P0womanat20weeks’ gestationcomplainsoft heacuteonsetofpleuri
ti
c
chestpainandsev eredyspnea.Shedeni esahi stor
yofreact i
veairwaydiseaseorcough.
Shehasnohi storyoftrauma.Onex amination, hert
emper aturei
s98° F(36.6°
C),heartr
ate
(HR)is120beat spermi nute,BPis130/ 70mm Hg, andrespirat
oryrateis40breathsper
minute.Thel ungex aminati
onr evealscl
earl ungsbilater
all
y.Thehear texaminati
onshows
tachycardi
a.Thef etalhearttonesareint her angeof140t o150bpm.Theoxy gensaturat
ion
l
ev el
is89%.Suppl ementalox ygenisgiv
en.

»Whatt
estwoul
dmostl
i
kel
yleadt
othedi
agnosi
s?pul
monar
yembol
usi
npr
egnancy
»Whati
stestmostl
i
kel
ytol
eadt
othedi
agnosi
s?spi
ral
comput
edt
omogr
aphy
,V/
Q
,i
magi
ngoflung

CASEOE-
4

Awomanaged26y earsisr eferredbyhergener alpr


acti
tioner.Sheis36weeks’ gestati
onin
herfourt
hpr egnancy,havi
nghadonemi scarr
iageandtwot ermv agi
nal deli
ver
ies.Forthe
l
ast2day sshehasbeenf eelinggener all
yunwel lwit
haf ever,decreasedappetiteanda
headacheaswel lasabdomi naldiscomf ort
.Shehasnotnot icedanyv aginalbl
eedi ngbuther
dischar
gehasbeenmor et hannor mal andt hereisanoffensiveodourt oit.Hert emperatur
e
i
s37. 8°
C,bloodpr essure106/ 68mmHgandhear trat
e109/ min.Thereisgener ali
zed
uteri
netendernessandi rr
itabilit
y.Fetalheartrateis170-180bpm.Onspecul um exami nati
on
thecervi
xisclosedandagr een/ greydischargeisseenwi thinthevagina.

I
nvest
igat
ion

Nor
mal
rangef
orpr
egnancy

Haemogl
obi
n

10.
9g/
dL

11–14g/
dL

Meancel
lvol
ume

80f
L

74.
4–95.
6fL

Whi
tecel
lcount

17.
3×109/
L

6–16×109/
L

Pl
atel
ets

327×109/
L

150–400×109/
L

C-
react
ivepr
otei
n

68mg/
L

<5mg/
L

»Whati
sthemostl
i
kel
ydi
agnosi
s?chor
ioamni
oni
ti
s
»Howwoul
dyoumanaget
hispat
ient
?ivbr
oadspect
rum ant
ibi
oti
cs,
I
Vfl
uids,
ster
oids

CASEOE-
5

A25-year-
oldG2P1womani sdeliv
eri
ngat42weeks’ gestat
ion.Thewomani snotedtohave
abodymassi ndexof42kg/ m2.Thefet
alweightcl
ini
call
yappearst obeabout3700g.Af t
er
a4-hourfi
rststageofl aboranda2-hoursecondstageoflabor,t
hef et
al headdel
i
v er
sbutis
notedtoberet r
actedbackt owardthepati
ent’
sint
roit
us.Thefetalshouldersdonotdeli
ver
,
evenwithmat ernalpushing.

»Whati
syournextst
epi
nmanagement
?mcr
ober
tsmaneuv
er

»Whatisali
kel
ycomplicat
ionthatcanoccurbecauseoft
hissi
tuat
ion?
post
par
tum
hemor
rhage,
neonat
alcomplicat
ions,

Mat
ernal
gest
ati
onal
diabet
is

Fet
albr
achi
alpl
exusi
njur
ies

CASEOE-
6

A23-year-
oldG2P1womanat16weeks’ gestati
oncompl ai
nsofa12- hourhistoryofcoli
cky,
ri
ghtlowerabdomi nalpai
n,andnauseawi thvomi ti
ng.Shedeni esv aginalbl
eedingor
l
eakageoff lui
dperv agi
na.Shedeniesdiarr
heaoreat i
ngst al
ef oods.Shehasahi stor
yofan
8-cm ovar
iancy st,andotherwi
sehasbeeni ngoodheal th.Shedeni esdy sur
iaorf ev
er,and
hashadnosur geries.HerBPis100/ 70mm Hg, heartr
at e-105bpm, respir
atoryrate-12
breat
hspermi nut e,andtemperatur
e99°F( 37.2°C).Onabdomi nalexami nat
ion,herbowel
soundsarehy poact i
ve.Theabdomeni stenderint herightlowerquadr antregionwith
si
gnifi
cantinvoluntaryguardi
ng.Thecervixisclosed.Thef etalhearttonesar eintherange
of140bpm.

»Whati
sthemostl
i
kel
ydi
agnosi
s?abdomi
nal
pai
ninpr
agnency(
ovar
ytor
sion)

»Whati
sthebestt
reat
mentf
ort
hiscondi
ti
on?
sur
ger
y,l
apar
otomy

POSTPARTUM HAMERRHAGE

Aft
era4- hourlabor,a31-year-
oldG4P3womanunder goesanuneventfulvagi
nal
deli
ver
yof
a7lb8ozi nfantoveranintactperineum.Duri
ngherl
abor
,sheisnotedtohavemildvar
iabl
e
decelerationsandaccel er
ati
onst hatincr
ease20beat
sperminute(bpm)abov et
hebaseli
ne
heartrate.Atdel i
very
,themal ebabyhasApgarscoresof8at1mi nute,and9at5minute.
Sli
ghtlengt
heningofthecordoccur
saft
er28minutesalongwit
hasmallgushofbl
oodper
vagina.Astheplacent
aisbeingdel
i
ver
ed,ashaggy
, r
eddish,
bul
gingmassisnot
edatthe
i
ntroit
usaroundt hepl
acent
a.

»Whati
sthemostl
i
kel
ydi
agnosi
s?

Ans-Mostl
i
kel
ydi
agnosi
s:Ut
eri
nei
nver
sion.

»Whati
sthemostl
i
kel
ycompl
i
cat
iont
ooccuri
nthi
spat
ient
?

Ans-
Mostl
i
kel
ycompl
i
cat
ion:
Post
par
tum hemor
rhage

3.1A23-y ear
-ol
dG1P0womanat38weeks’ gest
ati
ondeliver
eda7l b4ozbabyboy
vagi
nall
y.Upondeli
veryoftheplacenta,therewasnotedtobeani nver
tedut
erus,
whi
chwas
successfull
ymanagedincl
udingreplacementoftheuterus.Whichofthefol
l
owingpl
acent
al
i
mpl ant
ationsit
eswouldmostlikel
ypr edisposet
oani nv
erteduter
us?

A.Fundal+

B.Ant
eri
or

C.Post
eri
or

D.Lat
eral

E.Lowersegment

Whati
sthemostl
i
kel
ycompl
i
cat
ionoccuri
nthi
spat
ient
?

Secondar
ypph

3.2A24-year
-ol
dwomanunder wentanormalv
agi
naldel
i
veryofater
minfantf
emale.Aft
er
thedel
iv
ery,t
heplacent
adoesnotdeliv
erev
enaft
er30minutes.Whi
chofthef
oll
owing
wouldbethenextstepfort
hispat
ient
?

A.I
nit
iat
eoxy
toci
n

B.Wai
tforanaddi
ti
onal
30mi
nut
es

C.Hy
ster
ect
omy

D.At
temptamanual
ext
ract
ionoft
hepl
acent
a+

E.Mi
sopr
ost
olest
rogeni
ntr
avagi
nal
l
y

Whi
choft
hef
oll
owi
ngi
sbestexpl
ani
tat
ionofmechani
smsofhamer
rhage?

Ansi
nver
tdut
erus

A29-year-
oldparous( G5P4)womanat39weeks’ gestat
ionwi t
hpr eecl
ampsiadel
iv
er s
vagi
nall
y.Herprenatalcoursehasbeenuncompli
catedex ceptforasymptomati
cbact er
iur
ia
causedbyEscherichiacolii
nthefi
rstt
ri
mestert
reatedwithoral cephal
exi
n.Shedeniesa
f
ami
lyhistor
yofbl
eedi
ngdi
athesi
s.Af
tert
hepl
acent
aisdel
ivered,ther
ei sappr
eci
abl
e
v
agi
nalbleedi
ngest
imat
edat1000cc.»Whati
sthemostli
kelydiagnosis?

Ans-Mostl
i
kel
ydi
agnosi
s:Ut
eri
neat
ony
.

»Whati
sthenex
tst
epi
nther
apy
?

AnsNex tstepinther
apy:Diluteint
ravenous(I
V)oxyt
oci
n,bedsi
deuter
inemassageand
compression,andift
hisisineffect
ive,t
henint
ramuscul
arprost
agl
andi
nF2-al
pha(H
emabate)orrectalmisoprostol

6.
2A26- year-ol
dG2P1001womanunder wentanormal vaginal
deli
ver
y.Avi
able7l b4oz
maleinfantwasdel iver
ed.Theplacentadeli
veredspontaneousl
y.Theobst
etr
iciannoted
si
gnifi
cantbl oodlossfrom t
hevagina,tot
ali
ngapproximately700mL.Theuteri
nef undus
appearedt obewel lcontr
act
ed.Whi chofthefoll
owingisthemostcommonet iologyfort
he
bl
eedingint hispati
ent?

A.Ret
ainedpl
acent
a

B.Geni
tal
tractl
acer
ati
on+

C.Ut
eri
neat
ony

D.Coagul
opat
hy

E.Endomet
ri
alul
cer
ati
on

6.4A34-y
ear
-ol
dwomani snotedtohavesi
gnif
icantut
eri
nebleedingaf
teravaginaldel
iver
y
compli
cat
edbyplacentaabrupt
ion.Shei
snotedtobebleedi
ngf r
om multi
plevenipunct
ure
si
tes.Whi
choft
hef ol
lowingisthebestt
her
apy?

A.I
mmedi
atehy
ster
ect
omy

B.Packi
ngoft
heut
erus

C.Hy
pogast
ri
car
ter
yli
gat
ion

D.Li
gat
ionofut
ero-
ovar
ianl
i
gament
s

E.Cor
rect
ionofcoagul
opat
hy+

Pr
eecl
ampsi
A

A19- y
ear-oldG1P0womanat29weeks’ gestationar ri
vest othehospi t
albecauseofsev er
e
dyspneaof6hour s’ durati
on.Herpr enatal coursehasbeenunr emarkable, andshedeni es
anymedi cal problems.Herbl oodpr essure( BP)i s160/ 114mm Hg, heartrat e(HR)is105
beatspermi nute( bpm) ,r
espiratoryrate(RR)i s40br eathspermi nuteandl abor ed,and
oxygensat urationi s90%.Thef etalhearttonesar einther angeof140bpm.Aur i
neprotei
n
tocreati
niner atiois0. 6.Theser um alaninet r
ansami nase( ALT)i
s84I U/L( normal <35)and
aspartat
et ransami nase(AST)i s90I U/L( normal <35).Thepr enatalrecordsshowt he
fol
lowing:Gest ational AgeBP( mm Hg)Ur inePr otei
nFHT( bpm)Fundal Hei ght(cm)8weeks
100/60014012weeks110/
70014816weeks100/
76015020weeks105/
5801382026
weeks130/891+14225

»Whati
sthemostl
i
kel
ydi
agnosi
s?ansMostl
i
kel
ydi
agnosi
s:Pr
eecl
ampsi
awi
thsev
ere
f
eat
ures

»Whati
syouri
mmedi
atenextst
ep?

AnsImmedi atenex tstep:Thehi ghestpri


ori
tymustbetoimproveoxygenati
on.Suff
ici
ent
oxygenmustbepr ovidedtor aisetheO2saturat
ion>94%,andifthepati
entisti
ri
ng,
venti
lat
orsupportmayber equired.Thesecondpri
ori
tyi
stolowertheBPwi thi
ntrav
enous
(I
V)antihyper
tensi
v eagent s.Ifpulmonaryedemaisconfi
rmed,I
Vdi ur
esissuchas
fur
osemi deshouldbegi ven

CASEP-
2

A18- year
-ol
dG1P0womanat28weeks’ gest
ationisadmit
tedt
othehospitalwit
hablood
pressureof160/110mm Hg,el
evatedl
i
verfuncti
ontest
s,andaplat
eletcountof60000/μL.
Shecompl ai
nsofasever
eheadache.

»Whi
choft
hef
oll
owi
ngi
sthebestmanagement
?ansmagnesi
um sul
phat
e

»Whichoft
hef
oll
owi
ngwi
l
lbet
hemostcommonmechani
sm i
fsei
zur
esappeari
nthi
s
pat
ient
?

16.4A33-year
-ol
dwomanat29weeks’ gest
ati
onisnotedtohav eabloodpressur
eof150/
90mm Hgandapr ot
ein/creat
ini
nerati
oof0.6.Theplat
eletcount
,li
verfunct
iontest
s,and
creat
ini
nearenormal
.Whichofthefoll
owingisthebestmanagementf orthi
spati
ent?

A.I
nduct
ionofl
abor

B.Cesar
eansect
ion

C.Ant
ihy
per
tensi
vet
her
apy

D.Expect
antmanagement+

16.6Onpost part
um day1,a28-year-ol
dG1P1womanrepor
tssomeheadacheandprobl
ems
withhervi
sionbilat
eral
ly
.HerBPi s150/95mm HgandP/Cr at
ioi
s0.5.Herneurol
ogi
cal
examinati
oni snormalbutherv
isionisimpai
redi
nbot
heyes.Whi
chofthefol
lowi
ngisthe
bestnextstep?

A.Ant
ihy
per
tensi
veagent

B.I
VManni
tol

C.MRIoft
hebr
ain+

D.CTi
magi
ngoft
hebr
ain

E.Opht
hal
micey
edr
opst
obot
hey
es
16.
7A32-year
-ol
dG2P1womani
sat35weekswi
thchr
oni
chy
per
tensi
on.TheBPi
sint
he
140/95r
ange.

A.Cor
ti
cost
eroi
ds

B.Ant
ihy
per
tensi
veagent

C.Bi
ophy
sical
prof
il
e+

D.Magnesi
um sul
fat
eanddel
i
ver
y

E.Cont
inuedobser
vat
ion

16.
8A28-y
ear
-ol
dG1P0womani sat30weeks’gest
ati
onwit
hsuper
imposedpr
eecl
ampsi
a.
TheBPi
s150/100.Thepl
atel
etcounti
s95000andLFTis2×normal
.BPPis10/10.

A.Cor
ti
cost
eroi
ds+

B.Ant
ihy
per
tensi
veagent

C.Bi
ophy
sical
prof
il
e

D.Magnesi
um sul
fat
eanddel
i
ver
y

E.Cont
inuedobser
vat
ion

16.
9A30- year
-ol
dG2P1womani
sat31weekswit
hchroni
chyper
tensi
on,usi
ngoral
l
abetol
ol.HerBPint
heof
fi
cei
s160/95and162/90.Theur
inepr
otei
nisnegati
ve.

A.Cor
ti
cost
eroi
ds

B.Ant
ihy
per
tensi
veagent+

C.Bi
ophy
sical
prof
il
e

D.Magnesi
um sul
fat
eanddel
i
ver
y

E.Cont
inuedobser
vat
ion

16.10A24- y
ear-
oldG3P2womanat34weeks’gestat
ioni
snotedtohav
epreecl
ampsi
a.The
BPi s150/90andP/Cr at
ioi
s0.
5.Afet
alul
trasoundshowstheesti
matedf
etal
wei
ghtisat
the8thpercent
il
e.

A.Cor
ti
cost
eroi
ds

B.Ant
ihy
per
tensi
veagent

C.Bi
ophy
sical
prof
il
e+

D.Magnesi
um sul
fat
eanddel
i
ver
y

E.Cont
inuedobser
vat
ion
PRETERM DELI
VERY

Aheal thy19-year-oldG1P0womanat29weeks’ gestat


ionpresentst othel aboranddel iver
y
areacompl ainingofi nter mit
tentabdomi nalpai
n.Shedeni esleakageoff luidorbl eedingper
vagina.Herant enat alhistoryhasbeenunr emarkable.Shehasbeeneat inganddr inking
normal ly
.Onex ami nati
on, herbloodpressure(BP)is110/ 70mm Hg, hear trate(HR)i s90
beatspermi nut e( bpm) ,andt emperatureis99°F(37.2°C)
.Thef etalheartr at
et r
acing
revealsabasel inehear tr at
eof120bpm andar eacti
vepat t
ern.Uteri
necont ract
ionsar e
occurringevery3t o5mi nutes.Onpel vi
cexaminat i
on,hercervi
xis3cm di lated,90%
effaced,andt hefet alvertexispresentingat–1st ati
on.»Whati st hemostl i
kelydiagnosis?
AnsMostl i
kelydi agnosi s:Preter
ml abor.

»Whati
syournextst
epi
nmanagement
?

Nextst
epinmanagement:Tocol
ysi
s,tr
ytoident
if
yacauseoft
hepr
eter
mlabor
,ant
enat
al
st
eroi
ds,andant
ibi
oti
csf
orGBSpr ophyl
axi
s

17.
1A26- year
-ol
dwomani snot
edtobeat29weeks’gest
ation.Herl
astpregnancyended
i
ndeli
veryat30weeks’ gest
ati
on.I
nscreeni
ngforv
ari
oustypesofinf
ect
ion,whichofthe
fol
l
owingismostlikel
ytobeassociat
edwi t
hpr
eter
m del
iv
ery ?

Or

A26- year-
oldwomani snotedtobeat29weeks’ gest
ation.Herl
astpregnancyendedi n
deli
veryat30weeks’ gest
ation.Onexami nation,
hertemper at
ureis96.8°F(36.6°C),blood
pressure(BP)is100/60mm Hg, andheartrate(HR)is80beat spermi nut
e( bpm) .Herlungs
arecleartoauscult
ati
on.Nocost over
tebralangletendernessisfound.Theut eri
nef undal
heightis30cm, andtheuterusissli
ghtl
yt endertopalpati
on.Nol owerextr
emi tycordsare
palpated.Thefetal
heartt
onesar epersi
stent l
yintherangeof150t o155bpm wi thout
decelerati
ons.Ther
earenout eri
necontractions.

»Whatscreeni
ngt
estneedtobeprov
idedtoest
imat
ether
iskf
orpr
eter
m del
i
ver
y?usg,
l
ab
t
est,
uter
inemonit
eri
ng,pel
i
v cexami
nati
on

»Inscreeni
ngforv
arioustypesofinfect
ion,whi
choft
hef
oll
owi
ngi
smostl
i
kel
ytobe
associ
atedwit
hpreterm del
iver
y?gonococcal

A.Her
pessi
mpl
exv
irus

B.Candi
dav
agi
nit
is

C.Chl
amy
diacer
vici
ti
s

D.Gonococcal
cer
vici
ti
s+

E.Gr
oupBst
rept
ococcusoft
hev
agi
na

17.
2A25-
year
-ol
dG1P0womani
sat28weeks’
gest
ati
on.Shei
snot
edt
ohav
eregul
ar
uteri
necont r
acti
ons,andhercer
vixisdil
atedat2cm and80%effaced.Pret
ermlabori
s
diagnosed.Thephy si
cianr
evi
ewst herecordandnot
esthatt
hepat i
entshouldnothav
e
tocoly
tictherapy
.Whichoneofthef ol
l
owi ngi
sacontrai
ndi
cat
ionfortocoly
sis?

A.Suspect
edpl
acent
alabr
upt
ion

B.Gr
oupBst
rept
ococcal
bact
eri
uri
a+

C.Recentl
apar
otomy

D.Ut
eri
nef
ibr
oids

Whatmedi
cat
ioncanbegi
vent
odecr
easet
her
iskofneur
ologi
cal
impar
ementi
nthebaby
?

Ansant
enat
alst
eroi
d,mgso4

17.
3A35- year -
oldG1P0womanat32weeks’ gestati
onwasseeni ntheobst etri
c(OB)tri
age
uni
tthepr ev i
ousdaywi t
huteri
necont r
acti
ons.Onadmi ssion,
thefet
al heartrateis140bpm
wit
haccel erationsandnodecel erat
ions.Afetalfi
bronectinassayisperformed, whichwas
posit
ive.Ov erthecour seofthenext24hours, t
hepat i
entwasex aminedandnot edtohave
cerv
icaldilati
onf rom 1to2cm andef f
acementf r
om 30%t o90%.At ocolyt
icagenti sused.
Arepeatf etalhear tr
atepatt
ernrevealsabaselineof140bpm wi t
hmoder ater epeti
ti
ve
var
iabledecel erat
ions.Whichofthef ol
lowi
ngi sthemostl ikel
ytocol
y t
icagentused?

A.Ni
fedi
pine

B.I
ndomet
haci
n+

C.Magnesi
um sul
fat
e

D.Ter
but
ali
ne

»Whatar
ereasonsfort
ocol
ysi
sincaseofpr
eter
m cont
ract
ions?
anscor
dcompr
essi
on,
r
ept
ureofmembr ane

17.4A28- year-
oldwomanG1P0at29weeks’ gestat
ioni
st r
eatedwi t
hterbut
ali
nef or
preter
ml abor.Hercervi
xhaddilatedt
o3cm andwas90%ef faced.Shealsoreceived
betamethasonei nt
ramuscular
lytoenhancef
etall
ungmaturity.Thefoll
owingday,the
pati
entdev el
opsdy spnea,
tachypnea,
andanox y
gensaturati
onl ev
elof80%.Ox ygenisgi
ven.

Whatcompl
i
cat
ioni
smostl
i
kel
y?anspul
monar
yedema,
respi
rat
orydepr
essi
on,
duct
us
ar
ter
ious

»Whatar
ereasonsf
ort
ocol
ysi
sincaseofpr
eter
m cont
ract
ions?
anscor
dcompr
essi
on

A24-year-oldG2P1womanat30weeks’ gestati
onwasadmi ttedtothehospi
tal2daysago
forpr
emat ur eruptur
eofmembr anes.Herant enatal
historyhasbeenunr emarkabl
e.Today,
shestatest hatherbabyismov i
ngnor mall
y,andshedeni esanyf everorchi
ll
s.Herpast
medicalandsur gi
calhi
stor
iesareunremarkable.Onex ami nat
ion,hertemper
aturei
s100.8°F
(38.
2°C),bloodpr essur
e(BP)is100/60mm Hg, andhear trate(HR)is90beatsperminute
(bpm).Herlungsarecleartoauscultat
ion.Nocost ov
er t
ebralangl
etender
nessisfound.The
uter
inefundalhei
ghtis30cm, andtheuterusissl
ightlytendertopal
pati
on.Nolower
extr
emitycordsarepalpated.Thefetalheartt
onesar epersist
entl
yintherangeof170to
175bpm wi t
houtdecelerati
ons.Therearenout er
inecont r
acti
ons.

»Whatist
hemostl i
kel
ydi
agnosi
s?Mostl
i
kel
ydi
agnosi
s:I
ntr
a-amni
oti
cinf
ect
ion
(
chor
ioamnioni
ti
s).

»Whatisthebestmanagementf
orthi
spat
ient
?Bestmanagementforthispat
ient
:
I
ntr
avenous(I
V)anti
biot
ics(
ampi
cil
l
inandgent
amici
n)andinduct
ionoflabor

Puer
per
ium

A24- year -
oldG1P1womanunder wental ow-transv ersecesar eansect i
on2day sagof or
arrestofact i
vephaseofl abor .Sher equi redoxy t
ocinandani nternal ut
erinepressur e
catheter.Sher eachedandper sist
edat6- cm dilationfor3hour sdespi t
eadequat eut erine
contractionsasj udgedby240Mont ev ideouni t
s.Herbabywei ghed8l b9oz .Thepast
medi cal andsur gical hi
stori
eswer eunr emar kable.Shedeni esacoughordy suri
a.On
exami nat i
on,t
het emper aturei s102° F( 38.8°C), heartrateis80beat spermi nute, bl
ood
pressur eis120/ 70mm Hg, andr espirat oryratei s12br eathspermi nute.Thebr eastsar e
nontender .Thel ungsar ecleart oauscul tati
on.Ther ei snocost overtebralanglet enderness.
Theabdomenr ev ealsthattheski ninci sioniswi thouterythemaort enderness.Theut erine
fundusi sf i
rm,att heleveloftheumbi licus, andsomewhatt ender.Nol owerex tr
emi tycor ds
arepal pated.

»Whichofthefoll
owingi
sthemostsi
gnif
icantr
iskf
act
orinthi
spat
ient
?ansal
cohal
abuse,
i
nfer
ti
li
ty,
int
erv
oursesur
ingmenses,l
owbodyweight

»Whati
sthebestt
her
apyf
ort
hecondi
ti
on?
ansI
Vant
ibi
oti
csgent
amy
cinecl
i
ndamy
cine

25.
1A30-year
-ol
dG1P1whounderwentacesareansect
ion3dayspr
eviousl
yhasafev
erof
101°F(
38.
3°C)
.Theski
nincisi
oni
sindur
ated,
tender
,andery
themat
ous.Whichoft
he
fol
lowi
ngi
sthebestmanagement
?

A.I
nit
iat
ionofi
ntr
avenousampi
ci
ll
i
n

B.I
nit
iat
ionofi
ntr
avenoushepar
in

C.Pl
acementofawar
m compr
essont
hewound

D.Openi
ngoft
hewound+

»Whatisthebestt
herapyfort
heconditi
on?
ans-
prophl
act
icant
ibi
oti
csdur
ing
sur
ger
y,anti
micr
obi
alagentduri
ngdressi
ngs

25.2A29-year
-ol
dwomani sdiagnosedwithpostpartum endometri
ti
sbasedonfever
,
abdominal
pain,f
undal
tenderness,andeli
minati
onofot hereti
ologi
es.Whichoft
he
fol
lowi
ngisthemostsi
gnifi
cantriskfact
orforpostpart
um endomy ometr
it
is?

A.Numer
ousv
agi
nal
exami
nat
ions
B.Bact
eri
alv
agi
nosi
s

C.Cesar
eandel
i
ver
y+

D.I
nter
nal
uter
inepr
essur
emoni
tor
s

E.Pr
olongedr
upt
ureofmembr
anes

Whati
sthebestt
her
apyf
ort
hecondi
ti
on?
ansI
Vant
ibi
oti
cs

25.3A27- year-oldG1P0womanat39weeks’ gestati


oni snot
edtobei nlabor
.She
underwentar ti
fici
alruptureofmembr anes,andexper i
encesfetal
bradycardi
a.Palpat
ionof
thevaginar evealsar ope-l
ikestructureprolapsi
ngt hroughthecervi
x.Sheisdiagnosedwith
acordpr olapseandunder wentst atcesareandel i
very.Onpostoperati
veday2, t
hepatient
hasat emper atureof102° F( 38.8°
C) ,
andi sdiagnosedwi thendometri
ti
s.Thepat i
entwho
worksint hemi crobiologylaboratoryaskswhi choft hefoll
owingisthemostcommonl y
i
solatedbact eri
ai nherinfection?

A.Pept
ost
rept
ococcusspeci
es+

B.St
aphy
lococcusaur
eus

C.Gr
oupBSt
rept
ococcus

D.Escher
ichi
acol
i

Whichoft
hefol
lowi
ngi
sthemostsi
gni
fi
cantr
iskf
act
ori
nthi
spat
ient
?ans-Post
par
tum
endomyomet
ri
ti
s

25.4A22- year
-ol
dwomanwhounder wentcesar eandel
iv
eryhaspersi
stentfeverof102°
F
(38.
8°C),despit
etheuseoft r
ipl
eanti
bioti
ctherapy(ampici
l
li
n,gent
amicin,andcli
ndamycin)
.
Theurinalysi
s,wound,breast
s,anduter
inefundusarenormalonexamination.ACTscanof
thepelvi
sissuggestiv
eofsept i
cpelv
icthrombophlebi
ti
s.Whichofthefoll
owingisthebest
ther
apyf orthi
scondit
ion?

A.Hy
ster
ect
omy

B.Di
scont
inueant
ibi
oti
cther
apyandi
nit
iat
eint
rav
enoushepar
in

C.Cont
inueant
ibi
oti
cther
apyandbegi
nint
rav
enoushepar
in+

D.Sur
gical
embol
ect
omy

E.St
rept
oki
naset
her
apy

A20- year-ol
dparouswomancompl ainsofr i
ghtbreastpai
nandf ever.Shest atesthat3
weekspr evi
ousl
y,sheunderwentanor mal spontaneousvaginaldeliv
ery .Shehadbeen
breastfeedingwit
houtdiff
icult
yunti
l2day sago, whenshenot edpr ogressivepain,indurat
ion,
andr ednesst ot
her i
ghtbreast.Onexamination,hertemperatureis102° F(38.8°C),blood
pressurei s100/
70mm Hg, andheartrat
ei s110beat spermi nute.Hernecki ssupple.Her
ri
ghtbr easthasindurat
ionont heupperouterregionwithrednessandt enderness.Ther ei
s
al
sosigni
fi
cantfl
uctuancenotedinthebr
eastt
issue.Theabdomenisnontenderandt
her
eis
nocostov
ert
ebralangletender
ness.Thepel
vi
cex aminat
ioni
sunremarkabl
e.

»Whati
sthemostl
i
kel
ydi
agnosi
s?Mostl
i
kel
ydi
agnosi
s:Abscessoft
her
ightbr
east
.

»Whatisyournextst
epinther
apy
?Nextst
epi
nther
apy
:Inci
sionanddr
ainageoft
he
abscessandant
ibi
oti
cther
apy

26.3A28- year-
oldG1P1womanhasdel i
veredvagi
nal
ly3weeksago.Shei sbr eastfeeding
andnot esthatthebabypr ef
erstobr east
feedfr
om therightbreast
.Ontheleftbr east, she
notesa3- dayhistoryofatendermassont heupperouterquadrant.Onexami nati
on, shei s
afebri
le.Theleftbreasthasafluctuantmassof4×8cm oft heupperout
erquadr ant
withoutredness.Itissomewhatt ender.Whichofthefol
lowingisthebesttreatmentf orthi
s
conditi
on?

A.Or
alant
ibi
oti
cther
apy

B.Or
alant
if
ungal
ther
apy

C.Br
omocr
ipt
inet
her
apy

D.Aspi
rat
ion+

E.Mast
ect
omy

Whati
sthemostl
i
kel
ydi
agnosi
s?ansgal
act
ocel
e(mi
l
kret
ent
ioncy
st)

You might also like